ΠžΠ±Ρ‰Π΅Π΅ сопротивлСниС ΠΏΠ°Ρ€Π°Π»Π»Π΅Π»ΡŒΠ½ΠΎΠΉ рСзистивной Ρ†Π΅ΠΏΠΈ: РасчСт сопротивлСния для ΠΏΠ°Ρ€Π°Π»Π»Π΅Π»ΡŒΠ½ΠΎΠ³ΠΎ соСдинСния

Π‘ΠΎΠ΄Π΅Ρ€ΠΆΠ°Π½ΠΈΠ΅

ЀизичСская Ρ„ΠΎΡ€ΠΌΡƒΠ»Π° расчСта (опрСдСлСния) эквивалСнтного сопротивлСния Π² Ρ†Π΅ΠΏΠΈ

Если элСктричСская Ρ†Π΅ΠΏΡŒ содСрТит нСсколько рСзисторов, Ρ‚ΠΎ для подсчёта Π΅Ρ‘ основных ΠΏΠ°Ρ€Π°ΠΌΠ΅Ρ‚Ρ€ΠΎΠ² (силы Ρ‚ΠΎΠΊΠ°, напряТСния, мощности) ΡƒΠ΄ΠΎΠ±Π½ΠΎ всС рСзистивныС устройства Π·Π°ΠΌΠ΅Π½ΠΈΡ‚ΡŒ Π½Π° ΠΎΠ΄Π½ΠΎ эквивалСнтноС сопротивлСниС Ρ†Π΅ΠΏΠΈ. Волько для Π½Π΅Π³ΠΎ Π΄ΠΎΠ»ΠΆΠ½ΠΎ Π²Ρ‹ΠΏΠΎΠ»Π½ΡΡ‚ΡŒΡΡ ΡΠ»Π΅Π΄ΡƒΡŽΡ‰Π΅Π΅ Ρ‚Ρ€Π΅Π±ΠΎΠ²Π°Π½ΠΈΠ΅: Π΅Π³ΠΎ сопротивлСниС Π΄ΠΎΠ»ΠΆΠ½ΠΎ Π±Ρ‹Ρ‚ΡŒ Ρ€Π°Π²Π½Ρ‹ΠΌ суммарному Π·Π½Π°Ρ‡Π΅Π½ΠΈΡŽ сопротивлСний всСх элСмСнтов, Ρ‚ΠΎ Π΅ΡΡ‚ΡŒ показания Π°ΠΌΠΏΠ΅Ρ€ΠΌΠ΅Ρ‚Ρ€Π° ΠΈ Π²ΠΎΠ»ΡŒΡ‚ΠΌΠ΅Ρ‚Ρ€Π° Π² ΠΎΠ±Ρ‹Ρ‡Π½ΠΎΠΉ схСмС ΠΈ Π² ΠΏΡ€Π΅ΠΎΠ±Ρ€Π°Π·ΠΎΠ²Π°Π½Π½ΠΎΠΉ Π½Π΅ Π΄ΠΎΠ»ΠΆΠ½Ρ‹ ΠΈΠ·ΠΌΠ΅Π½ΠΈΡ‚ΡŒΡΡ. Π’Π°ΠΊΠΎΠΉ ΠΏΠΎΠ΄Ρ…ΠΎΠ΄ ΠΊ Ρ€Π΅ΡˆΠ΅Π½ΠΈΡŽ Π·Π°Π΄Π°Ρ‡ называСтся ΠΌΠ΅Ρ‚ΠΎΠ΄ΠΎΠΌ свёртывания Ρ†Π΅ΠΏΠΈ.

ΠœΠ΅Ρ‚ΠΎΠ΄ свёртывания Ρ†Π΅ΠΏΠΈ

Π’Π½ΠΈΠΌΠ°Π½ΠΈΠ΅! Расчёт эквивалСнтного (ΠΎΠ±Ρ‰Π΅Π³ΠΎ ΠΈΠ»ΠΈ суммарного) сопротивлСния Π² случаС ΠΏΠΎΡΠ»Π΅Π΄ΠΎΠ²Π°Ρ‚Π΅Π»ΡŒΠ½ΠΎΠ³ΠΎ ΠΈΠ»ΠΈ ΠΏΠ°Ρ€Π°Π»Π»Π΅Π»ΡŒΠ½ΠΎΠ³ΠΎ ΠΏΠΎΠ΄ΠΊΠ»ΡŽΡ‡Π΅Π½ΠΈΡ выполняСтся ΠΏΠΎ Ρ€Π°Π·Π½Ρ‹ΠΌ Ρ„ΠΎΡ€ΠΌΡƒΠ»Π°ΠΌ.

ΠŸΠΎΡΠ»Π΅Π΄ΠΎΠ²Π°Ρ‚Π΅Π»ΡŒΠ½ΠΎΠ΅ соСдинСниС элСмСнтов

Π’ случаС ΠΏΠΎΡΠ»Π΅Π΄ΠΎΠ²Π°Ρ‚Π΅Π»ΡŒΠ½ΠΎΠ³ΠΎ ΠΏΠΎΠ΄ΠΊΠ»ΡŽΡ‡Π΅Π½ΠΈΡ всС ΠΏΡ€ΠΈΠ±ΠΎΡ€Ρ‹ ΡΠΎΠ΅Π΄ΠΈΠ½ΡΡŽΡ‚ΡΡ ΠΏΠΎΡΠ»Π΅Π΄ΠΎΠ²Π°Ρ‚Π΅Π»ΡŒΠ½ΠΎ Π΄Ρ€ΡƒΠ³ с Π΄Ρ€ΡƒΠ³ΠΎΠΌ, Π° собранная Ρ†Π΅ΠΏΡŒ Π½Π΅ ΠΈΠΌΠ΅Π΅Ρ‚ Ρ€Π°Π·Π²Π΅Ρ‚Π²Π»Π΅Π½ΠΈΠΉ.

ΠŸΡ€ΠΈ Ρ‚Π°ΠΊΠΎΠΌ ΠΏΠΎΠ΄ΠΊΠ»ΡŽΡ‡Π΅Π½ΠΈΠΈ сила Ρ‚ΠΎΠΊΠ°, проходящая Ρ‡Π΅Ρ€Π΅Π· ΠΊΠ°ΠΆΠ΄Ρ‹ΠΉ рСзистор, Π±ΡƒΠ΄Π΅Ρ‚ одинаковая, Π° ΠΎΠ±Ρ‰Π΅Π΅ ΠΏΠ°Π΄Π΅Π½ΠΈΠ΅ напряТСния складываСтся ΠΈΠ· суммарных ΠΏΠ°Π΄Π΅Π½ΠΈΠΉ напряТСния Π½Π° ΠΊΠ°ΠΆΠ΄ΠΎΠΌ ΠΈΠ· ΠΏΡ€ΠΈΠ±ΠΎΡ€ΠΎΠ².

ΠŸΠΎΡΠ»Π΅Π΄ΠΎΠ²Π°Ρ‚Π΅Π»ΡŒΠ½ΠΎΠ΅ ΠΏΠΎΠ΄ΠΊΠ»ΡŽΡ‡Π΅Π½ΠΈΠ΅ ΠΏΡ€ΠΈΠ±ΠΎΡ€ΠΎΠ²

Π§Ρ‚ΠΎΠ±Ρ‹ ΠΎΠΏΡ€Π΅Π΄Π΅Π»ΠΈΡ‚ΡŒ суммарноС Π·Π½Π°Ρ‡Π΅Π½ΠΈΠ΅ Π² этом случаС, Π²ΠΎΡΠΏΠΎΠ»ΡŒΠ·ΡƒΠ΅ΠΌΡΡ Π·Π°ΠΊΠΎΠ½ΠΎΠΌ Ома, ΠΊΠΎΡ‚ΠΎΡ€Ρ‹ΠΉ записываСтся ΡΠ»Π΅Π΄ΡƒΡŽΡ‰ΠΈΠΌ ΠΎΠ±Ρ€Π°Π·ΠΎΠΌ:

I = U/R.

Из Π²Ρ‹ΡˆΠ΅ΡΡ‚ΠΎΡΡ‰Π΅Π³ΠΎ выраТСния ΠΏΠΎΠ»ΡƒΡ‡Π°Π΅ΠΌ Π·Π½Π°Ρ‡Π΅Π½ΠΈΠ΅ R:

R = U/I (1).

ΠŸΠΎΡΠΊΠΎΠ»ΡŒΠΊΡƒ ΠΏΡ€ΠΈ ΠΏΠΎΡΠ»Π΅Π΄ΠΎΠ²Π°Ρ‚Π΅Π»ΡŒΠ½ΠΎΠΌ соСдинСнии:

  • I = I1 = I2 =…= IN (2),
  • U = U1 + U2 +…+ UN (3),

Ρ„ΠΎΡ€ΠΌΡƒΠ»Π° для расчёта эквивалСнтного сопротивлСния (RΠΎΠ±Ρ‰ ΠΈΠ»ΠΈ Rэкв) ΠΈΠ· (1) – (3) Π±ΡƒΠ΄Π΅Ρ‚ ΠΈΠΌΠ΅Ρ‚ΡŒ Π²ΠΈΠ΄:

  • Rэкв = (U1 + U2 + …+ UN)/I,
  • Rэкв = R1 + R2 + … + RN (4).

Π’Π°ΠΊΠΈΠΌ ΠΎΠ±Ρ€Π°Π·ΠΎΠΌ, Ссли имССтся N ΠΏΠΎΡΠ»Π΅Π΄ΠΎΠ²Π°Ρ‚Π΅Π»ΡŒΠ½ΠΎ соСдинённых ΠΎΠ΄ΠΈΠ½Π°ΠΊΠΎΠ²Ρ‹Ρ… элСмСнтов, Ρ‚ΠΎ ΠΈΡ… ΠΌΠΎΠΆΠ½ΠΎ Π·Π°ΠΌΠ΅Π½ΠΈΡ‚ΡŒ Π½Π° ΠΎΠ΄Π½ΠΎ устройство, Ρƒ ΠΊΠΎΡ‚ΠΎΡ€ΠΎΠ³ΠΎ:

RΠΎΠ±Ρ‰ = NΒ·R (5).

ΠŸΠ°Ρ€Π°Π»Π»Π΅Π»ΡŒΠ½ΠΎΠ΅ соСдинСниС

ΠŸΡ€ΠΈ Ρ‚Π°ΠΊΠΎΠΌ ΠΏΠΎΠ΄ΠΊΠ»ΡŽΡ‡Π΅Π½ΠΈΠΈ Π²Ρ…ΠΎΠ΄Ρ‹ ΠΎΡ‚ всСх устройств соСдинСны Π² ΠΎΠ΄Π½ΠΎΠΉ Ρ‚ΠΎΡ‡ΠΊΠ΅, Π²Ρ‹Ρ…ΠΎΠ΄Ρ‹ – Π² Π΄Ρ€ΡƒΠ³ΠΎΠΉ Ρ‚ΠΎΡ‡ΠΊΠ΅. Π­Ρ‚ΠΈ Ρ‚ΠΎΡ‡ΠΊΠΈ Π² Ρ„ΠΈΠ·ΠΈΠΊΠ΅ ΠΈ элСктротСхникС Π½Π°Π·Ρ‹Π²Π°ΡŽΡ‚ΡΡ ΡƒΠ·Π»Π°ΠΌΠΈ. На элСктричСских схСмах ΡƒΠ·Π»Ρ‹ ΠΏΡ€Π΅Π΄ΡΡ‚Π°Π²Π»ΡΡŽΡ‚ собой мСста развСтвлСния ΠΏΡ€ΠΎΠ²ΠΎΠ΄Π½ΠΈΠΊΠΎΠ² ΠΈ ΠΎΠ±ΠΎΠ·Π½Π°Ρ‡Π°ΡŽΡ‚ΡΡ Ρ‚ΠΎΡ‡ΠΊΠ°ΠΌΠΈ.

ΠŸΠ°Ρ€Π°Π»Π»Π΅Π»ΡŒΠ½ΠΎΠ΅ соСдинСниС

РасчСт эквивалСнтного сопротивлСния Ρ‚Π°ΠΊΠΆΠ΅ выполняСм с ΠΏΠΎΠΌΠΎΡ‰ΡŒΡŽ Π·Π°ΠΊΠΎΠ½Π° Ома.

Π’ этом случаС ΠΎΠ±Ρ‰Π΅Π΅ Π·Π½Π°Ρ‡Π΅Π½ΠΈΠ΅ силы Ρ‚ΠΎΠΊΠ° складываСтся ΠΈΠ· суммы сил Ρ‚ΠΎΠΊΠΎΠ², ΠΏΡ€ΠΎΡ‚Π΅ΠΊΠ°ΡŽΡ‰ΠΈΡ… ΠΏΠΎ ΠΊΠ°ΠΆΠ΄ΠΎΠΉ Π²Π΅Ρ‚Π²ΠΈ, Π° Π²Π΅Π»ΠΈΡ‡ΠΈΠ½Π° падСния напряТСния для ΠΊΠ°ΠΆΠ΄ΠΎΠ³ΠΎ устройства ΠΈ ΠΎΠ±Ρ‰Π΅Π΅ напряТСниС ΠΎΠ΄ΠΈΠ½Π°ΠΊΠΎΠ²Ρ‹Π΅.

Если ΠΈΠΌΠ΅ΡŽΡ‚ΡΡ N рСзистивных устройств, ΠΏΠΎΠ΄ΠΊΠ»ΡŽΡ‡Π΅Π½Π½Ρ‹Ρ… Ρ‚Π°ΠΊΠΈΠΌ ΠΎΠ±Ρ€Π°Π·ΠΎΠΌ, Ρ‚ΠΎ:

I = I1 + I2 Β + … + IN (6),

U = U1 = U2 = … = UN (7).

Из Π²Ρ‹Ρ€Π°ΠΆΠ΅Π½ΠΈΠΉ (1), (6) ΠΈ (7) ΠΈΠΌΠ΅Π΅ΠΌ:

  • RΠΎΠ±Ρ‰ = U/(I1 + I2 + …+ IN),
  • 1/Rэкв = 1/R1 + 1/R2 +…+ 1/RN (8).

Если имССтся N ΠΎΠ΄ΠΈΠ½Π°ΠΊΠΎΠ²Ρ‹Ρ… рСзисторов, ΠΈΠΌΠ΅ΡŽΡ‰ΠΈΡ… ΠΏΠΎΠ΄ΠΊΠ»ΡŽΡ‡Π΅Π½ΠΈΠ΅ Π΄Π°Π½Π½ΠΎΠ³ΠΎ Ρ‚ΠΈΠΏΠ°, Ρ‚ΠΎ Ρ„ΠΎΡ€ΠΌΡƒΠ»Π° (8) прСобразуСтся ΡΠ»Π΅Π΄ΡƒΡŽΡ‰ΠΈΠΌ ΠΎΠ±Ρ€Π°Π·ΠΎΠΌ:

RΠΎΠ±Ρ‰ = R Β· R / NΒ·R = R / N (9).

Если соСдинСны нСсколько ΠΊΠ°Ρ‚ΡƒΡˆΠ΅ΠΊ индуктивности, Ρ‚ΠΎ ΠΈΡ… суммарноС ΠΈΠ½Π΄ΡƒΠΊΡ‚ΠΈΠ²Π½ΠΎΠ΅ сопротивлСниС рассчитываСтся Ρ‚Π°ΠΊ ΠΆΠ΅, ΠΊΠ°ΠΊ ΠΈ для рСзисторов.

Расчёт ΠΏΡ€ΠΈ смСшанном соСдинСнии устройств

Π’ случаС смСшанного ΠΏΠΎΠ΄ΠΊΠ»ΡŽΡ‡Π΅Π½ΠΈΡ ΠΏΡ€ΠΈΡΡƒΡ‚ΡΡ‚Π²ΡƒΡŽΡ‚ участки с ΠΏΠΎΡΠ»Π΅Π΄ΠΎΠ²Π°Ρ‚Π΅Π»ΡŒΠ½Ρ‹ΠΌ ΠΈ ΠΏΠ°Ρ€Π°Π»Π»Π΅Π»ΡŒΠ½Ρ‹ΠΌ ΠΏΠΎΠ΄ΠΊΠ»ΡŽΡ‡Π΅Π½ΠΈΡΠΌΠΈ элСмСнтов.

ΠŸΡ€ΠΈ Ρ€Π΅ΡˆΠ΅Π½ΠΈΠΈ Π·Π°Π΄Π°Ρ‡ΠΈ ΠΈΡΠΏΠΎΠ»ΡŒΠ·ΡƒΡŽΡ‚ ΠΌΠ΅Ρ‚ΠΎΠ΄ сворачивания Ρ†Π΅ΠΏΠΈ (ΠΌΠ΅Ρ‚ΠΎΠ΄ эквивалСнтных ΠΏΡ€Π΅ΠΎΠ±Ρ€Π°Π·ΠΎΠ²Π°Π½ΠΈΠΉ). Π•Π³ΠΎ ΠΈΡΠΏΠΎΠ»ΡŒΠ·ΡƒΡŽΡ‚ для вычислСния ΠΏΠ°Ρ€Π°ΠΌΠ΅Ρ‚Ρ€ΠΎΠ² Π² Ρ‚ΠΎΠΌ случаС, Ссли Π΅ΡΡ‚ΡŒ ΠΎΠ΄ΠΈΠ½ источник энСргии.

ΠŸΡ€Π΅Π΄ΠΏΠΎΠ»ΠΎΠΆΠΈΠΌ, Π·Π°Π΄Π°Π½Π° ΡΠ»Π΅Π΄ΡƒΡŽΡ‰Π°Ρ Π·Π°Π΄Π°Ρ‡Π°. ЭлСктричСская схСма (см. рис. Π½ΠΈΠΆΠ΅) состоит ΠΈΠ· 7 рСзисторов. РассчитайтС Ρ‚ΠΎΠΊΠΈ Π½Π° всСх рСзисторах, Ссли ΠΈΠΌΠ΅ΡŽΡ‚ΡΡ ΡΠ»Π΅Π΄ΡƒΡŽΡ‰ΠΈΠ΅ исходныС Π΄Π°Π½Π½Ρ‹Π΅:

  • R1 = 1Ом,
  • R2 = 2Ом,
  • R3 = 3Ом,
  • R4 = 6Ом,
  • R5 = 9Ом,
  • R6 = 18Ом,
  • R7 = 2,8Ом,
  • U = 32Π’.

ЭлСктричСская схСма

Из закона Ома имССм: 

I = U/R,

Π³Π΄Π΅ R – суммарноС сопротивлСниС всСх ΠΏΡ€ΠΈΠ±ΠΎΡ€ΠΎΠ².

Π•Π³ΠΎ Π±ΡƒΠ΄Π΅ΠΌ Π½Π°Ρ…ΠΎΠ΄ΠΈΡ‚ΡŒ, воспользовавшись ΠΌΠ΅Ρ‚ΠΎΠ΄ΠΎΠΌ сворачивания Ρ†Π΅ΠΏΠΈ.

Π­Π»Π΅ΠΌΠ΅Π½Ρ‚Ρ‹ R2 ΠΈ R3 ΠΏΠΎΠ΄ΠΊΠ»ΡŽΡ‡Π΅Π½Ρ‹ ΠΏΠ°Ρ€Π°Π»Π»Π΅Π»ΡŒΠ½ΠΎ, поэтому ΠΈΡ… ΠΌΠΎΠΆΠ½ΠΎ Π·Π°ΠΌΠ΅Π½ΠΈΡ‚ΡŒ Π½Π° R2,3, Π²Π΅Π»ΠΈΡ‡ΠΈΠ½Ρƒ ΠΊΠΎΡ‚ΠΎΡ€ΠΎΠ³ΠΎ ΠΌΠΎΠΆΠ½ΠΎ Ρ€Π°ΡΡΡ‡ΠΈΡ‚Π°Ρ‚ΡŒ ΠΏΠΎ Ρ„ΠΎΡ€ΠΌΡƒΠ»Π΅:

R2,3= R2Β·R3 / (R2+R3).

R4, R5 ΠΈ R6 Ρ‚Π°ΠΊΠΆΠ΅ Π²ΠΊΠ»ΡŽΡ‡Π΅Π½Ρ‹ ΠΏΠ°Ρ€Π°Π»Π»Π΅Π»ΡŒΠ½ΠΎ, ΠΈ ΠΈΡ… ΠΌΠΎΠΆΠ½ΠΎ Π·Π°ΠΌΠ΅Π½ΠΈΡ‚ΡŒ Π½Π° R4,5,6, ΠΊΠΎΡ‚ΠΎΡ€ΠΎΠ΅ вычисляСтся ΡΠ»Π΅Π΄ΡƒΡŽΡ‰ΠΈΠΌ ΠΎΠ±Ρ€Π°Π·ΠΎΠΌ:

1/R4,5,6 = 1/R4+1/R5+1/R6.

Π’Π°ΠΊΠΈΠΌ ΠΎΠ±Ρ€Π°Π·ΠΎΠΌ, схСму, ΠΈΠ·ΠΎΠ±Ρ€Π°ΠΆΡ‘Π½Π½ΡƒΡŽ Π½Π° ΠΊΠ°Ρ€Ρ‚ΠΈΠ½ΠΊΠ΅ Π²Ρ‹ΡˆΠ΅, ΠΌΠΎΠΆΠ½ΠΎ Π·Π°ΠΌΠ΅Π½ΠΈΡ‚ΡŒ Π½Π° ΡΠΊΠ²ΠΈΠ²Π°Π»Π΅Π½Ρ‚Π½ΡƒΡŽ, Π² ΠΊΠΎΡ‚ΠΎΡ€ΠΎΠΉ вмСсто рСзисторов R2, R3 ΠΈ R4, R5, R6 ΠΈΡΠΏΠΎΠ»ΡŒΠ·ΡƒΡŽΡ‚ΡΡ R2,3 ΠΈ R4,5,6.

ЭквивалСнтная схСма

Богласно ΠΊΠ°Ρ€Ρ‚ΠΈΠ½ΠΊΠ΅ Π²Ρ‹ΡˆΠ΅, Π² Ρ€Π΅Π·ΡƒΠ»ΡŒΡ‚Π°Ρ‚Π΅ ΠΏΡ€Π΅ΠΎΠ±Ρ€Π°Π·ΠΎΠ²Π°Π½ΠΈΠΉ ΠΏΠΎΠ»ΡƒΡ‡Π°Π΅ΠΌ ΠΏΠΎΡΠ»Π΅Π΄ΠΎΠ²Π°Ρ‚Π΅Π»ΡŒΠ½ΠΎΠ΅ соСдинСниС рСзисторов R1, R2,3, R4,5,6 ΠΈ R7.

RΠΎΠ±Ρ‰ ΠΌΠΎΠΆΠ΅Ρ‚ Π±Ρ‹Ρ‚ΡŒ Π½Π°ΠΉΠ΄Π΅Π½ΠΎ ΠΏΠΎ Ρ„ΠΎΡ€ΠΌΡƒΠ»Π΅:

RΠΎΠ±Ρ‰ = R1 + R2,3 + R4,5,6 + R7.

ΠŸΠΎΠ΄ΡΡ‚Π°Π²Π»ΡΠ΅ΠΌ числовыС значСния ΠΈ рассчитываСм R для ΠΎΠΏΡ€Π΅Π΄Π΅Π»Ρ‘Π½Π½Ρ‹Ρ… участков:

  • R2.3 = 2Ом·3Ом / (2Ом + 3Ом) = 1,2Ом,
  • 1/R4,5,6 = 1/6Ом + 1/9Ом + 1/18Ом = 1/3Ом,
  • R4,5,6 = 3Ом,
  • Rэкв = 1Ом + 1,2Ом + 3Ом + 2,8Ом= 8Ом.

Π’Π΅ΠΏΠ΅Ρ€ΡŒ, послС Ρ‚ΠΎΠ³ΠΎ, ΠΊΠ°ΠΊ нашли Rэкв, ΠΌΠΎΠΆΠ½ΠΎ Π²Ρ‹Ρ‡ΠΈΡΠ»ΡΡ‚ΡŒ Π·Π½Π°Ρ‡Π΅Π½ΠΈΠ΅ I:

I = 32Π’ / 8Ом = 4А.

ПослС Ρ‚ΠΎΠ³ΠΎ, ΠΊΠ°ΠΊ ΠΌΡ‹ ΠΏΠΎΠ»ΡƒΡ‡ΠΈΠ»ΠΈ Π²Π΅Π»ΠΈΡ‡ΠΈΠ½Ρƒ ΠΎΠ±Ρ‰Π΅Π³ΠΎ Ρ‚ΠΎΠΊΠ°, ΠΌΠΎΠΆΠ½ΠΎ Π²Ρ‹Ρ‡ΠΈΡΠ»ΠΈΡ‚ΡŒ силу Ρ‚ΠΎΠΊΠ°, ΠΏΡ€ΠΎΡ‚Π΅ΠΊΠ°ΡŽΡ‰ΡƒΡŽ Π½Π° ΠΊΠ°ΠΆΠ΄ΠΎΠΌ участкС.

ΠŸΠΎΡΠΊΠΎΠ»ΡŒΠΊΡƒ R1, R2,3, R4,5,6 ΠΈ R7 соСдинСны ΠΏΠΎΡΠ»Π΅Π΄ΠΎΠ²Π°Ρ‚Π΅Π»ΡŒΠ½ΠΎ, Ρ‚ΠΎ:

I1 = I2,3 = I4,5,6 = I7 = I = 4А.

На участкС R2,3 напряТСниС Π½Π°Ρ…ΠΎΠ΄ΠΈΠΌ ΠΏΠΎ Ρ„ΠΎΡ€ΠΌΡƒΠ»Π΅:

  • U2,3 = I2,3Β·R2,3,
  • U2,3 = 4А·1,2Ом = 4,8Π’.

ΠŸΠΎΡΠΊΠΎΠ»ΡŒΠΊΡƒ R2 ΠΈ R3 ΠΏΠΎΠ΄ΠΊΠ»ΡŽΡ‡Π΅Π½Ρ‹ ΠΏΠ°Ρ€Π°Π»Π»Π΅Π»ΡŒΠ½ΠΎ, Ρ‚ΠΎ U2,3 = U2 = U3, ΡΠ»Π΅Π΄ΠΎΠ²Π°Ρ‚Π΅Π»ΡŒΠ½ΠΎ:

  • I2 = U2 / R2,
  • I2 = 4,8Π’ / 2Ом = 2,4А,
  • I3 = U3 / R3,
  • I3 = 4,8Π’ / 3Ом = 1,6А.

ΠŸΡ€ΠΎΠ²Π΅Ρ€ΡΠ΅ΠΌ ΠΏΡ€Π°Π²ΠΈΠ»ΡŒΠ½ΠΎΡΡ‚ΡŒ Ρ€Π΅ΡˆΠ΅Π½ΠΈΡ:

  • I2,3 = I2 + I3,
  • I2,3 = 2,4А + 1,6А = 4А.

На участкС R4,5,Π± напряТСниС Ρ‚Π°ΠΊΠΆΠ΅ Π½Π°Ρ…ΠΎΠ΄ΠΈΠΌ, исходя ΠΈΠ· Π·Π°ΠΊΠΎΠ½Π° Ома:

  • U4,5,6 = I4,5,6Β·R4,5,6,
  • U4,5,6 = 4А·3Ом = 12Π’.

Π’Π°ΠΊ ΠΊΠ°ΠΊ R4, R5, RΠ± ΠΏΠΎΠ΄ΠΊΠ»ΡŽΡ‡Π΅Π½Ρ‹ ΠΏΠ°Ρ€Π°Π»Π»Π΅Π»ΡŒΠ½ΠΎ Π΄Ρ€ΡƒΠ³ ΠΊ Π΄Ρ€ΡƒΠ³Ρƒ, Ρ‚ΠΎ:

U4,5,6 = U4 = U5 = U6 = 12Π’.

ВычисляСм I4, I5, I6:

  • I4 = U4 / R4,
  • I4 = 12Π’ / 6Ом = 2А,
  • I5 = U5 / R5,
  • I5 = 12Π’ / 9Ом Β» 1,3А,
  • I6 = U6 / R6,
  • I5 = 12Π’ / 18Ом Β» 0,7А.

ΠŸΡ€ΠΎΠ²Π΅Ρ€ΡΠ΅ΠΌ ΠΏΡ€Π°Π²ΠΈΠ»ΡŒΠ½ΠΎΡΡ‚ΡŒ Ρ€Π΅ΡˆΠ΅Π½ΠΈΡ:

I4,5,6 = 2А + 1,3А + 0,7А = 4А.

Π§Ρ‚ΠΎΠ±Ρ‹ Π°Π²Ρ‚ΠΎΠΌΠ°Ρ‚ΠΈΠ·ΠΈΡ€ΠΎΠ²Π°Ρ‚ΡŒ Π²Ρ‹ΠΏΠΎΠ»Π½Π΅Π½ΠΈΠ΅ расчётов эквивалСнтных Π·Π½Π°Ρ‡Π΅Π½ΠΈΠΉ для Ρ€Π°Π·Π»ΠΈΡ‡Π½Ρ‹Ρ… участков Ρ†Π΅ΠΏΠΈ, ΠΌΠΎΠΆΠ½ΠΎ Π²ΠΎΡΠΏΠΎΠ»ΡŒΠ·ΠΎΠ²Π°Ρ‚ΡŒΡΡ сСрвисами сСти Π˜Π½Ρ‚Π΅Ρ€Π½Π΅Ρ‚, ΠΊΠΎΡ‚ΠΎΡ€Ρ‹Π΅ ΠΏΡ€Π΅Π΄Π»Π°Π³Π°ΡŽΡ‚ Π½Π° ΠΈΡ… сайтах Π²Ρ‹ΠΏΠΎΠ»Π½ΠΈΡ‚ΡŒ ΠΎΠ½Π»Π°ΠΉΠ½ вычислСния Π½ΡƒΠΆΠ½Ρ‹Ρ… элСктричСских характСристик. БСрвис ΠΎΠ±Ρ‹Ρ‡Π½ΠΎ ΠΈΠΌΠ΅Π΅Ρ‚ Π²ΡΡ‚Ρ€ΠΎΠ΅Π½Π½ΡƒΡŽ ΡΠΏΠ΅Ρ†ΠΈΠ°Π»ΡŒΠ½ΡƒΡŽ ΠΏΡ€ΠΎΠ³Ρ€Π°ΠΌΠΌΡƒ – ΠΊΠ°Π»ΡŒΠΊΡƒΠ»ΡΡ‚ΠΎΡ€, которая ΠΏΠΎΠΌΠΎΠ³Π°Π΅Ρ‚ быстро Π²Ρ‹ΠΏΠΎΠ»Π½ΠΈΡ‚ΡŒ расчСт сопротивлСния Ρ†Π΅ΠΏΠΈ любой слоТности.

Π’Π°ΠΊΠΈΠΌ ΠΎΠ±Ρ€Π°Π·ΠΎΠΌ, использованиС ΠΌΠ΅Ρ‚ΠΎΠ΄Π° эквивалСнтных ΠΏΡ€Π΅ΠΎΠ±Ρ€Π°Π·ΠΎΠ²Π°Π½ΠΈΠΉ ΠΏΡ€ΠΈ расчётС ΡΠΌΠ΅ΡˆΠ°Π½Π½Ρ‹Ρ… соСдинСний Ρ€Π°Π·Π»ΠΈΡ‡Π½Ρ‹Ρ… устройств позволяСт ΡƒΠΏΡ€ΠΎΡΡ‚ΠΈΡ‚ΡŒ ΠΈ ΡƒΡΠΊΠΎΡ€ΠΈΡ‚ΡŒ Π²Ρ‹ΠΏΠΎΠ»Π½Π΅Π½ΠΈΠ΅ вычислСний основных элСктричСских ΠΏΠ°Ρ€Π°ΠΌΠ΅Ρ‚Ρ€ΠΎΠ².

Π’ΠΈΠ΄Π΅ΠΎ

ΠžΡ†Π΅Π½ΠΈΡ‚Π΅ ΡΡ‚Π°Ρ‚ΡŒΡŽ:

Лабораторная Ρ€Π°Π±ΠΎΡ‚Π° ΠΏΠΎ исслСдованию свойств ΠΏΠ°Ρ€Π°Π»Π»Π΅Π»ΡŒΠ½ΠΎΠ³ΠΎ ΠΈ ΠΏΠΎΡΠ»Π΅Π΄ΠΎΠ²Π°Ρ‚Π΅Π»ΡŒΠ½ΠΎΠ³ΠΎ соСдинСния рСзисторов бСспаСчным способом

ο»Ώ

Π’ Ρ€Π°Π±ΠΎΡ‚Π΅ исслСдованы свойства ΠΈ физичСскиС Π·Π°ΠΊΠΎΠ½Ρ‹ соСдинСния рСзисторов Π±Π΅Π· ΠΏΠ°ΠΉΠΊΠΈ. ΠŸΡ€ΠΈ построСнии ΠΌΠ°ΠΊΠ΅Ρ‚ΠΎΠ² ΠΎΡ‚ ΠΌΠΎΠ½Ρ‚Π°ΠΆΠ° ΠΏΠ°ΠΉΠΊΠΎΠΉ удаСтся ΠΏΠΎΠ»Π½ΠΎΡΡ‚ΡŒΡŽ ΠΎΡ‚ΠΊΠ°Π·Π°Ρ‚ΡŒΡΡ ΠΈΠ»ΠΈ свСсти Π΅Π³ΠΎ ΠΊ ΠΌΠΈΠ½ΠΈΠΌΡƒΠΌΡƒ. Как ΠΏΠΎΠΊΠ°Π·Π°Π», ΠΎΠΏΡ‹Ρ‚ практичСскоС использованиС ΠΌΠ°ΠΊΠ΅Ρ‚Π½Ρ‹Ρ… ΠΏΠ»Π°Ρ‚ ΠΏΠΎΠ·Π²ΠΎΠ»ΡΡŽΡ‚, сущСствСнно ΡΡΠΊΠΎΠ½ΠΎΠΌΠΈΡ‚ΡŒ Π²Ρ€Π΅ΠΌΠ΅Π½Π½Ρ‹Π΅ ΠΈ ΠΌΠ°Ρ‚Π΅Ρ€ΠΈΠ°Π»ΡŒΠ½Ρ‹Π΅ Π·Π°Ρ‚Ρ€Π°Ρ‚Ρ‹.

ΠšΠ»ΡŽΡ‡Π΅Π²Ρ‹Π΅ слова: рСзисторы, макСтная ΠΏΠ»Π°Ρ‚Π°, соСдинСниС Π±Π΅Π· ΠΏΠ°ΠΉΠΊΠΈ, ΠΏΠ°Ρ€Π°Π»Π»Π΅Π»ΡŒΠ½ΠΎΠ΅ ΠΈ ΠΏΠΎΡΠ»Π΅Π΄ΠΎΠ²Π°Ρ‚Π΅Π»ΡŒΠ½ΠΎΠ΅ соСдинСниС, Ρ‚ΠΎΠΊΠΈ, напряТСниС ΠΈ сопротивлСния ΠΏΡ€ΠΈ соСдинСния рСзисторов.

The paper studies the properties and physical laws of connecting resistors without soldering. When building layouts, soldering installation can be completely, abandoned or reduced to a minimum. As experience has shown, the practical use of layout boards can significantly save time and material costs.

Keyword: resistors, layout Board, connection without soldering, parallel and serial connection, currents, voltage and resistance when connecting resistors.

Π’ соврСмСнном Π²Ρ‹ΡΡˆΠ΅ΠΌ ΡƒΡ‡Π΅Π±Π½ΠΎΠΌ Π·Π°Π²Π΅Π΄Π΅Π½ΠΈΠΈ ΠΏΠ΅Ρ€Π΅Π΄ ΠΊΠ°ΠΆΠ΄Ρ‹ΠΌ ΠΏΡ€Π΅ΠΏΠΎΠ΄Π°Π²Π°Ρ‚Π΅Π»Π΅ΠΌ стоит Ρ†Π΅Π»ΡŒ ΠΏΠΎ Ρ€Π°Π·Π²ΠΈΡ‚ΠΈΡŽ Π½Π°ΡƒΡ‡Π½ΠΎ-ΠΈΡΡΠ»Π΅Π΄ΠΎΠ²Π°Ρ‚Π΅Π»ΡŒΡΠΊΠΈΡ… Π½Π°Π²Ρ‹ΠΊΠΎΠ² студСнтов. Один ΠΈΠ· Π·Π°Π΄Π°Ρ‡ являСтсямотивации студСнта ΠΈ ΡΠΎΠ²Π΅Ρ€ΡˆΠ΅Π½ΡΡ‚Π²ΠΎΠ²Π°Π½ΠΈΠ΅ Π½Π°ΡƒΡ‡Π½ΠΎ-ΠΈΡΡΠ»Π΅Π΄ΠΎΠ²Π°Ρ‚Π΅Π»ΡŒΡΠΊΠΈΡ… Π½Π°Π²Ρ‹ΠΊΠΎΠ² Π² Ρ„ΠΎΡ€ΠΌΡƒΠ»ΠΈΡ€ΠΎΠ²ΠΊΠ΅ ΠΈΡΡΠ»Π΅Π΄ΠΎΠ²Π°Ρ‚Π΅Π»ΡŒΡΠΊΠΎΠ³ΠΎ вопроса Π²ΠΎ врСмя Π»Π°Π±ΠΎΡ€Π°Ρ‚ΠΎΡ€Π½ΠΎΠΉ Ρ€Π°Π±ΠΎΡ‚Ρ‹ Π½Π° ΡƒΡ€ΠΎΠΊΠ°Ρ… Ρ„ΠΈΠ·ΠΈΠΊΠΈ. Π’ Π΄Π°Π½Π½ΠΎΠΉ Ρ€Π°Π±ΠΎΡ‚Π΅ исслСдованы свойства ΠΈ физичСскиС Π·Π°ΠΊΠΎΠ½Ρ‹ соСдинСния рСзисторов Π±Π΅Π· ΠΏΠ°ΠΉΠΊΠΈ. Как ΠΏΠΎΠΊΠ°Π·Π°Π», ΠΎΠΏΡ‹Ρ‚ практичСскоС использованиС ΠΌΠ°ΠΊΠ΅Ρ‚Π½Ρ‹Ρ… ΠΏΠ»Π°Ρ‚ ΠΏΠΎΠ·Π²ΠΎΠ»ΡΡŽΡ‚, сущСствСнно ΡΡΠΊΠΎΠ½ΠΎΠΌΠΈΡ‚ΡŒ Π²Ρ€Π΅ΠΌΠ΅Π½Π½Ρ‹Π΅ ΠΈ ΠΌΠ°Ρ‚Π΅Ρ€ΠΈΠ°Π»ΡŒΠ½Ρ‹Π΅ Π·Π°Ρ‚Ρ€Π°Ρ‚Ρ‹. [1,2].

ЦСль Ρ€Π°Π±ΠΎΡ‚Ρ‹: ΠΎΠΏΡ‹Ρ‚Π½Ρ‹ΠΌ ΠΏΡƒΡ‚Π΅ΠΌ ΠΈΠ·ΡƒΡ‡ΠΈΡ‚ΡŒ Π·Π°ΠΊΠΎΠ½Ρ‹ протСкания Ρ‚ΠΎΠΊΠ° Ρ‡Π΅Ρ€Π΅Π· ΠΏΠΎΡΠ»Π΅Π΄ΠΎΠ²Π°Ρ‚Π΅Π»ΡŒΠ½ΠΎ ΠΈ ΠΏΠ°Ρ€Π°Π»Π»Π΅Π»ΡŒΠ½ΠΎ соСдинСниС Ρ€Π΅Π·ΠΈΡΡ‚ΠΎΡ€ΠΎΠ²ΡΠΏΠΎΠΌΠΎΡ‰ΡŒΡŽ бСспаСчноС соСдинСниС ΠΈ ΠΎΠΏΡ€Π΅Π΄Π΅Π»ΠΈΡ‚ΡŒ Ρ„ΠΎΡ€ΠΌΡƒΠ»Ρ‹ сопротивлСний Ρ‚Π°ΠΊΠΈΡ… участков. РСзисторы примСниСтся Π² элСктронных ΠΏΡ€ΠΈΠ±ΠΎΡ€Π°Ρ… ΠΈ Ρ€ΠΎΠ±ΠΎΡ‚ΠΎΡ‚Π΅Ρ…Π½ΠΈΠΊΠ΅. [3].

ΠŸΠΎΡΠ»Π΅Π΄ΠΎΠ²Π°Ρ‚Π΅Π»ΡŒΠ½Ρ‹ΠΌ соСдинСниСм сопротивлСний называСтся Ρ‚Π°ΠΊΠΎΠ΅ соСдинСниС, ΠΏΡ€ΠΈ ΠΊΠΎΡ‚ΠΎΡ€ΠΎΠΌ ΠΊΠΎΠ½Π΅Ρ† ΠΏΠ΅Ρ€Π²ΠΎΠ³ΠΎ сопротивлСния соСдиняСтся с Π½Π°Ρ‡Π°Π»ΠΎΠΌ Π²Ρ‚ΠΎΡ€ΠΎΠ³ΠΎ, ΠΊΠΎΠ½Π΅Ρ† Π²Ρ‚ΠΎΡ€ΠΎΠ³ΠΎ β€” с Π½Π°Ρ‡Π°Π»ΠΎΠΌ Ρ‚Ρ€Π΅Ρ‚ΡŒΠ΅Π³ΠΎ ΠΈ Ρ‚. Π΄. Рассмотрим физичСскиС Π·Π°ΠΊΠΎΠ½Ρ‹ соСдинСния рСзисторов. ΠŸΠ΅Ρ€Π΅Ρ‡Π΅Π½ΡŒ оборудования ΠΏΠΎΠΊΠ°Π·Π°Π½Ρ‹ Π½Π° рис.1,2,3.

ΠŸΠ΅Ρ€Π΅Ρ‡Π΅Π½ΡŒ оборудования

Рис. 1. РСзистор ΠΈ ΠΏΡ€ΠΎΠ²ΠΎΠ΄Π°

Рис. 2. ΠœΡƒΠ»ΡŒΡ‚ΠΈΠΌΠ΅Ρ‚Ρ€

Рис. 3. ΠœΠ°ΠΊΠ΅Ρ‚Π½Π°Ρ ΠΏΠ»Π°Ρ‚Π°

ΠŸΠΎΡΠ»Π΅Π΄ΠΎΠ²Π°Ρ‚Π΅Π»ΡŒΠ½ΠΎ соСдинСнных рСзисторов

Π‘ΠΎΠΏΡ€ΠΎΡ‚ΠΈΠ²Π»Π΅Π½ΠΈΠ΅ Π²ΠΏΠΎΡΠ»Π΅Π΄ΠΎΠ²Π°Ρ‚Π΅Π»ΡŒΠ½ΠΎΠΉ Ρ†Π΅ΠΏΠΈ. ΠžΠ±Ρ‰Π΅Π΅ сопротивлСниС ΠΏΠΎΡΠ»Π΅Π΄ΠΎΠ²Π°Ρ‚Π΅Π»ΡŒΠ½ΠΎ соСдинСнных рСзисторов Ρ€Π°Π²Π½ΠΎ суммС ΠΈΡ… сопротивлСний.

RΠΎΠ±Ρ‰ = R1+R2+R2+ β€’β€’β€’+Rn(1)

Если ΠΏΠΎΡΠ»Π΅Π΄ΠΎΠ²Π°Ρ‚Π΅Π»ΡŒΠ½ΠΎ Π²ΠΊΠ»ΡŽΡ‡Π΅Π½ΠΎ любоС количСство рСзисторов ΠΎΠ΄ΠΈΠ½Π°ΠΊΠΎΠ²Ρ‹Ρ… сопротивлСний, Ρ‚ΠΎ ΠΈΡ… ΠΎΠ±Ρ‰Π΅Π΅ сопротивлСниС ΠΌΠΎΠΆΠ½ΠΎ ΠΎΠΏΡ€Π΅Π΄Π΅Π»ΠΈΡ‚ΡŒ, ΡƒΠΌΠ½ΠΎΠΆΠΈΠ² сопротивлСниС ΠΎΠ΄Π½ΠΎΠ³ΠΎ рСзистора Π½Π° количСство рСзисторов.

RΠΎΠ±Ρ‰ = nR1 (2)

ΠŸΡ€ΠΈΠΌΠ΅Ρ€ ΠΏΠΎΡΠ»Π΅Π΄ΠΎΠ²Π°Ρ‚Π΅Π»ΡŒΠ½ΠΎΠ³ΠΎ соСдинСния Π΄Π²ΡƒΡ… рСзисторов ΠΏΠΎΠΊΠ°Π·Π°Π½ Π½Π° рис. 4.

ΠžΠ±Ρ‰Π΅Π΅ сопротивлСниС для Π΄Π²ΡƒΡ… рСзисторов.

RΠΎΠ±Ρ‰=R1+R2 (3)

RΠΎΠ±Ρ‰ = 25 Ом+35 Ом=60 Ом (4)

Рис. 4. ΠŸΠΎΡΠ»Π΅Π΄ΠΎΠ²Π°Ρ‚Π΅Π»ΡŒΠ½ΠΎΠ΅ соСдинСниС Π΄Π²ΡƒΡ… рСзисторов

Π’Π΅Π»ΠΈΡ‡ΠΈΠ½Π° Ρ‚ΠΎΠΊΠ° Π²ΠΏΠΎΡΠ»Π΅Π΄ΠΎΠ²Π°Ρ‚Π΅Π»ΡŒΠ½ΠΎΠΉ Ρ†Π΅ΠΏΠΈ

Π’Π°ΠΊ ΠΊΠ°ΠΊ Π² Π΄Π°Π½Π½ΠΎΠΉ Ρ†Π΅ΠΏΠΈ отсутствуСт ΠΎΡ‚Π²Π΅Ρ‚Π²Π»Π΅Π½ΠΈΠ΅ Ρ‚ΠΎΠΊΠ°, Ρ‚ΠΎ ΠΎΡ‡Π΅Π²ΠΈΠ΄Π½ΠΎ, Ρ‡Ρ‚ΠΎ количСство элСктричСства, ΠΏΡ€ΠΎΡ‚Π΅ΠΊΠ°ΡŽΡ‰Π΅Π΅ Ρ‡Π΅Ρ€Π΅Π· ΠΏΠΎΠΏΠ΅Ρ€Π΅Ρ‡Π½ΠΎΠ΅ сСчСниС ΠΏΡ€ΠΎΠ²ΠΎΠ΄Π½ΠΈΠΊΠ° Π·Π° Π΅Π΄ΠΈΠ½ΠΈΡ†Ρƒ Π²Ρ€Π΅ΠΌΠ΅Π½ΠΈ Π² любой Ρ‚ΠΎΡ‡ΠΊΠ΅ Ρ†Π΅ΠΏΠΈ, Π±ΡƒΠ΄Π΅Ρ‚ ΠΎΠ΄ΠΈΠ½Π°ΠΊΠΎΠ²Ρ‹ΠΌ. Π‘Π»Π΅Π΄ΠΎΠ²Π°Ρ‚Π΅Π»ΡŒΠ½ΠΎ, Π²ΠΎ всСх Ρ‚ΠΎΡ‡ΠΊΠ°Ρ… ΠΏΠΎΡΠ»Π΅Π΄ΠΎΠ²Π°Ρ‚Π΅Π»ΡŒΠ½ΠΎΠΉ Ρ†Π΅ΠΏΠΈ Π²Π΅Π»ΠΈΡ‡ΠΈΠ½Π° Ρ‚ΠΎΠΊΠ° ΠΎΠ΄ΠΈΠ½Π°ΠΊΠΎΠ²Π°.

ΠŸΠΎΡΡ‚ΠΎΠΌΡƒ ΠΏΡ€ΠΈ ΠΏΠΎΡΠ»Π΅Π΄ΠΎΠ²Π°Ρ‚Π΅Π»ΡŒΠ½ΠΎΠΌ соСдинСнии, для измСрСния Ρ‚ΠΎΠΊΠ° достаточно Π²ΠΊΠ»ΡŽΡ‡Π°Ρ‚ΡŒ ΠΎΠ΄ΠΈΠ½ Π°ΠΌΠΏΠ΅Ρ€ΠΌΠ΅Ρ‚Ρ€ Π½Π° любом участкС Ρ†Π΅ΠΏΠΈ.

Π’ качСствС ΠΏΡ€ΠΈΠΌΠ΅Ρ€Π°, ΠΌΠΎΠΆΠ½ΠΎ привСсти Ρ†Π΅ΠΏΡŒ ΠΏΠΎΡΠ»Π΅Π΄ΠΎΠ²Π°Ρ‚Π΅Π»ΡŒΠ½ΠΎΠ³ΠΎ соСдинСния Π΄Π²ΡƒΡ… рСзисторов, ΠΊΠΎΡ‚ΠΎΡ€Ρ‹ΠΉ ΠΏΠΎΠΊΠ°Π·Π°Π½ Π½Π° рис.5.

I=I1=I2=0,2A

Рис. 5. Π’Π΅Π»ΠΈΡ‡ΠΈΠ½Π° Ρ‚ΠΎΠΊΠ° Π² ΠΏΠΎΡΠ»Π΅Π΄ΠΎΠ²Π°Ρ‚Π΅Π»ΡŒΠ½ΠΎΠΉΡ†Π΅ΠΏΠΈ

РаспрСдСлСниС напряТСния Π²ΠΏΠΎΡΠ»Π΅Π΄ΠΎΠ²Π°Ρ‚Π΅Π»ΡŒΠ½ΠΎΠΉ Ρ†Π΅ΠΏΠΈ

НапряТСниС источника Ρ‚ΠΎΠΊΠ°, ΠΏΡ€ΠΈΠ»ΠΎΠΆΠ΅Π½Π½ΠΎΠ΅, ΠΊ Π²Π½Π΅ΡˆΠ½Π΅ΠΌΡƒ участку Ρ†Π΅ΠΏΠΈ распрСдСляСтся ΠΏΠΎ участкам Ρ†Π΅ΠΏΠΈ прямо ΠΏΡ€ΠΎΠΏΠΎΡ€Ρ†ΠΈΠΎΠ½Π°Π»ΡŒΠ½ΠΎ сопротивлСниям этих участков. НапряТСниС, ΠΏΡ€ΠΈΠ»ΠΎΠΆΠ΅Π½Π½ΠΎΠ΅, ΠΊ ΠΊΠ°ΠΆΠ΄ΠΎΠΌΡƒ ΠΈΠ· этих рСзисторов опрСдСляСтся ΠΏΠΎ Ρ„ΠΎΡ€ΠΌΡƒΠ»Π΅:

U=IR (5)

Π’Π°ΠΊ ΠΊΠ°ΠΊ Ρ‚ΠΎΠΊ Π² ΠΏΠΎΡΠ»Π΅Π΄ΠΎΠ²Π°Ρ‚Π΅Π»ΡŒΠ½ΠΎΠΉ Ρ†Π΅ΠΏΠΈ Π²Π΅Π·Π΄Π΅ ΠΎΠ΄ΠΈΠ½Π°ΠΊΠΎΠ², Π·Π½Π°Ρ‡ΠΈΡ‚, Π΄Π΅ΠΉΡΡ‚Π²ΠΈΡ‚Π΅Π»ΡŒΠ½ΠΎ напряТСниС Π½Π° Π΅Π΅ участках зависит ΠΎΡ‚ сопротивлСния, Ρ‡Π΅ΠΌ большС сопротивлСниС, Ρ‚Π΅ΠΌ большСС напряТСниС ΠΏΡ€ΠΈΠ»ΠΎΠΆΠ΅Π½ΠΎ ΠΊ Π΄Π°Π½Π½ΠΎΠΌΡƒ участку. Π‘ΡƒΠΌΠΌΠ° напряТСний Π½Π° участках ΠΏΠΎΡΠ»Π΅Π΄ΠΎΠ²Π°Ρ‚Π΅Π»ΡŒΠ½ΠΎΠΉ Ρ†Π΅ΠΏΠΈ Ρ€Π°Π²Π½Π° Π½Π°ΠΏΡ€ΡΠΆΠ΅Π½ΠΈΡŽ источника Ρ‚ΠΎΠΊΠ°.

UΠΎΠ±Ρ‰=U1+ U2+ U3+ β€’β€’β€’ +Un(6)

ΠŸΡ€ΠΈΠΌΠ΅Ρ€, распрСдСлСниС напряТСния Π² ΠΏΠΎΡΠ»Π΅Π΄ΠΎΠ²Π°Ρ‚Π΅Π»ΡŒΠ½ΠΎΠΉ Ρ†Π΅ΠΏΠΈ соСдинСния Π΄Π²ΡƒΡ… рСзисторов ΠΏΠΎΠΊΠ°Π·Π°Π½ Π½Π° рис.6.

UΠΎΠ±Ρ‰=U1+ U2(7)

12Π’=5Π’+7Π’

ΠŸΠ°Ρ€Π°Π»Π»Π΅Π»ΡŒΠ½ΠΎΠ΅ соСдинСниС сопротивлСний

ΠŸΠ°Ρ€Π°Π»Π»Π΅Π»ΡŒΠ½Ρ‹ΠΌ соСдинСниСм сопротивлСний называСтся Ρ‚Π°ΠΊΠΎΠ΅ соСдинСниС, ΠΏΡ€ΠΈ ΠΊΠΎΡ‚ΠΎΡ€ΠΎΠΌ ΠΊ ΠΎΠ΄Π½ΠΎΠΌΡƒ Π·Π°ΠΆΠΈΠΌΡƒ источника ΠΏΠΎΠ΄ΠΊΠ»ΡŽΡ‡Π°ΡŽΡ‚ΡΡ Π½Π°Ρ‡Π°Π»Π° сопротивлСний, Π° ΠΊ Π΄Ρ€ΡƒΠ³ΠΎΠΌΡƒ Π·Π°ΠΆΠΈΠΌΡƒ β€” ΠΊΠΎΠ½Ρ†Ρ‹.

Рис.

6. РаспрСдСлСниС напряТСния Π² ΠΏΠΎΡΠ»Π΅Π΄ΠΎΠ²Π°Ρ‚Π΅Π»ΡŒΠ½ΠΎΠΉ Ρ†Π΅ΠΏΠΈ

Π‘ΠΎΠΏΡ€ΠΎΡ‚ΠΈΠ²Π»Π΅Π½ΠΈΠ΅Π² ΠΏΠ°Ρ€Π°Π»Π»Π΅Π»ΡŒΠ½ΠΎΠΉ Ρ†Π΅ΠΏΠΈ

ΠžΠ±Ρ‰Π΅Π΅ сопротивлСниС ΠΏΠ°Ρ€Π°Π»Π»Π΅Π»ΡŒΠ½ΠΎ Π²ΠΊΠ»ΡŽΡ‡Π΅Π½Π½Ρ‹Ρ… сопротивлСний опрСдСляСтся ΠΏΠΎ Ρ„ΠΎΡ€ΠΌΡƒΠ»Π΅:

(8)

Если ΠΏΠ°Ρ€Π°Π»Π»Π΅Π»ΡŒΠ½ΠΎ Π²ΠΊΠ»ΡŽΡ‡Π΅Π½ΠΎ любоС количСство рСзисторов ΠΎΠ΄ΠΈΠ½Π°ΠΊΠΎΠ²Ρ‹Ρ… сопротивлСний, Ρ‚ΠΎ ΠΈΡ… ΠΎΠ±Ρ‰Π΅Π΅ сопротивлСниС ΠΌΠΎΠΆΠ½ΠΎ ΠΎΠΏΡ€Π΅Π΄Π΅Π»ΠΈΡ‚ΡŒ, Ссли сопротивлСниС ΠΎΠ΄Π½ΠΎΠ³ΠΎ рСзистора Ρ€Π°Π·Π΄Π΅Π»ΠΈΡ‚ΡŒ Π½Π° количСство рСзисторов.

ΠžΠ±Ρ‰Π΅Π΅ сопротивлСниС ΠΏΠ°Ρ€Π°Π»Π»Π΅Π»ΡŒΠ½ΠΎ Π²ΠΊΠ»ΡŽΡ‡Π΅Π½Π½Ρ‹Ρ… сопротивлСний всСгда мСньшС наимСньшСго сопротивлСния, входящСго Π² Π΄Π°Π½Π½ΠΎΠ΅ соСдинСниС. ΠŸΡ€ΠΈΠΌΠ΅Ρ€, Ссли ΠΏΠ°Ρ€Π°Π»Π»Π΅Π»ΡŒΠ½ΠΎ Π²ΠΊΠ»ΡŽΡ‡Π΅Π½ΠΎ Ρ‚ΠΎΠ»ΡŒΠΊΠΎ Π΄Π²Π° рСзистора (рис.7), Ρ‚ΠΎ ΠΈΡ… ΠΎΠ±Ρ‰Π΅Π΅ сопротивлСниС ΠΌΠΎΠΆΠ½ΠΎ ΠΎΠΏΡ€Π΅Π΄Π΅Π»ΠΈΡ‚ΡŒ ΠΏΠΎ Ρ„ΠΎΡ€ΠΌΡƒΠ»Π΅:

(10)

Рис. 7. ΠŸΠ°Ρ€Π°Π»Π»Π΅Π»ΡŒΠ½ΠΎΠ΅ соСдинСниС Π΄Π²ΡƒΡ… рСзисторов

РаспрСдСлСниС Ρ‚ΠΎΠΊΠ° Π²ΠΏΠ°Ρ€Π°Π»Π»Π΅Π»ΡŒΠ½ΠΎΠΉ Ρ†Π΅ΠΏΠΈ

Π’ Ρ†Π΅ΠΏΠΈ с ΠΏΠ°Ρ€Π°Π»Π»Π΅Π»ΡŒΠ½Ρ‹ΠΌ соСдинСниСм, Ρ‡Ρ‚ΠΎ элСктричСский Ρ‚ΠΎΠΊ распрСдСляСтся ΠΏΠΎ ΠΏΠ°Ρ€Π°Π»Π»Π΅Π»ΡŒΠ½Ρ‹ΠΌ вСтвям ΠΎΠ±Ρ€Π°Ρ‚Π½ΠΎ ΠΏΡ€ΠΎΠΏΠΎΡ€Ρ†ΠΈΠΎΠ½Π°Π»ΡŒΠ½ΠΎ ΠΈΡ… сопротивлСниям.

Π­Ρ‚ΠΎ Π·Π½Π°Ρ‡ΠΈΡ‚, Ρ‡Ρ‚ΠΎ Ρ‡Π΅ΠΌ большС сопротивлСниС, Ρ‚Π΅ΠΌ мСньшС ΠΏΠΎ Π½Π΅ΠΌΡƒ ΠΏΠΎΠΉΠ΄Π΅Ρ‚ Ρ‚ΠΎΠΊ.

(11)

Рассматривая Ρ‚ΠΎΡ‡ΠΊΡƒ развСтвлСния А, Π·Π°ΠΌΠ΅Ρ‡Π°Π΅ΠΌ, Ρ‡Ρ‚ΠΎ ΠΊ Π½Π΅ΠΉ ΠΏΡ€ΠΈΡ‚Π΅ΠΊΠ°Π΅Ρ‚ Ρ‚ΠΎΠΊ I, Π° Ρ‚ΠΎΠΊΠΈ I1, I2, ΡƒΡ‚Π΅ΠΊΠ°ΡŽΡ‚ ΠΈΠ· Π½Π΅Π΅. Π’Π°ΠΊ ΠΊΠ°ΠΊ двиТущиСся элСктричСскиС заряды Π½Π΅ ΡΠΊΠ°ΠΏΠ»ΠΈΠ²Π°ΡŽΡ‚ΡΡ Π² Ρ‚ΠΎΡ‡ΠΊΠ΅, Ρ‚ΠΎ ΠΎΡ‡Π΅Π²ΠΈΠ΄Π½ΠΎ, Ρ‡Ρ‚ΠΎ суммарный заряд, ΠΏΡ€ΠΈΡ‚Π΅ΠΊΠ°ΡŽΡ‰ΠΈΠΉ ΠΊ Ρ‚ΠΎΡ‡ΠΊΠ΅ развСтвлСния, Ρ€Π°Π²Π΅Π½ суммарному заряду, ΡƒΡ‚Π΅ΠΊΠ°ΡŽΡ‰Π΅ΠΌΡƒ ΠΎΡ‚ Π½Π΅Π΅:

I=I1+ I2+ I3 +β€’β€’β€’ +In(12)

Π‘Π»Π΅Π΄ΠΎΠ²Π°Ρ‚Π΅Π»ΡŒΠ½ΠΎ, Π²Π΅Π»ΠΈΡ‡ΠΈΠ½Π° Ρ‚ΠΎΠΊΠ° Π² Π½Π΅ Ρ€Π°Π·Π²Π΅Ρ‚Π²Π»Π΅Π½Π½ΠΎΠΉ части Ρ†Π΅ΠΏΠΈ Ρ€Π°Π²Π½Π° суммС Ρ‚ΠΎΠΊΠΎΠ² Π² ΠΏΠ°Ρ€Π°Π»Π»Π΅Π»ΡŒΠ½Ρ‹Ρ… вСтвях.

ΠŸΡ€ΠΈΠΌΠ΅Ρ€, распрСдСлСниС Ρ‚ΠΎΠΊΠ° Π² ΠΏΠ°Ρ€Π°Π»Π»Π΅Π»ΡŒΠ½ΠΎΠΉ Ρ†Π΅ΠΏΠΈ соСдинСния Π΄Π²ΡƒΡ… рСзисторов ΠΏΠΎΠΊΠ°Π·Π°Π½ Π½Π° рис. 8.

I=I1+ I2

Рис. 8.

РаспрСдСлСниС Ρ‚ΠΎΠΊΠ° Π² ΠΏΠ°Ρ€Π°Π»Π»Π΅Π»ΡŒΠ½ΠΎΠΉ Ρ†Π΅ΠΏΠΈ

Π’Π΅Π»ΠΈΡ‡ΠΈΠ½Π° напряТСния Π²ΠΏΠ°Ρ€Π°Π»Π»Π΅Π»ΡŒΠ½Ρ‹Ρ… цСпях

Π’Π°ΠΊ ΠΊΠ°ΠΊ Π½Π°Ρ‡Π°Π»Π° всСх сопротивлСний свСдСны Π² ΠΎΠ΄Π½Ρƒ ΠΎΠ±Ρ‰ΡƒΡŽ Ρ‚ΠΎΡ‡ΠΊΡƒ, Π° ΠΊΠΎΠ½Ρ†Ρ‹ β€” Π² Π΄Ρ€ΡƒΠ³ΡƒΡŽ, Ρ‚ΠΎ ΠΎΡ‡Π΅Π²ΠΈΠ΄Π½ΠΎ, Ρ‡Ρ‚ΠΎ Ρ€Π°Π·Π½ΠΎΡΡ‚ΡŒ ΠΏΠΎΡ‚Π΅Π½Ρ†ΠΈΠ°Π»ΠΎΠ² Π½Π° ΠΊΠΎΠ½Ρ†Π°Ρ… любого ΠΈΠ· ΠΏΠ°Ρ€Π°Π»Π»Π΅Π»ΡŒΠ½ΠΎ Π²ΠΊΠ»ΡŽΡ‡Π΅Π½Π½Ρ‹Ρ… сопротивлСний Ρ€Π°Π²Π½Π° разности ΠΏΠΎΡ‚Π΅Π½Ρ†ΠΈΠ°Π»ΠΎΠ² ΠΌΠ΅ΠΆΠ΄Ρƒ ΠΎΠ±Ρ‰ΠΈΠΌΠΈ Ρ‚ΠΎΡ‡ΠΊΠ°ΠΌΠΈ.

Π˜Ρ‚Π°ΠΊ, ΠΏΡ€ΠΈ ΠΏΠ°Ρ€Π°Π»Π»Π΅Π»ΡŒΠ½ΠΎΠΌ соСдинСнии сопротивлСний напряТСния Π½Π° Π½ΠΈΡ… Ρ€Π°Π²Π½Ρ‹ ΠΌΠ΅ΠΆΠ΄Ρƒ собой.

U=U1=U2=U3=β€’β€’β€’=Un (13)

Π‘ΠΎΠ΄Π΅Ρ€ΠΆΠ°Π½ΠΈΠ΅ Ρ…ΠΎΠ΄Π° Ρ€Π°Π±ΠΎΡ‚Ρ‹ ΠΈΠΏΠΎΡΠ»Π΅Π΄ΠΎΠ²Π°Ρ‚Π΅Π»ΡŒΠ½ΠΎΡΡ‚ΡŒ дСйствий

ВСхничСскоС Π·Π°Π΄Π°Π½ΠΈΠ΅

1) Π‘ΠΎΠ±Ρ€Π°Ρ‚ΡŒ ΡΠ»Π΅ΠΊΡ‚Ρ€ΠΈΡ‡Π΅ΡΠΊΡƒΡŽ Ρ†Π΅ΠΏΡŒ Π² ΠΌΠ°ΠΊΠ΅Ρ‚Π½ΡƒΡŽ ΠΏΠ»Π°Ρ‚Ρƒ (рис.9.)

2) Π‘Π½ΡΡ‚ΡŒ показания ΠΏΡ€ΠΈΠ±ΠΎΡ€ΠΎΠ² ΠΈ Π·Π°ΠΏΠΈΡΠ°Ρ‚ΡŒ ΠΈΡ… Π² Ρ‚Π°Π±Π»ΠΈΡ†Ρƒ.

3) ΠŸΡ€ΠΎΠΈΠ·Π²Π΅ΡΡ‚ΠΈ расчСты.

4) ΠžΡ‚Π²Π΅Ρ‚ΠΈΡ‚ΡŒ Π½Π° ΠΊΠΎΠ½Ρ‚Ρ€ΠΎΠ»ΡŒΠ½Ρ‹Π΅ вопросы.

5) Π‘Π΄Π΅Π»Π°Ρ‚ΡŒ Π²Ρ‹Π²ΠΎΠ΄.

ΠŸΠΎΡ€ΡΠ΄ΠΎΠΊ выполнСния Ρ€Π°Π±ΠΎΡ‚Ρ‹

1) Π‘ΠΎΠ±Ρ€Π°Ρ‚ΡŒ схСму Π² ΠΌΠ°ΠΊΠ΅Ρ‚Π½ΡƒΡŽ ΠΏΠ»Π°Ρ‚Ρƒ (Рис. 9.).

2) Π£ΡΡ‚Π°Π½ΠΎΠ²ΠΈΡ‚ΡŒ Π½Π° схСмС Π²Π΅Π»ΠΈΡ‡ΠΈΠ½Ρ‹ R1=1 кОм + N, R2=2 кОм + N, R3=3 кОм + N ΠΈ R4=4 кОм+ N, Π³Π΄Π΅ N β€” Π½ΠΎΠΌΠ΅Ρ€ студСнта ΠΏΠΎ ΠΆΡƒΡ€Π½Π°Π»Ρƒ (ΠΌΠΎΡ‰Π½ΠΎΡΡ‚ΡŒ рСзисторов Π±ΠΎΠ»Π΅Π΅ 1 Π’Ρ‚).

3) Π’ΠΊΠ»ΡŽΡ‡ΠΈΡ‚ΡŒ источник ΠΈ ΡƒΡΡ‚Π°Π½ΠΎΠ²ΠΈΡ‚ΡŒ напряТСниС U=5Π’, 9Π’, 12Π’, 15Π’.

4) ΠžΠΏΡ€Π΅Π΄Π΅Π»ΠΈΡ‚Π΅ ΡΠΊΡΠΏΠ΅Ρ€ΠΈΠΌΠ΅Π½Ρ‚Π°Π»ΡŒΠ½ΠΎ с ΠΏΠΎΠΌΠΎΡ‰ΡŒΡŽ ΠΌΡƒΠ»ΡŒΡ‚ΠΈΠΌΠ΅Ρ‚Ρ€Π° (Π² Ρ€Π΅ΠΆΠΈΠΌΠ΅ измСрСния сопротивлСний) сопротивлСниС ΠΌΠ΅ΠΆΠ΄Ρƒ Ρ‚ΠΎΡ‡ΠΊΠ°ΠΌΠΈ:

А ΠΈΠ’; Π’ΠΈ Π‘; Аи Π‘; DΠΈ Π•.

Π—Π°ΠΏΠΈΡˆΠΈΡ‚Π΅ эти показания Π² Ρ‚Π°Π±Π»ΠΈΡ†Ρƒ 1, 2.

Рис. 9. Π‘Ρ…Π΅ΠΌΠ° исслСдования: Π°) ΠΏΠΎΡΠ»Π΅Π΄ΠΎΠ²Π°Ρ‚Π΅Π»ΡŒΠ½ΠΎΠ΅ соСдинСниС, Π±) ΠΏΠ°Ρ€Π°Π»Π»Π΅Π»ΡŒΠ½ΠΎΠ΅ соСдинСниС

5) РассчитайтС тСорСтичСскиС значСния сопротивлСний ΠΌΠ΅ΠΆΠ΄Ρƒ ΡƒΠΊΠ°Π·Π°Π½Π½Ρ‹ΠΌΠΈ Ρ‚ΠΎΡ‡ΠΊΠ°ΠΌΠΈ схСмы ΠΈ сравнитС ΠΈΡ… с ΠΈΠ·ΠΌΠ΅Ρ€Π΅Π½Π½Ρ‹ΠΌΠΈ.

6) Π˜Π·ΠΌΠ΅Ρ€ΡŒΡ‚Π΅ с ΠΏΠΎΠΌΠΎΡ‰ΡŒΡŽ ΠΌΡƒΠ»ΡŒΡ‚ΠΈΠΌΠ΅Ρ‚Ρ€Π° (Π² Ρ€Π΅ΠΆΠΈΠΌΠ΅ измСрСния постоянного Ρ‚ΠΎΠΊΠ°) Ρ‚ΠΎΠΊΠΈ, Ρ‚Π΅ΠΊΡƒΡ‰ΠΈΠ΅ Ρ‡Π΅Ρ€Π΅Π· ΠΊΠ°ΠΆΠ΄ΠΎΠ΅ сопротивлСниС. Π—Π°ΠΏΠΈΡˆΠΈΡ‚Π΅ показания ΠΏΡ€ΠΈΠ±ΠΎΡ€Π° Π² Ρ‚Π°Π±Π»ΠΈΡ†Ρƒ 1.

7) ΠŸΡ€ΠΎΠ²Π΅Ρ€ΡŒΡ‚Π΅ ΡΠΊΡΠΏΠ΅Ρ€ΠΈΠΌΠ΅Π½Ρ‚Π°Π»ΡŒΠ½ΠΎ, Ρ‡Ρ‚ΠΎ Π² ΠΏΠΎΡΠ»Π΅Π΄ΠΎΠ²Π°Ρ‚Π΅Π»ΡŒΠ½ΠΎΠΉ Ρ†Π΅ΠΏΠΈ Ρ‚ΠΎΠΊ ΠΎΠ΄ΠΈΠ½Π°ΠΊΠΎΠ² Ρ‡Π΅Ρ€Π΅Π· всС сопротивлСния, Π° Π² ΠΏΠ°Ρ€Π°Π»Π»Π΅Π»ΡŒΠ½ΠΎΠΉ Ρ†Π΅ΠΏΠΈ раздСляСтся Ρ‚Π°ΠΊ, Ρ‡Ρ‚ΠΎ сумма всСх Ρ‚ΠΎΠΊΠΎΠ² Ρ‡Π΅Ρ€Π΅Π· ΠΏΠ°Ρ€Π°Π»Π»Π΅Π»ΡŒΠ½ΠΎ соСдинСнныС элСмСнты, Ρ€Π°Π²Π½Π° ΠΏΠΎΠ»Π½ΠΎΠΌΡƒ Ρ‚ΠΎΠΊΡƒ Ρ‡Π΅Ρ€Π΅Π· вСсь участок.

8) Π˜Π·ΠΌΠ΅Ρ€ΡŒΡ‚Π΅ с ΠΏΠΎΠΌΠΎΡ‰ΡŒΡŽ ΠΌΡƒΠ»ΡŒΡ‚ΠΈΠΌΠ΅Ρ‚Ρ€Π° (Π² Ρ€Π΅ΠΆΠΈΠΌΠ΅ измСрСния постоянного напряТСния) напряТСния Π½Π° ΠΊΠ°ΠΆΠ΄ΠΎΠΌ сопротивлСнии. Π—Π°ΠΏΠΈΡˆΠΈΡ‚Π΅ показания ΠΏΡ€ΠΈΠ±ΠΎΡ€Π° Π² Ρ‚Π°Π±Π»ΠΈΡ†Ρƒ 2.

9) ΠŸΡ€ΠΎΠ²Π΅Ρ€ΡŒΡ‚Π΅ ΡΠΊΡΠΏΠ΅Ρ€ΠΈΠΌΠ΅Π½Ρ‚Π°Π»ΡŒΠ½ΠΎ, Ρ‡Ρ‚ΠΎ Π² ΠΏΠΎΡΠ»Π΅Π΄ΠΎΠ²Π°Ρ‚Π΅Π»ΡŒΠ½ΠΎΠΉ Ρ†Π΅ΠΏΠΈ напряТСниС Π½Π° всСм участкС Ρ€Π°Π²Π½ΠΎ суммС напряТСний Π½Π° ΠΊΠ°ΠΆΠ΄ΠΎΠΌ элСмСнтС, Π° Π² ΠΏΠ°Ρ€Π°Π»Π»Π΅Π»ΡŒΠ½ΠΎΠΉ Ρ†Π΅ΠΏΠΈ, напряТСниС ΠΎΠ΄Π½ΠΎ ΠΈ Ρ‚ΠΎ ΠΆΠ΅ Π½Π° ΠΊΠ°ΠΆΠ΄ΠΎΠΌ элСмСнтС.

10) ΠžΡ‚ΠΊΠ»ΡŽΡ‡ΠΈΡ‚ΡŒ схСму.

Π’Π°Π±Π»ΠΈΡ†Π° 1

РассчитанныС ΠΈΠΈΠ·ΠΌΠ΅Ρ€Π΅Π½Π½Ρ‹Π΅ ΠΏΠ°Ρ€Π°ΠΌΠ΅Ρ‚Ρ€Ρ‹ для ΠΏΠΎΡΠ»Π΅Π΄ΠΎΠ²Π°Ρ‚Π΅Π»ΡŒΠ½ΠΎΠ³ΠΎ соСдинСния рСзисторов: Аи Π’; Π’ΠΈ Π‘; Аи Π‘.

β„–Π²Π°Ρ€.

УстановлСно

Рассчитано

Π˜Π·ΠΌΡ‘Ρ€Π΅Π½Π½Ρ‹Π΅ ΠΏΠ°Ρ€Π°ΠΌΠ΅Ρ‚Ρ€Ρ‹

R1

R2

А— Π’

Π’β€” Π‘

А— Π‘

I

U

U1

U2

кОм

кОм

кОм

кОм

кОм

А

Π’

Π’

Π’

Π’Π°Π±Π»ΠΈΡ†Π° 2

РассчитанныС ΠΈΠΈΠ·ΠΌΠ΅Ρ€Π΅Π½Π½Ρ‹Π΅ ΠΏΠ°Ρ€Π°ΠΌΠ΅Ρ‚Ρ€Ρ‹ для ΠΏΠ°Ρ€Π°Π»Π»Π΅Π»ΡŒΠ½ΠΎΠ³ΠΎ соСдинСния рСзисторов: DΠΈ Π•.

β„–Π²Π°Ρ€.

УстановлСно

Рассчитано

Π˜Π·ΠΌΠ΅Ρ€Ρ‘Π½Π½Ρ‹Π΅ ΠΏΠ°Ρ€Π°ΠΌΠ΅Ρ‚Ρ€Ρ‹

R3

R4

D ΠΈΠ•

U

I

I1

I2

кОм

кОм

кОм

Π’

А

А

А

Π‘ΠΎΠ΄Π΅Ρ€ΠΆΠ°Π½ΠΈΠ΅ ΠΎΡ‚Ρ‡Π΅Ρ‚Π°

Π°) Π½Π°Π·Π²Π°Π½ΠΈΠ΅ ΠΈ Ρ†Π΅Π»ΡŒ Ρ€Π°Π±ΠΎΡ‚Ρ‹;

Π±) схСмы экспСримСнтов ΠΈ Ρ‚Π°Π±Π»ΠΈΡ†Ρ‹ ΠΏΠΎΠ»ΡƒΡ‡Π΅Π½Π½Ρ‹Ρ… ΡΠΊΡΠΏΠ΅Ρ€ΠΈΠΌΠ΅Π½Ρ‚Π°Π»ΡŒΠ½Ρ‹Ρ… Π΄Π°Π½Π½Ρ‹Ρ…;

Π²) Ρ€Π΅Π·ΡƒΠ»ΡŒΡ‚Π°Ρ‚Ρ‹ расчётов:

Π³) Π²Ρ‹Π²ΠΎΠ΄Ρ‹ ΠΏΠΎ Ρ€Π°Π±ΠΎΡ‚Π΅.

ΠšΠΎΠ½Ρ‚Ρ€ΠΎΠ»ΡŒΠ½Ρ‹Π΅ вопросы

1) ΠœΠΎΠΆΠ΅Ρ‚ Π»ΠΈ сопротивлСниС участка Π΄Π²ΡƒΡ… ΠΏΠ°Ρ€Π°Π»Π»Π΅Π»ΡŒΠ½ΠΎ соСдинСнных ΠΏΡ€ΠΎΠ²ΠΎΠ΄Π½ΠΈΠΊΠΎΠ² Π±Ρ‹Ρ‚ΡŒ большС (мСньшС) любого ΠΈΠ· Π½ΠΈΡ…? ΠžΠ±ΡŠΡΡΠ½ΠΈΡ‚Π΅ ΠΎΡ‚Π²Π΅Ρ‚.

2) Как ΠΏΠΎ Π²ΠΎΠ»ΡŒΡ‚Π°ΠΌΠΏΠ΅Ρ€Π½ΠΎΠΉ характСристикС ΠΎΠΏΡ€Π΅Π΄Π΅Π»ΠΈΡ‚ΡŒ Π²Π΅Π»ΠΈΡ‡ΠΈΠ½Ρƒ сопротивлСния Ρ†Π΅ΠΏΠΈ?

3) Как ΠΏΠΎ показаниям ΠΌΡƒΠ»ΡŒΡ‚ΠΈΠΌΠ΅Ρ‚Ρ€Π° ΠΌΠΎΠΆΠ½ΠΎ ΠΎΠΏΡ€Π΅Π΄Π΅Π»ΠΈΡ‚ΡŒ Π²Π΅Π»ΠΈΡ‡ΠΈΠ½Ρƒ сопротивлСния участка элСктричСской Ρ†Π΅ΠΏΠΈ ΠΈ потрСбляСмая ΠΌΠΎΡ‰Π½ΠΎΡΡ‚ΡŒ?

4) НарисуйтС схСмы ΠΏΠΎΡΠ»Π΅Π΄ΠΎΠ²Π°Ρ‚Π΅Π»ΡŒΠ½ΠΎΠ΅ ΠΈ ΠΏΠ°Ρ€Π°Π»Π»Π΅Π»ΡŒΠ½ΠΎΠ΅ соСдинСниС Π΄Π²ΡƒΡ… рСзисторов.

5) ΠžΠ±ΡŠΡΡΠ½ΠΈΡ‚Π΅ физичСскиС Π·Π°ΠΊΠΎΠ½Ρ‹ соСдинСния рСзисторов.

Π’Ρ‹Π²ΠΎΠ΄Ρ‹

Π’ Ρ€Π°Π±ΠΎΡ‚Π΅ исслСдованы свойства ΠΈ физичСскиС Π·Π°ΠΊΠΎΠ½Ρ‹ ΠΏΠ°Ρ€Π°Π»Π»Π΅Π»ΡŒΠ½ΠΎΠ³ΠΎ ΠΈ ΠΏΠΎΡΠ»Π΅Π΄ΠΎΠ²Π°Ρ‚Π΅Π»ΡŒΠ½ΠΎΠ³ΠΎ соСдинСния рСзисторов бСс примСнСния ΠΏΠ°ΠΉΠΊΠΈ.

ΠŸΡ€ΠΈ построСнии ΠΌΠ°ΠΊΠ΅Ρ‚ΠΎΠ² ΠΎΡ‚ ΠΌΠΎΠ½Ρ‚Π°ΠΆΠ° ΠΏΠ°ΠΉΠΊΠΎΠΉ удаСтся ΠΏΠΎΠ»Π½ΠΎΡΡ‚ΡŒΡŽ ΠΎΡ‚ΠΊΠ°Π·Π°Ρ‚ΡŒΡΡ ΠΈΠ»ΠΈ свСсти Π΅Π³ΠΎ ΠΊ ΠΌΠΈΠ½ΠΈΠΌΡƒΠΌΡƒ. Как ΠΏΠΎΠΊΠ°Π·Π°Π», ΠΎΠΏΡ‹Ρ‚ практичСскоС использованиС ΠΌΠ°ΠΊΠ΅Ρ‚Π½Ρ‹Ρ… ΠΏΠ»Π°Ρ‚ ΠΏΠΎΠ·Π²ΠΎΠ»ΡΡŽΡ‚, сущСствСнно ΡΡΠΊΠΎΠ½ΠΎΠΌΠΈΡ‚ΡŒ Π²Ρ€Π΅ΠΌΠ΅Π½Π½Ρ‹Π΅ ΠΈ ΠΌΠ°Ρ‚Π΅Ρ€ΠΈΠ°Π»ΡŒΠ½Ρ‹Π΅ Π·Π°Ρ‚Ρ€Π°Ρ‚Ρ‹.

Π›ΠΈΡ‚Π΅Ρ€Π°Ρ‚ΡƒΡ€Π°:

  1. А. Π›. ΠœΠ°Ρ€Ρ‡Π΅Π½ΠΊΠΎ, Освальд Π‘. Π’. Π›Π°Π±ΠΎΡ€Π°Ρ‚ΠΎΡ€Π½Ρ‹ΠΉ ΠΏΡ€Π°ΠΊΡ‚ΠΈΠΊΡƒΠΌ ΠΏΠΎ элСктротСхникС ΠΈ элСктроникС Π² срСдСMULTISIM. Π£Ρ‡Π΅Π±Π½ΠΎΠ΅ пособиС для Π²ΡƒΠ·ΠΎΠ².-М.:Π”ΠœΠš ΠŸΡ€Π΅ΡΡ.2010.-448Π‘.
  2. Π›. Π“. БСлиовская, БСлиовский Н. А. Π ΠΎΠ±ΠΎΡ‚ΠΈΠ·ΠΈΡ€ΠΎΠ²Π°Π½Π½Ρ‹Π΅ Π»Π°Π±ΠΎΡ€Π°Ρ‚ΠΎΡ€Π½Ρ‹Π΅Π΅ Ρ€Π°Π±ΠΎΡ‚Ρ‹ ΠΏΠΎ Ρ„ΠΈΠ·ΠΈΠΊΠΈ, ΠŸΡ€ΠΎΠΏΠ΅Π΄Π΅Π²Ρ‚Π΅Ρ‚ΠΈΡ‡Π΅ΡΠΊΠΈΠΉ курс Ρ„ΠΈΠ·ΠΈΠΊΠ°. -М.:Π”ΠœΠš ΠŸΡ€Π΅ΡΡ.2016.-164Π‘.
  3. ВикипСдия [Π­Π»Π΅ΠΊΡ‚Ρ€ΠΎΠ½Π½Ρ‹ΠΉ рСсурс]. http://ru.wikipedia.org/wiki/Processing.

ΠžΡΠ½ΠΎΠ²Π½Ρ‹Π΅ Ρ‚Π΅Ρ€ΠΌΠΈΠ½Ρ‹ (Π³Π΅Π½Π΅Ρ€ΠΈΡ€ΡƒΡŽΡ‚ΡΡ автоматичСски): ΠΎΠ±Ρ‰Π΅Π΅ сопротивлСниС, рСзистор, сопротивлСниС, ΠΏΠΎΡΠ»Π΅Π΄ΠΎΠ²Π°Ρ‚Π΅Π»ΡŒΠ½Π°Ρ Ρ†Π΅ΠΏΡŒ, ΠΏΠΎΡΠ»Π΅Π΄ΠΎΠ²Π°Ρ‚Π΅Π»ΡŒΠ½ΠΎΠ΅ соСдинСниС, соСдинСниС рСзисторов, Π²Π΅Π»ΠΈΡ‡ΠΈΠ½Π° Ρ‚ΠΎΠΊΠ°, ΠΊΠΎΠΌ, макСтная ΠΏΠ»Π°Ρ‚Π°, ΠΏΠ°Ρ€Π°Π»Π»Π΅Π»ΡŒΠ½Π°Ρ Ρ†Π΅ΠΏΡŒ, ΠΏΠ°Ρ€Π°Π»Π»Π΅Π»ΡŒΠ½ΠΎΠ΅ соСдинСниС.

Π‘Ρ…Π΅ΠΌΠ½Ρ‹Π΅ соСдинСния Π² рСзисторах — CoderLessons.com

РСзистор ΠΏΡ€ΠΈ ΠΏΠΎΠ΄ΠΊΠ»ΡŽΡ‡Π΅Π½ΠΈΠΈ Π² Ρ†Π΅ΠΏΠΈ, это соСдинСниС ΠΌΠΎΠΆΠ΅Ρ‚ Π±Ρ‹Ρ‚ΡŒ ΠΏΠΎΡΠ»Π΅Π΄ΠΎΠ²Π°Ρ‚Π΅Π»ΡŒΠ½Ρ‹ΠΌ ΠΈΠ»ΠΈ ΠΏΠ°Ρ€Π°Π»Π»Π΅Π»ΡŒΠ½Ρ‹ΠΌ. Π”Π°Π²Π°ΠΉΡ‚Π΅ Ρ‚Π΅ΠΏΠ΅Ρ€ΡŒ ΡƒΠ·Π½Π°Π΅ΠΌ, Ρ‡Ρ‚ΠΎ ΠΏΡ€ΠΎΠΈΠ·ΠΎΠΉΠ΄Π΅Ρ‚ с суммарными значСниями Ρ‚ΠΎΠΊΠ°, напряТСния ΠΈ сопротивлСния, Ссли ΠΎΠ½ΠΈ Π±ΡƒΠ΄ΡƒΡ‚ соСдинСны ΠΏΠΎΡΠ»Π΅Π΄ΠΎΠ²Π°Ρ‚Π΅Π»ΡŒΠ½ΠΎ, Π° Ρ‚Π°ΠΊΠΆΠ΅ ΠΏΡ€ΠΈ ΠΏΠ°Ρ€Π°Π»Π»Π΅Π»ΡŒΠ½ΠΎΠΌ соСдинСнии.

РСзисторы Π² сСрии

Π”Π°Π²Π°ΠΉΡ‚Π΅ посмотрим, Ρ‡Ρ‚ΠΎ происходит, ΠΊΠΎΠ³Π΄Π° нСсколько ΠΏΠΎΡΠ»Π΅Π΄ΠΎΠ²Π°Ρ‚Π΅Π»ΡŒΠ½ΠΎ соСдинСнных рСзисторов. Рассмотрим Ρ‚Ρ€ΠΈ рСзистора с Ρ€Π°Π·Π½Ρ‹ΠΌΠΈ значСниями, ΠΊΠ°ΠΊ ΠΏΠΎΠΊΠ°Π·Π°Π½ΠΎ Π½Π° рисункС Π½ΠΈΠΆΠ΅.

сопротивлСниС

ΠžΠ±Ρ‰Π΅Π΅ сопротивлСниС Ρ†Π΅ΠΏΠΈ, ΠΈΠΌΠ΅ΡŽΡ‰Π΅ΠΉ ΠΏΠΎΡΠ»Π΅Π΄ΠΎΠ²Π°Ρ‚Π΅Π»ΡŒΠ½Ρ‹Π΅ рСзисторы, Ρ€Π°Π²Π½ΠΎ суммС ΠΎΡ‚Π΄Π΅Π»ΡŒΠ½Ρ‹Ρ… сопротивлСний. Π­Ρ‚ΠΎ ΠΎΠ·Π½Π°Ρ‡Π°Π΅Ρ‚, Ρ‡Ρ‚ΠΎ Π½Π° ΠΏΡ€ΠΈΠ²Π΅Π΄Π΅Π½Π½ΠΎΠΌ Π²Ρ‹ΡˆΠ΅ рисункС Ρ‚Ρ€ΠΈ рСзистора ΠΈΠΌΠ΅ΡŽΡ‚ значСния 1 кОм, 5 кОм ΠΈ 9 кОм соотвСтствСнно.

ΠžΠ±Ρ‰Π΅Π΅ Π·Π½Π°Ρ‡Π΅Π½ΠΈΠ΅ сопротивлСния рСзисторной сСти составляСт β€”

R=R1Β +R2Β +R3

Π­Ρ‚ΠΎ ΠΎΠ·Π½Π°Ρ‡Π°Π΅Ρ‚, Ρ‡Ρ‚ΠΎ 1 + 5 + 9 = 15 кОм β€” это ΠΏΠΎΠ»Π½ΠΎΠ΅ сопротивлСниС.

Π“Π΄Π΅ R 1 β€” это сопротивлСниС 1- Π³ΠΎ рСзистора, R 2 β€” это сопротивлСниС 2- Π³ΠΎ рСзистора, Π° R 3 β€” это сопротивлСниС 3- Π³ΠΎ рСзистора Π² Π²Ρ‹ΡˆΠ΅ΡƒΠΊΠ°Π·Π°Π½Π½ΠΎΠΉ сСти рСзисторов.

Π²ΠΎΠ»ΡŒΡ‚Π°ΠΆ

ΠžΠ±Ρ‰Π΅Π΅ напряТСниС, ΠΊΠΎΡ‚ΠΎΡ€ΠΎΠ΅ появляСтся Π² сСти ΠΏΠΎΡΠ»Π΅Π΄ΠΎΠ²Π°Ρ‚Π΅Π»ΡŒΠ½Ρ‹Ρ… рСзисторов, являСтся суммой ΠΏΠ°Π΄Π΅Π½ΠΈΠΉ напряТСния Π½Π° ΠΊΠ°ΠΆΠ΄ΠΎΠΌ ΠΎΡ‚Π΄Π΅Π»ΡŒΠ½ΠΎΠΌ сопротивлСнии. На рисункС Π²Ρ‹ΡˆΠ΅ Ρƒ нас Π΅ΡΡ‚ΡŒ Ρ‚Ρ€ΠΈ Ρ€Π°Π·Π½Ρ‹Ρ… рСзистора, ΠΊΠΎΡ‚ΠΎΡ€Ρ‹Π΅ ΠΈΠΌΠ΅ΡŽΡ‚ Ρ‚Ρ€ΠΈ Ρ€Π°Π·Π½Ρ‹Ρ… значСния падСния напряТСния Π½Π° ΠΊΠ°ΠΆΠ΄ΠΎΠΉ ступСни.

ΠžΠ±Ρ‰Π΅Π΅ напряТСниС, ΠΊΠΎΡ‚ΠΎΡ€ΠΎΠ΅ появляСтся Π½Π° Ρ†Π΅ΠΏΠΈ β€”

V=V1Β +V2Β +V3

Π­Ρ‚ΠΎ ΠΎΠ·Π½Π°Ρ‡Π°Π΅Ρ‚, Ρ‡Ρ‚ΠΎ ΠΏΠΎΠ»Π½ΠΎΠ΅ напряТСниС Ρ€Π°Π²Π½ΠΎ 1v + 5v + 9v = 15v.

Π“Π΄Π΅ V 1 β€” ΠΏΠ°Π΄Π΅Π½ΠΈΠ΅ напряТСния 1- Π³ΠΎ рСзистора, V 2 β€” ΠΏΠ°Π΄Π΅Π½ΠΈΠ΅ напряТСния 2- Π³ΠΎ рСзистора, Π° V 3 β€” ΠΏΠ°Π΄Π΅Π½ΠΈΠ΅ напряТСния 3- Π³ΠΎ рСзистора Π² Π²Ρ‹ΡˆΠ΅ΡƒΠΊΠ°Π·Π°Π½Π½ΠΎΠΉ сСти рСзисторов.

Π’Π΅ΠΊΡƒΡ‰ΠΈΠΉ

ΠžΠ±Ρ‰Π΅Π΅ количСство Ρ‚ΠΎΠΊΠ°, ΠΏΡ€ΠΎΡ‚Π΅ΠΊΠ°ΡŽΡ‰Π΅Π³ΠΎ Ρ‡Π΅Ρ€Π΅Π· Π½Π°Π±ΠΎΡ€ ΠΏΠΎΡΠ»Π΅Π΄ΠΎΠ²Π°Ρ‚Π΅Π»ΡŒΠ½ΠΎ Π²ΠΊΠ»ΡŽΡ‡Π΅Π½Π½Ρ‹Ρ… рСзисторов, ΠΎΠ΄ΠΈΠ½Π°ΠΊΠΎΠ²ΠΎ Π²ΠΎ всСх Ρ‚ΠΎΡ‡ΠΊΠ°Ρ… сСти рСзисторов. Π‘Π»Π΅Π΄ΠΎΠ²Π°Ρ‚Π΅Π»ΡŒΠ½ΠΎ, Ρ‚ΠΎΠΊ Ρ€Π°Π²Π΅Π½ 5А ΠΏΡ€ΠΈ ΠΈΠ·ΠΌΠ΅Ρ€Π΅Π½ΠΈΠΈ Π½Π° Π²Ρ…ΠΎΠ΄Π΅ ΠΈΠ»ΠΈ Π² любой Ρ‚ΠΎΡ‡ΠΊΠ΅ ΠΌΠ΅ΠΆΠ΄Ρƒ рСзисторами ΠΈΠ»ΠΈ Π΄Π°ΠΆΠ΅ Π½Π° Π²Ρ‹Ρ…ΠΎΠ΄Π΅.

Π’ΠΎΠΊ Ρ‡Π΅Ρ€Π΅Π· ΡΠ΅Ρ‚ΡŒ β€”

I=I1=I2=I3

Π­Ρ‚ΠΎ ΠΎΠ·Π½Π°Ρ‡Π°Π΅Ρ‚, Ρ‡Ρ‚ΠΎ Ρ‚ΠΎΠΊ Π²ΠΎ всСх Ρ‚ΠΎΡ‡ΠΊΠ°Ρ… составляСт 5А.

Π“Π΄Π΅ I 1 β€” Ρ‚ΠΎΠΊ Ρ‡Π΅Ρ€Π΅Π· 1- ΠΉ рСзистор, I 2 β€” Ρ‚ΠΎΠΊ Ρ‡Π΅Ρ€Π΅Π· 2- ΠΉ рСзистор, Π° I 3 β€” Ρ‚ΠΎΠΊ Ρ‡Π΅Ρ€Π΅Π· 3- ΠΉ рСзистор Π² Π²Ρ‹ΡˆΠ΅ΡƒΠΏΠΎΠΌΡΠ½ΡƒΡ‚ΠΎΠΉ сСти рСзисторов.

РСзисторы ΠΏΠ°Ρ€Π°Π»Π»Π΅Π»ΡŒΠ½ΠΎ

Π”Π°Π²Π°ΠΉΡ‚Π΅ посмотрим, Ρ‡Ρ‚ΠΎ происходит, ΠΊΠΎΠ³Π΄Π° нСсколько рСзисторов ΠΏΠΎΠ΄ΠΊΠ»ΡŽΡ‡Π΅Π½Ρ‹ ΠΏΠ°Ρ€Π°Π»Π»Π΅Π»ΡŒΠ½ΠΎ. Рассмотрим Ρ‚Ρ€ΠΈ рСзистора с Ρ€Π°Π·Π½Ρ‹ΠΌΠΈ значСниями, ΠΊΠ°ΠΊ ΠΏΠΎΠΊΠ°Π·Π°Π½ΠΎ Π½Π° рисункС Π½ΠΈΠΆΠ΅.

сопротивлСниС

ΠžΠ±Ρ‰Π΅Π΅ сопротивлСниС Ρ†Π΅ΠΏΠΈ, ΠΈΠΌΠ΅ΡŽΡ‰Π΅ΠΉ ΠΏΠ°Ρ€Π°Π»Π»Π΅Π»ΡŒΠ½Ρ‹Π΅ рСзисторы, рассчитываСтся ΠΈΠ½Π°Ρ‡Π΅, Ρ‡Π΅ΠΌ ΠΌΠ΅Ρ‚ΠΎΠ΄ ΠΏΠΎΡΠ»Π΅Π΄ΠΎΠ²Π°Ρ‚Π΅Π»ΡŒΠ½Ρ‹Ρ… рСзисторных сСтСй. Π—Π΄Π΅ΡΡŒ ΠΎΠ±Ρ€Π°Ρ‚Π½ΠΎΠ΅ (1 / R) Π·Π½Π°Ρ‡Π΅Π½ΠΈΠ΅ ΠΎΡ‚Π΄Π΅Π»ΡŒΠ½Ρ‹Ρ… сопротивлСний складываСтся с ΠΎΠ±Ρ€Π°Ρ‚Π½ΠΎΠΉ Π²Π΅Π»ΠΈΡ‡ΠΈΠ½ΠΎΠΉ алгСбраичСской суммы, Ρ‡Ρ‚ΠΎΠ±Ρ‹ ΠΏΠΎΠ»ΡƒΡ‡ΠΈΡ‚ΡŒ ΠΎΠ±Ρ‰Π΅Π΅ Π·Π½Π°Ρ‡Π΅Π½ΠΈΠ΅ сопротивлСния.

ΠžΠ±Ρ‰Π΅Π΅ Π·Π½Π°Ρ‡Π΅Π½ΠΈΠ΅ сопротивлСния рСзисторной сСти составляСт β€”

Β Π³ΠΈΠ΄Ρ€ΠΎΡ€Π°Π·Ρ€Ρ‹Π²Π°1R,=Β Π³ΠΈΠ΄Ρ€ΠΎΡ€Π°Π·Ρ€Ρ‹Π²Π°1R1Β +Β Π³ΠΈΠ΄Ρ€ΠΎΡ€Π°Π·Ρ€Ρ‹Π²Π°1R2Β +Β Π³ΠΈΠ΄Ρ€ΠΎΡ€Π°Π·Ρ€Ρ‹Π²Π°1R3

Π“Π΄Π΅ R 1 β€” это сопротивлСниС 1- Π³ΠΎ рСзистора, R 2 β€” это сопротивлСниС 2- Π³ΠΎ рСзистора, Π° R 3 β€” это сопротивлСниС 3- Π³ΠΎ рСзистора Π² Π²Ρ‹ΡˆΠ΅ΡƒΠΊΠ°Π·Π°Π½Π½ΠΎΠΉ сСти рСзисторов.

НапримСр, Ссли Ρ€Π°ΡΡΠΌΠ°Ρ‚Ρ€ΠΈΠ²Π°ΡŽΡ‚ΡΡ значСния сопротивлСния ΠΈΠ· ΠΏΡ€Π΅Π΄Ρ‹Π΄ΡƒΡ‰Π΅Π³ΠΎ ΠΏΡ€ΠΈΠΌΠ΅Ρ€Π°, это ΠΎΠ·Π½Π°Ρ‡Π°Π΅Ρ‚, Ρ‡Ρ‚ΠΎ R 1 = 1 кОм, R 2 = 5 кОм ΠΈ R 3 = 9 кОм. ΠžΠ±Ρ‰Π΅Π΅ сопротивлСниС ΠΏΠ°Ρ€Π°Π»Π»Π΅Π»ΡŒΠ½ΠΎΠΉ рСзисторной сСти составит β€”

Β Π³ΠΈΠ΄Ρ€ΠΎΡ€Π°Π·Ρ€Ρ‹Π²Π°1R,=Β Π³ΠΈΠ΄Ρ€ΠΎΡ€Π°Π·Ρ€Ρ‹Π²Π°11Β +Β Π³ΠΈΠ΄Ρ€ΠΎΡ€Π°Π·Ρ€Ρ‹Π²Π°15Β +Β Π³ΠΈΠ΄Ρ€ΠΎΡ€Π°Π·Ρ€Ρ‹Π²Π°19

=Β Π³ΠΈΠ΄Ρ€ΠΎΡ€Π°Π·Ρ€Ρ‹Π²Π°45Β +9Β +545=Β Π³ΠΈΠ΄Ρ€ΠΎΡ€Π°Π·Ρ€Ρ‹Π²Π°5945

R=Β Π³ΠΈΠ΄Ρ€ΠΎΡ€Π°Π·Ρ€Ρ‹Π²Π°4559=0.762KΒ Omega=76,2Β Omega

Из ΠΌΠ΅Ρ‚ΠΎΠ΄Π° расчСта ΠΏΠ°Ρ€Π°Π»Π»Π΅Π»ΡŒΠ½ΠΎΠ³ΠΎ сопротивлСния ΠΌΡ‹ ΠΌΠΎΠΆΠ΅ΠΌ вывСсти простоС ΡƒΡ€Π°Π²Π½Π΅Π½ΠΈΠ΅ для двухрСзисторной ΠΏΠ°Ρ€Π°Π»Π»Π΅Π»ΡŒΠ½ΠΎΠΉ сСти. Π­Ρ‚ΠΎ β€”

R=Β Π³ΠΈΠ΄Ρ€ΠΎΡ€Π°Π·Ρ€Ρ‹Π²Π°R1Β Ρ€Π°Π·R2R1Β +R2Β 

Π²ΠΎΠ»ΡŒΡ‚Π°ΠΆ

ΠžΠ±Ρ‰Π΅Π΅ напряТСниС, ΠΊΠΎΡ‚ΠΎΡ€ΠΎΠ΅ появляСтся Π² сСти ΠΏΠ°Ρ€Π°Π»Π»Π΅Π»ΡŒΠ½Ρ‹Ρ… рСзисторов, совпадаСт с ΠΏΠ°Π΄Π΅Π½ΠΈΠ΅ΠΌ напряТСния Π½Π° ΠΊΠ°ΠΆΠ΄ΠΎΠΌ ΠΎΡ‚Π΄Π΅Π»ΡŒΠ½ΠΎΠΌ сопротивлСнии.

НапряТСниС, ΠΊΠΎΡ‚ΠΎΡ€ΠΎΠ΅ появляСтся Π½Π° Ρ†Π΅ΠΏΠΈ β€”

V=V1=V2=V3

Π“Π΄Π΅ V 1 β€” ΠΏΠ°Π΄Π΅Π½ΠΈΠ΅ напряТСния 1- Π³ΠΎ рСзистора, V 2 β€” ΠΏΠ°Π΄Π΅Π½ΠΈΠ΅ напряТСния 2- Π³ΠΎ рСзистора, Π° V 3 β€” ΠΏΠ°Π΄Π΅Π½ΠΈΠ΅ напряТСния 3- Π³ΠΎ рСзистора Π² Π²Ρ‹ΡˆΠ΅ΡƒΠΊΠ°Π·Π°Π½Π½ΠΎΠΉ сСти рСзисторов. Π‘Π»Π΅Π΄ΠΎΠ²Π°Ρ‚Π΅Π»ΡŒΠ½ΠΎ, напряТСниС ΠΎΠ΄ΠΈΠ½Π°ΠΊΠΎΠ²ΠΎ Π²ΠΎ всСх Ρ‚ΠΎΡ‡ΠΊΠ°Ρ… ΠΏΠ°Ρ€Π°Π»Π»Π΅Π»ΡŒΠ½ΠΎΠΉ сСти рСзисторов.

Π’Π΅ΠΊΡƒΡ‰ΠΈΠΉ

ΠžΠ±Ρ‰Π΅Π΅ количСство Ρ‚ΠΎΠΊΠ°, ΠΏΠΎΡΡ‚ΡƒΠΏΠ°ΡŽΡ‰Π΅Π³ΠΎ Π² ΠΏΠ°Ρ€Π°Π»Π»Π΅Π»ΡŒΠ½ΡƒΡŽ Ρ€Π΅Π·ΠΈΡΡ‚ΠΈΠ²Π½ΡƒΡŽ ΡΠ΅Ρ‚ΡŒ, прСдставляСт собой сумму всСх ΠΎΡ‚Π΄Π΅Π»ΡŒΠ½Ρ‹Ρ… Ρ‚ΠΎΠΊΠΎΠ², ΠΏΡ€ΠΎΡ‚Π΅ΠΊΠ°ΡŽΡ‰ΠΈΡ… Π²ΠΎ всСх ΠΏΠ°Ρ€Π°Π»Π»Π΅Π»ΡŒΠ½Ρ‹Ρ… вСтвях. Π—Π½Π°Ρ‡Π΅Π½ΠΈΠ΅ сопротивлСния ΠΊΠ°ΠΆΠ΄ΠΎΠΉ Π²Π΅Ρ‚Π²ΠΈ опрСдСляСт Π·Π½Π°Ρ‡Π΅Π½ΠΈΠ΅ Ρ‚ΠΎΠΊΠ°, ΠΏΡ€ΠΎΡ‚Π΅ΠΊΠ°ΡŽΡ‰Π΅Π³ΠΎ Ρ‡Π΅Ρ€Π΅Π· Π½Π΅Π΅. ΠžΠ±Ρ‰ΠΈΠΉ Ρ‚ΠΎΠΊ Ρ‡Π΅Ρ€Π΅Π· ΡΠ΅Ρ‚ΡŒ

I=I1Β +I2Β +I3

Π“Π΄Π΅ I 1 β€” Ρ‚ΠΎΠΊ Ρ‡Π΅Ρ€Π΅Π· 1- ΠΉ рСзистор, I 2 β€” Ρ‚ΠΎΠΊ Ρ‡Π΅Ρ€Π΅Π· 2- ΠΉ рСзистор, Π° I 3 β€” Ρ‚ΠΎΠΊ Ρ‡Π΅Ρ€Π΅Π· 3- ΠΉ рСзистор Π² Π²Ρ‹ΡˆΠ΅ΡƒΠΏΠΎΠΌΡΠ½ΡƒΡ‚ΠΎΠΉ сСти рСзисторов. Π‘Π»Π΅Π΄ΠΎΠ²Π°Ρ‚Π΅Π»ΡŒΠ½ΠΎ, сумма ΠΎΡ‚Π΄Π΅Π»ΡŒΠ½Ρ‹Ρ… Ρ‚ΠΎΠΊΠΎΠ² Π² Ρ€Π°Π·Π½Ρ‹Ρ… вСтвях позволяСт ΠΏΠΎΠ»ΡƒΡ‡ΠΈΡ‚ΡŒ суммарный Ρ‚ΠΎΠΊ Π² ΠΏΠ°Ρ€Π°Π»Π»Π΅Π»ΡŒΠ½ΠΎΠΉ рСзистивной сСти.

РСзистор особСнно ΠΈΡΠΏΠΎΠ»ΡŒΠ·ΡƒΠ΅Ρ‚ΡΡ Π² качСствС Π½Π°Π³Ρ€ΡƒΠ·ΠΊΠΈ Π½Π° Π²Ρ‹Ρ…ΠΎΠ΄Π΅ ΠΌΠ½ΠΎΠ³ΠΈΡ… Ρ†Π΅ΠΏΠ΅ΠΉ. Если рСзистивная Π½Π°Π³Ρ€ΡƒΠ·ΠΊΠ° Π²ΠΎΠΎΠ±Ρ‰Π΅ Π½Π΅ ΠΈΡΠΏΠΎΠ»ΡŒΠ·ΡƒΠ΅Ρ‚ΡΡ, ΠΏΠ΅Ρ€Π΅Π΄ Π½Π°Π³Ρ€ΡƒΠ·ΠΊΠΎΠΉ устанавливаСтся рСзистор. РСзистор ΠΎΠ±Ρ‹Ρ‡Π½ΠΎ являСтся основным ΠΊΠΎΠΌΠΏΠΎΠ½Π΅Π½Ρ‚ΠΎΠΌ Π² любой Ρ†Π΅ΠΏΠΈ.

ΠŸΠ°Ρ€Π°Π»Π»Π΅Π»ΡŒΠ½ΠΎΠ΅ Π²ΠΊΠ»ΡŽΡ‡Π΅Π½ΠΈΠ΅ рСзисторов. Как Ρ€Π°ΡΡΡ‡ΠΈΡ‚Π°Ρ‚ΡŒ слоТныС схСмы соСдинСния рСзисторов. Бпособы соСдинСния ΠΏΡ€ΠΎΠ²ΠΎΠ΄Π½ΠΈΠΊΠΎΠ²

ΠžΠΏΡ€Π΅Π΄Π΅Π»Π΅Π½ΠΈΠ΅ ΠΏΠ°Ρ€Π°Π»Π»Π΅Π»ΡŒΠ½ΠΎΠ³ΠΎ соСдинСния

ΠŸΠ°Ρ€Π°Π»Π»Π΅Π»ΡŒΠ½ΠΎΠ΅ соСдинСниС элСктричСских элСмСнтов (ΠΏΡ€ΠΎΠ²ΠΎΠ΄Π½ΠΈΠΊΠΎΠ², сопротивлСний, СмкостСй, индуктивностСй) — это Ρ‚Π°ΠΊΠΎΠ΅ соСдинСниС, ΠΏΡ€ΠΈ ΠΊΠΎΡ‚ΠΎΡ€ΠΎΠΌ ΠΏΠΎΠ΄ΠΊΠ»ΡŽΡ‡Π΅Π½Π½Ρ‹Π΅ элСмСнты Ρ†Π΅ΠΏΠΈ ΠΈΠΌΠ΅ΡŽΡ‚ Π΄Π²Π° ΠΎΠ±Ρ‰ΠΈΡ… ΡƒΠ·Π»Π° ΠΏΠΎΠ΄ΠΊΠ»ΡŽΡ‡Π΅Π½ΠΈΡ.

РСзисторы ΠΏΡ€Π΅Π΄ΡΡ‚Π°Π²Π»ΡΡŽΡ‚ собой пассивныС элСктронныС ΠΊΠΎΠΌΠΏΠΎΠ½Π΅Π½Ρ‚Ρ‹, основным ΠΏΠ°Ρ€Π°ΠΌΠ΅Ρ‚Ρ€ΠΎΠΌ ΠΊΠΎΡ‚ΠΎΡ€Ρ‹Ρ… являСтся сопротивлСниС, Ρ‚Π°ΠΊΠΆΠ΅ Π½Π°Π·Ρ‹Π²Π°Π΅ΠΌΠΎΠ΅ элСктричСским сопротивлСниСм, ΠΎΡ‚ΡΡŽΠ΄Π° ΠΈ Π΄Ρ€ΡƒΠ³ΠΎΠΉ Π½Π°Π·Ρ‹Π²Π°Π΅ΠΌΡ‹ΠΉ рСзистор. Π‘ΠΎΠΏΡ€ΠΎΡ‚ΠΈΠ²Π»Π΅Π½ΠΈΠ΅ — это ΠΏΠ°Ρ€Π°ΠΌΠ΅Ρ‚Ρ€, ΠΏΠΎΠ»Π½ΠΎΡΡ‚ΡŒΡŽ Π½Π΅ зависящий ΠΎΡ‚ напряТСния.

Богласно «старой школС» рСзисторы Π±Ρ‹Π»ΠΈ ΠΎΡ‚ΠΌΠ΅Ρ‡Π΅Π½Ρ‹ «сломанной» Π²Π΅Ρ‚Π²ΡŒΡŽ схСмы, ΡΠΈΠΌΠ²ΠΎΠ»ΠΈΠ·ΠΈΡ€ΡƒΡŽΡ‰Π΅ΠΉ ΠΈΠ·Π±Ρ‹Ρ‚ΠΎΠΊ Π½Π° ΠΏΡ€ΠΎΠ²ΠΎΠ΄Π½ΠΈΠΊΠ΅. Π’ настоящСС врСмя прСдставлСниС ΠΈΠ΄Π΅Π°Π»ΡŒΠ½Ρ‹Ρ… рСзисторов Π² схСмах замСщСния ΠΈΡΠΏΠΎΠ»ΡŒΠ·ΡƒΠ΅Ρ‚ Ρ‚ΠΎΡ‚ ΠΆΠ΅ символ, Ρ‡Ρ‚ΠΎ ΠΈ Π² случаС идСального ΠΎΠ΄Π½ΠΎΠΎΠ±ΠΎΡ€ΠΎΡ‚Π½ΠΎΠ³ΠΎ элСмСнта, описываСмого импСдансом. Π­Ρ‚ΠΎ связано с Ρ‚Π΅ΠΌ, Ρ‡Ρ‚ΠΎ рСзистор являСтся самым элСмСнтарным элСктронным ΠΊΠΎΠΌΠΏΠΎΠ½Π΅Π½Ρ‚ΠΎΠΌ, Π° ΠΎΠΏΡ€Π΅Π΄Π΅Π»Π΅Π½ΠΈΠ΅ импСданса идСального рСзистора Π½Π° основС рСзистора сводится ΠΊ ΠΎΠΏΡ€Π΅Π΄Π΅Π»Π΅Π½ΠΈΡŽ сопротивлСния рСзистора, ΠΊΠΎΡ‚ΠΎΡ€ΠΎΠ΅ рассматриваСтся Π² ΠΊΠΎΠ½Ρ†Π΅ Ρ€Π°Π·Π΄Π΅Π»Π° ΠΏΠΎΠ΄ Π½Π°Π·Π²Π°Π½ΠΈΠ΅ΠΌ «ИмпСданс».

Π”Ρ€ΡƒΠ³ΠΎΠ΅ ΠΎΠΏΡ€Π΅Π΄Π΅Π»Π΅Π½ΠΈΠ΅: сопротивлСния ΠΏΠΎΠ΄ΠΊΠ»ΡŽΡ‡Π΅Π½Ρ‹ ΠΏΠ°Ρ€Π°Π»Π»Π΅Π»ΡŒΠ½ΠΎ, Ссли ΠΎΠ½ΠΈ ΠΏΠΎΠ΄ΠΊΠ»ΡŽΡ‡Π΅Π½Ρ‹ ΠΎΠ΄Π½ΠΎ ΠΈ Ρ‚ΠΎΠΉ ΠΆΠ΅ ΠΏΠ°Ρ€Π΅ ΡƒΠ·Π»ΠΎΠ².

ГрафичСскоС ΠΎΠ±ΠΎΠ·Π½Π°Ρ‡Π΅Π½ΠΈΠ΅ схСмы ΠΏΠ°Ρ€Π°Π»Π»Π΅Π»ΡŒΠ½ΠΎΠ³ΠΎ соСднинСния

На ΠΏΡ€ΠΈΠ²Π΅Π΄Π΅Π½Π½ΠΎΠΌ рисункС ΠΏΠΎΠΊΠ°Π·Π°Π½Π° схСма ΠΏΠ°Ρ€Π°Π»Π»Π΅Π»ΡŒΠ½ΠΎΠ΅ ΠΏΠΎΠ΄ΠΊΠ»ΡŽΡ‡Π΅Π½ΠΈΡ сопротивлСний R1, R2, R3, R4. Из схСмы Π²ΠΈΠ΄Π½ΠΎ, Ρ‡Ρ‚ΠΎ всС эти Ρ‡Π΅Ρ‚Ρ‹Ρ€Π΅ сопротивлСния ΠΈΠΌΠ΅ΡŽΡ‚ Π΄Π²Π΅ ΠΎΠ±Ρ‰ΠΈΠ΅ Ρ‚ΠΎΡ‡ΠΊΠΈ (ΡƒΠ·Π»Π° ΠΏΠΎΠ΄ΠΊΠ»ΡŽΡ‡Π΅Π½ΠΈΡ).

Богласно ΠΎΠ±ΠΎΠ±Ρ‰Π΅Π½Π½ΠΎΠΌΡƒ Π·Π°ΠΊΠΎΠ½Ρƒ Ома, сопротивлСниС опрСдСляСтся. Если Π²Ρ‹ измСряСтС напряТСниС ΠΈ Ρ‚ΠΎΠΊ, ΠΏΡ€ΠΎΡ‚Π΅ΠΊΠ°ΡŽΡ‰ΠΈΠ΅ Π² рСзистивный ΠΊΠΎΠ½Ρ‚ΡƒΡ€, Ρ‡Π΅Ρ€Π½Ρ‹ΠΉ ящик, Ρ‚ΠΎΠ³Π΄Π° Π·Π°ΠΊΠΎΠ½ Ома ΠΌΠΎΠΆΠ΅Ρ‚ Π±Ρ‹Ρ‚ΡŒ Π·Π°ΠΌΠ΅Π½Π΅Π½ ΠΎΠ΄Π½ΠΈΠΌ рСзистором. ВнутрСнняя структура Ρ‚Π°ΠΊΠΎΠ³ΠΎ Ρ‡Π΅Ρ€Π½ΠΎΠ³ΠΎ ящика Π½Π΅ Π΄ΠΎΠ»ΠΆΠ½Π° ΠΈΠ³Ρ€Π°Ρ‚ΡŒ Π°Π±ΡΠΎΠ»ΡŽΡ‚Π½ΠΎ Π½ΠΈΠΊΠ°ΠΊΠΎΠΉ Ρ€ΠΎΠ»ΠΈ, ΠΎΠ½ ΠΌΠΎΠΆΠ΅Ρ‚ ΡΠΎΠ΄Π΅Ρ€ΠΆΠ°Ρ‚ΡŒ любоС количСство рСзисторов, свободно взаимосвязанных — Π·Π°ΠΌΠ΅Π½Π° ΠΈΡ… ΠΎΠ΄Π½ΠΈΠΌ рСзистором всСгда Π²ΠΎΠ·ΠΌΠΎΠΆΠ½Π°.

РСзисторы-рСзисторы, соСдинСнныС ΠΏΠΎΡΠ»Π΅Π΄ΠΎΠ²Π°Ρ‚Π΅Π»ΡŒΠ½ΠΎ Π² Π΄Π°Π½Π½ΠΎΠΉ Π²Π΅Ρ‚Π²ΠΈ Ρ†Π΅ΠΏΠΈ, Ρ€Π°Π²Π½Ρ‹ суммС ΠΈΡ… сопротивлСний. ΠŸΡ€ΠΈ ΠΏΠΎΡΠ»Π΅Π΄ΠΎΠ²Π°Ρ‚Π΅Π»ΡŒΠ½ΠΎΠΌ ΠΏΠΎΠ΄ΠΊΠ»ΡŽΡ‡Π΅Π½ΠΈΠΈ рСзисторов Ρ‡Π΅Ρ€Π΅Π· Π²Π΅Ρ‚Π²ΡŒ, согласно ΠΏΠ΅Ρ€Π²ΠΎΠΌΡƒ Π·Π°ΠΊΠΎΠ½Ρƒ ΠšΠΈΡ€Ρ…Π³ΠΎΡ„Π°, Ρ‚Π΅Ρ‡Π΅Ρ‚ Ρ‚ΠΎΠΊ, Ρ‚.Π΅. ΠΊΠ°ΠΆΠ΄Ρ‹ΠΉ ΠΈΠ· ΠΏΠ°Ρ€Π°Π»Π»Π΅Π»ΡŒΠ½Ρ‹Ρ… ΠΏΠΎΠ΄ΠΊΠ»ΡŽΡ‡Π΅Π½Π½Ρ‹Ρ… рСзисторов Π±ΡƒΠ΄Π΅Ρ‚ Ρ€Π°Π²Π½ΠΎΠΌΠ΅Ρ€Π½ΠΎ Ρ‚Π΅Ρ‡ΡŒ для ΠΊΠ°ΠΆΠ΄ΠΎΠ³ΠΎ Ρ‚ΠΎΠΊΠ°.

Π’ элСктротСхникС принято, Π½ΠΎ Π½Π΅ строго трСбуСтся, Ρ€ΠΈΡΠΎΠ²Π°Ρ‚ΡŒ ΠΏΡ€ΠΎΠ²ΠΎΠ΄Π° Π³ΠΎΡ€ΠΈΠ·ΠΎΠ½Ρ‚Π°Π»ΡŒΠ½ΠΎ ΠΈ Π²Π΅Ρ€Ρ‚ΠΈΠΊΠ°Π»ΡŒΠ½ΠΎ. ΠŸΠΎΡΡ‚ΠΎΠΌΡƒ эту ΠΆΠ΅ схСму ΠΌΠΎΠΆΠ½ΠΎ ΠΈΠ·ΠΎΠ±Ρ€Π°Π·ΠΈΡ‚ΡŒ, ΠΊΠ°ΠΊ Π½Π° рисункС Π½ΠΈΠΆΠ΅. Π­Ρ‚ΠΎ Ρ‚ΠΎΠΆΠ΅ ΠΏΠ°Ρ€Π°Π»Π»Π΅Π»ΡŒΠ½ΠΎΠ΅ соСдинСниС Ρ‚Π΅Ρ… ΠΆΠ΅ самых сопротивлСний.

Π€ΠΎΡ€ΠΌΡƒΠ»Π° для расчСта ΠΏΠ°Ρ€Π°Π»Π»Π΅Π»ΡŒΠ½ΠΎΠ³ΠΎ соСдинСния сопротивлСний

ΠŸΡ€ΠΈ ΠΏΠ°Ρ€Π°Π»Π»Π΅Π»ΡŒΠ½ΠΎΠΌ соСдинСнии обратная Π²Π΅Π»ΠΈΡ‡ΠΈΠ½Π° ΠΎΡ‚ эквивалСнтного сопротивлСния Ρ€Π°Π²Π½Π° суммС ΠΎΠ±Ρ€Π°Ρ‚Π½Ρ‹Ρ… Π²Π΅Π»ΠΈΡ‡ΠΈΠ½ всСх ΠΏΠ°Ρ€Π°Π»Π»Π΅Π»ΡŒΠ½ΠΎ ΠΏΠΎΠ΄ΠΊΠ»ΡŽΡ‡Π΅Π½Π½Ρ‹Ρ… сопротивлСний. ЭквивалСнтная ΠΏΡ€ΠΎΠ²ΠΎΠ΄ΠΈΠΌΠΎΡΡ‚ΡŒ Ρ€Π°Π²Π½Π° суммС всСх ΠΏΠ°Ρ€Π°Π»Π»Π΅Π»ΡŒΠ½ΠΎ ΠΏΠΎΠ΄ΠΊΠ»ΡŽΡ‡Π΅Π½Π½Ρ‹Ρ… проводимостСй элСктричСской схСмы.

ПослС прСобразования ΠΌΡ‹ ΠΏΠΎΠ»ΡƒΡ‡Π°Π΅ΠΌ. Π Π°Π·Π΄Π΅Π»ΠΈΡ‚Π΅ Π΄Π²Π΅ стороны Ρ‚ΠΎΠΊΠΎΠΌ, ΠΊΠΎΡ‚ΠΎΡ€Ρ‹ΠΉ ΠΌΡ‹ ΠΏΠΎΠ»ΡƒΡ‡Π°Π΅ΠΌ. Для Π΄Π²ΡƒΡ… рСзисторов этот шаблон ΠΈΠΌΠ΅Π΅Ρ‚ Π²ΠΈΠ΄. ΠŸΡ€ΠΈ ΠΏΠ°Ρ€Π°Π»Π»Π΅Π»ΡŒΠ½ΠΎΠΌ соСдинСнии рСзисторов напряТСниС Π½Π° Π½ΠΈΡ… Ρ€Π°Π²Π½ΠΎ, Ρ€Π°Π²Π½ΠΎ. Богласно Π·Π°ΠΊΠΎΠ½Ρƒ ΠšΠΈΡ€Ρ…Π³ΠΎΡ„Π° сумма Ρ‚Π΅Ρ‡Π΅Π½ΠΈΠΉ, ΠΏΡ€ΠΎΡ‚Π΅ΠΊΠ°ΡŽΡ‰ΠΈΡ… ΠΈΠ· Β«Π²Π΅Ρ€Ρ…Π½Π΅Π³ΠΎΒ» ΡƒΠ·Π»Π°, Ρ€Π°Π²Π½Π° влиянию, ΠΊΠΎΡ‚ΠΎΡ€ΠΎΠ΅ Π²Ρ‹Ρ€Π°ΠΆΠ°Π΅Ρ‚ Π·Π°Π²ΠΈΡΠΈΠΌΠΎΡΡ‚ΡŒ. Π Π°Π·Π΄Π΅Π»ΠΈΡ‚Π΅ с ΠΎΠ±Π΅ΠΈΡ… сторон напряТСниС.

РСзисторы, ΠΎΡ‚ΠΌΠ΅Ρ‡Π΅Π½Π½Ρ‹Π΅ Ρ†Π²Π΅Ρ‚ΠΎΠ²Ρ‹ΠΌ ΠΊΠΎΠ΄ΠΎΠΌ. РСзисторы Ρ‡Π°Ρ‰Π΅ всСго Π²ΡΡ‚Ρ€Π΅Ρ‡Π°ΡŽΡ‚ΡΡ Π² Π²ΠΈΠ΄Π΅ дискрСтных элСмСнтов, ΠΏΡ€ΠΎΠ΄Π°Π²Π°Π΅ΠΌΡ‹Ρ… ΠΎΡ‚Π΄Π΅Π»ΡŒΠ½ΠΎ ΠΈΠ»ΠΈ Π² Β«Π»Π΅Π½Ρ‚Π΅Β» с Π±ΡƒΠΌΠ°ΠΆΠ½ΠΎΠΉ связью. РСзисторы ΠΈΠΌΠ΅ΡŽΡ‚ Ρ„ΠΎΡ€ΠΌΡƒ нСбольшого корпуса, Π½Π°ΠΏΠΎΠΌΠΈΠ½Π°ΡŽΡ‰Π΅Π³ΠΎ ΠΏΠΎΠΏΠ΅Ρ€Π΅Ρ‡Π½ΠΎΠ΅ сСчСниС ΠΏΡ€ΠΈΠ½Ρ†ΠΈΠΏΠΈΠ°Π»ΡŒΠ½ΠΎΠΉ схСмы Π·Π°ΠΌΠ΅Π½Ρ‹, ΠΈΠ· ΠΊΠΎΡ‚ΠΎΡ€ΠΎΠΉ выводятся Π΄Π²Π° ΠΏΡ€ΠΎΠ²ΠΎΠ΄Π°, Ρ‡Ρ‚ΠΎΠ±Ρ‹ ΠΎΠ½ΠΈ ΠΌΠΎΠ³Π»ΠΈ Π±Ρ‹Ρ‚ΡŒ Π²ΠΊΠ»ΡŽΡ‡Π΅Π½Ρ‹ Π² схСму.


Для ΠΏΡ€ΠΈΠ²Π΅Π΄Π΅Π½Π½ΠΎΠΉ Π²Ρ‹ΡˆΠ΅ схСмы эквивалСнтноС сопротивлСниС ΠΌΠΎΠΆΠ½ΠΎ Ρ€Π°ΡΡΡ‡ΠΈΡ‚Π°Ρ‚ΡŒ ΠΏΠΎ Ρ„ΠΎΡ€ΠΌΡƒΠ»Π΅:


Π’ частном случаС ΠΏΡ€ΠΈ ΠΏΠΎΠ΄ΠΊΠ»ΡŽΡ‡Π΅Π½ΠΈΠΈ ΠΏΠ°Ρ€Π°Π»Π»Π΅Π»ΡŒΠ½ΠΎ Π΄Π²ΡƒΡ… сопротивлСний:

РСзисторы с Ρ€Π°Π·Π»ΠΈΡ‡Π½Ρ‹ΠΌΠΈ сопротивлСниями доступны. Из-Π·Π° ΠΈΡ… часто Π½Π΅Π±ΠΎΠ»ΡŒΡˆΠΈΡ… Ρ€Π°Π·ΠΌΠ΅Ρ€ΠΎΠ² ΠΈ цилиндричСских характСристик, ΠΏΡ€Π΅ΠΏΡΡ‚ΡΡ‚Π²ΡƒΡŽΡ‰ΠΈΡ… ΠΎΠΏΠΈΡΠ°Ρ‚Π΅Π»ΡŒΠ½ΠΎΠΌΡƒ процСссу, Π±Ρ‹Π» принят ΠΎΠ±Ρ‰ΠΈΠΉ стандарт для описания рСзисторов, Ρ‡Ρ‚ΠΎΠ±Ρ‹ ΠΈΠ·Π±Π΅ΠΆΠ°Ρ‚ΡŒ ошибок внСдрСния. ΠœΠ°Ρ€ΠΊΠΈΡ€ΠΎΠ²ΠΊΠ° выполняСтся с ΠΏΠΎΠΌΠΎΡ‰ΡŒΡŽ систСмы Ρ†Π²Π΅Ρ‚ΠΎΠ²ΠΎΠ³ΠΎ кодирования, ΠΏΠΎΠΊΠ°Π·Π°Π½Π½ΠΎΠΉ Π² Ρ‚Π°Π±Π»ΠΈΡ†Π΅ Π½ΠΈΠΆΠ΅. ΠšΠΎΠ΄Ρ‹ ΠΎΠ±Ρ‹Ρ‡Π½ΠΎ ΡΡ‡ΠΈΡ‚Ρ‹Π²Π°ΡŽΡ‚ΡΡ с самых ΡΠΊΡΡ‚Ρ€Π΅ΠΌΠ°Π»ΡŒΠ½Ρ‹Ρ… полос — Ρ‡Π°Ρ‰Π΅ всСго ΠΏΠ΅Ρ€Π²Ρ‹Π΅ Π΄Π²Π΅ полосы ΠΎΠΏΡ€Π΅Π΄Π΅Π»ΡΡŽΡ‚ сопротивлСниС, Ρ‚Ρ€Π΅Ρ‚ΠΈΠΉ ΠΌΠ½ΠΎΠΆΠΈΡ‚Π΅Π»ΡŒ ΠΈ ΡΠ»Π΅Π΄ΡƒΡŽΡ‰ΠΈΠΉ допуск, Π° ΠΈΠ½ΠΎΠ³Π΄Π° ΠΈ Ρ‚Π΅ΠΌΠΏΠ΅Ρ€Π°Ρ‚ΡƒΡ€Π½Ρ‹ΠΉ коэффициСнт сопротивлСния.

Π”ΠΎΠΏΠΎΠ»Π½ΠΈΡ‚Π΅Π»ΡŒΠ½Π°Ρ информация ΠΏΡ€ΠΈΠ²Π΅Π΄Π΅Π½Π° Π½ΠΈΠΆΠ΅ Ρ‚Π°Π±Π»ΠΈΡ†Ρ‹. ΠŸΠΎΠ»ΠΎΡΡ‹ ΠΎΠ±Ρ‹Ρ‡Π½ΠΎ Ρ‚Ρ€ΠΈ, Ρ‡Π΅Ρ‚Ρ‹Ρ€Π΅ ΠΈΠ»ΠΈ ΡˆΠ΅ΡΡ‚ΡŒ, Ссли Π΅ΡΡ‚ΡŒ 3 полосы — Ρ‚ΠΎΠ³Π΄Π° всС Ρ‚Ρ€ΠΈ ΡΠ²Π»ΡΡŽΡ‚ΡΡ сопротивлСниСм, Π° допуск Ρ€Π°Π²Π΅Π½ Β± 20%, Ссли Π΅ΡΡ‚ΡŒ 4 полосы — Ρ‚ΠΎΠ³Π΄Π° ΠΏΠ΅Ρ€Π²Ρ‹Π΅ Ρ‚Ρ€ΠΈ ΠΎΠ·Π½Π°Ρ‡Π°ΡŽΡ‚ сопротивлСниС, Π° Ρ‡Π΅Ρ‚Π²Π΅Ρ€Ρ‚Ρ‹ΠΉ — Ρ‚ΠΎΠ»Π΅Ρ€Π°Π½Ρ‚Π½ΠΎΡΡ‚ΡŒ, Ссли ΠΈΡ… ΡˆΠ΅ΡΡ‚ΡŒ, Π·Π½Π°Ρ‡ΠΈΡ‚, ΠΌΡ‹ ΠΈΠΌΠ΅Π΅ΠΌ Ρ€Π°Π±ΠΎΡ‚Π° с ΠΏΡ€Π΅Ρ†ΠΈΠ·ΠΈΠΎΠ½Π½Ρ‹ΠΌ рСзистором, Π° ΠΏΠ΅Ρ€Π²Ρ‹Π΅ Ρ‚Ρ€ΠΈ ΠΎΠ±ΠΎΠ·Π½Π°Ρ‡Π°ΡŽΡ‚ Π½ΠΎΠΌΠ΅Ρ€Π° сопротивлСний, Ρ‡Π΅Ρ‚Π²Π΅Ρ€Ρ‚Ρ‹ΠΉ — ΠΌΠ½ΠΎΠΆΠΈΡ‚Π΅Π»ΡŒ, пятый — допуск, ΡˆΠ΅ΡΡ‚ΠΎΠΉ — коэффициСнт сопротивлСния Ρ‚Π΅ΠΌΠΏΠ΅Ρ€Π°Ρ‚ΡƒΡ€Ρ‹. ΠŸΡƒΠ±Π»ΠΈΠΊΡƒΠ΅Ρ‚ΡΡ для обСспСчСния согласованности ΠΌΠ°Ρ‚Π΅Ρ€ΠΈΠ°Π»Π° ΠΈ самого быстрого доступа.

Π­ΠΊΠ²ΠΈΠ²Π°Π»Π΅Π½Ρ‚Π½ΠΎΠ΅ сопротивлСниС Ρ†Π΅ΠΏΠΈ опрСдСляСтся ΠΏΠΎ Ρ„ΠΎΡ€ΠΌΡƒΠ»Π΅:

Π’ случаС ΠΏΠΎΠ΄ΠΊΠ»ΡŽΡ‡Π΅Π½ΠΈΡ «n» ΠΎΠ΄ΠΈΠ½Π°ΠΊΠΎΠ²Ρ‹Ρ… сопротивлСний, эквивалСнтноС сопротивлСниС ΠΌΠΎΠΆΠ½ΠΎ Ρ€Π°ΡΡΡ‡ΠΈΡ‚Π°Ρ‚ΡŒ ΠΏΠΎ частной Ρ„ΠΎΡ€ΠΌΡƒΠ»Π΅:

Π˜Π½Ρ„ΠΎΡ€ΠΌΠ°Ρ†ΠΈΡ ΠΎ сопротивлСнии рСзистора кодируСтся Π² Ρ†Π²Π΅Ρ‚Π΅. Π’ Π˜Π½Ρ‚Π΅Ρ€Π½Π΅Ρ‚Π΅ Π²Ρ‹ ΠΌΠΎΠΆΠ΅Ρ‚Π΅ Π½Π°ΠΉΡ‚ΠΈ нСсколько ΠΊΠ°Π»ΡŒΠΊΡƒΠ»ΡΡ‚ΠΎΡ€ΠΎΠ², ΠΊΠΎΡ‚ΠΎΡ€Ρ‹Π΅ ΠΏΠΎΠ·Π²ΠΎΠ»ΡΡŽΡ‚ ΠΊΠΎΠ½Π²Π΅Ρ€Ρ‚ΠΈΡ€ΠΎΠ²Π°Ρ‚ΡŒ ΠΈΠ· слайдов Π² слСнг ΠΈ Π½Π°ΠΎΠ±ΠΎΡ€ΠΎΡ‚. НиТС Π³Ρ€Π°Ρ„ΠΈΠΊΠ° ΠΏΠΎΠΊΠ°Π·Π°Π½Ρ‹ 4 Π½Π°ΠΈΠ±ΠΎΠ»Π΅Π΅ популярных значСния сопротивлСния: 100 Ом, 220 Ом, 4, 7 кОм, 10 кОм. Золотая полоса справа ΡƒΠΊΠ°Π·Ρ‹Π²Π°Π΅Ρ‚ Π½Π° сопротивлСниС рСзистора Β± 5%.

Π”Ρ€ΡƒΠ³ΠΈΠ΅ Π»ΡŽΠ±ΠΈΡ‚Π΅Π»ΠΈ — 10% сСрСбра ΠΈ 2% красныС. Π¦Π²Π΅Ρ‚ΠΎΠ²Ρ‹Π΅ ΠΊΠΎΠ΄Ρ‹ Π½Π°ΠΈΠ±ΠΎΠ»Π΅Π΅ часто ΠΈΡΠΏΠΎΠ»ΡŒΠ·ΡƒΠ΅ΠΌΡ‹Ρ… Π·Π½Π°Ρ‡Π΅Π½ΠΈΠΉ рСзисторов. ΠžΠ±Ρ€Π°Ρ‚ΠΈΡ‚Π΅ Π²Π½ΠΈΠΌΠ°Π½ΠΈΠ΅, Ρ‡Ρ‚ΠΎ, комбинируя рСзисторы, ΠΌΡ‹ ΠΌΠΎΠΆΠ΅ΠΌ ΠΏΠΎΠ»ΡƒΡ‡ΠΈΡ‚ΡŒ Π΄Ρ€ΡƒΠ³ΠΈΠ΅ ΠΎΡ‚ΡΡƒΡ‚ΡΡ‚Π²ΡƒΡŽΡ‰ΠΈΠ΅ значСния. РСзистивноС сопротивлСниС рСзисторов, соСдинСнных ΠΏΠ°Ρ€Π°Π»Π»Π΅Π»ΡŒΠ½ΠΎ, Ρ€Π°Π²Π½ΠΎ инвСрсии ΠΎΠ±Ρ€Π°Ρ‚Π½ΠΎΠΉ суммы Π΅Π΄ΠΈΠ½ΠΈΡ‡Π½Ρ‹Ρ… сопротивлСний. Π”ΠΈΠΎΠ΄Ρ‹ Π½Π΅ Π΄ΠΎΠ»ΠΆΠ½Ρ‹ ΡΠΎΠ΅Π΄ΠΈΠ½ΡΡ‚ΡŒΡΡ ΠΏΠ°Ρ€Π°Π»Π»Π΅Π»ΡŒΠ½ΠΎ Π΄Ρ€ΡƒΠ³ с Π΄Ρ€ΡƒΠ³ΠΎΠΌ — Π² этом случаС ΠΊΠ°ΠΆΠ΄Ρ‹ΠΉ Π΄ΠΎΠ»ΠΆΠ΅Π½ ΠΈΠΌΠ΅Ρ‚ΡŒ свой собствСнный рСзистор. Π”ΠΈΠΎΠ΄Ρ‹ ΠΌΠΎΠΆΠ½ΠΎ ΠΏΠΎΠ΄ΠΊΠ»ΡŽΡ‡Π°Ρ‚ΡŒ ΠΏΠΎΡΠ»Π΅Π΄ΠΎΠ²Π°Ρ‚Π΅Π»ΡŒΠ½ΠΎ ΠΈ ΠΏΠ°Ρ€Π°Π»Π»Π΅Π»ΡŒΠ½ΠΎ-ΠΏΠ°Ρ€Π°Π»Π»Π΅Π»ΡŒΠ½ΠΎ, Π½ΠΎ Π²Π°ΠΌ Π½ΡƒΠΆΠ½ΠΎ Π²Ρ‹Π±Ρ€Π°Ρ‚ΡŒ ΠΏΡ€Π°Π²ΠΈΠ»ΡŒΠ½Ρ‹Π΅ рСзисторы.

Π€ΠΎΡ€ΠΌΡƒΠ»Ρ‹ для частного рассчСта Π²Ρ‹Ρ‚Π΅ΠΊΠ°ΡŽΡ‚ ΠΈΠ· основной Ρ„ΠΎΡ€ΠΌΡƒΠ»Ρ‹.

Π€ΠΎΡ€ΠΌΡƒΠ»Π° для расчСта ΠΏΠ°Ρ€Π°Π»Π»Π΅Π»ΡŒΠ½ΠΎΠ³ΠΎ соСдинСния СмкостСй (кондСнсаторов)

ΠŸΡ€ΠΈ ΠΏΠ°Ρ€Π°Π»Π»Π΅Π»ΡŒΠ½ΠΎΠΌ ΠΏΠΎΠ΄ΠΊΠ»ΡŽΡ‡Π΅Π½ΠΈΠΈ СмкостСй (кондСнсаторов) эквивалСнтная Π΅ΠΌΠΊΠΎΡΡ‚ΡŒ Ρ€Π°Π²Π½Π° суммС ΠΏΠ°Ρ€Π°Π»Π»Π΅Π»ΡŒΠ½ΠΎ ΠΏΠΎΠ΄ΠΊΠ»ΡŽΡ‡Π΅Π½Π½Ρ‹Ρ… СмкостСй:

Π€ΠΎΡ€ΠΌΡƒΠ»Π° для расчСта ΠΏΠ°Ρ€Π°Π»Π»Π΅Π»ΡŒΠ½ΠΎΠ³ΠΎ соСдинСния индуктивностСй

ΠŸΡ€ΠΈ ΠΏΠ°Ρ€Π°Π»Π»Π΅Π»ΡŒΠ½ΠΎΠΌ ΠΏΠΎΠ΄ΠΊΠ»ΡŽΡ‡Π΅Π½ΠΈΠΈ индуктивностСй, эквивалСнтная ΠΈΠ½Π΄ΡƒΠΊΡ‚ΠΈΠ²Π½ΠΎΡΡ‚ΡŒ рассчитываСтся Ρ‚Π°ΠΊ ΠΆΠ΅, ΠΊΠ°ΠΊ ΠΈ эквивалСнтноС сопротивлСниС ΠΏΡ€ΠΈ ΠΏΠ°Ρ€Π°Π»Π»Π΅Π»ΡŒΠ½ΠΎΠΌ соСдинСнии:

Π‘Ρ‚ΠΎΠ» со стандартными значСниями Π΄ΠΎΠ»ΠΆΠ΅Π½ Π±Ρ‹Ρ‚ΡŒ достаточным. Π’Π°Π±Π»ΠΈΡ†Π°: ЗначСния рСзисторов для ΠΎΡ‚Π΄Π΅Π»ΡŒΠ½Ρ‹Ρ… свСтодиодов. Π’ΠΎΠΊ, ΠΏΡ€ΠΎΡ‚Π΅ΠΊΠ°ΡŽΡ‰ΠΈΠΉ Ρ‡Π΅Ρ€Π΅Π· Π΄ΠΈΠΎΠ΄: 20 мА. ΠŸΠΎΠ΄Ρ€ΠΎΠ±Π½Ρ‹Π΅ способы расчСта ΠΈ объСдинСния ΠΌΠΎΠΆΠ½ΠΎ Π½Π°ΠΉΡ‚ΠΈ Π² Π˜Π½Ρ‚Π΅Ρ€Π½Π΅Ρ‚Π΅. Π₯отя ΡΠ½Π΅Ρ€Π³ΠΎΡΠ±Π΅Ρ€Π΅Π³Π°ΡŽΡ‰ΠΈΠ΅ Ρ‚Π΅Ρ…Π½ΠΎΠ»ΠΎΠ³ΠΈΠΈ Π² элСктроникС становятся всС Π±ΠΎΠ»Π΅Π΅ популярными, маловСроятно, Ρ‡Ρ‚ΠΎ мСханичСскиС ΠΎΡ…Π»Π°ΠΆΠ΄Π°ΡŽΡ‰ΠΈΠ΅ устройства ΠΊΠΎΠ³Π΄Π°-Π»ΠΈΠ±ΠΎ Π±ΡƒΠ΄ΡƒΡ‚ ΠΏΠΎΠ»Π½ΠΎΡΡ‚ΡŒΡŽ оставлСны. Π’ настоящСС врСмя, ΠΏΠΎΠΌΠΈΠΌΠΎ Тидкостных систСм охлаТдСния, вСнтиляторы Ρ‡Π°Ρ‰Π΅ всСго ΠΈΡΠΏΠΎΠ»ΡŒΠ·ΡƒΡŽΡ‚ΡΡ Π² корпусах. Π’ ΡΡ‚Π°Ρ‚ΡŒΠ΅ прСдставлСны основныС способы ΠΈΡ… классификации, ΠΎΡ†Π΅Π½ΠΊΠΈ ΠΏΡ€ΠΎΠΈΠ·Π²ΠΎΠ΄ΠΈΡ‚Π΅Π»ΡŒΠ½ΠΎΡΡ‚ΠΈ Π² ΠΎΡ‚Π½ΠΎΡˆΠ΅Π½ΠΈΠΈ Ρ‚Ρ€Π΅Π±ΠΎΠ²Π°Π½ΠΈΠΉ ΠΊΠΎΠ½ΠΊΡ€Π΅Ρ‚Π½ΠΎΠ³ΠΎ прилоТСния ΠΈ элСмСнтов Π΄ΠΈΠ·Π°ΠΉΠ½Π°, Π½Π° ΠΊΠΎΡ‚ΠΎΡ€Ρ‹Π΅ Π²Ρ‹ Π΄ΠΎΠ»ΠΆΠ½Ρ‹ ΠΎΠ±Ρ€Π°Ρ‚ΠΈΡ‚ΡŒ Π²Π½ΠΈΠΌΠ°Π½ΠΈΠ΅.


НСобходимо ΠΎΠ±Ρ€Π°Ρ‚ΠΈΡ‚ΡŒ Π²Π½ΠΈΠΌΠ°Π½ΠΈΠ΅, Ρ‡Ρ‚ΠΎ Π² Ρ„ΠΎΡ€ΠΌΡƒΠ»Π΅ Π½Π΅ ΡƒΡ‡Ρ‚Π΅Π½Ρ‹ Π²Π·Π°ΠΈΠΌΠ½Ρ‹Π΅ индуктивности.

ΠŸΡ€ΠΈΠΌΠ΅Ρ€ свСртывания ΠΏΠ°Ρ€Π°Π»Π»Π΅Π»ΡŒΠ½ΠΎΠ³ΠΎ сопротивлСния

Для участка элСктричСской Ρ†Π΅ΠΏΠΈ Π½Π΅ΠΎΠ±Ρ…ΠΎΠ΄ΠΈΠΌΠΎ Π½Π°ΠΉΡ‚ΠΈ ΠΏΠ°Ρ€Π°Π»Π»Π΅Π»ΡŒΠ½ΠΎΠ΅ соСдинСниС сопротивлСний Π²Ρ‹ΠΏΠΎΠ»Π½ΠΈΡ‚ΡŒ ΠΈΡ… ΠΏΡ€Π΅ΠΎΠ±Ρ€Π°Π·ΠΎΠ²Π°Π½ΠΈΠ΅ Π΄ΠΎ ΠΎΠ΄Π½ΠΎΠ³ΠΎ.

Π¦Π΅Π½Ρ‚Ρ€ΠΎΠ±Π΅ΠΆΠ½Ρ‹Π΅ вСнтиляторы Ρ€Π°Π·Π΄Π΅Π»Π΅Π½Ρ‹ Π½Π° Π²Π΅Π»ΠΈΡ‡ΠΈΠ½Ρƒ ΡƒΠ³Π»Π° Π²Ρ‹Ρ…ΠΎΠ΄Π° лопастСй для вСнтиляторов с ΠΈΠ·ΠΎΠ³Π½ΡƒΡ‚Ρ‹ΠΌΠΈ ΠΏΠ΅Ρ€Π΅Π΄Π½ΠΈΠΌΠΈ ΠΈ Π·Π°Π΄Π½ΠΈΠΌΠΈ лСзвиями. ВСнтиляторы Π² основном Ρ€Π°Π·Π΄Π΅Π»Π΅Π½Ρ‹ Π² зависимости ΠΎΡ‚ направлСния ΠΏΠΎΡ‚ΠΎΠΊΠ° Π²ΠΎΠ·Π΄ΡƒΡˆΠ½ΠΎΠ³ΠΎ ΠΏΠΎΡ‚ΠΎΠΊΠ° ΠΎΡ‚Π½ΠΎΡΠΈΡ‚Π΅Π»ΡŒΠ½ΠΎ оси вСнтилятора, выдСляя Π² этом ΠΎΡ‚Π½ΠΎΡˆΠ΅Π½ΠΈΠΈ Ρ‚Ρ€ΠΈ Π³Ρ€ΡƒΠΏΠΏΡ‹: осСвыС, Ρ€Π°Π΄ΠΈΠ°Π»ΡŒΠ½Ρ‹Π΅ ΠΈ Π΄ΠΈΠ°Π³ΠΎΠ½Π°Π»ΡŒΠ½Ρ‹Π΅. ΠŸΡ€ΠΈ ΠΎΡ…Π»Π°ΠΆΠ΄Π΅Π½ΠΈΠΈ элСктронных устройств ΠΈΡΠΏΠΎΠ»ΡŒΠ·ΡƒΡŽΡ‚ΡΡ вСнтиляторы ΠΏΠ΅Ρ€Π²ΠΎΠ³ΠΎ Ρ‚ΠΈΠΏΠ°, Π² ΠΊΠΎΡ‚ΠΎΡ€Ρ‹Ρ… ΠΏΠΎΡ‚ΠΎΠΊ Π²ΠΎΠ·Π΄ΡƒΡ…Π° ΠΏΡ€ΠΎΡ‚Π΅ΠΊΠ°Π΅Ρ‚ Π² осСвом Π½Π°ΠΏΡ€Π°Π²Π»Π΅Π½ΠΈΠΈ.

Π’ примСнСниях этого Ρ‚ΠΈΠΏΠ° вСнтилятор, характСрная ΠΎΡΠΎΠ±Π΅Π½Π½ΠΎΡΡ‚ΡŒ ΠΊΠΎΡ‚ΠΎΡ€ΠΎΠ³ΠΎ прСдставляСт собой лопасти Ρ€ΠΎΡ‚ΠΎΡ€Π° Π² Π²ΠΈΠ΄Π΅ ΠΏΡ€ΠΎΠΏΠ΅Π»Π»Π΅Ρ€Π°, Ρ‡Π°Ρ‰Π΅ всСго размСщаСтся Π² корпусС вмСстС с элСктродвигатСлСм, встроСнным Π² ступицу Ρ€ΠΎΡ‚ΠΎΡ€Π°. Вакая компактная конструкция с ΠΌΠΎΠ½Ρ‚Π°ΠΆΠ½Ρ‹ΠΌΠΈ отвСрстиями Π½Π° Ρ„Π»Π°Π½Ρ†Π΅ вСнтилятора позволяСт ΡƒΡΡ‚Π°Π½Π°Π²Π»ΠΈΠ²Π°Ρ‚ΡŒ Π΄Π°ΠΆΠ΅ Π² ΠΎΠ³Ρ€Π°Π½ΠΈΡ‡Π΅Π½Π½Ρ‹Ρ… пространствСнных условиях. Однако ΠΏΡ€ΠΎΠ±Π»Π΅ΠΌΠ° с ΠΊΠΎΠΌΠΏΠ°ΠΊΡ‚Π½Ρ‹ΠΌ Π΄ΠΈΠ·Π°ΠΉΠ½ΠΎΠΌ — это распорки, ΡΠΎΠ΅Π΄ΠΈΠ½ΡΡŽΡ‰ΠΈΠ΅ корпус двигатСля с Ρ„Π»Π°Π½Ρ†Π΅ΠΌ.

Из схСмы Π²ΠΈΠ΄Π½ΠΎ, Ρ‡Ρ‚ΠΎ ΠΏΠ°Ρ€Π°Π»Π»Π΅Π»ΡŒΠ½ΠΎ ΠΏΠΎΠ΄ΠΊΠ»ΡŽΡ‡Π΅Π½Ρ‹ Ρ‚ΠΎΠ»ΡŒΠΊΠΎ R2 ΠΈ R4. R3 Π½Π΅ ΠΏΠ°Ρ€Π°Π»Π»Π΅Π»ΡŒΠ½ΠΎ, Ρ‚.ΠΊ. ΠΎΠ΄Π½ΠΈΠΌ ΠΊΠΎΠ½Ρ†ΠΎΠΌ ΠΎΠ½ΠΎ ΠΏΠΎΠ΄ΠΊΠ»ΡŽΡ‡Π΅Π½ΠΎ ΠΊ E1. R1 — ΠΎΠ΄Π½ΠΈΠΌ ΠΊΠΎΠ½Ρ†ΠΎΠΌ ΠΏΠΎΠ΄ΠΊΠ»ΡŽΡ‡Π΅Π½ΠΎ ΠΊ R5, Π° Π½Π΅ ΠΊ ΡƒΠ·Π»Ρƒ. R5 — ΠΎΠ΄Π½ΠΈΠΌ ΠΊΠΎΠ½Ρ†ΠΎΠΌ ΠΏΠΎΠ΄ΠΊΠ»ΡŽΡ‡Π΅Π½ΠΎ ΠΊ R1, Π° Π½Π΅ ΠΊ ΡƒΠ·Π»Ρƒ. МоТно Ρ‚Π°ΠΊ ΠΆΠ΅ Π³ΠΎΠ²ΠΎΡ€ΠΈΡ‚ΡŒ, Ρ‡Ρ‚ΠΎ ΠΏΠΎΡΠ»Π΅Π΄ΠΎΠ²Π°Ρ‚Π΅Π»ΡŒΠ½ΠΎΠ΅ соСдинСниС сопротивлСний R1 ΠΈ R5 ΠΏΠΎΠ΄ΠΊΠ»ΡŽΡ‡Π΅Π½ΠΎ ΠΏΠ°Ρ€Π°Π»Π»Π΅Π»ΡŒΠ½ΠΎ с R2 ΠΈ R4.

Они ΠΎΠ±Π΅ΡΠΏΠ΅Ρ‡ΠΈΠ²Π°ΡŽΡ‚ ΡΡ‚Π°Π±ΠΈΠ»ΡŒΠ½ΠΎΡΡ‚ΡŒ, Π½ΠΎ Π² Ρ‚ΠΎ ΠΆΠ΅ врСмя ΠΏΡ€Π΅ΠΏΡΡ‚ΡΡ‚Π²ΡƒΡŽΡ‚ ΠΏΠΎΡ‚ΠΎΠΊΡƒ Π²ΠΎΠ·Π΄ΡƒΡ…Π°. Π’ΠΎ врСмя Ρ€Π°Π±ΠΎΡ‚Ρ‹ лопасти Ρ€ΠΎΡ‚ΠΎΡ€Π° ΠΏΠΎΠ΄ΠΌΠ΅Ρ‚Π°ΡŽΡ‚ стойки, вызывая скачки давлСния Π² ΠΈΡ… окрСстностях. Π­Ρ‚ΠΎ ΠΎΠ΄ΠΈΠ½ ΠΈΠ· основных источников ΡˆΡƒΠΌΠ°, создаваСмых осСвыми вСнтиляторами. ΠŸΠΎΡΡ‚ΠΎΠΌΡƒ Π½Π° этапС проСктирования этого устройства Ρ„ΠΎΡ€ΠΌΡƒ ΠΈ количСство распорок Ρ‚Ρ‰Π°Ρ‚Π΅Π»ΡŒΠ½ΠΎ Π°Π½Π°Π»ΠΈΠ·ΠΈΡ€ΡƒΡŽΡ‚ Π½Π° ΠΏΡ€Π΅Π΄ΠΌΠ΅Ρ‚ ΠΈΡ… влияния Π½Π° ΠΏΠΎΡ‚ΠΎΠΊ Π²ΠΎΠ·Π΄ΡƒΡ…Π° Ρ‡Π΅Ρ€Π΅Π· вСнтилятор.

Устройство Π² ΠΎΠ΄Π½ΠΎΠΌ корпусС вСнтилятора. НаиболСС распространСнными ΡΠ²Π»ΡΡŽΡ‚ΡΡ осСвыС вСнтиляторы со ΡΠΏΠΈΡ€Π°Π»ΡŒΠ½ΠΎ ΠΈΠ·ΠΎΠ³Π½ΡƒΡ‚Ρ‹ΠΌΠΈ распорками. На ΠΏΡ€Π°ΠΊΡ‚ΠΈΠΊΠ΅ ΡΠΎΠ±Π»ΡŽΠ΄Π°ΡŽΡ‚ΡΡ Π½Π΅ΠΊΠΎΡ‚ΠΎΡ€Ρ‹Π΅ основныС ΠΏΡ€Π°Π²ΠΈΠ»Π° ΠΎΡ‚Π½ΠΎΡΠΈΡ‚Π΅Π»ΡŒΠ½ΠΎ ΠΈΡ… установки. Один ΠΈΠ· Π½ΠΈΡ… касаСтся Ρ€Π°Π·Ρ€Ρ‹Π²Π° ΠΌΠ΅ΠΆΠ΄Ρƒ лСзвиями ΠΈ стойками — Ρ‡Π΅ΠΌ ΠΎΠ½ΠΈ ΠΊΡ€ΡƒΠΏΠ½Π΅Π΅, Ρ‚Π΅ΠΌ мСньшС ΡˆΡƒΠΌΠ°. Π—Π½Π°Ρ‡ΠΈΡ‚Π΅Π»ΡŒΠ½ΠΎ большоС количСство Π»Π΅Π·Π²ΠΈΠΉ ΠΈ ΠΏΡ€ΠΎΠΊΠ»Π°Π΄ΠΎΠΊ. Π§Π΅ΠΌ мСньшС стойки, Ρ‚Π΅ΠΌ мСньшС ΡˆΡƒΠΌΠ°, особСнно ΠΏΡ€ΠΈ высоких скоростях вращСния Ρ€ΠΎΡ‚ΠΎΡ€Π°.

Π’ΠΎΠΊ ΠΏΡ€ΠΈ ΠΏΠ°Ρ€Π°Π»Π»Π΅Π»ΡŒΠ½ΠΎΠΌ соСдинСнии

ΠŸΡ€ΠΈ ΠΏΠ°Ρ€Π°Π»Π»Π΅Π»ΡŒΠ½ΠΎΠΌ соСдинСнии сопротивлСний Ρ‚ΠΎΠΊ Ρ‡Π΅Ρ€Π΅Π· ΠΊΠ°ΠΆΠ΄ΠΎΠ΅ сопротивлСниС Π² ΠΎΠ±Ρ‰Π΅ΠΌ случаС Ρ€Π°Π·Π½Ρ‹ΠΉ. Π’Π΅Π»ΠΈΡ‡ΠΈΠ½Π° Ρ‚ΠΎΠΊΠ° ΠΎΠ±Ρ€Π°Ρ‚Π½ΠΎ ΠΏΡ€ΠΎΠΏΠΎΡ€Ρ†ΠΈΠΎΠ½Π°Π»ΡŒΠ½Π° Π²Π΅Π»ΠΈΡ‡ΠΈΠ½Π΅ сопротивлСния.

НапряТСниС ΠΏΡ€ΠΈ ΠΏΠ°Ρ€Π°Π»Π»Π΅Π»ΡŒΠ½ΠΎΠΌ соСдинСнии

ΠŸΡ€ΠΈ ΠΏΠ°Ρ€Π°Π»Π»Π΅Π»ΡŒΠ½ΠΎΠΌ соСдинСнии Ρ€Π°Π·Π½ΠΎΡΡ‚ΡŒ ΠΏΠΎΡ‚Π΅Π½Ρ†ΠΈΠ°Π»ΠΎΠ² ΠΌΠ΅ΠΆΠ΄Ρƒ ΡƒΠ·Π»Π°ΠΌΠΈ, ΠΎΠ±ΡŠΠ΅Π΄ΠΈΠ½ΡΡŽΡ‰ΠΈΠΌΠΈ элСмСнты Ρ†Π΅ΠΏΠΈ, ΠΎΠ΄ΠΈΠ½Π°ΠΊΠΎΠ²Π° для всСх элСмСнтов.

Π’ свою ΠΎΡ‡Π΅Ρ€Π΅Π΄ΡŒ, Ρ‡Π΅ΠΌ большС Π»Π΅Π·Π²ΠΈΠΉ, Ρ‚Π΅ΠΌ большС ΠΏΠΎΡ‚ΠΎΠΊ Π²ΠΎΠ·Π΄ΡƒΡ…Π°, Π½ΠΎ Π² Ρ‚ΠΎ ΠΆΠ΅ врСмя лСзвия Ρ‡Π°Ρ‰Π΅ ΡΠΌΠ΅Ρ‚Π°ΡŽΡ‚ стойки, Ρ‡Ρ‚ΠΎ создаСт большС ΡˆΡƒΠΌΠ°. Π’Π°ΠΊΠΆΠ΅ Π²Π°ΠΆΠ½ΠΎ, Ρ‡Ρ‚ΠΎΠ±Ρ‹ стойки Ρ€Π°ΡΠΏΠΎΠ»Π°Π³Π°Π»ΠΈΡΡŒ Π½Π° Π²Ρ‹Ρ…ΠΎΠ΄Π½ΠΎΠΉ сторонС Π²ΠΎΠ·Π΄ΡƒΡ…Π° ΠΎΡ‚ вСнтилятора, Ρ‡Ρ‚ΠΎ ΠΎΠΏΡ€Π°Π²Π΄Π°Π½ΠΎ характСристиками ΠΏΠΎΡ‚ΠΎΠΊΠ°. Π’ΠΎΠ·Π΄ΡƒΡˆΠ½Ρ‹ΠΉ ΠΏΠΎΡ‚ΠΎΠΊ Ρƒ Π²Ρ…ΠΎΠ΄Π° вСнтилятора являСтся Π»Π°ΠΌΠΈΠ½Π°Ρ€Π½Ρ‹ΠΌ ΠΏΠΎΡ‚ΠΎΠΊΠΎΠΌ, Π° Π½Π° Π²Ρ‹Ρ…ΠΎΠ΄Π΅ вСнтилятора ΠΏΠΎΡ‚ΠΎΠΊ измСняСт свой Ρ…Π°Ρ€Π°ΠΊΡ‚Π΅Ρ€ Π½Π° Ρ‚ΡƒΡ€Π±ΡƒΠ»Π΅Π½Ρ‚Π½Ρ‹ΠΉ.

Когда Π½Π° ΠΏΡƒΡ‚ΠΈ Π»Π°ΠΌΠΈΠ½Π°Ρ€Π½ΠΎΠ³ΠΎ ΠΏΠΎΡ‚ΠΎΠΊΠ° Π²ΠΎΠ·Π½ΠΈΠΊΠ°Π΅Ρ‚ прСпятствиС, ΡΠ»Ρ‹ΡˆΠΈΡ‚ΡΡ опрСдСлСнная частота, ΡΠ»Ρ‹ΡˆΠΈΡ‚ΡΡ ΠΊΠ°ΠΊ свисток. Π‘ Π΄Ρ€ΡƒΠ³ΠΎΠΉ стороны, ΠΊΠΎΠ³Π΄Π° Ρ‚ΡƒΡ€Π±ΡƒΠ»Π΅Π½Ρ‚Π½Ρ‹ΠΉ ΠΏΠΎΡ‚ΠΎΠΊ сталкиваСтся с прСпятствиСм, ΠΏΠΎΡΠ²Π»ΡΡŽΡ‚ΡΡ Π·Π²ΡƒΠΊΠΈ Ρ€Π°Π·Π½Ρ‹Ρ… частот, Ρ‡Ρ‚ΠΎ Π΄Π΅Π»Π°Π΅Ρ‚ ΡˆΡƒΠΌ ΠΌΠ΅Π½Π΅Π΅ Ρ€Π°Π·Π΄Ρ€Π°ΠΆΠ°ΡŽΡ‰ΠΈΠΌ для людСй.

ΠŸΡ€ΠΈΠΌΠ΅Π½Π΅Π½ΠΈΠ΅ ΠΏΠ°Ρ€Π°Π»Π»Π΅Π»ΡŒΠ½ΠΎΠ³ΠΎ соСдинСния

1. Π’ ΠΏΡ€ΠΎΠΌΡ‹ΡˆΠ»Π΅Π½Π½ΠΎΡΡ‚ΠΈ ΠΈΠ·Π³ΠΎΡ‚Π°Π²Π»ΠΈΠ²Π°ΡŽΡ‚ΡΡ сопротивлСния ΠΎΠΏΡ€Π΅Π΄Π΅Π»Π΅Π½Π½Ρ‹Ρ… Π²Π΅Π»ΠΈΡ‡ΠΈΠ½. Иногда Π½Π΅ΠΎΠ±Ρ…ΠΎΠ΄ΠΈΠΌΠΎ ΠΏΠΎΠ»ΡƒΡ‡ΠΈΡ‚ΡŒ Π·Π½Π°Ρ‡Π΅Π½ΠΈΠ΅ сопротивлСния Π²Π½Π΅ Π΄Π°Π½Π½Ρ‹Ρ… рядов. Для этого ΠΌΠΎΠΆΠ½ΠΎ ΠΏΠΎΠ΄ΠΊΠ»ΡŽΡ‡ΠΈΡ‚ΡŒ нСсколько сопротивлСний ΠΏΠ°Ρ€Π°Π»Π»Π΅Π»ΡŒΠ½ΠΎ. Π­ΠΊΠ²ΠΈΠ²Π°Π»Π΅Π½Ρ‚Π½ΠΎΠ΅ сопротивлСниС всСгда Π±ΡƒΠ΄Π΅Ρ‚ мСньшС самого большого Π½ΠΎΠΌΠΈΠ½Π°Π»Π° сопротивлСния.

Π¦Π΅Π½Ρ‚Ρ€ΠΎΠ±Π΅ΠΆΠ½Ρ‹Π΅ ΠΈ Π΄ΠΈΠ°Π³ΠΎΠ½Π°Π»ΡŒΠ½Ρ‹Π΅ вСнтиляторы

Π’ Π΄ΠΎΠΏΠΎΠ»Π½Π΅Π½ΠΈΠ΅ ΠΊ осСвым вСнтиляторам ΠΏΡ€ΠΈ ΠΎΡ…Π»Π°ΠΆΠ΄Π΅Π½ΠΈΠΈ элСктронных ΠΊΠΎΠΌΠΏΠΎΠ½Π΅Π½Ρ‚ΠΎΠ² ΠΈΡΠΏΠΎΠ»ΡŒΠ·ΡƒΡŽΡ‚ΡΡ, хотя ΠΈ Ρ€Π΅ΠΆΠ΅, Ρ€Π°Π΄ΠΈΠ°Π»ΡŒΠ½Ρ‹Π΅ вСнтиляторы. Π’ этом Ρ‚ΠΈΠΏΠ΅ устройства Π²Ρ…ΠΎΠ΄ Π²ΠΎΠ·Π΄ΡƒΡ…Π° находится Π² осСвом Π½Π°ΠΏΡ€Π°Π²Π»Π΅Π½ΠΈΠΈ, Π° Π²Ρ‹Ρ…ΠΎΠ΄ Π² Π½Π°ΠΏΡ€Π°Π²Π»Π΅Π½ΠΈΠΈ, пСрпСндикулярном оси вСнтилятора. Π˜Ρ… Ρ…Π°Ρ€Π°ΠΊΡ‚Π΅Ρ€Π½ΠΎΠΉ ΠΎΡΠΎΠ±Π΅Π½Π½ΠΎΡΡ‚ΡŒΡŽ являСтся Ρ€ΠΎΡ‚ΠΎΡ€, Π½Π°ΠΏΠΎΠΌΠΈΠ½Π°ΡŽΡ‰ΠΈΠΉ водяноС колСсо. НаиболСС часто встрСчаСтся структура, Π² ΠΊΠΎΡ‚ΠΎΡ€ΠΎΠΉ Π΄Π²ΠΈΠ³Π°Ρ‚Π΅Π»ΡŒ находится Π² области всасывания вСнтилятора. Π’ ΠΎΠΏΡ€Π΅Π΄Π΅Π»Π΅Π½Π½ΠΎΠΉ стСпСни ΠΎΠ½ ΠΎΠ³Ρ€Π°Π½ΠΈΡ‡ΠΈΠ²Π°Π΅Ρ‚ ΠΏΠΎΡ‚ΠΎΠΊ Π²ΠΎΠ·Π΄ΡƒΡ…Π°, Π½ΠΎ Π² Ρ‚ΠΎ ΠΆΠ΅ врСмя ΠΎΠ½ допускаСт ΠΊΠΎΠΌΠΏΠ°ΠΊΡ‚Π½Ρ‹ΠΉ Π΄ΠΈΠ·Π°ΠΉΠ½.

2. Π”Π΅Π»ΠΈΡ‚Π΅Π»ΡŒ Ρ‚ΠΎΠΊΠΎΠ².

ΠŸΠ°Ρ€Π°Π»Π»Π΅Π»ΡŒΠ½ΠΎΠ΅ соСдинСниС рСзисторов β€” ΠΎΠ΄Π½ΠΎ ΠΈΠ· Π΄Π²ΡƒΡ… Π²ΠΈΠ΄ΠΎΠ² элСктричСских соСдинСний, ΠΊΠΎΠ³Π΄Π° ΠΎΠ±Π° Π²Ρ‹Π²ΠΎΠ΄Π° ΠΎΠ΄Π½ΠΎΠ³ΠΎ рСзистора соСдинСны с ΡΠΎΠΎΡ‚Π²Π΅Ρ‚ΡΡ‚Π²ΡƒΡŽΡ‰ΠΈΠΌΠΈ Π²Ρ‹Π²ΠΎΠ΄Π°ΠΌΠΈ Π΄Ρ€ΡƒΠ³ΠΎΠ³ΠΎ рСзистора ΠΈΠ»ΠΈ рСзисторов. Π—Π°Ρ‡Π°ΡΡ‚ΡƒΡŽ ΠΈΠ»ΠΈ ΠΏΠ°Ρ€Π°Π»Π»Π΅Π»ΡŒΠ½ΠΎ для Ρ‚ΠΎΠ³ΠΎ, Ρ‡Ρ‚ΠΎΠ±Ρ‹ ΡΠΎΠ·Π΄Π°Ρ‚ΡŒ Π±ΠΎΠ»Π΅Π΅ слоТныС элСктронныС схСмы.

Π’Ρ€Π΅Ρ‚ΠΈΠΉ Ρ‚ΠΈΠΏ — вСнтиляторы с ΡΠΌΠ΅ΡˆΠ°Π½Π½Ρ‹ΠΌ ΠΏΠΎΡ‚ΠΎΠΊΠΎΠΌ. диагональ, которая, хотя с Ρ‚ΠΎΡ‡ΠΊΠΈ зрСния внСшнСго Π²ΠΈΠ΄Π°, Π±ΠΎΠ»Π΅Π΅ ΠΏΠΎΡ…ΠΎΠΆΠ° Π½Π° осСвыС вСнтиляторы, благодаря ΠΏΡ€ΠΈΠ½Ρ†ΠΈΠΏΡƒ Ρ€Π°Π±ΠΎΡ‚Ρ‹ прСдставляСт собой ΠΏΡ€ΠΎΠΌΠ΅ΠΆΡƒΡ‚ΠΎΡ‡Π½ΡƒΡŽ ΠΊΠΎΠ½ΡΡ‚Ρ€ΡƒΠΊΡ†ΠΈΡŽ ΠΌΠ΅ΠΆΠ΄Ρƒ осСвыми ΠΈ Ρ€Π°Π΄ΠΈΠ°Π»ΡŒΠ½Ρ‹ΠΌΠΈ вСнтиляторами: Π²Ρ…ΠΎΠ΄Π½ΠΎΠ΅ отвСрстиС для Π²ΠΎΠ·Π΄ΡƒΡ…Π° находится Π² осСвом Π½Π°ΠΏΡ€Π°Π²Π»Π΅Π½ΠΈΠΈ Π½Π° Π²Ρ‹Ρ…ΠΎΠ΄Π΅, Π½ΠΎ Π²Ρ‹Ρ…ΠΎΠ΄ ΠΏΠΎ Π΄ΠΈΠ°Π³ΠΎΠ½Π°Π»ΠΈ. Π’ Ρ€Π΅Π·ΡƒΠ»ΡŒΡ‚Π°Ρ‚Π΅ вСнтиляторы смСшанного ΠΏΠΎΡ‚ΠΎΠΊΠ° ΡΠΎΠ·Π΄Π°ΡŽΡ‚ Π±ΠΎΠ»Π΅Π΅ высокоС Π΄Π°Π²Π»Π΅Π½ΠΈΠ΅ ΠΏΡ€ΠΈ Ρ‚Π΅Ρ… ΠΆΠ΅ Ρ€Π°Π·ΠΌΠ΅Ρ€Π°Ρ… ΠΈ Ρ€Π°Π±ΠΎΡ‡ΠΈΡ… скоростях, Ρ‡Ρ‚ΠΎ ΠΈ осСвыС вСнтиляторы, Π° ΠΏΠΎΠ΄ этим ΡƒΠ³Π»ΠΎΠΌ ΠΎΠ±Ρ€Π°Π·ΡƒΡŽΡ‚ΡΡ Ρ€Π°Π΄ΠΈΠ°Π»ΡŒΠ½Ρ‹Π΅ вСнтиляторы.

Π‘Ρ…Π΅ΠΌΠ° Π·Π°ΠΌΠ΅Π½Ρ‹ устройства Π½Π° рис. РСзисторы ΡΠΎΠΎΡ‚Π²Π΅Ρ‚ΡΡ‚Π²ΡƒΡŽΡ‚ ΡΠΎΠΏΡ€ΠΎΡ‚ΠΈΠ²Π»Π΅Π½ΠΈΡŽ ΠΏΠΎΡ‚ΠΎΠΊΠ° ΠΎΡ‚Π΄Π΅Π»ΡŒΠ½Ρ‹Ρ… ΠΊΠΎΠΌΠΏΠΎΠ½Π΅Π½Ρ‚ΠΎΠ² устройства. ΠŸΡ€ΠΎΠΈΠ·Π²ΠΎΠ΄ΠΈΡ‚Π΅Π»ΡŒΠ½ΠΎΡΡ‚ΡŒ вСнтилятора описываСт Π΅Π³ΠΎ Ρ…Π°Ρ€Π°ΠΊΡ‚Π΅Ρ€ΠΈΡΡ‚ΠΈΡ‡Π΅ΡΠΊΡƒΡŽ ΠΊΡ€ΠΈΠ²ΡƒΡŽ, которая прСдставляСт ΠΎΠ±ΡŠΠ΅ΠΌΠ½Ρ‹ΠΉ расход ΠΏΠ΅Ρ€Π΅ΠΊΠ°Ρ‡ΠΈΠ²Π°Π΅ΠΌΠΎΠ³ΠΎ Π³Π°Π·Π°, Π² зависимости ΠΎΡ‚ увСличСния давлСния, создаваСмого вСнтилятором. ΠžΡΠ½ΠΎΠ²Ρ‹Π²Π°ΡΡΡŒ Π½Π° этой характСристикС, ΠΌΠΎΠΆΠ½ΠΎ ΡΡ€Π°Π²Π½ΠΈΠ²Π°Ρ‚ΡŒ Ρ€Π°Π·Π½Ρ‹Π΅ Ρ‚ΠΈΠΏΡ‹ вСнтиляторов, Π½ΠΎ Π½Π° ΠΏΡ€Π°ΠΊΡ‚ΠΈΠΊΠ΅ Π΅Π΅ Ρ‡Π°Ρ‰Π΅ всСго ΠΈΡΠΏΠΎΠ»ΡŒΠ·ΡƒΡŽΡ‚ для ΠΎΡ†Π΅Π½ΠΊΠΈ ΠΏΡ€ΠΎΠΈΠ·Π²ΠΎΠ΄ΠΈΡ‚Π΅Π»ΡŒΠ½ΠΎΡΡ‚ΠΈ вСнтилятора Π² соотвСтствии с трСбованиями ΠΊΠΎΠ½ΠΊΡ€Π΅Ρ‚Π½ΠΎΠ³ΠΎ прилоТСния.

Π‘Ρ…Π΅ΠΌΠ° ΠΏΠ°Ρ€Π°Π»Π»Π΅Π»ΡŒΠ½ΠΎΠ³ΠΎ соСдинСния ΠΏΠΎΠΊΠ°Π·Π°Π½ Π½Π° рисункС Π½ΠΈΠΆΠ΅. ΠŸΡ€ΠΈ ΠΏΠ°Ρ€Π°Π»Π»Π΅Π»ΡŒΠ½ΠΎΠΌ соСдинСнии рСзисторов, напряТСниС Π½Π° всСх рСзисторах Π±ΡƒΠ΄Π΅Ρ‚ ΠΎΠ΄ΠΈΠ½Π°ΠΊΠΎΠ²Ρ‹ΠΌ, Π° ΠΏΡ€ΠΎΡ‚Π΅ΠΊΠ°ΡŽΡ‰ΠΈΠΉ Ρ‡Π΅Ρ€Π΅Π· Π½ΠΈΡ… Ρ‚ΠΎΠΊ Π±ΡƒΠ΄Π΅Ρ‚ ΠΏΡ€ΠΎΠΏΠΎΡ€Ρ†ΠΈΠΎΠ½Π°Π»Π΅Π½ ΠΈΡ… ΡΠΎΠΏΡ€ΠΎΡ‚ΠΈΠ²Π»Π΅Π½ΠΈΡŽ:

Π€ΠΎΡ€ΠΌΡƒΠ»Π° ΠΏΠ°Ρ€Π°Π»Π»Π΅Π»ΡŒΠ½ΠΎΠ³ΠΎ соСдинСния рСзисторов

ΠžΠ±Ρ‰Π΅Π΅ сопротивлСниС Π½Π΅ΡΠΊΠΎΠ»ΡŒΠΊΠΈΡ… рСзисторов соСдинСнных ΠΏΠ°Ρ€Π°Π»Π»Π΅Π»ΡŒΠ½ΠΎ опрСдСляСтся ΠΏΠΎ ΡΠ»Π΅Π΄ΡƒΡŽΡ‰Π΅ΠΉ Ρ„ΠΎΡ€ΠΌΡƒΠ»Π΅:

Π’ этом случаС характСристики вСнтилятора Π°Π½Π°Π»ΠΈΠ·ΠΈΡ€ΡƒΡŽΡ‚ΡΡ вмСстС с ΠΊΡ€ΠΈΠ²ΠΎΠΉ, которая ΠΏΠΎΠΊΠ°Π·Ρ‹Π²Π°Π΅Ρ‚ ΠΏΠ°Π΄Π΅Π½ΠΈΠ΅ давлСния Π² ΠΎΡ…Π»Π°ΠΆΠ΄Π°Π΅ΠΌΠΎΠΌ устройствС Π² зависимости ΠΎΡ‚ количСства проходящСго Ρ‡Π΅Ρ€Π΅Π· Π½Π΅Π³ΠΎ Π²ΠΎΠ·Π΄ΡƒΡ…Π°. Π§Ρ‚ΠΎΠ±Ρ‹ ΠΎΠΏΡ€Π΅Π΄Π΅Π»ΠΈΡ‚ΡŒ характСристики устройства, сначала ΠΎΠΏΡ€Π΅Π΄Π΅Π»ΠΈΡ‚Π΅ Ρ‚Π΅ΠΊΡƒΡ‰Π΅Π΅ сопротивлСниС ΠΏΠΎΡ‚ΠΎΠΊΠ°. Π’ΠΎΡ‡Π½Ρ‹Π΅ расчСты сопротивлСния ΠΏΠΎΡ‚ΠΎΠΊΡƒ ΠΎΠ±Ρ‹Ρ‡Π½ΠΎ ΠΈΡΠΏΠΎΠ»ΡŒΠ·ΡƒΡŽΡ‚ спСциализированноС ΠΏΡ€ΠΎΠ³Ρ€Π°ΠΌΠΌΠ½ΠΎΠ΅ обСспСчСниС, ΠΊΠΎΡ‚ΠΎΡ€ΠΎΠ΅ ΠΈΡΠΏΠΎΠ»ΡŒΠ·ΡƒΠ΅Ρ‚ слоТныС ΠΌΠΎΠ΄Π΅Π»ΠΈ ΠΈ числСнныС ΠΌΠ΅Ρ‚ΠΎΠ΄Ρ‹.

Однако Π½Π° ΠΏΡ€Π°ΠΊΡ‚ΠΈΠΊΠ΅ Ρ‚Π°ΠΊΠΆΠ΅ ΠΈΡΠΏΠΎΠ»ΡŒΠ·ΡƒΡŽΡ‚ΡΡ ΡƒΠΏΡ€ΠΎΡ‰Π΅Π½Π½Ρ‹Π΅ ΠΌΠ΅Ρ‚ΠΎΠ΄Ρ‹. Один ΠΈΠ· Π½ΠΈΡ… Π±ΡƒΠ΄Π΅Ρ‚ рассмотрСн Π½Π° ΠΏΡ€ΠΈΠΌΠ΅Ρ€Π΅ устройства, Π² ΠΊΠΎΡ‚ΠΎΡ€ΠΎΠΌ монтируСтся ΠΎΠ΄ΠΈΠ½ вСнтилятор ΠΈ вСнтиляционная Ρ€Π΅ΡˆΠ΅Ρ‚ΠΊΠ°. Он состоит ΠΈΠ· пяти ΠΏΠ°Ρ€Π°Π»Π»Π΅Π»ΡŒΠ½Ρ‹Ρ… ΠΌΠΎΠ½Ρ‚Π°ΠΆΠ½Ρ‹Ρ… ΠΏΠ»Π°Ρ‚ с ΠΊΠΎΠΌΠΏΠΎΠ½Π΅Π½Ρ‚Π°ΠΌΠΈ ΠΈ Π±Π»ΠΎΠΊΠΎΠΌ питания, Π·Π°ΠΊΠ»ΡŽΡ‡Π΅Π½Π½Ρ‹Ρ… Π² ΠΎΡ‚Π΄Π΅Π»ΡŒΠ½Ρ‹ΠΉ корпус с отвСрстиями Π½Π° ΠΏΠ΅Ρ€Π΅Π΄Π½Π΅ΠΉ ΠΈ Π·Π°Π΄Π½Π΅ΠΉ панСлях. На рисункС 3 ΠΏΠΎΠΊΠ°Π·Π°Π½Π° схСма Π·Π°ΠΌΠ΅Π½Ρ‹ рассматриваСмого устройства. ΠžΡ‚Π΄Π΅Π»ΡŒΠ½Ρ‹Π΅ ΠΊΠΎΠΌΠΏΠΎΠ½Π΅Π½Ρ‚Ρ‹ ΠΈ ΠΊΠΎΠΌΠΏΠΎΠ½Π΅Π½Ρ‚Ρ‹ корпуса ΠΏΠΎΠΊΠ°Π·Π°Π½Ρ‹ Π½Π° Π½Π΅ΠΌ Π² Π²ΠΈΠ΄Π΅ рСзисторов с ΠΎΠΏΡ€Π΅Π΄Π΅Π»Π΅Π½Π½Ρ‹ΠΌ сопротивлСниСм ΠΏΠΎΡ‚ΠΎΠΊΡƒ.

Π’ΠΎΠΊ, ΠΏΡ€ΠΎΡ‚Π΅ΠΊΠ°ΡŽΡ‰ΠΈΠΉ Ρ‡Π΅Ρ€Π΅Π· ΠΎΡ‚Π΄Π΅Π»ΡŒΠ½ΠΎ взятый рСзистор, согласно , ΠΌΠΎΠΆΠ½ΠΎ Π½Π°ΠΉΡ‚ΠΈ ΠΏΠΎ Ρ„ΠΎΡ€ΠΌΡƒΠ»Π΅:

ΠŸΠ°Ρ€Π°Π»Π»Π΅Π»ΡŒΠ½ΠΎΠ΅ соСдинСниС рСзисторов β€” расчСт

ΠŸΡ€ΠΈΠΌΠ΅Ρ€ β„–1

ΠŸΡ€ΠΈ Ρ€Π°Π·Ρ€Π°Π±ΠΎΡ‚ΠΊΠ΅ устройства, Π²ΠΎΠ·Π½ΠΈΠΊΠ»Π° Π½Π΅ΠΎΠ±Ρ…ΠΎΠ΄ΠΈΠΌΠΎΡΡ‚ΡŒ ΡƒΡΡ‚Π°Π½ΠΎΠ²ΠΈΡ‚ΡŒ рСзистор с сопротивлСниСм 8 Ом. Если ΠΌΡ‹ просмотрим вСсь Π½ΠΎΠΌΠΈΠ½Π°Π»ΡŒΠ½Ρ‹ΠΉ ряд стандартных Π·Π½Π°Ρ‡Π΅Π½ΠΈΠΉ рСзисторов, Ρ‚ΠΎ ΠΌΡ‹ ΡƒΠ²ΠΈΠ΄ΠΈΠΌ, Ρ‡Ρ‚ΠΎ рСзистора с сопротивлСниСм Π² 8 Ом Π² Π½Π΅ΠΌ Π½Π΅Ρ‚.

ΠžΠΏΡ€Π΅Π΄Π΅Π»Π΅Π½ΠΈΠ΅ характСристик устройства

Π₯арактСристики систСмы вСнтиляции: ΠΎΡ…Π»Π°ΠΆΠ΄Π΅Π½Π½ΠΎΠ΅ устройство — вСнтилятор. Π‘ΠΎΠΎΡ‚Π²Π΅Ρ‚ΡΡ‚Π²ΡƒΡŽΡ‰ΠΈΠ΅ Ρ„ΠΎΡ€ΠΌΡƒΠ»Ρ‹ ΠΈΡΠΏΠΎΠ»ΡŒΠ·ΡƒΡŽΡ‚ΡΡ для опрСдСлСния Π·Π½Π°Ρ‡Π΅Π½ΠΈΠΉ сопротивлСний, Π² ΠΊΠΎΡ‚ΠΎΡ€Ρ‹Ρ… прСдполагаСтся, Ρ‡Ρ‚ΠΎ сопротивлСниС зависит ΠΎΡ‚ ΠΏΠ»ΠΎΡ‰Π°Π΄ΠΈ ΠΏΠΎΠΏΠ΅Ρ€Π΅Ρ‡Π½ΠΎΠ³ΠΎ сСчСния прСпятствия, Π° Π² случаС вСнтиляционной Ρ€Π΅ΡˆΠ΅Ρ‚ΠΊΠΈ — ΠΎΡ‚ ΠΏΠ»ΠΎΡ‰Π°Π΄ΠΈ повСрхности, Ρ‡Π΅Ρ€Π΅Π· ΠΊΠΎΡ‚ΠΎΡ€ΡƒΡŽ ΠΏΡ€ΠΎΡ‚Π΅ΠΊΠ°Π΅Ρ‚ ΠΏΠΎΡ‚ΠΎΠΊ Π²ΠΎΠ·Π΄ΡƒΡ…Π°. Π‘Π»Π΅Π΄ΡƒΡŽΡ‰ΠΈΠΉ шаг — Ρ€Π°ΡΡΡ‡ΠΈΡ‚Π°Ρ‚ΡŒ ΠΎΠ±Ρ‰Π΅Π΅ сопротивлСниС всСй систСмы.

Для этой Ρ†Π΅Π»ΠΈ ΠΈΡΠΏΠΎΠ»ΡŒΠ·ΡƒΡŽΡ‚ΡΡ Π°Π½Π°Π»ΠΎΠ³ΠΈ, примСняСмыС ΠΏΡ€ΠΈ Π·Π°ΠΌΠ΅Π½Π΅ ΠΏΠΎΡΠ»Π΅Π΄ΠΎΠ²Π°Ρ‚Π΅Π»ΡŒΠ½ΠΎΠ³ΠΎ сопротивлСния ΠΈ ΠΏΠ°Ρ€Π°Π»Π»Π΅Π»ΡŒΠ½Ρ‹Ρ… элСктричСских сопротивлСний. Π—Π°Ρ‚Π΅ΠΌ эту характСристику слСдуСт Ρ€Π°Π·ΠΌΠ΅ΡΡ‚ΠΈΡ‚ΡŒ Π½Π° ΠΎΠ±Ρ‰Π΅ΠΌ Π³Ρ€Π°Ρ„ΠΈΠΊΠ΅ с характСристиками вСнтилятора. Π’ΠΎΠ³Π΄Π° Π² Ρ‚ΠΎΡ‡ΠΊΠ΅ пСрСсСчСния Π΄Π²ΡƒΡ… ΠΊΡ€ΠΈΠ²Ρ‹Ρ… опрСдСляСтся Ρ‚ΠΎΡ‡ΠΊΠ° дСйствия вСнтилятора. Π’ Π½Π΅ΠΌ описываСтся фактичСская ΠΏΡ€ΠΎΠΈΠ·Π²ΠΎΠ΄ΠΈΡ‚Π΅Π»ΡŒΠ½ΠΎΡΡ‚ΡŒ ΠΊΠΎΠ½ΠΊΡ€Π΅Ρ‚Π½ΠΎΠ³ΠΎ вСнтилятора Π² этой установкС, Ρ‚ΠΎ Π΅ΡΡ‚ΡŒ количСство Π²ΠΎΠ·Π΄ΡƒΡ…Π°, ΠΊΠΎΡ‚ΠΎΡ€ΠΎΠ΅ вСнтилятор ΠΌΠΎΠΆΠ΅Ρ‚ ΠΎΡ…Π»Π°ΠΆΠ΄Π°Ρ‚ΡŒ ΠΏΡ€ΠΈ ΠΎΡ…Π»Π°ΠΆΠ΄Π΅Π½ΠΈΠΈ устройства.

Π’Ρ‹Ρ…ΠΎΠ΄ΠΎΠΌ ΠΈΠ· Π΄Π°Π½Π½ΠΎΠΉ ситуации Π±ΡƒΠ΄Π΅Ρ‚ использованиС Π΄Π²ΡƒΡ… ΠΏΠ°Ρ€Π°Π»Π»Π΅Π»ΡŒΠ½ΠΎ соСдинСнных рСзисторов. Π­ΠΊΠ²ΠΈΠ²Π°Π»Π΅Π½Ρ‚Π½ΠΎΠ΅ Π·Π½Π°Ρ‡Π΅Π½ΠΈΠ΅ сопротивлСния для Π΄Π²ΡƒΡ… рСзисторов соСдинСнных ΠΏΠ°Ρ€Π°Π»Π»Π΅Π»ΡŒΠ½ΠΎ рассчитываСтся ΡΠ»Π΅Π΄ΡƒΡŽΡ‰ΠΈΠΌ ΠΎΠ±Ρ€Π°Π·ΠΎΠΌ:

Π”Π°Π½Π½ΠΎΠ΅ ΡƒΡ€Π°Π²Π½Π΅Π½ΠΈΠ΅ ΠΏΠΎΠΊΠ°Π·Ρ‹Π²Π°Π΅Ρ‚, Ρ‡Ρ‚ΠΎ Ссли R1 Ρ€Π°Π²Π΅Π½ R2, Ρ‚ΠΎ сопротивлСниС R составляСт ΠΏΠΎΠ»ΠΎΠ²ΠΈΠ½Ρƒ сопротивлСния ΠΎΠ΄Π½ΠΎΠ³ΠΎ ΠΈΠ· Π΄Π²ΡƒΡ… рСзисторов. ΠŸΡ€ΠΈ R = 8 Ом, R1 ΠΈ R2 Π΄ΠΎΠ»ΠΆΠ½Ρ‹, ΡΠ»Π΅Π΄ΠΎΠ²Π°Ρ‚Π΅Π»ΡŒΠ½ΠΎ, ΠΈΠΌΠ΅Ρ‚ΡŒ Π·Π½Π°Ρ‡Π΅Π½ΠΈΠ΅ 2 Γ— 8 = 16 Ом.
Π’Π΅ΠΏΠ΅Ρ€ΡŒ ΠΏΡ€ΠΎΠ²Π΅Π΄Π΅ΠΌ ΠΏΡ€ΠΎΠ²Π΅Ρ€ΠΊΡƒ, рассчитав ΠΎΠ±Ρ‰Π΅Π΅ сопротивлСниС Π΄Π²ΡƒΡ… рСзисторов:

Π’Π°ΠΊΠΈΠΌ ΠΎΠ±Ρ€Π°Π·ΠΎΠΌ, ΠΌΡ‹ ΠΏΠΎΠ»ΡƒΡ‡ΠΈΠ»ΠΈ Π½Π΅ΠΎΠ±Ρ…ΠΎΠ΄ΠΈΠΌΠΎΠ΅ сопротивлСниС 8 Ом, соСдинив ΠΏΠ°Ρ€Π°Π»Π»Π΅Π»ΡŒΠ½ΠΎ Π΄Π²Π° рСзистора ΠΏΠΎ 16 Ом.

ΠŸΡ€ΠΈΠΌΠ΅Ρ€ расчСта β„–2

Найти ΠΎΠ±Ρ‰Π΅Π΅ сопротивлСниС R ΠΈΠ· Ρ‚Ρ€Π΅Ρ… ΠΏΠ°Ρ€Π°Π»Π»Π΅Π»ΡŒΠ½ΠΎ соСдинСнных рСзисторов:

ΠžΠ±Ρ‰Π΅Π΅ сопротивлСниС R рассчитываСтся ΠΏΠΎ Ρ„ΠΎΡ€ΠΌΡƒΠ»Π΅:

Π­Ρ‚ΠΎΡ‚ ΠΌΠ΅Ρ‚ΠΎΠ΄ расчСта ΠΌΠΎΠΆΠ΅Ρ‚ Π±Ρ‹Ρ‚ΡŒ ΠΈΡΠΏΠΎΠ»ΡŒΠ·ΠΎΠ²Π°Π½Ρ‹ для расчСта любого количСства ΠΎΡ‚Π΄Π΅Π»ΡŒΠ½Ρ‹Ρ… сопротивлСний соСдинСнных ΠΏΠ°Ρ€Π°Π»Π»Π΅Π»ΡŒΠ½ΠΎ.

Один Π²Π°ΠΆΠ½Ρ‹ΠΉ ΠΌΠΎΠΌΠ΅Π½Ρ‚, ΠΊΠΎΡ‚ΠΎΡ€Ρ‹ΠΉ Π½Π΅ΠΎΠ±Ρ…ΠΎΠ΄ΠΈΠΌΠΎ Π·Π°ΠΏΠΎΠΌΠ½ΠΈΡ‚ΡŒ ΠΏΡ€ΠΈ расчСтС ΠΏΠ°Ρ€Π°Π»Π»Π΅Π»ΡŒΠ½ΠΎ соСдинСнных рСзисторов – это Ρ‚ΠΎ, Ρ‡Ρ‚ΠΎ ΠΎΠ±Ρ‰Π΅Π΅ сопротивлСниС всСгда Π±ΡƒΠ΄Π΅Ρ‚ мСньшС, Ρ‡Π΅ΠΌ Π·Π½Π°Ρ‡Π΅Π½ΠΈΠ΅ наимСньшСго сопротивлСния Π² этой ΠΊΠΎΠΌΠ±ΠΈΠ½Π°Ρ†ΠΈΠΈ.

Как Ρ€Π°ΡΡΡ‡ΠΈΡ‚Π°Ρ‚ΡŒ слоТныС схСмы соСдинСния рСзисторов

Π‘ΠΎΠ»Π΅Π΅ слоТныС соСдинСния рСзисторов ΠΌΠΎΠ³ΡƒΡ‚ Π±Ρ‹Ρ‚ΡŒ рассчитаны ΠΏΡƒΡ‚Π΅ΠΌ систСматичСской Π³Ρ€ΡƒΠΏΠΏΠΈΡ€ΠΎΠ²ΠΊΠΈ рСзисторов. На рисункС Π½ΠΈΠΆΠ΅ Π½Π΅ΠΎΠ±Ρ…ΠΎΠ΄ΠΈΠΌΠΎ ΠΏΠΎΡΡ‡ΠΈΡ‚Π°Ρ‚ΡŒ ΠΎΠ±Ρ‰Π΅Π΅ сопротивлСниС Ρ†Π΅ΠΏΠΈ, состоящСй ΠΈΠ· Ρ‚Ρ€Π΅Ρ… рСзисторов:



Для простоты расчСта, сначала сгруппируСм рСзисторы ΠΏΠΎ ΠΏΠ°Ρ€Π°Π»Π»Π΅Π»ΡŒΠ½ΠΎΠΌΡƒ ΠΈ ΠΏΠΎΡΠ»Π΅Π΄ΠΎΠ²Π°Ρ‚Π΅Π»ΡŒΠ½ΠΎΠΌΡƒ Ρ‚ΠΈΠΏΡƒ соСдинСния.
РСзисторы R2 ΠΈ R3 соСдинСны ΠΏΠΎΡΠ»Π΅Π΄ΠΎΠ²Π°Ρ‚Π΅Π»ΡŒΠ½ΠΎ (Π³Ρ€ΡƒΠΏΠΏΠ° 2). Они Π² свою ΠΎΡ‡Π΅Ρ€Π΅Π΄ΡŒ соСдинСны ΠΏΠ°Ρ€Π°Π»Π»Π΅Π»ΡŒΠ½ΠΎ с рСзистором R1 (Π³Ρ€ΡƒΠΏΠΏΠ° 1).

ΠŸΠΎΡΠ»Π΅Π΄ΠΎΠ²Π°Ρ‚Π΅Π»ΡŒΠ½ΠΎΠ΅ соСдинСниС рСзисторов Π³Ρ€ΡƒΠΏΠΏΡ‹ 2 вычисляСтся ΠΊΠ°ΠΊ сумма сопротивлСний R2 ΠΈ R3:

Π’ Ρ€Π΅Π·ΡƒΠ»ΡŒΡ‚Π°Ρ‚Π΅ ΠΌΡ‹ ΡƒΠΏΡ€ΠΎΡ‰Π°Π΅ΠΌ схСму Π² Π²ΠΈΠ΄Π΅ Π΄Π²ΡƒΡ… ΠΏΠ°Ρ€Π°Π»Π»Π΅Π»ΡŒΠ½Ρ‹Ρ… рСзисторов. Π’Π΅ΠΏΠ΅Ρ€ΡŒ ΠΎΠ±Ρ‰Π΅Π΅ сопротивлСниС всСй схСмы ΠΌΠΎΠΆΠ½ΠΎ ΠΏΠΎΡΡ‡ΠΈΡ‚Π°Ρ‚ΡŒ ΡΠ»Π΅Π΄ΡƒΡŽΡ‰ΠΈΠΌ ΠΎΠ±Ρ€Π°Π·ΠΎΠΌ:

РасчСт Π±ΠΎΠ»Π΅Π΅ слоТных соСдинСний рСзисторов ΠΌΠΎΠΆΠ½ΠΎ Π²Ρ‹ΠΏΠΎΠ»Π½ΠΈΡ‚ΡŒ ΠΈΡΠΏΠΎΠ»ΡŒΠ·ΡƒΡ Π·Π°ΠΊΠΎΠ½Ρ‹ ΠšΠΈΡ€Ρ…Π³ΠΎΡ„Π°.

Π’ΠΎΠΊ, ΠΏΡ€ΠΎΡ‚Π΅ΠΊΠ°ΡŽΡ‰ΠΈΠΉ Π² Ρ†Π΅ΠΏΠΈ ΠΏΠ°Ρ€Π°Π»Π»Π΅Π»ΡŒΠ½ΠΎ соСдинСнных рСзисторах

ΠžΠ±Ρ‰ΠΈΠΉ Ρ‚ΠΎΠΊ I ΠΏΡ€ΠΎΡ‚Π΅ΠΊΠ°ΡŽΡ‰ΠΈΠΉ Π² Ρ†Π΅ΠΏΠΈ ΠΏΠ°Ρ€Π°Π»Π»Π΅Π»ΡŒΠ½Ρ‹Ρ… рСзисторов равняСтся суммС ΠΎΡ‚Π΄Π΅Π»ΡŒΠ½Ρ‹Ρ… Ρ‚ΠΎΠΊΠΎΠ², ΠΏΡ€ΠΎΡ‚Π΅ΠΊΠ°ΡŽΡ‰ΠΈΡ… Π²ΠΎ всСх ΠΏΠ°Ρ€Π°Π»Π»Π΅Π»ΡŒΠ½Ρ‹Ρ… вСтвях, ΠΏΡ€ΠΈΡ‡Π΅ΠΌ Ρ‚ΠΎΠΊ Π² ΠΎΡ‚Π΄Π΅Π»ΡŒΠ½ΠΎ взятой Π²Π΅Ρ‚Π²ΠΈ Π½Π΅ ΠΎΠ±ΡΠ·Π°Ρ‚Π΅Π»ΡŒΠ½ΠΎ Π΄ΠΎΠ»ΠΆΠ΅Π½ Π±Ρ‹Ρ‚ΡŒ Ρ€Π°Π²Π΅Π½ Ρ‚ΠΎΠΊΡƒ Π² сосСдних вСтвях.

НСсмотря Π½Π° ΠΏΠ°Ρ€Π°Π»Π»Π΅Π»ΡŒΠ½ΠΎΠ΅ соСдинСниС, ΠΊ ΠΊΠ°ΠΆΠ΄ΠΎΠΌΡƒ рСзистору ΠΏΡ€ΠΈΠ»ΠΎΠΆΠ΅Π½ΠΎ ΠΎΠ΄Π½ΠΎ ΠΈ Ρ‚ΠΎ ΠΆΠ΅ напряТСниС. А ΠΏΠΎΡΠΊΠΎΠ»ΡŒΠΊΡƒ Π²Π΅Π»ΠΈΡ‡ΠΈΠ½Π° сопротивлСний Π² ΠΏΠ°Ρ€Π°Π»Π»Π΅Π»ΡŒΠ½ΠΎΠΉ Ρ†Π΅ΠΏΠΈ ΠΌΠΎΠΆΠ΅Ρ‚ Π±Ρ‹Ρ‚ΡŒ Ρ€Π°Π·Π½ΠΎΠΉ, Ρ‚ΠΎ ΠΈ Π²Π΅Π»ΠΈΡ‡ΠΈΠ½Π° ΠΏΡ€ΠΎΡ‚Π΅ΠΊΠ°ΡŽΡ‰Π΅Π³ΠΎ Ρ‚ΠΎΠΊΠ° Ρ‡Π΅Ρ€Π΅Π· ΠΊΠ°ΠΆΠ΄Ρ‹ΠΉ рСзистор Ρ‚ΠΎΠΆΠ΅ Π±ΡƒΠ΄Π΅Ρ‚ ΠΎΡ‚Π»ΠΈΡ‡Π°Ρ‚ΡŒΡΡ (ΠΏΠΎ ΠΎΠΏΡ€Π΅Π΄Π΅Π»Π΅Π½ΠΈΡŽ Π·Π°ΠΊΠΎΠ½Π° Ома).

Рассмотрим это Π½Π° ΠΏΡ€ΠΈΠΌΠ΅Ρ€Π΅ Π΄Π²ΡƒΡ… ΠΏΠ°Ρ€Π°Π»Π»Π΅Π»ΡŒΠ½ΠΎ соСдинСнных рСзисторов. Π’ΠΎΠΊ, ΠΊΠΎΡ‚ΠΎΡ€Ρ‹ΠΉ Ρ‚Π΅Ρ‡Π΅Ρ‚ Ρ‡Π΅Ρ€Π΅Π· ΠΊΠ°ΠΆΠ΄Ρ‹ΠΉ ΠΈΠ· рСзисторов (I1 ΠΈ I2) Π±ΡƒΠ΄Π΅Ρ‚ ΠΎΡ‚Π»ΠΈΡ‡Π°Ρ‚ΡŒΡΡ Π΄Ρ€ΡƒΠ³ ΠΎΡ‚ Π΄Ρ€ΡƒΠ³Π° ΠΏΠΎΡΠΊΠΎΠ»ΡŒΠΊΡƒ сопротивлСния рСзисторов R1 ΠΈ R2 Π½Π΅ Ρ€Π°Π²Π½Ρ‹.
Однако ΠΌΡ‹ Π·Π½Π°Π΅ΠΌ, Ρ‡Ρ‚ΠΎ Ρ‚ΠΎΠΊ, ΠΊΠΎΡ‚ΠΎΡ€Ρ‹ΠΉ поступаСт Π² Ρ†Π΅ΠΏΡŒ Π² Ρ‚ΠΎΡ‡ΠΊΠ΅ «А» Π΄ΠΎΠ»ΠΆΠ΅Π½ Π²Ρ‹ΠΉΡ‚ΠΈ ΠΈΠ· Ρ†Π΅ΠΏΠΈ Π² Ρ‚ΠΎΡ‡ΠΊΠ΅ Β«BΒ» .

ΠŸΠ΅Ρ€Π²ΠΎΠ΅ ΠΏΡ€Π°Π²ΠΈΠ»ΠΎ ΠšΠΈΡ€Ρ…Π³ΠΎΡ„Π° гласит: Β«ΠžΠ±Ρ‰ΠΈΠΉ Ρ‚ΠΎΠΊ, выходящий ΠΈΠ· Ρ†Π΅ΠΏΠΈ Ρ€Π°Π²Π΅Π½ Ρ‚ΠΎΠΊΡƒ входящий Π² Ρ†Π΅ΠΏΡŒΒ».

Π’Π°ΠΊΠΈΠΌ ΠΎΠ±Ρ€Π°Π·ΠΎΠΌ, ΠΏΡ€ΠΎΡ‚Π΅ΠΊΠ°ΡŽΡ‰ΠΈΠΉ ΠΎΠ±Ρ‰ΠΈΠΉ Ρ‚ΠΎΠΊ Π² Ρ†Π΅ΠΏΠΈ ΠΌΠΎΠΆΠ½ΠΎ ΠΎΠΏΡ€Π΅Π΄Π΅Π»ΠΈΡ‚ΡŒ ΠΊΠ°ΠΊ:

Π—Π°Ρ‚Π΅ΠΌ с ΠΏΠΎΠΌΠΎΡ‰ΡŒΡŽ Π·Π°ΠΊΠΎΠ½Π° Ома ΠΌΠΎΠΆΠ½ΠΎ Π²Ρ‹Ρ‡ΠΈΡΠ»ΠΈΡ‚ΡŒ Ρ‚ΠΎΠΊ, ΠΊΠΎΡ‚ΠΎΡ€Ρ‹ΠΉ ΠΏΡ€ΠΎΡ‚Π΅ΠΊΠ°Π΅Ρ‚ Ρ‡Π΅Ρ€Π΅Π· ΠΊΠ°ΠΆΠ΄Ρ‹ΠΉ рСзистор:

Π’ΠΎΠΊ, ΠΏΡ€ΠΎΡ‚Π΅ΠΊΠ°ΡŽΡ‰ΠΈΠΉ Π² R1 = U Γ· R1 = 12 Γ· 22 кОм = 0,545 мА

Π’ΠΎΠΊ, ΠΏΡ€ΠΎΡ‚Π΅ΠΊΠ°ΡŽΡ‰ΠΈΠΉ Π² R 2 = U Γ· R2 = 12 Γ· 47 кОм = 0,255 мА

Π’Π°ΠΊΠΈΠΌ ΠΎΠ±Ρ€Π°Π·ΠΎΠΌ, ΠΎΠ±Ρ‰ΠΈΠΉ Ρ‚ΠΎΠΊ Π±ΡƒΠ΄Π΅Ρ‚ Ρ€Π°Π²Π΅Π½:

I = 0,545 мА + 0,255 мА = 0,8 мА

Π­Ρ‚ΠΎ Ρ‚Π°ΠΊΠΆΠ΅ ΠΌΠΎΠΆΠ½ΠΎ ΠΏΡ€ΠΎΠ²Π΅Ρ€ΠΈΡ‚ΡŒ, ΠΈΡΠΏΠΎΠ»ΡŒΠ·ΡƒΡ Π·Π°ΠΊΠΎΠ½ Ома:

I = U Γ· R = 12 Π’ Γ· 15 кОм = 0,8 мА (Ρ‚ΠΎ ΠΆΠ΅ самоС)

Π³Π΄Π΅ 15кОм β€” это ΠΎΠ±Ρ‰Π΅Π΅ сопротивлСниС Π΄Π²ΡƒΡ… ΠΏΠ°Ρ€Π°Π»Π»Π΅Π»ΡŒΠ½ΠΎ соСдинСнных рСзисторов (22 кОм ΠΈ 47 кОм)

И Π² Π·Π°Π²Π΅Ρ€ΡˆΠ΅Π½ΠΈΠΈ хочСтся ΠΎΡ‚ΠΌΠ΅Ρ‚ΠΈΡ‚ΡŒ, Ρ‡Ρ‚ΠΎ Π±ΠΎΠ»ΡŒΡˆΠΈΠ½ΡΡ‚Π²ΠΎ соврСмСнных рСзисторов ΠΌΠ°Ρ€ΠΊΠΈΡ€ΡƒΡŽΡ‚ΡΡ Ρ†Π²Π΅Ρ‚Π½Ρ‹ΠΌΠΈ полосками ΠΈ Π½Π°Π·Π½Π°Ρ‡Π΅Π½ΠΈΠ΅ Π΅Π΅ ΠΌΠΎΠΆΠ½ΠΎ ΡƒΠ·Π½Π°Ρ‚ΡŒ .

ΠŸΠ°Ρ€Π°Π»Π»Π΅Π»ΡŒΠ½ΠΎΠ΅ соСдинСниС рСзисторов β€” ΠΎΠ½Π»Π°ΠΉΠ½ ΠΊΠ°Π»ΡŒΠΊΡƒΠ»ΡΡ‚ΠΎΡ€

Π§Ρ‚ΠΎΠ±Ρ‹ быстро Π²Ρ‹Ρ‡ΠΈΡΠ»ΠΈΡ‚ΡŒ ΠΎΠ±Ρ‰Π΅Π΅ сопротивлСниС Π΄Π²ΡƒΡ… ΠΈ Π±ΠΎΠ»Π΅Π΅ рСзисторов, соСдинСнных ΠΏΠ°Ρ€Π°Π»Π»Π΅Π»ΡŒΠ½ΠΎ, Π²Ρ‹ ΠΌΠΎΠΆΠ΅Ρ‚Π΅ Π²ΠΎΡΠΏΠΎΠ»ΡŒΠ·ΠΎΠ²Π°Ρ‚ΡŒΡΡ ΡΠ»Π΅Π΄ΡƒΡŽΡ‰ΠΈΠΌ ΠΎΠ½Π»Π°ΠΉΠ½ ΠΊΠ°Π»ΡŒΠΊΡƒΠ»ΡΡ‚ΠΎΡ€ΠΎΠΌ:

ПодвСдСм ΠΈΡ‚ΠΎΠ³

Когда Π΄Π²Π° ΠΈΠ»ΠΈ Π±ΠΎΠ»Π΅Π΅ рСзистора соСдинСны Ρ‚Π°ΠΊ, Ρ‡Ρ‚ΠΎ ΠΎΠ±Π° Π²Ρ‹Π²ΠΎΠ΄Π° ΠΎΠ΄Π½ΠΎΠ³ΠΎ рСзистора соСдинСны с ΡΠΎΠΎΡ‚Π²Π΅Ρ‚ΡΡ‚Π²ΡƒΡŽΡ‰ΠΈΠΌΠΈ Π²Ρ‹Π²ΠΎΠ΄Π°ΠΌΠΈ Π΄Ρ€ΡƒΠ³ΠΎΠ³ΠΎ рСзистора ΠΈΠ»ΠΈ рСзисторов, Ρ‚ΠΎ говорят, Ρ‡Ρ‚ΠΎ ΠΎΠ½ΠΈ соСдинСны ΠΌΠ΅ΠΆΠ΄Ρƒ собой ΠΏΠ°Ρ€Π°Π»Π»Π΅Π»ΡŒΠ½ΠΎ. НапряТСниС Π½Π° ΠΊΠ°ΠΆΠ΄ΠΎΠΌ рСзисторС Π²Π½ΡƒΡ‚Ρ€ΠΈ ΠΏΠ°Ρ€Π°Π»Π»Π΅Π»ΡŒΠ½ΠΎΠΉ ΠΊΠΎΠΌΠ±ΠΈΠ½Π°Ρ†ΠΈΠΈ ΠΎΠ΄ΠΈΠ½Π°ΠΊΠΎΠ²ΠΎΠ΅, Π½ΠΎ Ρ‚ΠΎΠΊΠΈ, ΠΏΡ€ΠΎΡ‚Π΅ΠΊΠ°ΡŽΡ‰ΠΈΠ΅ Ρ‡Π΅Ρ€Π΅Π· Π½ΠΈΡ…, ΠΌΠΎΠ³ΡƒΡ‚ ΠΎΡ‚Π»ΠΈΡ‡Π°Ρ‚ΡŒΡΡ Π΄Ρ€ΡƒΠ³ ΠΎΡ‚ Π΄Ρ€ΡƒΠ³Π°, Π² зависимости ΠΎΡ‚ Π²Π΅Π»ΠΈΡ‡ΠΈΠ½Ρ‹ сопротивлСний ΠΊΠ°ΠΆΠ΄ΠΎΠ³ΠΎ рСзистора.

Π­ΠΊΠ²ΠΈΠ²Π°Π»Π΅Π½Ρ‚Π½ΠΎΠ΅ ΠΈΠ»ΠΈ ΠΏΠΎΠ»Π½ΠΎΠ΅ сопротивлСниС ΠΏΠ°Ρ€Π°Π»Π»Π΅Π»ΡŒΠ½ΠΎΠΉ ΠΊΠΎΠΌΠ±ΠΈΠ½Π°Ρ†ΠΈΠΈ всСгда Π±ΡƒΠ΄Π΅Ρ‚ мСньшС минимального сопротивлСния рСзистора входящСго Π² ΠΏΠ°Ρ€Π°Π»Π»Π΅Π»ΡŒΠ½ΠΎΠ΅ соСдинСниС.

ΠŸΠ°Ρ€Π°Π»Π»Π΅Π»ΡŒΠ½ΠΎΠ΅ соСдинСниС рСзисторов, Π° Ρ‚Π°ΠΊΠΆΠ΅ ΠΏΠΎΡΠ»Π΅Π΄ΠΎΠ²Π°Ρ‚Π΅Π»ΡŒΠ½ΠΎΠ΅

Π’Ρ‹Π²ΠΎΠ΄ Ρ„ΠΎΡ€ΠΌΡƒΠ» для опрСдСлСния ΠΎΠ±Ρ‰Π΅Π³ΠΎ сопротивлСния. ΠŸΡ€Π°ΠΊΡ‚ΠΈΡ‡Π΅ΡΠΊΠΈΠ΅ ΠΏΡ€ΠΈΠΌΠ΅Ρ€Ρ‹ ΠΏΠΎΡΠ»Π΅Π΄ΠΎΠ²Π°Ρ‚Π΅Π»ΡŒΠ½ΠΎΠ³ΠΎ, ΠΏΠ°Ρ€Π°Π»Π»Π΅Π»ΡŒΠ½ΠΎΠ³ΠΎ ΠΈ смСшанного соСдинСния рСзисторов.

ΠžΠΏΡ€Π΅Π΄Π΅Π»Π΅Π½ΠΈΠ΅ эквивалСнтного сопротивлСния

ΠŸΡ€ΠΈ рассмотрСнии схСм Π»ΡŽΠ±Ρ‹Ρ… элСктричСских ΠΈΠ»ΠΈ элСктронных устройств ΠΌΠΎΠΆΠ½ΠΎ ΡƒΠ²ΠΈΠ΄Π΅Ρ‚ΡŒ, Ρ‡Ρ‚ΠΎ Ρ‚Π°ΠΊΠΈΠ΅ ΠΊΠΎΠΌΠΏΠΎΠ½Π΅Π½Ρ‚Ρ‹, ΠΊΠ°ΠΊ рСзисторы, ΠΈΠΌΠ΅ΡŽΡ‚ Ρ€Π°Π·Π½Ρ‹Π΅ Ρ‚ΠΈΠΏΡ‹ соСдинСний ΠΌΠ΅ΠΆΠ΄Ρƒ собой. Π§Ρ‚ΠΎΠ±Ρ‹ ΠΎΠΏΡ€Π΅Π΄Π΅Π»ΠΈΡ‚ΡŒ эквивалСнтноС соСдинСниС, Π½Π΅ΠΎΠ±Ρ…ΠΎΠ΄ΠΈΠΌΠΎ Ρ€Π°ΡΡΠΌΠ°Ρ‚Ρ€ΠΈΠ²Π°Ρ‚ΡŒ Π΄Π²Π° элСмСнта, Π²ΠΊΠ»ΡŽΡ‡Π΅Π½Π½Ρ‹Ρ… Π² ΠΎΠΏΡ€Π΅Π΄Π΅Π»Ρ‘Π½Π½ΠΎΠΌ порядкС. НСсмотря Π½Π° Ρ‚ΠΎ, Ρ‡Ρ‚ΠΎ Π½Π° Ρ‡Π΅Ρ€Ρ‚Π΅ΠΆΠ΅ ΠΈΡ… ΠΌΠΎΠΆΠ΅Ρ‚ Π±Ρ‹Ρ‚ΡŒ нСсколько дСсятков, ΠΈ соСдинСны ΠΎΠ½ΠΈ ΠΏΠΎ-Ρ€Π°Π·Π½ΠΎΠΌΡƒ, Π΅ΡΡ‚ΡŒ Ρ‚ΠΎΠ»ΡŒΠΊΠΎ Π΄Π²Π° Ρ‚ΠΈΠΏΠ° Π²ΠΊΠ»ΡŽΡ‡Π΅Π½ΠΈΡ ΠΈΡ… Π΄Ρ€ΡƒΠ³ с Π΄Ρ€ΡƒΠ³ΠΎΠΌ: ΠΏΠΎΡΠ»Π΅Π΄ΠΎΠ²Π°Ρ‚Π΅Π»ΡŒΠ½ΠΎΠ΅ ΠΈ ΠΏΠ°Ρ€Π°Π»Π»Π΅Π»ΡŒΠ½ΠΎΠ΅. ΠžΡΡ‚Π°Π»ΡŒΠ½Ρ‹Π΅ ΠΊΠΎΠ½Ρ„ΠΈΠ³ΡƒΡ€Π°Ρ†ΠΈΠΈ – это лишь ΠΈΡ… Π²Π°Ρ€ΠΈΠ°Ρ†ΠΈΠΈ.

ΠŸΠΎΡΠ»Π΅Π΄ΠΎΠ²Π°Ρ‚Π΅Π»ΡŒΠ½ΠΎΠ΅ соСдинСниС[ΠΏΡ€Π°Π²ΠΈΡ‚ΡŒ | ΠΏΡ€Π°Π²ΠΈΡ‚ΡŒ ΠΊΠΎΠ΄]

ΠŸΡ€ΠΈ ΠΏΠΎΡΠ»Π΅Π΄ΠΎΠ²Π°Ρ‚Π΅Π»ΡŒΠ½ΠΎΠΌ соСдинСнии ΠΏΡ€ΠΎΠ²ΠΎΠ΄Π½ΠΈΠΊΠΎΠ² сила Ρ‚ΠΎΠΊΠ° Π² Π»ΡŽΠ±Ρ‹Ρ… частях Ρ†Π΅ΠΏΠΈ ΠΎΠ΄Π½Π° ΠΈ Ρ‚Π° ΠΆΠ΅: I = I 1 = I 2 = β‹― = I n {displaystyle Imathrm {=} I_{1}=I_{2}=cdots =I_{n}} (Ρ‚Π°ΠΊ ΠΊΠ°ΠΊ сила Ρ‚ΠΎΠΊΠ° опрСдСляСтся количСством элСктронов, проходящих Ρ‡Π΅Ρ€Π΅Π· ΠΏΠΎΠΏΠ΅Ρ€Π΅Ρ‡Π½ΠΎΠ΅ сСчСниС ΠΏΡ€ΠΎΠ²ΠΎΠ΄Π½ΠΈΠΊΠ°, ΠΈ Ссли Π² Ρ†Π΅ΠΏΠΈ Π½Π΅Ρ‚ ΡƒΠ·Π»ΠΎΠ², Ρ‚ΠΎ всС элСктроны Π² Π½Π΅ΠΉ Π±ΡƒΠ΄ΡƒΡ‚ Ρ‚Π΅Ρ‡ΡŒ ΠΏΠΎ ΠΎΠ΄Π½ΠΎΠΌΡƒ ΠΏΡ€ΠΎΠ²ΠΎΠ΄Π½ΠΈΠΊΡƒ).

ПолноС напряТСниС Π² Ρ†Π΅ΠΏΠΈ ΠΏΡ€ΠΈ ΠΏΠΎΡΠ»Π΅Π΄ΠΎΠ²Π°Ρ‚Π΅Π»ΡŒΠ½ΠΎΠΌ соСдинСнии, ΠΈΠ»ΠΈ напряТСниС Π½Π° ΠΏΠΎΠ»ΡŽΡΠ°Ρ… источника питания, Ρ€Π°Π²Π½ΠΎ суммС напряТСний Π½Π° ΠΎΡ‚Π΄Π΅Π»ΡŒΠ½Ρ‹Ρ… участках Ρ†Π΅ΠΏΠΈ: U = U 1 + U 2 + β‹― + U n {displaystyle Umathrm {=} U_{1}+U_{2}+cdots +U_{n}} .

РСзисторы[ΠΏΡ€Π°Π²ΠΈΡ‚ΡŒ | ΠΏΡ€Π°Π²ΠΈΡ‚ΡŒ ΠΊΠΎΠ΄]

R = R 1 + R 2 + β‹― + R n {displaystyle R=R_{1}+R_{2}+cdots +R_{n}}

ΠšΠ°Ρ‚ΡƒΡˆΠΊΠ° индуктивности[ΠΏΡ€Π°Π²ΠΈΡ‚ΡŒ | ΠΏΡ€Π°Π²ΠΈΡ‚ΡŒ ΠΊΠΎΠ΄]

L = L 1 + L 2 + β‹― + L n {displaystyle L=L_{1}+L_{2}+cdots +L_{n}}

ЭлСктричСский кондСнсатор[ΠΏΡ€Π°Π²ΠΈΡ‚ΡŒ | ΠΏΡ€Π°Π²ΠΈΡ‚ΡŒ ΠΊΠΎΠ΄]

1 C = 1 C 1 + 1 C 2 + β‹― + 1 C n {displaystyle {frac {1}{C}}={frac {1}{C_{1}}}+{frac {1}{C_{2}}}+cdots +{frac {1}{C_{n}}}} .

ΠœΠ΅ΠΌΡ€ΠΈΡΡ‚ΠΎΡ€Ρ‹[ΠΏΡ€Π°Π²ΠΈΡ‚ΡŒ | ΠΏΡ€Π°Π²ΠΈΡ‚ΡŒ ΠΊΠΎΠ΄]

M = M 1 + M 2 + β‹― + M n {displaystyle M=M_{1}+M_{2}+cdots +M_{n}}

Π’Ρ‹ΠΊΠ»ΡŽΡ‡Π°Ρ‚Π΅Π»ΠΈ[ΠΏΡ€Π°Π²ΠΈΡ‚ΡŒ | ΠΏΡ€Π°Π²ΠΈΡ‚ΡŒ ΠΊΠΎΠ΄]

ЦСпь Π·Π°ΠΌΠΊΠ½ΡƒΡ‚Π°, ΠΊΠΎΠ³Π΄Π° Π·Π°ΠΌΠΊΠ½ΡƒΡ‚Ρ‹ всС Π²Ρ‹ΠΊΠ»ΡŽΡ‡Π°Ρ‚Π΅Π»ΠΈ. ЦСпь Ρ€Π°Π·ΠΎΠΌΠΊΠ½ΡƒΡ‚Π°, ΠΊΠΎΠ³Π΄Π° Ρ€Π°Π·ΠΎΠΌΠΊΠ½ΡƒΡ‚ хотя Π±Ρ‹ ΠΎΠ΄ΠΈΠ½ Π²Ρ‹ΠΊΠ»ΡŽΡ‡Π°Ρ‚Π΅Π»ΡŒ. (Π‘ΠΌ.Ρ‚Π°ΠΊΠΆΠ΅ ЛогичСская опСрация И).

Π§Ρ‚ΠΎ Ρ‚Π°ΠΊΠΎΠ΅ рСзистор ΠΈ для Ρ‡Π΅Π³ΠΎ ΠΎΠ½ Π½ΡƒΠΆΠ΅Π½

РСзистор – пассивный элСмСнт элСктричСской Ρ†Π΅ΠΏΠΈ, ΠΊΠΎΡ‚ΠΎΡ€Ρ‹ΠΉ ΠΏΠΎΠ³Π»ΠΎΡ‰Π°Π΅Ρ‚ ΡΠ½Π΅Ρ€Π³ΠΈΡŽ Ρ‚ΠΎΠΊΠ° ΠΈ ΠΏΡ€Π΅ΠΎΠ±Ρ€Π°Π·ΠΎΠ²Ρ‹Π²Π°Π΅Ρ‚ Π΅Ρ‘ Π² Ρ‚Π΅ΠΏΠ»ΠΎ Π·Π° счСт сопротивлСния ΠΏΠΎΡ‚ΠΎΠΊΡƒ элСктронов Π² Ρ†Π΅ΠΏΠΈ.

Π—Π°Π²ΠΈΡΠΈΠΌΠΎΡΡ‚ΡŒ Ρ‚ΠΎΠΊΠ° ΠΎΡ‚ сопротивлСния описываСтся Π·Π°ΠΊΠΎΠ½ΠΎΠΌ Ома ΠΈ рассчитываСтся ΠΏΠΎ Ρ„ΠΎΡ€ΠΌΡƒΠ»Π΅ I = U/R.

Бвойство рСзисторов ΠΎΠ³Ρ€Π°Π½ΠΈΡ‡ΠΈΠ²Π°Ρ‚ΡŒ Ρ‚ΠΎΠΊ ΠΈ ΡΠ½ΠΈΠΆΠ°Ρ‚ΡŒ напряТСниС ΠΈΡΠΏΠΎΠ»ΡŒΠ·ΡƒΠ΅Ρ‚ΡΡ Π²ΠΎ ΠΌΠ½ΠΎΠ³ΠΈΡ… элСктронных устройствах ΠΈ Π±Ρ‹Ρ‚ΠΎΠ²Ρ‹Ρ… ΠΏΡ€ΠΈΠ±ΠΎΡ€Π°Ρ….

Π‘ΠΏΡ€Π°Π²ΠΊΠ°: РСзисторы Π±Ρ‹Π²Π°ΡŽΡ‚ Π΄Π²ΡƒΡ… Π²ΠΈΠ΄ΠΎΠ² – постоянныС ΠΈ ΠΏΠ΅Ρ€Π΅ΠΌΠ΅Π½Π½Ρ‹Π΅, Π²ΠΎ Π²Ρ‚ΠΎΡ€ΠΎΠΌ случаС сопротивлСниС ΠΏΡ€ΠΎΠ²ΠΎΠ΄Π½ΠΈΠΊΠ° измСняСтся мСханичСским ΠΏΡƒΡ‚Π΅ΠΌ (Π²Ρ€ΡƒΡ‡Π½ΡƒΡŽ).

ΠŸΠΎΡΠ»Π΅Π΄ΠΎΠ²Π°Ρ‚Π΅Π»ΡŒΠ½ΠΎΠ΅ ΠΈ ΠΏΠ°Ρ€Π°Π»Π»Π΅Π»ΡŒΠ½ΠΎΠ΅ соСдинСниС рСзисторов – основныС способы соСдинСния рСзистивных элСмСнтов.

Π’Π½ΠΈΠΌΠ°Π½ΠΈΠ΅! РСзистор Π½Π΅ ΠΈΠΌΠ΅Π΅Ρ‚ полярности, Π΄Π»ΠΈΠ½Π° Π²Ρ‹Π²ΠΎΠ΄ΠΎΠ² с ΠΎΠ±ΠΎΠΈΡ… ΠΊΠΎΠ½Ρ†ΠΎΠ² ΠΎΠ΄ΠΈΠ½Π°ΠΊΠΎΠ²Π°, поэтому для Π»ΡƒΡ‡ΡˆΠ΅Π³ΠΎ понимания сути соСдинСния прСдлагаСтся Π½Π°Π·Ρ‹Π²Π°Ρ‚ΡŒ Π²Ρ‹Π²ΠΎΠ΄Ρ‹:

  1. Π‘ ΠΏΡ€Π°Π²ΠΎΠ³ΠΎ края – ΠΏΡ€Π°Π²Ρ‹ΠΉ.
  2. Π‘ Π»Π΅Π²ΠΎΠ³ΠΎ края – Π»Π΅Π²Ρ‹ΠΉ.

ΠŸΠΎΠ½ΡΡ‚ΠΈΠ΅ ΠΏΠ°Ρ€Π°Π»Π»Π΅Π»ΡŒΠ½ΠΎΠ³ΠΎ ΠΏΠΎΠ΄ΠΊΠ»ΡŽΡ‡Π΅Π½ΠΈΡ рСзисторов

ΠŸΡ€ΠΈ ΠΏΠ°Ρ€Π°Π»Π»Π΅Π»ΡŒΠ½ΠΎΠΌ ΠΏΠΎΠ΄ΠΊΠ»ΡŽΡ‡Π΅Π½ΠΈΠΈ ΠΏΡ€Π°Π²Ρ‹Π΅ Π²Ρ‹Π²ΠΎΠ΄Ρ‹ всСх рСзисторов ΡΠΎΠ΅Π΄ΠΈΠ½ΡΡŽΡ‚ΡΡ Π² ΠΎΠ΄ΠΈΠ½ ΡƒΠ·Π΅Π», Π»Π΅Π²Ρ‹Π΅ – Π²ΠΎ Π²Ρ‚ΠΎΡ€ΠΎΠΉ ΡƒΠ·Π΅Π».

ΠŸΡ€ΠΈ ΠΏΠ°Ρ€Π°Π»Π»Π΅Π»ΡŒΠ½ΠΎΠΌ Π²ΠΊΠ»ΡŽΡ‡Π΅Π½ΠΈΠΈ рСзисторов Ρ‚ΠΎΠΊ Π² Ρ†Π΅ΠΏΡŒ развСтвляСтся ΠΏΠΎ ΠΎΡ‚Π΄Π΅Π»ΡŒΠ½Ρ‹ΠΌ вСтвям, протСкая Ρ‡Π΅Ρ€Π΅Π· ΠΊΠ°ΠΆΠ΄Ρ‹ΠΉ элСмСнт – ΠΏΠΎ Π·Π°ΠΊΠΎΠ½Ρƒ Ома Π²Π΅Π»ΠΈΡ‡ΠΈΠ½Π° Ρ‚ΠΎΠΊΠ° ΠΎΠ±Ρ€Π°Ρ‚Π½ΠΎ ΠΏΡ€ΠΎΠΏΠΎΡ€Ρ†ΠΈΠΎΠ½Π°Π»ΡŒΠ½Π° ΡΠΎΠΏΡ€ΠΎΡ‚ΠΈΠ²Π»Π΅Π½ΠΈΡŽ, напряТСниС Π½Π° всСх элСмСнтах ΠΎΠ΄ΠΈΠ½Π°ΠΊΠΎΠ²ΠΎΠ΅.

Π‘ΠΏΡ€Π°Π²ΠΊΠ°: Π’Π΅Ρ‚Π²ΡŒ – Ρ„Ρ€Π°Π³ΠΌΠ΅Π½Ρ‚ элСктричСской Ρ†Π΅ΠΏΠΈ, содСрТащий ΠΎΠ΄ΠΈΠ½ ΠΈΠ»ΠΈ нСсколько ΠΏΠΎΡΠ»Π΅Π΄ΠΎΠ²Π°Ρ‚Π΅Π»ΡŒΠ½ΠΎ соСдинСнных ΠΊΠΎΠΌΠΏΠΎΠ½Π΅Π½Ρ‚ΠΎΠ² ΠΎΡ‚ ΡƒΠ·Π»Π° Π΄ΠΎ ΡƒΠ·Π»Π°.

Π€ΠΎΡ€ΠΌΡƒΠ»Π° сопротивлСния

Π€ΠΎΡ€ΠΌΡƒΠ»Π° ёмкостного сопротивлСния выводится ΡΠ»Π΅Π΄ΡƒΡŽΡ‰ΠΈΠΌ ΠΎΠ±Ρ€Π°Π·ΠΎΠΌ:

  • Π’Π½Π°Ρ‡Π°Π»Π΅ слСдуСт Π²Ρ‹Ρ‡ΠΈΡΠ»ΠΈΡ‚ΡŒ ΡƒΠ³Π»ΠΎΠ²ΡƒΡŽ частоту. Для этого частоту ΠΏΡ€ΠΎΡ‚Π΅ΠΊΠ°ΡŽΡ‰Π΅Π³ΠΎ ΠΏΠΎ Ρ†Π΅ΠΏΠΈ Ρ‚ΠΎΠΊΠ° (Π² Π³Π΅Ρ€Ρ†Π°Ρ…) Π½Π΅ΠΎΠ±Ρ…ΠΎΠ΄ΠΈΠΌΠΎ ΡƒΠΌΠ½ΠΎΠΆΠΈΡ‚ΡŒ Π½Π° ΡƒΠ΄Π²ΠΎΠ΅Π½Π½ΠΎΠ΅ число Β«ΠΏΠΈΒ».
  • Π—Π°Ρ‚Π΅ΠΌ ΠΏΠΎΠ»ΡƒΡ‡Π΅Π½Π½ΠΎΠ΅ число слСдуСт ΠΏΠ΅Ρ€Π΅ΠΌΠ½ΠΎΠΆΠΈΡ‚ΡŒ Π½Π° Ρ‘ΠΌΠΊΠΎΡΡ‚ΡŒ кондСнсатора Π² Ρ„Π°Ρ€Π°Π΄Π°Ρ….

Π§Ρ‚ΠΎΠ±Ρ‹ ΠΏΠΎΠ»ΡƒΡ‡ΠΈΡ‚ΡŒ Π·Π½Π°Ρ‡Π΅Π½ΠΈΠ΅ ёмкостного сопротивлСния Π² ΠΎΠΌΠ°Ρ…, слСдуСт Ρ€Π°Π·Π΄Π΅Π»ΠΈΡ‚ΡŒ Π΅Π΄ΠΈΠ½ΠΈΡ†Ρƒ Π½Π° число, ΠΏΠΎΠ»ΡƒΡ‡Π΅Π½Π½ΠΎΠ΅ послС умноТСния ΡƒΠ³Π»ΠΎΠ²ΠΎΠΉ частоты Π½Π° Ρ‘ΠΌΠΊΠΎΡΡ‚ΡŒ. Из этой Ρ„ΠΎΡ€ΠΌΡƒΠ»Ρ‹ Π²Ρ‹Ρ‚Π΅ΠΊΠ°Π΅Ρ‚, Ρ‡Ρ‚ΠΎ Ρ‡Π΅ΠΌ большС Ρ‘ΠΌΠΊΠΎΡΡ‚ΡŒ кондСнсатора ΠΈΠ»ΠΈ частота ΠΏΠ΅Ρ€Π΅ΠΌΠ΅Π½Π½ΠΎΠ³ΠΎ Ρ‚ΠΎΠΊΠ°, Ρ‚Π΅ΠΌ мСньшС Π΅Π³ΠΎ сопротивлСниС.

Когда частота Π±ΡƒΠ΄Π΅Ρ‚ Ρ€Π°Π²Π½Π° Π½ΡƒΠ»ΡŽ (постоянный Ρ‚ΠΎΠΊ), ёмкостноС сопротивлСниС станСт бСсконСчно большим. ΠšΠΎΠ½Π΄Π΅Π½ΡΠ°Ρ‚ΠΎΡ€ ΠΎΡ‡Π΅Π½ΡŒ большой ёмкости Π±ΡƒΠ΄Π΅Ρ‚ ΠΏΡ€ΠΎΠ²ΠΎΠ΄ΠΈΡ‚ΡŒ Ρ‚ΠΎΠΊ Π² ΡˆΠΈΡ€ΠΎΠΊΠΎΠΌ Π΄ΠΈΠ°ΠΏΠ°Π·ΠΎΠ½Π΅ частот.

ΠŸΠΎΡΠ»Π΅Π΄ΠΎΠ²Π°Ρ‚Π΅Π»ΡŒΠ½ΠΎΠ΅ соСдинСниС элСмСнтов

ПодобноС Π²ΠΊΠ»ΡŽΡ‡Π΅Π½ΠΈΠ΅ ΠΏΠΎΠ΄Ρ€Π°Π·ΡƒΠΌΠ΅Π²Π°Π΅Ρ‚ ΠΊΠΎΠΌΠ±ΠΈΠ½Π°Ρ†ΠΈΡŽ Π΄Π΅Ρ‚Π°Π»Π΅ΠΉ Π² прямой ΠΏΠΎΡΠ»Π΅Π΄ΠΎΠ²Π°Ρ‚Π΅Π»ΡŒΠ½ΠΎΡΡ‚ΠΈ. Π’Ρ‹Ρ…ΠΎΠ΄ ΠΎΠ΄Π½ΠΎΠ³ΠΎ сопротивлСния ΠΏΠΎΠ΄ΠΊΠ»ΡŽΡ‡Π°Π΅Ρ‚ΡΡ ΠΊ Π²Ρ…ΠΎΠ΄Ρƒ Π΄Ρ€ΡƒΠ³ΠΎΠ³ΠΎ. ΠŸΡ€ΠΈ этом ΠΎΡ‚ΡΡƒΡ‚ΡΡ‚Π²ΡƒΡŽΡ‚ ΠΊΠ°ΠΊΠΈΠ΅-Π»ΠΈΠ±ΠΎ отвСтвлСния Π½Π° участкС. Π’Π΅Π»ΠΈΡ‡ΠΈΠ½Π° Ρ‚ΠΎΠΊΠ°, ΠΊΠΎΡ‚ΠΎΡ€Ρ‹ΠΉ ΠΏΡ€ΠΎΡ…ΠΎΠ΄ΠΈΡ‚ Ρ‡Π΅Ρ€Π΅Π· всС соСдинённыС ΠΏΠΎΡΠ»Π΅Π΄ΠΎΠ²Π°Ρ‚Π΅Π»ΡŒΠ½ΠΎ ΠΊΠΎΠΌΠΏΠΎΠ½Π΅Π½Ρ‚Ρ‹, Π±ΡƒΠ΄Π΅Ρ‚ ΠΎΠ΄Π½Π° ΠΈ Ρ‚Π° ΠΆΠ΅.

Π’Π½ΠΈΠΌΠ°Π½ΠΈΠ΅! Π‘Π½ΠΈΠΆΠ΅Π½ΠΈΠ΅ ΠΏΠΎΡ‚Π΅Π½Ρ†ΠΈΠ°Π»Π° Π½Π° ΠΊΠ°ΠΆΠ΄ΠΎΠΌ рСзистивном элСмСнтС Π² суммС даст ΠΏΠΎΠ»Π½ΠΎΠ΅ напряТСниС, ΠΏΡ€ΠΈΠ»ΠΎΠΆΠ΅Π½Π½ΠΎΠ΅ ΠΊ ΠΏΠΎΡΠ»Π΅Π΄ΠΎΠ²Π°Ρ‚Π΅Π»ΡŒΠ½ΠΎΠΉ Ρ†Π΅ΠΏΠΈ.

ΠŸΠΎΡΠ»Π΅Π΄ΠΎΠ²Π°Ρ‚Π΅Π»ΡŒΠ½ΠΎΠ΅ Π²ΠΊΠ»ΡŽΡ‡Π΅Π½ΠΈΠ΅ рСзисторов

Π’ случаС постоянного Ρ‚ΠΎΠΊΠ° Ρ„ΠΎΡ€ΠΌΡƒΠ»Π° Π·Π°ΠΊΠΎΠ½Π° Ома для ΠΎΡ‚Ρ€Π΅Π·ΠΊΠ° Ρ†Π΅ΠΏΠΈ ΠΈΠΌΠ΅Π΅Ρ‚ Π²ΠΈΠ΄:

I = U/R.

Π‘ΠΈΠ»Π° Ρ‚ΠΎΠΊΠ° зависит ΠΎΡ‚ ΠΏΡ€ΠΈΠ»ΠΎΠΆΠ΅Π½Π½ΠΎΠ³ΠΎ напряТСния ΠΈ ΠΎΠΊΠ°Π·Π°Π½Π½ΠΎΠ³ΠΎ Π΅ΠΌΡƒ сопротивлСния. Если Π²Ρ‹Ρ€Π°Π·ΠΈΡ‚ΡŒ R, Π΅Π³ΠΎ Ρ„ΠΎΡ€ΠΌΡƒΠ»Π°:

R = U/I.

ΠŸΠ°Ρ€Π°ΠΌΠ΅Ρ‚Ρ€Ρ‹ ΠΏΠΎΡΠ»Π΅Π΄ΠΎΠ²Π°Ρ‚Π΅Π»ΡŒΠ½ΠΎΠΉ Ρ†Π΅ΠΏΠΈ, Π²ΠΊΠ»ΡŽΡ‡Π°ΡŽΡ‰Π΅ΠΉ n соСдинённых Π΄Ρ€ΡƒΠ³ с Π΄Ρ€ΡƒΠ³ΠΎΠΌ элСмСнтов, ΠΈΠΌΠ΅ΡŽΡ‚ свои особСнности.

ΠŸΡ€ΠΎΡ…ΠΎΠ΄ΡΡ‰ΠΈΠΉ ΠΏΠΎ Ρ†Π΅ΠΏΠΈ Ρ‚ΠΎΠΊ Π²Π΅Π·Π΄Π΅ ΠΎΠ΄ΠΈΠ½Π°ΠΊΠΎΠ²Ρ‹ΠΉ:

I = I1= I2= … = In.

ΠŸΡ€ΠΈΠΊΠ»Π°Π΄Ρ‹Π²Π°Π΅ΠΌΠΎΠ΅ напряТСниС являСтся суммой напряТСний Π½Π° ΠΊΠ°ΠΆΠ΄ΠΎΠΌ рСзисторС:

U = U1 + U2+ … + Un.

Π‘Π»Π΅Π΄ΠΎΠ²Π°Ρ‚Π΅Π»ΡŒΠ½ΠΎ, Ρ€Π°ΡΡΡ‡ΠΈΡ‚Π°Ρ‚ΡŒ ΠΌΠΎΠΆΠ½ΠΎ ΠΎΠ±Ρ‰Π΅Π΅:

Rэкв.= U1/I + U2/I + … +Un/I) = R1 + R2 + … +Rn.

Π’Π°ΠΆΠ½ΠΎ! ΠŸΠΎΡΠ»Π΅Π΄ΠΎΠ²Π°Ρ‚Π΅Π»ΡŒΠ½Π°Ρ Ρ†Π΅ΠΏΡŒ, ΠΈΠΌΠ΅ΡŽΡ‰Π°Ρ Π² своём составС N рСзисторов Ρ€Π°Π²Π½ΠΎΠ³ΠΎ Π½ΠΎΠΌΠΈΠ½Π°Π»Π°, ΠΈΠΌΠ΅Π΅Ρ‚ эквивалСнтноС сопротивлСниС Rэкв. = N*R.

Как ΠΏΠΎ Π²Π°Ρ… ΠΎΠΏΡ€Π΅Π΄Π΅Π»ΠΈΡ‚ΡŒ сопротивлСниС Ρ†Π΅ΠΏΠΈ

Π’ Π»ΠΈΠ½Π΅ΠΉΠ½ΠΎΠΉ элСктричСской Ρ†Π΅ΠΏΠΈ сопротивлСния Π΅Π΅ элСмСнтов Π½Π΅ зависят ΠΎΡ‚ Π²Π΅Π»ΠΈΡ‡ΠΈΠ½Ρ‹ ΠΈΠ»ΠΈ направлСния Ρ‚ΠΎΠΊΠ° ΠΈΠ»ΠΈ напряТСния. Π’ΠΎΠ»ΡŒΡ‚Π°ΠΌΠΏΠ΅Ρ€Π½Ρ‹Π΅ характСристики Π»ΠΈΠ½Π΅ΠΉΠ½Ρ‹Ρ… элСмСнтов (Π·Π°Π²ΠΈΡΠΈΠΌΠΎΡΡ‚ΡŒ напряТСния Π½Π° элСмСнтС ΠΎΡ‚ Ρ‚ΠΎΠΊΠ°) ΡΠ²Π»ΡΡŽΡ‚ΡΡ прямыми линиями.

Π’ Π½Π΅Π»ΠΈΠ½Π΅ΠΉΠ½ΠΎΠΉ элСктричСской Ρ†Π΅ΠΏΠΈ сопротивлСния Π΅Π΅ элСмСнтов зависят ΠΎΡ‚ Π²Π΅Π»ΠΈΡ‡ΠΈΠ½Ρ‹ ΠΈΠ»ΠΈ направлСния Ρ‚ΠΎΠΊΠ° ΠΈΠ»ΠΈ напряТСния. НСлинСйныС элСмСнты ΠΈΠΌΠ΅ΡŽΡ‚ ΠΊΡ€ΠΈΠ²ΠΎΠ»ΠΈΠ½Π΅ΠΉΠ½Ρ‹Π΅ Π²ΠΎΠ»ΡŒΡ‚Π°ΠΌΠΏΠ΅Ρ€Π½Ρ‹Π΅ характСристики, симмСтричныС ΠΈΠ»ΠΈ нСсиммСтричныС ΠΎΡ‚Π½ΠΎΡΠΈΡ‚Π΅Π»ΡŒΠ½ΠΎ осСй ΠΊΠΎΠΎΡ€Π΄ΠΈΠ½Π°Ρ‚. БопротивлСния Π½Π΅Π»ΠΈΠ½Π΅ΠΉΠ½Ρ‹Ρ… элСмСнтов с симмСтричной характСристикой Π½Π΅ зависят ΠΎΡ‚ направлСния Ρ‚ΠΎΠΊΠ°. БопротивлСния Π½Π΅Π»ΠΈΠ½Π΅ΠΉΠ½Ρ‹Ρ… элСмСнтов с нСсиммСтричной характСристикой зависят ΠΎΡ‚ направлСния Ρ‚ΠΎΠΊΠ°. НапримСр, элСктролампы, тСрмисторы ΠΈΠΌΠ΅ΡŽΡ‚ симмСтричныС Π²ΠΎΠ»ΡŒΡ‚Π°ΠΌΠΏΠ΅Ρ€Π½Ρ‹Π΅ характСристики (рис. 5.1), Π° ΠΏΠΎΠ»ΡƒΠΏΡ€ΠΎΠ²ΠΎΠ΄Π½ΠΈΠΊΠΎΠ²Ρ‹Π΅ Π΄ΠΈΠΎΠ΄Ρ‹ β€” нСсиммСтричныС характСристики (рис. 5.2).

БтатичСским ΠΈΠ»ΠΈ ΠΈΠ½Ρ‚Π΅Π³Ρ€Π°Π»ΡŒΠ½Ρ‹ΠΌ сопротивлСниСм Π½Π΅Π»ΠΈΠ½Π΅ΠΉΠ½ΠΎΠ³ΠΎ элСмСнта называСтся ΠΎΡ‚Π½ΠΎΡˆΠ΅Π½ΠΈΠ΅ напряТСния Π½Π° элСмСнтС ΠΊ Π²Π΅Π»ΠΈΡ‡ΠΈΠ½Π΅ Ρ‚ΠΎΠΊΠ°. Π­Ρ‚ΠΎ сопротивлСниС ΠΏΡ€ΠΎΠΏΠΎΡ€Ρ†ΠΈΠΎΠ½Π°Π»ΡŒΠ½ΠΎ тангСнсу ΡƒΠ³Π»Π° Π½Π°ΠΊΠ»ΠΎΠ½Π° Ξ± ΠΌΠ΅ΠΆΠ΄Ρƒ осью Ρ‚ΠΎΠΊΠ° ΠΈ прямой, ΠΏΡ€ΠΎΠ²Π΅Π΄Π΅Π½Π½ΠΎΠΉ ΠΈΠ· Π½Π°Ρ‡Π°Π»Π° ΠΊΠΎΠΎΡ€Π΄ΠΈΠ½Π°Ρ‚ Π² Ρ‚ΠΎΡ‡ΠΊΡƒ Π° характСристики (рис. 5.3)

.

Π”ΠΈΡ„Ρ„Π΅Ρ€Π΅Π½Ρ†ΠΈΠ°Π»ΡŒΠ½ΠΎΠ΅ ΠΈΠ»ΠΈ динамичСскоС сопротивлСниС Π½Π΅Π»ΠΈΠ½Π΅ΠΉΠ½ΠΎΠ³ΠΎ элСмСнта β€” это Π²Π΅Π»ΠΈΡ‡ΠΈΠ½Π°, равная ΠΎΡ‚Π½ΠΎΡˆΠ΅Π½ΠΈΡŽ бСсконСчно ΠΌΠ°Π»ΠΎΠ³ΠΎ приращСния напряТСния Π½Π° Π½Π΅Π»ΠΈΠ½Π΅ΠΉΠ½ΠΎΠΌ сопротивлСнии ΠΊ ΡΠΎΠΎΡ‚Π²Π΅Ρ‚ΡΡ‚Π²ΡƒΡŽΡ‰Π΅ΠΌΡƒ ΠΏΡ€ΠΈΡ€Π°Ρ‰Π΅Π½ΠΈΡŽ Ρ‚ΠΎΠΊΠ°.

Π­Ρ‚ΠΎ сопротивлСниС ΠΏΡ€ΠΎΠΏΠΎΡ€Ρ†ΠΈΠΎΠ½Π°Π»ΡŒΠ½ΠΎ тангСнсу ΡƒΠ³Π»Π° Π½Π°ΠΊΠ»ΠΎΠ½Π° Ξ² ΠΌΠ΅ΠΆΠ΄Ρƒ осью Ρ‚ΠΎΠΊΠ° ΠΈ ΠΊΠ°ΡΠ°Ρ‚Π΅Π»ΡŒΠ½ΠΎΠΉ ΠΊ Ρ‚ΠΎΡ‡ΠΊΠ΅ a характСристики (рис. 5.4).

.

ΠŸΡ€ΠΈ ΠΏΠ΅Ρ€Π΅Ρ…ΠΎΠ΄Π΅ ΠΎΡ‚ ΠΎΠ΄Π½ΠΎΠΉ Ρ‚ΠΎΡ‡ΠΊΠΈ Π²ΠΎΠ»ΡŒΡ‚Π°ΠΌΠΏΠ΅Ρ€Π½ΠΎΠΉ характСристики ΠΊ сосСднСй статичСскоС ΠΈ динамичСскоС сопротивлСния Π½Π΅Π»ΠΈΠ½Π΅ΠΉΠ½ΠΎΠ³ΠΎ элСмСнта ΠΌΠ΅Π½ΡΡŽΡ‚ΡΡ. БтатичСскоС ΠΈ динамичСскоС сопротивлСния Π»ΠΈΠ½Π΅ΠΉΠ½ΠΎΠ³ΠΎ элСмСнта ΠΎΠ΄ΠΈΠ½Π°ΠΊΠΎΠ²Ρ‹ ΠΈ Π½Π΅ зависят ΠΎΡ‚ Ρ‚ΠΎΠΊΠ° ΠΈΠ»ΠΈ напряТСния.

5.2. ГрафичСский ΠΌΠ΅Ρ‚ΠΎΠ΄ расчСта Π½Π΅Π»ΠΈΠ½Π΅ΠΉΠ½Ρ‹Ρ… Ρ†Π΅ΠΏΠ΅ΠΉ постоянного Ρ‚ΠΎΠΊΠ°

Π˜Π·Π²Π΅ΡΡ‚Π½Ρ‹Π΅ аналитичСскиС ΠΌΠ΅Ρ‚ΠΎΠ΄Ρ‹ Π½Π΅ΠΏΡ€ΠΈΠ³ΠΎΠ΄Π½Ρ‹ для расчСта Π½Π΅Π»ΠΈΠ½Π΅ΠΉΠ½Ρ‹Ρ… элСктричСских Ρ†Π΅ΠΏΠ΅ΠΉ, Ρ‚Π°ΠΊ ΠΊΠ°ΠΊ сопротивлСния Π½Π΅Π»ΠΈΠ½Π΅ΠΉΠ½Ρ‹Ρ… элСмСнтов зависят ΠΎΡ‚ направлСния ΠΈ значСния Ρ‚ΠΎΠΊΠ° ΠΈΠ»ΠΈ напряТСния. ΠŸΡ€ΠΈΠΌΠ΅Π½ΡΡŽΡ‚ΡΡ графоаналитичСскиС ΠΌΠ΅Ρ‚ΠΎΠ΄Ρ‹, основанныС Π½Π° ΠΏΡ€ΠΈΠΌΠ΅Π½Π΅Π½ΠΈΠΈ Π·Π°ΠΊΠΎΠ½ΠΎΠ² ΠšΠΈΡ€Ρ…Π³ΠΎΡ„Π° ΠΈ использовании Π·Π°Π΄Π°Π½Π½Ρ‹Ρ… Π²ΠΎΠ»ΡŒΡ‚Π°ΠΌΠΏΠ΅Ρ€Π½Ρ‹Ρ… характСристик (ВАΠ₯) этих элСмСнтов. Рассмотрим ΡΠ»Π΅ΠΊΡ‚Ρ€ΠΈΡ‡Π΅ΡΠΊΡƒΡŽ Ρ†Π΅ΠΏΡŒ, ΡΠΎΡΡ‚ΠΎΡΡ‰ΡƒΡŽ ΠΈΠ· Π΄Π²ΡƒΡ… ΠΏΠΎΡΠ»Π΅Π΄ΠΎΠ²Π°Ρ‚Π΅Π»ΡŒΠ½ΠΎ соСдинСнных Π½Π΅Π»ΠΈΠ½Π΅ΠΉΠ½Ρ‹Ρ… сопротивлСний Π½.с.1 ΠΈ Π½.с.2 (рис. 5.5). ВАΠ₯ 1 ΠΈ ВАΠ₯ 2 ΠΏΡ€ΠΈΠ²Π΅Π΄Π΅Π½Ρ‹ Π½Π° рис. 5.6.

К Ρ†Π΅ΠΏΠΈ ΠΏΠΎΠ΄Π²Π΅Π΄Π΅Π½ΠΎ напряТСниС U, ΠΈ ΠΎΠ½ΠΎ Ρ€Π°Π²Π½ΠΎ суммС ΠΏΠ°Π΄Π΅Π½ΠΈΠΉ напряТСний Π½Π° Π½.с.1 ΠΈ Π½.с.2:

(5.1)

По всСй Ρ†Π΅ΠΏΠΈ ΠΏΡ€ΠΎΡ‚Π΅ΠΊΠ°Π΅Ρ‚ ΠΎΠ΄ΠΈΠ½ ΠΈ Ρ‚ΠΎΡ‚ ΠΆΠ΅ Ρ‚ΠΎΠΊ I, Ρ‚Π°ΠΊ ΠΊΠ°ΠΊ Π½.с.1 ΠΈ Π½.с.2 соСдинСны ΠΌΠ΅ΠΆΠ΄Ρƒ собой ΠΏΠΎΡΠ»Π΅Π΄ΠΎΠ²Π°Ρ‚Π΅Π»ΡŒΠ½ΠΎ. Для опрСдСлСния Ρ‚ΠΎΠΊΠ° Π² элСктричСской Ρ†Π΅ΠΏΠΈ Π½ΡƒΠΆΠ½ΠΎ ΠΏΠΎΡΡ‚Ρ€ΠΎΠΈΡ‚ΡŒ Ρ€Π΅Π·ΡƒΠ»ΡŒΡ‚ΠΈΡ€ΡƒΡŽΡ‰ΡƒΡŽ ВАΠ₯ Ρ†Π΅ΠΏΠΈ. Для построСния этой характСристики слСдуСт ΡΡƒΠΌΠΌΠΈΡ€ΠΎΠ²Π°Ρ‚ΡŒ абсциссы ΠΊΡ€ΠΈΠ²Ρ‹Ρ… 1 ΠΈ 2 (Π°Π³ = Π°Π± + Π°Π²), ΡΠΎΠΎΡ‚Π²Π΅Ρ‚ΡΡ‚Π²ΡƒΡŽΡ‰ΠΈΠ΅ ΠΎΠ΄Π½ΠΈΠΌ ΠΈ Ρ‚Π΅ ΠΆΠ΅ значСниям Ρ‚ΠΎΠΊΠ°. Π”Π°Π»Π΅Π΅, задаваясь ΠΏΡ€ΠΎΠΈΠ·Π²ΠΎΠ»ΡŒΠ½Ρ‹ΠΌ Π·Π½Π°Ρ‡Π΅Π½ΠΈΠ΅ΠΌ Ρ‚ΠΎΠΊΠ° (Π½Π°ΠΏΡ€ΠΈΠΌΠ΅Ρ€, большС I’ ΠΈ мСньшС I’ ) ΠΌΠΎΠΆΠ½ΠΎ ΠΏΠΎΡΡ‚Ρ€ΠΎΠΈΡ‚ΡŒ ВАΠ₯ всСй Ρ†Π΅ΠΏΠΈ (рис. 5.6, кривая 3).

ΠŸΡ€ΠΈ ΠΏΠ°Ρ€Π°Π»Π»Π΅Π»ΡŒΠ½ΠΎΠΌ соСдинСнии Π΄Π²ΡƒΡ… Π½Π΅Π»ΠΈΠ½Π΅ΠΉΠ½Ρ‹Ρ… элСмСнтов (рис. 5.7) Ρ‚ΠΎΠΊ Π² Π½Π΅Ρ€Π°Π·Π²Π΅Ρ‚Π²Π»Π΅Π½Π½ΠΎΠΉ части элСктричСской Ρ†Π΅ΠΏΠΈ Ρ€Π°Π²Π΅Π½ суммС Ρ‚ΠΎΠΊΠΎΠ² Π² ΠΏΠ°Ρ€Π°Π»Π»Π΅Π»ΡŒΠ½Ρ‹Ρ… ΠΎΠΏΡ€Π΅Π΄Π΅Π»Π΅Π½Π½Ρ‹Ρ… вСтвях. ΠŸΠΎΡΡ‚ΠΎΠΌΡƒ ΠΏΡ€ΠΈ построСнии Ρ€Π΅Π·ΡƒΠ»ΡŒΡ‚ΠΈΡ€ΡƒΡŽΡ‰Π΅ΠΉ ВАΠ₯ всСй Ρ†Π΅ΠΏΠΈ слСдуСт ΡΡƒΠΌΠΌΠΈΡ€ΠΎΠ²Π°Ρ‚ΡŒ ΠΎΡ€Π΄ΠΈΠ½Π°Ρ‚Ρ‹ Π³Ρ€Π°Ρ„ΠΈΠΊΠΎΠ² 1 ΠΈ 2 (рис. 5.8), ΡΠΎΠΎΡ‚Π²Π΅Ρ‚ΡΡ‚Π²ΡƒΡŽΡ‰ΠΈΠ΅ ΠΎΠ΄Π½ΠΈΠΌ ΠΈ Ρ‚Π΅ ΠΆΠ΅ значСниям напряТСния, Ρ‚Π°ΠΊ ΠΊΠ°ΠΊ ΠΊ этим Π½Π΅Π»ΠΈΠ½Π΅ΠΉΠ½Ρ‹ΠΌ элСмСнтам ΠΏΡ€ΠΈΠ»ΠΎΠΆΠ΅Π½ΠΎ ΠΎΠ΄Π½ΠΎ ΠΈ Ρ‚ΠΎ ΠΆΠ΅ напряТСниС, Ρ€Π°Π²Π½ΠΎΠ΅ Π½Π°ΠΏΡ€ΡΠΆΠ΅Π½ΠΈΡŽ внСшнСй сСти, Ρ‚.Π΅. источника питания. НапримСр, для ΠΏΡ€ΠΎΠΈΠ·Π²ΠΎΠ»ΡŒΠ½ΠΎΠ³ΠΎ значСния напряТСния Π½Π°Ρ…ΠΎΠ΄ΠΈΠΌ ΠΎΡ€Π΄ΠΈΠ½Π°Ρ‚Ρƒ Π°Π³ Ρ‚ΠΎΡ‡ΠΊΠΈ для Ρ€Π΅Π·ΡƒΠ»ΡŒΡ‚ΠΈΡ€ΡƒΡŽΡ‰Π΅ΠΉ ΠΊΡ€ΠΈΠ²ΠΎΠΉ 3. (Π°Π³ = Π°Π² + Π°Π±)

Π”Π°Π»Π΅Π΅ задаваясь ΠΏΡ€ΠΎΠΈΠ·Π²ΠΎΠ»ΡŒΠ½Ρ‹ΠΌ Π·Π½Π°Ρ‡Π΅Π½ΠΈΠ΅ΠΌ напряТСния большС ΠΈ мСньшС U’, ΠΌΠΎΠΆΠ½ΠΎ ΠΏΠΎΡΡ‚Ρ€ΠΎΠΈΡ‚ΡŒ ВАΠ₯ всСй Ρ†Π΅ΠΏΠΈ (кривая 3). Π—Π°Ρ‚Π΅ΠΌ, ΠΏΠΎΠ»ΡŒΠ·ΡƒΡΡΡŒ ВАΠ₯, ΠΌΠΎΠΆΠ½ΠΎ ΠΏΡ€ΠΈ любом Π·Π½Π°Ρ‡Π΅Π½ΠΈΠΈ ΠΏΡ€ΠΈΠ»ΠΎΠΆΠ΅Π½Π½ΠΎΠ³ΠΎ напряТСния U (ΠΎΡ‚Ρ€Π΅Π·ΠΎΠΊ ΠΎΡ€) Π½Π°ΠΉΡ‚ΠΈ Π²Π΅Π»ΠΈΡ‡ΠΈΠ½Ρƒ ΠΎΠ±Ρ‰Π΅Π³ΠΎ Ρ‚ΠΎΠΊΠ° I (pn = oΠΊ). Π­Ρ‚ΠΎ напряТСниС Ρ‚Π°ΠΊΠΆΠ΅ опрСдСляСт значСния Ρ‚ΠΎΠΊΠΎΠ² I1 ΠΈ I2 Π² ΠΎΡ‚Π΄Π΅Π»ΡŒΠ½Ρ‹Ρ… вСтвях с ΡƒΡ‡Π΅Ρ‚ΠΎΠΌ ΠΌΠ°ΡΡˆΡ‚Π°Π±Π° Ρ‚ΠΎΠΊΠ° mI.

Π§Ρ‚ΠΎ Π΅Ρ‰Ρ‘ Π½ΡƒΠΆΠ½ΠΎ ΡƒΡ‡ΠΈΡ‚Ρ‹Π²Π°Ρ‚ΡŒ ΠΏΡ€ΠΈ ΠΏΠΎΠ΄ΠΊΠ»ΡŽΡ‡Π΅Π½ΠΈΠΈ рСзисторов

Π’Π°ΠΆΠ½Ρ‹ΠΉ ΠΏΠΎΠΊΠ°Π·Π°Ρ‚Π΅Π»ΡŒ Π² Ρ€Π°Π±ΠΎΡ‚Π΅ рСзистивного элСмСнта ΠΌΠΎΡ‰Π½ΠΎΡΡ‚ΡŒ рассСивания – ΠΏΠ΅Ρ€Π΅Ρ…ΠΎΠ΄ элСктричСской энСргии Π² Ρ‚Π΅ΠΏΠ»ΠΎΠ²ΡƒΡŽ, Π²Ρ‹Π·Ρ‹Π²Π°ΡŽΡ‰ΡƒΡŽ Π½Π°Π³Ρ€Π΅Π² элСмСнта.

ΠŸΡ€ΠΈ ΠΏΡ€Π΅Π²Ρ‹ΡˆΠ΅Π½ΠΈΠΈ допустимой мощности рассСивания рСзисторы Π±ΡƒΠ΄ΡƒΡ‚ сильно Π³Ρ€Π΅Ρ‚ΡŒΡΡ ΠΈ ΠΌΠΎΠ³ΡƒΡ‚ ΡΠ³ΠΎΡ€Π΅Ρ‚ΡŒ, поэтому ΠΏΡ€ΠΈ расчСтС схСм соСдинСния Π½Π°Π΄ΠΎ ΡƒΡ‡ΠΈΡ‚Ρ‹Π²Π°Ρ‚ΡŒ этот ΠΏΠ°Ρ€Π°ΠΌΠ΅Ρ‚Ρ€ – Π²Π°ΠΆΠ½ΠΎ Π·Π½Π°Ρ‚ΡŒ насколько измСнится ΠΌΠΎΡ‰Π½ΠΎΡΡ‚ΡŒ рСзистивных элСмСнтов ΠΏΡ€ΠΈ Π²ΠΊΠ»ΡŽΡ‡Π΅Π½ΠΈΠΈ Π² ΡΠ»Π΅ΠΊΡ‚Ρ€ΠΈΡ‡Π΅ΡΠΊΡƒΡŽ Ρ†Π΅ΠΏΡŒ.

Как ΠΏΡ€Π°Π²ΠΈΠ»ΡŒΠ½ΠΎ Ρ€Π°ΡΡΡ‡ΠΈΡ‚Π°Ρ‚ΡŒ сопротивлСниС

ΠŸΡ€ΠΈΠΌΠ΅Π½ΡΠ΅Ρ‚ΡΡ Π·Π°ΠΊΠΎΠ½ Ома для участка Ρ†Π΅ΠΏΠΈ – расчСт сопротивлСния дСлаСтся ΠΏΠΎ Ρ„ΠΎΡ€ΠΌΡƒΠ»Π΅ R = U/I, Π³Π΄Π΅

  • U – ΠΏΠ°Π΄Π΅Π½ΠΈΠ΅ напряТСниС Π½Π° ΠΊΠΎΠ½ΠΊΡ€Π΅Ρ‚Π½ΠΎΠΌ рСзистивном элСмСнтС;
  • I – Ρ‚ΠΎΠΊ, ΠΏΡ€ΠΎΡ‚Π΅ΠΊΠ°ΡŽΡ‰ΠΈΠΉ Ρ‡Π΅Ρ€Π΅Π· Π½Π΅Π³ΠΎ.

ΠŸΡ€ΠΈ ΠΏΠΎΡΠ»Π΅Π΄ΠΎΠ²Π°Ρ‚Π΅Π»ΡŒΠ½ΠΎΠΌ соСдинСнии

Для Π΄Π²ΡƒΡ… элСмСнтов считаСм RΠΎΠ±Ρ‰ = R1+R2.

Для Π½Π΅ΡΠΊΠΎΠ»ΡŒΠΊΠΈΡ… сопротивлСний Ρ€Π°Π·Π½ΠΎΠ³ΠΎ Π½ΠΎΠΌΠΈΠ½Π°Π»Π° RΠΎΠ±Ρ‰ = R1+R2+R3+…+Rn.

ΠŸΡ€ΠΈ ΠΏΠ°Ρ€Π°Π»Π»Π΅Π»ΡŒΠ½ΠΎΠΌ соСдинСнии

РасчСт для Π΄Π²ΡƒΡ… рСзисторов Π΄Π΅Π»Π°Π΅ΠΌ ΠΏΠΎ Ρ„ΠΎΡ€ΠΌΡƒΠ»Π΅ RΠΎΠ±Ρ‰ = (R1Γ—R2)/(R1+R2).

Π‘ΠΎΠΏΡ€ΠΎΡ‚ΠΈΠ²Π»Π΅Π½ΠΈΠ΅ ΠΏΠ°Ρ€Π°Π»Π»Π΅Π»ΡŒΠ½Ρ‹Ρ… рСзисторов с Ρ€Π°Π·Π½Ρ‹ΠΌ Π½ΠΎΠΌΠΈΠ½Π°Π»ΠΎΠΌ рассчитываСм ΠΏΠΎ Ρ„ΠΎΡ€ΠΌΡƒΠ»Π΅

RΠΎΠ±Ρ‰ = 1/(1/R1+1/R2+1/R3+…+1/Rn).

Для элСмСнтов, соСдинСнных Π² ΠΏΠ°Ρ€Π°Π»Π»Π΅Π»ΡŒ, суммарноС сопротивлСниС всСгда Π½ΠΈΠΆΠ΅ наимСньшСго номинального.

ΠŸΠΎΡΠ»Π΅Π΄ΠΎΠ²Π°Ρ‚Π΅Π»ΡŒΠ½ΠΎΠ΅ соСдинСниС сопротивлСний Π² Ρ†Π΅ΠΏΠΈ постоянного Ρ‚ΠΎΠΊΠ°. ΠŸΠΎΡΠ»Π΅Π΄ΠΎΠ²Π°Ρ‚Π΅Π»ΡŒΠ½ΠΎΠ΅ ΠΈ ΠΏΠ°Ρ€Π°Π»Π»Π΅Π»ΡŒΠ½ΠΎΠ΅ соСдинСния ΠΏΡ€ΠΎΠ²ΠΎΠ΄Π½ΠΈΠΊΠΎΠ²

ΠŸΠΎΡΠ»Π΅Π΄ΠΎΠ²Π°Ρ‚Π΅Π»ΡŒΠ½ΠΎΠ΅, ΠΏΠ°Ρ€Π°Π»Π»Π΅Π»ΡŒΠ½ΠΎΠ΅ ΠΈ смСшанноС соСдинСния рСзисторов. Π—Π½Π°Ρ‡ΠΈΡ‚Π΅Π»ΡŒΠ½ΠΎΠ΅ число ΠΏΡ€ΠΈΠ΅ΠΌΠ½ΠΈΠΊΠΎΠ², Π²ΠΊΠ»ΡŽΡ‡Π΅Π½Π½Ρ‹Ρ… Π² ΡΠ»Π΅ΠΊΡ‚Ρ€ΠΈΡ‡Π΅ΡΠΊΡƒΡŽ Ρ†Π΅ΠΏΡŒ (элСктричСскиС Π»Π°ΠΌΠΏΡ‹, ΡΠ»Π΅ΠΊΡ‚Ρ€ΠΎΠ½Π°Π³Ρ€Π΅Π²Π°Ρ‚Π΅Π»ΡŒΠ½Ρ‹Π΅ ΠΏΡ€ΠΈΠ±ΠΎΡ€Ρ‹ ΠΈ Π΄Ρ€.), ΠΌΠΎΠΆΠ½ΠΎ Ρ€Π°ΡΡΠΌΠ°Ρ‚Ρ€ΠΈΠ²Π°Ρ‚ΡŒ ΠΊΠ°ΠΊ Π½Π΅ΠΊΠΎΡ‚ΠΎΡ€Ρ‹Π΅ элСмСнты, ΠΈΠΌΠ΅ΡŽΡ‰ΠΈΠ΅ ΠΎΠΏΡ€Π΅Π΄Π΅Π»Π΅Π½Π½ΠΎΠ΅ сопротивлСниС. Π­Ρ‚ΠΎ ΠΎΠ±ΡΡ‚ΠΎΡΡ‚Π΅Π»ΡŒΡΡ‚Π²ΠΎ Π΄Π°Π΅Ρ‚ Π½Π°ΠΌ Π²ΠΎΠ·ΠΌΠΎΠΆΠ½ΠΎΡΡ‚ΡŒ ΠΏΡ€ΠΈ составлСнии ΠΈ ΠΈΠ·ΡƒΡ‡Π΅Π½ΠΈΠΈ элСктричСских схСм Π·Π°ΠΌΠ΅Π½ΡΡ‚ΡŒ ΠΊΠΎΠ½ΠΊΡ€Π΅Ρ‚Π½Ρ‹Π΅ ΠΏΡ€ΠΈΠ΅ΠΌΠ½ΠΈΠΊΠΈ рСзисторами с ΠΎΠΏΡ€Π΅Π΄Π΅Π»Π΅Π½Π½Ρ‹ΠΌΠΈ сопротивлСниями. Π Π°Π·Π»ΠΈΡ‡Π°ΡŽΡ‚ ΡΠ»Π΅Π΄ΡƒΡŽΡ‰ΠΈΠ΅ способы соСдинСния рСзисторов (ΠΏΡ€ΠΈΠ΅ΠΌΠ½ΠΈΠΊΠΎΠ² элСктричСской энСргии): ΠΏΠΎΡΠ»Π΅Π΄ΠΎΠ²Π°Ρ‚Π΅Π»ΡŒΠ½ΠΎΠ΅, ΠΏΠ°Ρ€Π°Π»Π»Π΅Π»ΡŒΠ½ΠΎΠ΅ ΠΈ смСшанноС.

ΠŸΠΎΡΠ»Π΅Π΄ΠΎΠ²Π°Ρ‚Π΅Π»ΡŒΠ½ΠΎΠ΅ соСдинСниС рСзисторов . ΠŸΡ€ΠΈ ΠΏΠΎΡΠ»Π΅Π΄ΠΎΠ²Π°Ρ‚Π΅Π»ΡŒΠ½ΠΎΠΌ соСдинСнии Π½Π΅ΡΠΊΠΎΠ»ΡŒΠΊΠΈΡ… рСзисторов ΠΊΠΎΠ½Π΅Ρ† ΠΏΠ΅Ρ€Π²ΠΎΠ³ΠΎ рСзистора ΡΠΎΠ΅Π΄ΠΈΠ½ΡΡŽΡ‚ с Π½Π°Ρ‡Π°Π»ΠΎΠΌ Π²Ρ‚ΠΎΡ€ΠΎΠ³ΠΎ, ΠΊΠΎΠ½Π΅Ρ† Π²Ρ‚ΠΎΡ€ΠΎΠ³ΠΎ — с Π½Π°Ρ‡Π°Π»ΠΎΠΌ Ρ‚Ρ€Π΅Ρ‚ΡŒΠ΅Π³ΠΎ ΠΈ Ρ‚. Π΄. ΠŸΡ€ΠΈ Ρ‚Π°ΠΊΠΎΠΌ соСдинСнии ΠΏΠΎ всСм элСмСнтам ΠΏΠΎΡΠ»Π΅Π΄ΠΎΠ²Π°Ρ‚Π΅Π»ΡŒΠ½ΠΎΠΉ Ρ†Π΅ΠΏΠΈ ΠΏΡ€ΠΎΡ…ΠΎΠ΄ΠΈΡ‚
ΠΎΠ΄ΠΈΠ½ ΠΈ Ρ‚ΠΎΡ‚ ΠΆΠ΅ Ρ‚ΠΎΠΊ I.
ΠŸΠΎΡΠ»Π΅Π΄ΠΎΠ²Π°Ρ‚Π΅Π»ΡŒΠ½ΠΎΠ΅ соСдинСниС ΠΏΡ€ΠΈΠ΅ΠΌΠ½ΠΈΠΊΠΎΠ² поясняСт рис. 25, Π°.
.ЗамСняя Π»Π°ΠΌΠΏΡ‹ рСзисторами с сопротивлСниями R1, R2 ΠΈ R3, ΠΏΠΎΠ»ΡƒΡ‡ΠΈΠΌ схСму, ΠΏΠΎΠΊΠ°Π·Π°Π½Π½ΡƒΡŽ Π½Π° рис. 25, Π±.
Если ΠΏΡ€ΠΈΠ½ΡΡ‚ΡŒ, Ρ‡Ρ‚ΠΎ Π² источникС Ro = 0, Ρ‚ΠΎ для Ρ‚Ρ€Π΅Ρ… ΠΏΠΎΡΠ»Π΅Π΄ΠΎΠ²Π°Ρ‚Π΅Π»ΡŒΠ½ΠΎ соСдинСнных рСзисторов согласно Π²Ρ‚ΠΎΡ€ΠΎΠΌΡƒ Π·Π°ΠΊΠΎΠ½Ρƒ ΠšΠΈΡ€Ρ…Π³ΠΎΡ„Π° ΠΌΠΎΠΆΠ½ΠΎ Π½Π°ΠΏΠΈΡΠ°Ρ‚ΡŒ:

E = IR 1 + IR 2 + IR 3 = I(R 1 + R 2 + R 3) = IR эк (19)

гдС R эк = R 1 + R 2 + R 3 .
Π‘Π»Π΅Π΄ΠΎΠ²Π°Ρ‚Π΅Π»ΡŒΠ½ΠΎ, эквивалСнтноС сопротивлСниС ΠΏΠΎΡΠ»Π΅Π΄ΠΎΠ²Π°Ρ‚Π΅Π»ΡŒΠ½ΠΎΠΉ Ρ†Π΅ΠΏΠΈ Ρ€Π°Π²Π½ΠΎ суммС сопротивлСний всСх ΠΏΠΎΡΠ»Π΅Π΄ΠΎΠ²Π°Ρ‚Π΅Π»ΡŒΠ½ΠΎ соСдинСнных рСзисторов.Π’Π°ΠΊ ΠΊΠ°ΠΊ напряТСния Π½Π° ΠΎΡ‚Π΄Π΅Π»ΡŒΠ½Ρ‹Ρ… участках Ρ†Π΅ΠΏΠΈ согласно Π·Π°ΠΊΠΎΠ½Ρƒ Ома: U 1 =IR 1 ; U 2 = IR 2 , U 3 = IR Π· ΠΈ Π² Π΄Π°Π½Π½ΠΎΠΌ случаС E = U, Ρ‚ΠΎ длярассматриваСмой Ρ†Π΅ΠΏΠΈ

U = U 1 + U 2 +U 3 (20)

Π‘Π»Π΅Π΄ΠΎΠ²Π°Ρ‚Π΅Π»ΡŒΠ½ΠΎ, напряТСниС U Π½Π° Π·Π°ΠΆΠΈΠΌΠ°Ρ… источника Ρ€Π°Π²Π½ΠΎ суммС напряТСний Π½Π° ΠΊΠ°ΠΆΠ΄ΠΎΠΌ ΠΈΠ· ΠΏΠΎΡΠ»Π΅Π΄ΠΎΠ²Π°Ρ‚Π΅Π»ΡŒΠ½ΠΎ Π²ΠΊΠ»ΡŽΡ‡Π΅Π½Π½Ρ‹Ρ… рСзисторов.
Из ΡƒΠΊΠ°Π·Π°Π½Π½Ρ‹Ρ… Ρ„ΠΎΡ€ΠΌΡƒΠ» слСдуСт Ρ‚Π°ΠΊΠΆΠ΅, Ρ‡Ρ‚ΠΎ напряТСния Ρ€Π°ΡΠΏΡ€Π΅Π΄Π΅Π»ΡΡŽΡ‚ΡΡ ΠΌΠ΅ΠΆΠ΄Ρƒ ΠΏΠΎΡΠ»Π΅Π΄ΠΎΠ²Π°Ρ‚Π΅Π»ΡŒΠ½ΠΎ соСдинСнными рСзисторами ΠΏΡ€ΠΎΠΏΠΎΡ€Ρ†ΠΈΠΎΠ½Π°Π»ΡŒΠ½ΠΎ ΠΈΡ… сопротивлСниям:

U 1: U 2: U 3 = R 1: R 2: R 3 (21)

Ρ‚. Π΅. Ρ‡Π΅ΠΌ большС сопротивлСниС ΠΊΠ°ΠΊΠΎΠ³ΠΎ-Π»ΠΈΠ±ΠΎ ΠΏΡ€ΠΈΠ΅ΠΌΠ½ΠΈΠΊΠ° Π² ΠΏΠΎΡΠ»Π΅Π΄ΠΎΠ²Π°Ρ‚Π΅Π»ΡŒΠ½ΠΎΠΉ Ρ†Π΅ΠΏΠΈ, Ρ‚Π΅ΠΌ большС ΠΏΡ€ΠΈΠ»ΠΎΠΆΠ΅Π½Π½ΠΎΠ΅ ΠΊ Π½Π΅ΠΌΡƒ напряТСниС.

Π’ случаС Ссли ΠΏΠΎΡΠ»Π΅Π΄ΠΎΠ²Π°Ρ‚Π΅Π»ΡŒΠ½ΠΎ ΡΠΎΠ΅Π΄ΠΈΠ½ΡΡŽΡ‚ΡΡ нСсколько, Π½Π°ΠΏΡ€ΠΈΠΌΠ΅Ρ€ ΠΏ, рСзисторов с ΠΎΠ΄ΠΈΠ½Π°ΠΊΠΎΠ²Ρ‹ΠΌ сопротивлСниСм R1, эквивалСнтноС сопротивлСниС Ρ†Π΅ΠΏΠΈ Rэк Π±ΡƒΠ΄Π΅Ρ‚ Π² ΠΏ Ρ€Π°Π· большС сопротивлСния R1, Ρ‚. Π΅. Rэк = nR1. НапряТСниС U1 Π½Π° ΠΊΠ°ΠΆΠ΄ΠΎΠΌ рСзисторС Π² этом случаС Π² ΠΏ Ρ€Π°Π· мСньшС ΠΎΠ±Ρ‰Π΅Π³ΠΎ напряТСния U:

ΠŸΡ€ΠΈ ΠΏΠΎΡΠ»Π΅Π΄ΠΎΠ²Π°Ρ‚Π΅Π»ΡŒΠ½ΠΎΠΌ соСдинСнии ΠΏΡ€ΠΈΠ΅ΠΌΠ½ΠΈΠΊΠΎΠ² ΠΈΠ·ΠΌΠ΅Π½Π΅Π½ΠΈΠ΅ сопротивлСния ΠΎΠ΄Π½ΠΎΠ³ΠΎ ΠΈΠ· Π½ΠΈΡ… тотчас ΠΆΠ΅ Π²Π»Π΅Ρ‡Π΅Ρ‚ Π·Π° собой ΠΈΠ·ΠΌΠ΅Π½Π΅Π½ΠΈΠ΅ напряТСния Π½Π° Π΄Ρ€ΡƒΠ³ΠΈΡ… связанных с Π½ΠΈΠΌ ΠΏΡ€ΠΈΠ΅ΠΌΠ½ΠΈΠΊΠ°Ρ…. ΠŸΡ€ΠΈ Π²Ρ‹ΠΊΠ»ΡŽΡ‡Π΅Π½ΠΈΠΈ ΠΈΠ»ΠΈ ΠΎΠ±Ρ€Ρ‹Π²Π΅ элСктричСской Ρ†Π΅ΠΏΠΈ Π² ΠΎΠ΄Π½ΠΎΠΌ ΠΈΠ· ΠΏΡ€ΠΈΠ΅ΠΌΠ½ΠΈΠΊΠΎΠ² ΠΈ Π² ΠΎΡΡ‚Π°Π»ΡŒΠ½Ρ‹Ρ… ΠΏΡ€ΠΈΠ΅ΠΌΠ½ΠΈΠΊΠ°Ρ… прСкращаСтся Ρ‚ΠΎΠΊ. ΠŸΠΎΡΡ‚ΠΎΠΌΡƒ ΠΏΠΎΡΠ»Π΅Π΄ΠΎΠ²Π°Ρ‚Π΅Π»ΡŒΠ½ΠΎΠ΅ соСдинСниС ΠΏΡ€ΠΈΠ΅ΠΌΠ½ΠΈΠΊΠΎΠ² ΠΏΡ€ΠΈΠΌΠ΅Π½ΡΡŽΡ‚ Ρ€Π΅Π΄ΠΊΠΎ — Ρ‚ΠΎΠ»ΡŒΠΊΠΎ Π² Ρ‚ΠΎΠΌ случаС, ΠΊΠΎΠ³Π΄Π° напряТСниС источника элСктричСской энСргии большС номинального напряТСния, Π½Π° ΠΊΠΎΡ‚ΠΎΡ€ΠΎΠ΅ рассчитан ΠΏΠΎΡ‚Ρ€Π΅Π±ΠΈΡ‚Π΅Π»ΡŒ. НапримСр, напряТСниС Π² элСктричСской сСти, ΠΎΡ‚ ΠΊΠΎΡ‚ΠΎΡ€ΠΎΠΉ ΠΏΠΈΡ‚Π°ΡŽΡ‚ΡΡ Π²Π°Π³ΠΎΠ½Ρ‹ ΠΌΠ΅Ρ‚Ρ€ΠΎΠΏΠΎΠ»ΠΈΡ‚Π΅Π½Π°, составляСт 825 Π’, номинальноС ΠΆΠ΅ напряТСниС элСктричСских Π»Π°ΠΌΠΏ, примСняСмых Π² этих Π²Π°Π³ΠΎΠ½Π°Ρ…, 55 Π’. ΠŸΠΎΡΡ‚ΠΎΠΌΡƒ Π² Π²Π°Π³ΠΎΠ½Π°Ρ… ΠΌΠ΅Ρ‚Ρ€ΠΎΠΏΠΎΠ»ΠΈΡ‚Π΅Π½Π° элСктричСскиС Π»Π°ΠΌΠΏΡ‹ Π²ΠΊΠ»ΡŽΡ‡Π°ΡŽΡ‚ ΠΏΠΎΡΠ»Π΅Π΄ΠΎΠ²Π°Ρ‚Π΅Π»ΡŒΠ½ΠΎ ΠΏΠΎ 15 Π»Π°ΠΌΠΏ Π² ΠΊΠ°ΠΆΠ΄ΠΎΠΉ Ρ†Π΅ΠΏΠΈ.
ΠŸΠ°Ρ€Π°Π»Π»Π΅Π»ΡŒΠ½ΠΎΠ΅ соСдинСниС рСзисторов . ΠŸΡ€ΠΈ ΠΏΠ°Ρ€Π°Π»Π»Π΅Π»ΡŒΠ½ΠΎΠΌ соСдинСнии Π½Π΅ΡΠΊΠΎΠ»ΡŒΠΊΠΈΡ… ΠΏΡ€ΠΈΠ΅ΠΌΠ½ΠΈΠΊΠΎΠ² ΠΎΠ½ΠΈ Π²ΠΊΠ»ΡŽΡ‡Π°ΡŽΡ‚ΡΡ ΠΌΠ΅ΠΆΠ΄Ρƒ двумя Ρ‚ΠΎΡ‡ΠΊΠ°ΠΌΠΈ элСктричСской Ρ†Π΅ΠΏΠΈ, образуя ΠΏΠ°Ρ€Π°Π»Π»Π΅Π»ΡŒΠ½Ρ‹Π΅ Π²Π΅Ρ‚Π²ΠΈ (рис. 26, Π°). ЗамСняя

Π»Π°ΠΌΠΏΡ‹ рСзисторами с сопротивлСниями R1, R2, R3, ΠΏΠΎΠ»ΡƒΡ‡ΠΈΠΌ схСму, ΠΏΠΎΠΊΠ°Π·Π°Π½Π½ΡƒΡŽ Π½Π° рис. 26, Π±.
ΠŸΡ€ΠΈ ΠΏΠ°Ρ€Π°Π»Π»Π΅Π»ΡŒΠ½ΠΎΠΌ соСдинСнии ΠΊΠΎ всСм рСзисторам ΠΏΡ€ΠΈΠ»ΠΎΠΆΠ΅Π½ΠΎ ΠΎΠ΄ΠΈΠ½Π°ΠΊΠΎΠ²ΠΎΠ΅ напряТСниС U. ΠŸΠΎΡΡ‚ΠΎΠΌΡƒ согласно Π·Π°ΠΊΠΎΠ½Ρƒ Ома:

I 1 =U/R 1 ; I 2 =U/R 2 ; I 3 =U/R 3 .

Π’ΠΎΠΊ Π² Π½Π΅Ρ€Π°Π·Π²Π΅Ρ‚Π²Π»Π΅Π½Π½ΠΎΠΉ части Ρ†Π΅ΠΏΠΈ согласно ΠΏΠ΅Ρ€Π²ΠΎΠΌΡƒ Π·Π°ΠΊΠΎΠ½Ρƒ ΠšΠΈΡ€Ρ…Π³ΠΎΡ„Π° I = I 1 +I 2 +I 3 , ΠΈΠ»ΠΈ

I = U / R 1 + U / R 2 + U / R 3 = U (1/R 1 + 1/R 2 + 1/R 3) = U / R эк (23)

Π‘Π»Π΅Π΄ΠΎΠ²Π°Ρ‚Π΅Π»ΡŒΠ½ΠΎ, эквивалСнтноС сопротивлСниС рассматриваСмой Ρ†Π΅ΠΏΠΈ ΠΏΡ€ΠΈ ΠΏΠ°Ρ€Π°Π»Π»Π΅Π»ΡŒΠ½ΠΎΠΌ соСдинСнии Ρ‚Ρ€Π΅Ρ… рСзисторов опрСдСляСтся Ρ„ΠΎΡ€ΠΌΡƒΠ»ΠΎΠΉ

1/R эк = 1/R 1 + 1/R 2 + 1/R 3 (24)

Вводя Π² Ρ„ΠΎΡ€ΠΌΡƒΠ»Ρƒ (24) вмСсто Π·Π½Π°Ρ‡Π΅Π½ΠΈΠΉ 1/R эк, 1/R 1 , 1/R 2 ΠΈ 1/R 3 ΡΠΎΠΎΡ‚Π²Π΅Ρ‚ΡΡ‚Π²ΡƒΡŽΡ‰ΠΈΠ΅ проводимости G эк, G 1 , G 2 ΠΈ G 3 , ΠΏΠΎΠ»ΡƒΡ‡ΠΈΠΌ: эквивалСнтная ΠΏΡ€ΠΎΠ²ΠΎΠ΄ΠΈΠΌΠΎΡΡ‚ΡŒ ΠΏΠ°Ρ€Π°Π»Π»Π΅Π»ΡŒΠ½ΠΎΠΉ Ρ†Π΅ΠΏΠΈ Ρ€Π°Π²Π½Π° суммС проводимостСй ΠΏΠ°Ρ€Π°Π»Π»Π΅Π»ΡŒΠ½ΠΎ соСдинСнных рСзисторов :

G эк = G 1 + G 2 +G 3 (25)

Π’Π°ΠΊΠΈΠΌ ΠΎΠ±Ρ€Π°Π·ΠΎΠΌ, ΠΏΡ€ΠΈ ΡƒΠ²Π΅Π»ΠΈΡ‡Π΅Π½ΠΈΠΈ числа ΠΏΠ°Ρ€Π°Π»Π»Π΅Π»ΡŒΠ½ΠΎ Π²ΠΊΠ»ΡŽΡ‡Π°Π΅ΠΌΡ‹Ρ… рСзисторов Ρ€Π΅Π·ΡƒΠ»ΡŒΡ‚ΠΈΡ€ΡƒΡŽΡ‰Π°Ρ ΠΏΡ€ΠΎΠ²ΠΎΠ΄ΠΈΠΌΠΎΡΡ‚ΡŒ элСктричСской Ρ†Π΅ΠΏΠΈ увСличиваСтся, Π° Ρ€Π΅Π·ΡƒΠ»ΡŒΡ‚ΠΈΡ€ΡƒΡŽΡ‰Π΅Π΅ сопротивлСниС ΡƒΠΌΠ΅Π½ΡŒΡˆΠ°Π΅Ρ‚ΡΡ.
Из ΠΏΡ€ΠΈΠ²Π΅Π΄Π΅Π½Π½Ρ‹Ρ… Ρ„ΠΎΡ€ΠΌΡƒΠ» слСдуСт, Ρ‡Ρ‚ΠΎ Ρ‚ΠΎΠΊΠΈ Ρ€Π°ΡΠΏΡ€Π΅Π΄Π΅Π»ΡΡŽΡ‚ΡΡ ΠΌΠ΅ΠΆΠ΄Ρƒ ΠΏΠ°Ρ€Π°Π»Π»Π΅Π»ΡŒΠ½Ρ‹ΠΌΠΈ вСтвями ΠΎΠ±Ρ€Π°Ρ‚Π½ΠΎ ΠΏΡ€ΠΎΠΏΠΎΡ€Ρ†ΠΈΠΎΠ½Π°Π»ΡŒΠ½ΠΎ ΠΈΡ… элСктричСским сопротивлСниям ΠΈΠ»ΠΈ прямо ΠΏΡ€ΠΎΠΏΠΎΡ€Ρ†ΠΈΠΎΠ½Π°Π»ΡŒΠ½ΠΎ ΠΈΡ… проводимостям. НапримСр, ΠΏΡ€ΠΈ Ρ‚Ρ€Π΅Ρ… вСтвях

I 1: I 2: I 3 = 1/R 1: 1/R 2: 1/R 3 = G 1 + G 2 + G 3 (26)

Π’ этом ΠΎΡ‚Π½ΠΎΡˆΠ΅Π½ΠΈΠΈ ΠΈΠΌΠ΅Π΅Ρ‚ мСсто полная аналогия ΠΌΠ΅ΠΆΠ΄Ρƒ распрСдСлСниСм Ρ‚ΠΎΠΊΠΎΠ² ΠΏΠΎ ΠΎΡ‚Π΄Π΅Π»ΡŒΠ½Ρ‹ΠΌ вСтвям ΠΈ распрСдСлСниСм ΠΏΠΎΡ‚ΠΎΠΊΠΎΠ² Π²ΠΎΠ΄Ρ‹ ΠΏΠΎ Ρ‚Ρ€ΡƒΠ±Π°ΠΌ.
ΠŸΡ€ΠΈΠ²Π΅Π΄Π΅Π½Π½Ρ‹Π΅ Ρ„ΠΎΡ€ΠΌΡƒΠ»Ρ‹ Π΄Π°ΡŽΡ‚ Π²ΠΎΠ·ΠΌΠΎΠΆΠ½ΠΎΡΡ‚ΡŒ ΠΎΠΏΡ€Π΅Π΄Π΅Π»ΠΈΡ‚ΡŒ эквивалСнтноС сопротивлСниС Ρ†Π΅ΠΏΠΈ для Ρ€Π°Π·Π»ΠΈΡ‡Π½Ρ‹Ρ… ΠΊΠΎΠ½ΠΊΡ€Π΅Ρ‚Π½Ρ‹Ρ… случаСв. НапримСр, ΠΏΡ€ΠΈ Π΄Π²ΡƒΡ… ΠΏΠ°Ρ€Π°Π»Π»Π΅Π»ΡŒΠ½ΠΎ Π²ΠΊΠ»ΡŽΡ‡Π΅Π½Π½Ρ‹Ρ… рСзисторах Ρ€Π΅Π·ΡƒΠ»ΡŒΡ‚ΠΈΡ€ΡƒΡŽΡ‰Π΅Π΅ сопротивлСниС Ρ†Π΅ΠΏΠΈ

R эк =R 1 R 2 /(R 1 +R 2)

ΠΏΡ€ΠΈ Ρ‚Ρ€Π΅Ρ… ΠΏΠ°Ρ€Π°Π»Π»Π΅Π»ΡŒΠ½ΠΎ Π²ΠΊΠ»ΡŽΡ‡Π΅Π½Π½Ρ‹Ρ… рСзисторах

R эк =R 1 R 2 R 3 /(R 1 R 2 +R 2 R 3 +R 1 R 3)

ΠŸΡ€ΠΈ ΠΏΠ°Ρ€Π°Π»Π»Π΅Π»ΡŒΠ½ΠΎΠΌ соСдинСнии Π½Π΅ΡΠΊΠΎΠ»ΡŒΠΊΠΈΡ…, Π½Π°ΠΏΡ€ΠΈΠΌΠ΅Ρ€ n, рСзисторов с ΠΎΠ΄ΠΈΠ½Π°ΠΊΠΎΠ²Ρ‹ΠΌ сопротивлСниСм R1 Ρ€Π΅Π·ΡƒΠ»ΡŒΡ‚ΠΈΡ€ΡƒΡŽΡ‰Π΅Π΅ сопротивлСниС Ρ†Π΅ΠΏΠΈ Rэк Π±ΡƒΠ΄Π΅Ρ‚ Π² n Ρ€Π°Π· мСньшС сопротивлСния R1, Ρ‚.Π΅.

R эк = R1 / n (27)

ΠŸΡ€ΠΎΡ…ΠΎΠ΄ΡΡ‰ΠΈΠΉ ΠΏΠΎ ΠΊΠ°ΠΆΠ΄ΠΎΠΉ Π²Π΅Ρ‚Π²ΠΈ Ρ‚ΠΎΠΊ I1, Π² этом случаС Π±ΡƒΠ΄Π΅Ρ‚ Π² ΠΏ Ρ€Π°Π· мСньшС ΠΎΠ±Ρ‰Π΅Π³ΠΎ Ρ‚ΠΎΠΊΠ°:

I1 = I / n (28)

ΠŸΡ€ΠΈ ΠΏΠ°Ρ€Π°Π»Π»Π΅Π»ΡŒΠ½ΠΎΠΌ соСдинСнии ΠΏΡ€ΠΈΠ΅ΠΌΠ½ΠΈΠΊΠΎΠ², всС ΠΎΠ½ΠΈ находятся ΠΏΠΎΠ΄ ΠΎΠ΄Π½ΠΈΠΌ ΠΈ Ρ‚Π΅ΠΌ ΠΆΠ΅ напряТСниСм, ΠΈ Ρ€Π΅ΠΆΠΈΠΌ Ρ€Π°Π±ΠΎΡ‚Ρ‹ ΠΊΠ°ΠΆΠ΄ΠΎΠ³ΠΎ ΠΈΠ· Π½ΠΈΡ… Π½Π΅ зависит ΠΎΡ‚ ΠΎΡΡ‚Π°Π»ΡŒΠ½Ρ‹Ρ…. Π­Ρ‚ΠΎ ΠΎΠ·Π½Π°Ρ‡Π°Π΅Ρ‚, Ρ‡Ρ‚ΠΎ Ρ‚ΠΎΠΊ, проходящий ΠΏΠΎ ΠΊΠ°ΠΊΠΎΠΌΡƒ-Π»ΠΈΠ±ΠΎ ΠΈΠ· ΠΏΡ€ΠΈΠ΅ΠΌΠ½ΠΈΠΊΠΎΠ², Π½Π΅ Π±ΡƒΠ΄Π΅Ρ‚ ΠΎΠΊΠ°Π·Ρ‹Π²Π°Ρ‚ΡŒ сущСствСнного влияния Π½Π° Π΄Ρ€ΡƒΠ³ΠΈΠ΅ ΠΏΡ€ΠΈΠ΅ΠΌΠ½ΠΈΠΊΠΈ. ΠŸΡ€ΠΈ всяком Π²Ρ‹ΠΊΠ»ΡŽΡ‡Π΅Π½ΠΈΠΈ ΠΈΠ»ΠΈ Π²Ρ‹Ρ…ΠΎΠ΄Π΅ ΠΈΠ· строя любого ΠΏΡ€ΠΈΠ΅ΠΌΠ½ΠΈΠΊΠ° ΠΎΡΡ‚Π°Π»ΡŒΠ½Ρ‹Π΅ ΠΏΡ€ΠΈΠ΅ΠΌΠ½ΠΈΠΊΠΈ ΠΎΡΡ‚Π°ΡŽΡ‚ΡΡ вклю-

Ρ‡Π΅Π½Π½Ρ‹ΠΌΠΈ. ΠŸΠΎΡΡ‚ΠΎΠΌΡƒ ΠΏΠ°Ρ€Π°Π»Π»Π΅Π»ΡŒΠ½ΠΎΠ΅ соСдинСниС ΠΈΠΌΠ΅Π΅Ρ‚ сущСствСнныС прСимущСства ΠΏΠ΅Ρ€Π΅Π΄ ΠΏΠΎΡΠ»Π΅Π΄ΠΎΠ²Π°Ρ‚Π΅Π»ΡŒΠ½Ρ‹ΠΌ, вслСдствиС Ρ‡Π΅Π³ΠΎ ΠΎΠ½ΠΎ ΠΏΠΎΠ»ΡƒΡ‡ΠΈΠ»ΠΎ Π½Π°ΠΈΠ±ΠΎΠ»Π΅Π΅ ΡˆΠΈΡ€ΠΎΠΊΠΎΠ΅ распространСниС. Π’ частности, элСктричСскиС Π»Π°ΠΌΠΏΡ‹ ΠΈ Π΄Π²ΠΈΠ³Π°Ρ‚Π΅Π»ΠΈ, ΠΏΡ€Π΅Π΄Π½Π°Π·Π½Π°Ρ‡Π΅Π½Π½Ρ‹Π΅ для Ρ€Π°Π±ΠΎΡ‚Ρ‹ ΠΏΡ€ΠΈ ΠΎΠΏΡ€Π΅Π΄Π΅Π»Π΅Π½Π½ΠΎΠΌ (номинальном) напряТСнии, всСгда Π²ΠΊΠ»ΡŽΡ‡Π°ΡŽΡ‚ ΠΏΠ°Ρ€Π°Π»Π»Π΅Π»ΡŒΠ½ΠΎ.
На элСктровозах постоянного Ρ‚ΠΎΠΊΠ° ΠΈ Π½Π΅ΠΊΠΎΡ‚ΠΎΡ€Ρ‹Ρ… Ρ‚Π΅ΠΏΠ»ΠΎΠ²ΠΎΠ·Π°Ρ… тяговыС Π΄Π²ΠΈΠ³Π°Ρ‚Π΅Π»ΠΈ Π² процСссС рСгулирования скорости двиТСния Π½ΡƒΠΆΠ½ΠΎ Π²ΠΊΠ»ΡŽΡ‡Π°Ρ‚ΡŒ ΠΏΠΎΠ΄ Ρ€Π°Π·Π»ΠΈΡ‡Π½Ρ‹Π΅ напряТСния, поэтому ΠΎΠ½ΠΈ Π² процСссС Ρ€Π°Π·Π³ΠΎΠ½Π° ΠΏΠ΅Ρ€Π΅ΠΊΠ»ΡŽΡ‡Π°ΡŽΡ‚ΡΡ с ΠΏΠΎΡΠ»Π΅Π΄ΠΎΠ²Π°Ρ‚Π΅Π»ΡŒΠ½ΠΎΠ³ΠΎ соСдинСния Π½Π° ΠΏΠ°Ρ€Π°Π»Π»Π΅Π»ΡŒΠ½ΠΎΠ΅.

БмСшанноС соСдинСниС рСзисторов . Π‘ΠΌΠ΅ΡˆΠ°Π½Π½Ρ‹ΠΌ соСдинСниСм называСтся Ρ‚Π°ΠΊΠΎΠ΅ соСдинСниС, ΠΏΡ€ΠΈ ΠΊΠΎΡ‚ΠΎΡ€ΠΎΠΌ Ρ‡Π°ΡΡ‚ΡŒ рСзисторов Π²ΠΊΠ»ΡŽΡ‡Π°Π΅Ρ‚ΡΡ ΠΏΠΎΡΠ»Π΅Π΄ΠΎΠ²Π°Ρ‚Π΅Π»ΡŒΠ½ΠΎ, Π° Ρ‡Π°ΡΡ‚ΡŒ — ΠΏΠ°Ρ€Π°Π»Π»Π΅Π»ΡŒΠ½ΠΎ. НапримСр, Π² схСмС рис. 27, Π° ΠΈΠΌΠ΅ΡŽΡ‚ΡΡ Π΄Π²Π° ΠΏΠΎΡΠ»Π΅Π΄ΠΎΠ²Π°Ρ‚Π΅Π»ΡŒΠ½ΠΎ Π²ΠΊΠ»ΡŽΡ‡Π΅Π½Π½Ρ‹Ρ… рСзистора сопротивлСниями R1 ΠΈ R2, ΠΏΠ°Ρ€Π°Π»Π»Π΅Π»ΡŒΠ½ΠΎ ΠΈΠΌ Π²ΠΊΠ»ΡŽΡ‡Π΅Π½ рСзистор сопротивлСниСм RΠ·, Π° рСзистор сопротивлСниСм R4 Π²ΠΊΠ»ΡŽΡ‡Π΅Π½ ΠΏΠΎΡΠ»Π΅Π΄ΠΎΠ²Π°Ρ‚Π΅Π»ΡŒΠ½ΠΎ с Π³Ρ€ΡƒΠΏΠΏΠΎΠΉ рСзисторов сопротивлСниями R1, R2 ΠΈ R3.
Π­ΠΊΠ²ΠΈΠ²Π°Π»Π΅Π½Ρ‚Π½ΠΎΠ΅ сопротивлСниС Ρ†Π΅ΠΏΠΈ ΠΏΡ€ΠΈ смСшанном соСдинСнии ΠΎΠ±Ρ‹Ρ‡Π½ΠΎ ΠΎΠΏΡ€Π΅Π΄Π΅Π»ΡΡŽΡ‚ ΠΌΠ΅Ρ‚ΠΎΠ΄ΠΎΠΌ прСобразования, ΠΏΡ€ΠΈ ΠΊΠΎΡ‚ΠΎΡ€ΠΎΠΌ ΡΠ»ΠΎΠΆΠ½ΡƒΡŽ Ρ†Π΅ΠΏΡŒ ΠΏΠΎΡΠ»Π΅Π΄ΠΎΠ²Π°Ρ‚Π΅Π»ΡŒΠ½Ρ‹ΠΌΠΈ этапами ΠΏΡ€Π΅ΠΎΠ±Ρ€Π°Π·ΠΎΠ²Ρ‹Π²Π°ΡŽΡ‚ Π² ΠΏΡ€ΠΎΡΡ‚Π΅ΠΉΡˆΡƒΡŽ. НапримСр, для схСмы рис. 27, Π° Π²Π½Π°Ρ‡Π°Π»Π΅ ΠΎΠΏΡ€Π΅Π΄Π΅Π»ΡΡŽΡ‚ эквивалСнтноС сопротивлСниС R12 ΠΏΠΎΡΠ»Π΅Π΄ΠΎΠ²Π°Ρ‚Π΅Π»ΡŒΠ½ΠΎ Π²ΠΊΠ»ΡŽΡ‡Π΅Π½Π½Ρ‹Ρ… рСзисторов с сопротивлСниями R1 ΠΈ R2: R12 = R1 + R2. ΠŸΡ€ΠΈ этом схСма рис. 27, Π° замСняСтся эквивалСнтной схСмой рис. 27, Π±. Π—Π°Ρ‚Π΅ΠΌ ΠΎΠΏΡ€Π΅Π΄Π΅Π»ΡΡŽΡ‚ эквивалСнтноС сопротивлСниС R123 ΠΏΠ°Ρ€Π°Π»Π»Π΅Π»ΡŒΠ½ΠΎ Π²ΠΊΠ»ΡŽΡ‡Π΅Π½Π½Ρ‹Ρ… сопротивлСний ΠΈ R3 ΠΏΠΎ Ρ„ΠΎΡ€ΠΌΡƒΠ»Π΅

R 123 =R 12 R 3 /(R 12 +R 3)=(R 1 +R 2)R 3 /(R 1 +R 2 +R 3).

ΠŸΡ€ΠΈ этом схСма рис. 27, Π± замСняСтся эквивалСнтной схСмой рис. 27, Π². ПослС этого находят эквивалСнтноС сопротивлСниС всСй Ρ†Π΅ΠΏΠΈ суммированиСм сопротивлСния R123 ΠΈ ΠΏΠΎΡΠ»Π΅Π΄ΠΎΠ²Π°Ρ‚Π΅Π»ΡŒΠ½ΠΎ Π²ΠΊΠ»ΡŽΡ‡Π΅Π½Π½ΠΎΠ³ΠΎ с Π½ΠΈΠΌ сопротивлСния R4:

R эк = R 123 + R 4 = (R 1 + R 2) R 3 / (R 1 + R 2 + R 3) + R 4

ΠŸΠΎΡΠ»Π΅Π΄ΠΎΠ²Π°Ρ‚Π΅Π»ΡŒΠ½ΠΎΠ΅, ΠΏΠ°Ρ€Π°Π»Π»Π΅Π»ΡŒΠ½ΠΎΠ΅ ΠΈ смСшанноС соСдинСния ΡˆΠΈΡ€ΠΎΠΊΠΎ ΠΏΡ€ΠΈΠΌΠ΅Π½ΡΡŽΡ‚ для измСнСния сопротивлСния пусковых рСостатов ΠΏΡ€ΠΈ пускС э. ΠΏ. с. постоянного Ρ‚ΠΎΠΊΠ°.

ВсСм Π΄ΠΎΠ±Ρ€ΠΎΠ³ΠΎ Π²Ρ€Π΅ΠΌΠ΅Π½ΠΈ суток. Π’ ΠΏΡ€ΠΎΡˆΠ»ΠΎΠΉ ΡΡ‚Π°Ρ‚ΡŒΠ΅ я рассмотрСл , ΠΏΡ€ΠΈΠΌΠ΅Π½ΠΈΡ‚Π΅Π»ΡŒΠ½ΠΎ ΠΊ элСктричСским цСпям, содСрТащиС источники энСргии. Но Π² основС Π°Π½Π°Π»ΠΈΠ·Π° ΠΈ проСктирования элСктронных схСм вмСстС с Π·Π°ΠΊΠΎΠ½ΠΎΠΌ Ома Π»Π΅ΠΆΠ°Ρ‚ Ρ‚Π°ΠΊΠΆΠ΅ Π·Π°ΠΊΠΎΠ½Ρ‹ баланса , Π½Π°Π·Ρ‹Π²Π°Π΅ΠΌΡ‹ΠΌ ΠΏΠ΅Ρ€Π²Ρ‹ΠΌ Π·Π°ΠΊΠΎΠ½ΠΎΠΌ ΠšΠΈΡ€Ρ…Π³ΠΎΡ„Π°, ΠΈ баланса напряТСния Π½Π° участках Ρ†Π΅ΠΏΠΈ, Π½Π°Π·Ρ‹Π²Π°Π΅ΠΌΡ‹ΠΌ Π²Ρ‚ΠΎΡ€Ρ‹ΠΌ Π·Π°ΠΊΠΎΠ½ΠΎΠΌ ΠšΠΈΡ€Ρ…Π³ΠΎΡ„Π°, ΠΊΠΎΡ‚ΠΎΡ€Ρ‹Π΅ рассмотрим Π² Π΄Π°Π½Π½ΠΎΠΉ ΡΡ‚Π°Ρ‚ΡŒΠ΅. Но для Π½Π°Ρ‡Π°Π»Π° выясним, ΠΊΠ°ΠΊ ΡΠΎΠ΅Π΄ΠΈΠ½ΡΡŽΡ‚ΡΡ ΠΌΠ΅ΠΆΠ΄Ρƒ собой ΠΏΡ€ΠΈΡ‘ΠΌΠ½ΠΈΠΊΠΈ энСргии ΠΈ ΠΊΠ°ΠΊΠΈΠ΅ ΠΏΡ€ΠΈ этом Π²Π·Π°ΠΈΠΌΠΎΠΎΡ‚Π½ΠΎΡˆΠ΅Π½ΠΈΡ ΠΌΠ΅ΠΆΠ΄Ρƒ Ρ‚ΠΎΠΊΠ°ΠΌΠΈ, напряТСниями ΠΈ .

ΠŸΡ€ΠΈΠ΅ΠΌΠ½ΠΈΠΊΠΈ элСктричСской энСргии ΠΌΠΎΠΆΠ½ΠΎ ΡΠΎΠ΅Π΄ΠΈΠ½ΠΈΡ‚ΡŒ ΠΌΠ΅ΠΆΠ΄Ρƒ собой трСмя Ρ€Π°Π·Π»ΠΈΡ‡Π½Ρ‹ΠΌΠΈ способами: ΠΏΠΎΡΠ»Π΅Π΄ΠΎΠ²Π°Ρ‚Π΅Π»ΡŒΠ½ΠΎ, ΠΏΠ°Ρ€Π°Π»Π»Π΅Π»ΡŒΠ½ΠΎ ΠΈΠ»ΠΈ смСшано (ΠΏΠΎΡΠ»Π΅Π΄ΠΎΠ²Π°Ρ‚Π΅Π»ΡŒΠ½ΠΎ β€” ΠΏΠ°Ρ€Π°Π»Π»Π΅Π»ΡŒΠ½ΠΎ). Π’Π½Π°Ρ‡Π°Π»Π΅ рассмотрим ΠΏΠΎΡΠ»Π΅Π΄ΠΎΠ²Π°Ρ‚Π΅Π»ΡŒΠ½Ρ‹ΠΉ способ соСдинСния, ΠΏΡ€ΠΈ ΠΊΠΎΡ‚ΠΎΡ€ΠΎΠΌ ΠΊΠΎΠ½Π΅Ρ† ΠΎΠ΄Π½ΠΎΠ³ΠΎ ΠΏΡ€ΠΈΠ΅ΠΌΠ½ΠΈΠΊΠ° ΡΠΎΠ΅Π΄ΠΈΠ½ΡΡŽΡ‚ с Π½Π°Ρ‡Π°Π»ΠΎΠΌ Π²Ρ‚ΠΎΡ€ΠΎΠ³ΠΎ ΠΏΡ€ΠΈΠ΅ΠΌΠ½ΠΈΠΊΠ°, Π° ΠΊΠΎΠ½Π΅Ρ† Π²Ρ‚ΠΎΡ€ΠΎΠ³ΠΎ ΠΏΡ€ΠΈΠ΅ΠΌΠ½ΠΈΠΊΠ° – с Π½Π°Ρ‡Π°Π»ΠΎΠΌ Ρ‚Ρ€Π΅Ρ‚ΡŒΠ΅Π³ΠΎ ΠΈ Ρ‚Π°ΠΊ Π΄Π°Π»Π΅Π΅. На рисункС Π½ΠΈΠΆΠ΅ ΠΏΠΎΠΊΠ°Π·Π°Π½ΠΎ ΠΏΠΎΡΠ»Π΅Π΄ΠΎΠ²Π°Ρ‚Π΅Π»ΡŒΠ½ΠΎΠ΅ соСдинСниС ΠΏΡ€ΠΈΠ΅ΠΌΠ½ΠΈΠΊΠΎΠ² энСргии с ΠΈΡ… ΠΏΠΎΠ΄ΠΊΠ»ΡŽΡ‡Π΅Π½ΠΈΠ΅ΠΌ ΠΊ источнику энСргии

ΠŸΡ€ΠΈΠΌΠ΅Ρ€ ΠΏΠΎΡΠ»Π΅Π΄ΠΎΠ²Π°Ρ‚Π΅Π»ΡŒΠ½ΠΎΠ³ΠΎ ΠΏΠΎΠ΄ΠΊΠ»ΡŽΡ‡Π΅Π½ΠΈΡ ΠΏΡ€ΠΈΠ΅ΠΌΠ½ΠΈΠΊΠΎΠ² энСргии.

Π’ Π΄Π°Π½Π½ΠΎΠΌ случаС Ρ†Π΅ΠΏΡŒ состоит ΠΈΠ· Ρ‚Ρ€Ρ‘Ρ… ΠΏΠΎΡΠ»Π΅Π΄ΠΎΠ²Π°Ρ‚Π΅Π»ΡŒΠ½Ρ‹Ρ… ΠΏΡ€ΠΈΠ΅ΠΌΠ½ΠΈΠΊΠΎΠ² энСргии с сопротивлСниСм R1, R2, R3 подсоСдинСнных ΠΊ источнику энСргии с U. Π§Π΅Ρ€Π΅Π· Ρ†Π΅ΠΏΡŒ ΠΏΡ€ΠΎΡ‚Π΅ΠΊΠ°Π΅Ρ‚ элСктричСский Ρ‚ΠΎΠΊ силой I, Ρ‚ΠΎ Π΅ΡΡ‚ΡŒ, напряТСниС Π½Π° ΠΊΠ°ΠΆΠ΄ΠΎΠΌ сопротивлСнии Π±ΡƒΠ΄Π΅Ρ‚ Ρ€Π°Π²Π½ΡΡ‚ΡŒΡΡ ΠΏΡ€ΠΎΠΈΠ·Π²Π΅Π΄Π΅Π½ΠΈΡŽ силы Ρ‚ΠΎΠΊΠ° ΠΈ сопротивлСния

Π’Π°ΠΊΠΈΠΌ ΠΎΠ±Ρ€Π°Π·ΠΎΠΌ, ΠΏΠ°Π΄Π΅Π½ΠΈΠ΅ напряТСния Π½Π° ΠΏΠΎΡΠ»Π΅Π΄ΠΎΠ²Π°Ρ‚Π΅Π»ΡŒΠ½ΠΎ соСдинённых сопротивлСниях ΠΏΡ€ΠΎΠΏΠΎΡ€Ρ†ΠΈΠΎΠ½Π°Π»ΡŒΠ½Ρ‹ Π²Π΅Π»ΠΈΡ‡ΠΈΠ½Π°ΠΌ этих сопротивлСний.

Из Π²Ρ‹ΡˆΠ΅ΡΠΊΠ°Π·Π°Π½Π½ΠΎΠ³ΠΎ Π²Ρ‹Ρ‚Π΅ΠΊΠ°Π΅Ρ‚ ΠΏΡ€Π°Π²ΠΈΠ»ΠΎ эквивалСнтного ΠΏΠΎΡΠ»Π΅Π΄ΠΎΠ²Π°Ρ‚Π΅Π»ΡŒΠ½ΠΎΠ³ΠΎ сопротивлСния, ΠΊΠΎΡ‚ΠΎΡ€ΠΎΠ΅ гласит, Ρ‡Ρ‚ΠΎ ΠΏΠΎΡΠ»Π΅Π΄ΠΎΠ²Π°Ρ‚Π΅Π»ΡŒΠ½ΠΎ соСдинённыС сопротивлСния ΠΌΠΎΠΆΠ½ΠΎ ΠΏΡ€Π΅Π΄ΡΡ‚Π°Π²ΠΈΡ‚ΡŒ эквивалСнтным ΠΏΠΎΡΠ»Π΅Π΄ΠΎΠ²Π°Ρ‚Π΅Π»ΡŒΠ½Ρ‹ΠΌ сопротивлСниСм Π²Π΅Π»ΠΈΡ‡ΠΈΠ½Π°, ΠΊΠΎΡ‚ΠΎΡ€ΠΎΠ³ΠΎ Ρ€Π°Π²Π½Π° суммС ΠΏΠΎΡΠ»Π΅Π΄ΠΎΠ²Π°Ρ‚Π΅Π»ΡŒΠ½ΠΎ соСдинённых сопротивлСний. Π­Ρ‚ΠΎ Π·Π°Π²ΠΈΡΠΈΠΌΠΎΡΡ‚ΡŒ прСдставлСна ΡΠ»Π΅Π΄ΡƒΡŽΡ‰ΠΈΠΌΠΈ ΡΠΎΠΎΡ‚Π½ΠΎΡˆΠ΅Π½ΠΈΡΠΌΠΈ

Π³Π΄Π΅ R – эквивалСнтноС ΠΏΠΎΡΠ»Π΅Π΄ΠΎΠ²Π°Ρ‚Π΅Π»ΡŒΠ½ΠΎΠ΅ сопротивлСниС.

ΠŸΡ€ΠΈΠΌΠ΅Π½Π΅Π½ΠΈΠ΅ ΠΏΠΎΡΠ»Π΅Π΄ΠΎΠ²Π°Ρ‚Π΅Π»ΡŒΠ½ΠΎΠ³ΠΎ соСдинСния

ΠžΡΠ½ΠΎΠ²Π½Ρ‹ΠΌ Π½Π°Π·Π½Π°Ρ‡Π΅Π½ΠΈΠ΅ΠΌ ΠΏΠΎΡΠ»Π΅Π΄ΠΎΠ²Π°Ρ‚Π΅Π»ΡŒΠ½ΠΎΠ³ΠΎ соСдинСния ΠΏΡ€ΠΈΠ΅ΠΌΠ½ΠΈΠΊΠΎΠ² энСргии являСтся обСспСчСниС Ρ‚Ρ€Π΅Π±ΡƒΠ΅ΠΌΠΎΠ³ΠΎ напряТСния мСньшС, Ρ‡Π΅ΠΌ напряТСниС источника энСргии. Одними ΠΈΠ· Ρ‚Π°ΠΊΠΈΡ… ΠΏΡ€ΠΈΠΌΠ΅Π½Π΅Π½ΠΈΠΉ являСтся Π΄Π΅Π»ΠΈΡ‚Π΅Π»ΡŒ напряТСния ΠΈ ΠΏΠΎΡ‚Π΅Π½Ρ†ΠΈΠΎΠΌΠ΅Ρ‚Ρ€


Π”Π΅Π»ΠΈΡ‚Π΅Π»ΡŒ напряТСния (слСва) ΠΈ ΠΏΠΎΡ‚Π΅Π½Ρ†ΠΈΠΎΠΌΠ΅Ρ‚Ρ€ (справа).

Π’ качСствС Π΄Π΅Π»ΠΈΡ‚Π΅Π»Π΅ΠΉ напряТСния ΠΈΡΠΏΠΎΠ»ΡŒΠ·ΡƒΡŽΡ‚ ΠΏΠΎΡΠ»Π΅Π΄ΠΎΠ²Π°Ρ‚Π΅Π»ΡŒΠ½ΠΎ соСдинённыС рСзисторы, Π² Π΄Π°Π½Π½ΠΎΠΌ случаС R1 ΠΈ R2, ΠΊΠΎΡ‚ΠΎΡ€Ρ‹Π΅ дСлят напряТСниС источника энСргии Π½Π° Π΄Π²Π΅ части U1 ΠΈ U2. НапряТСния U1 ΠΈ U2 ΠΌΠΎΠΆΠ½ΠΎ ΠΈΡΠΏΠΎΠ»ΡŒΠ·ΠΎΠ²Π°Ρ‚ΡŒ для Ρ€Π°Π±ΠΎΡ‚Ρ‹ Ρ€Π°Π·Π½Ρ‹Ρ… ΠΏΡ€ΠΈΠ΅ΠΌΠ½ΠΈΠΊΠΎΠ² энСргии.

Π”ΠΎΠ²ΠΎΠ»ΡŒΠ½ΠΎ часто ΠΈΡΠΏΠΎΠ»ΡŒΠ·ΡƒΡŽΡ‚ Ρ€Π΅Π³ΡƒΠ»ΠΈΡ€ΡƒΠ΅ΠΌΡ‹ΠΉ Π΄Π΅Π»ΠΈΡ‚Π΅Π»ΡŒ напряТСния, Π² качСствС ΠΊΠΎΡ‚ΠΎΡ€ΠΎΠ³ΠΎ ΠΏΡ€ΠΈΠΌΠ΅Π½ΡΡŽΡ‚ ΠΏΠ΅Ρ€Π΅ΠΌΠ΅Π½Π½Ρ‹ΠΉ рСзистор R. Π‘ΡƒΠΌΠΌΠ°Ρ€Π½ΠΎΠ΅ сопротивлСниС, ΠΊΠΎΡ‚ΠΎΡ€ΠΎΠ³ΠΎ дСлится Π½Π° Π΄Π²Π΅ части с ΠΏΠΎΠΌΠΎΡ‰ΡŒΡŽ ΠΏΠΎΠ΄Π²ΠΈΠΆΠ½ΠΎΠ³ΠΎ ΠΊΠΎΠ½Ρ‚Π°ΠΊΡ‚Π°, ΠΈ Ρ‚Π°ΠΊΠΈΠΌ ΠΎΠ±Ρ€Π°Π·ΠΎΠΌ ΠΌΠΎΠΆΠ½ΠΎ ΠΏΠ»Π°Π²Π½ΠΎ ΠΈΠ·ΠΌΠ΅Π½ΡΡ‚ΡŒ напряТСниС U2 Π½Π° ΠΏΡ€ΠΈΠ΅ΠΌΠ½ΠΈΠΊΠ΅ энСргии.

Π•Ρ‰Ρ‘ ΠΎΠ΄Π½ΠΈΠΌ способом соСдинСния ΠΏΡ€ΠΈΠ΅ΠΌΠ½ΠΈΠΊΠΎΠ² элСктричСской энСргии являСтся ΠΏΠ°Ρ€Π°Π»Π»Π΅Π»ΡŒΠ½ΠΎΠ΅ соСдинСниС, ΠΊΠΎΡ‚ΠΎΡ€ΠΎΠ΅ характСризуСтся Ρ‚Π΅ΠΌ, Ρ‡Ρ‚ΠΎ ΠΊ ΠΎΠ΄Π½ΠΈΠΌ ΠΈ Ρ‚Π΅ΠΌ ΠΆΠ΅ ΡƒΠ·Π»Π°ΠΌ элСктричСской Ρ†Π΅ΠΏΠΈ присоСдинСны нСсколько ΠΏΡ€Π΅Π΅ΠΌΠ½ΠΈΠΊΠΎΠ² энСргии. ΠŸΡ€ΠΈΠΌΠ΅Ρ€ Ρ‚Π°ΠΊΠΎΠ³ΠΎ соСдинСния ΠΏΠΎΠΊΠ°Π·Π°Π½ Π½Π° рисункС Π½ΠΈΠΆΠ΅


ΠŸΡ€ΠΈΠΌΠ΅Ρ€ ΠΏΠ°Ρ€Π°Π»Π»Π΅Π»ΡŒΠ½ΠΎΠ³ΠΎ соСдинСния ΠΏΡ€ΠΈΠ΅ΠΌΠ½ΠΈΠΊΠΎΠ² энСргии.

ЭлСктричСская Ρ†Π΅ΠΏΡŒ Π½Π° рисункС состоит ΠΈΠ· Ρ‚Ρ€Ρ‘Ρ… ΠΏΠ°Ρ€Π°Π»Π»Π΅Π»ΡŒΠ½Ρ‹Ρ… Π²Π΅Ρ‚Π²Π΅ΠΉ с сопротивлСниями Π½Π°Π³Ρ€ΡƒΠ·ΠΊΠΈ R1, R2 ΠΈ R3. ЦСпь ΠΏΠΎΠ΄ΠΊΠ»ΡŽΡ‡Π΅Π½Π° ΠΊ источнику энСргии с напряТСниСм U, Ρ‡Π΅Ρ€Π΅Π· Ρ†Π΅ΠΏΡŒ ΠΏΡ€ΠΎΡ‚Π΅ΠΊΠ°Π΅Ρ‚ элСктричСский Ρ‚ΠΎΠΊ с силой I. Π’Π°ΠΊΠΈΠΌ ΠΎΠ±Ρ€Π°Π·ΠΎΠΌ, Ρ‡Π΅Ρ€Π΅Π· ΠΊΠ°ΠΆΠ΄ΡƒΡŽ Π²Π΅Ρ‚Π²ΡŒ ΠΏΡ€ΠΎΡ‚Π΅ΠΊΠ°Π΅Ρ‚ Ρ‚ΠΎΠΊ Ρ€Π°Π²Π½Ρ‹ΠΉ ΠΎΡ‚Π½ΠΎΡˆΠ΅Π½ΠΈΡŽ напряТСния ΠΊ ΡΠΎΠΏΡ€ΠΎΡ‚ΠΈΠ²Π»Π΅Π½ΠΈΡŽ ΠΊΠ°ΠΆΠ΄ΠΎΠΉ Π²Π΅Ρ‚Π²ΠΈ

Π’Π°ΠΊ ΠΊΠ°ΠΊ всС Π²Π΅Ρ‚Π²ΠΈ Ρ†Π΅ΠΏΠΈ находятся ΠΏΠΎΠ΄ ΠΎΠ΄Π½ΠΈΠΌ напряТСниСм U, Ρ‚ΠΎ Ρ‚ΠΎΠΊΠΈ ΠΏΡ€ΠΈΠ΅ΠΌΠ½ΠΈΠΊΠΎΠ² энСргии ΠΎΠ±Ρ€Π°Ρ‚Π½ΠΎ ΠΏΡ€ΠΎΠΏΠΎΡ€Ρ†ΠΈΠΎΠ½Π°Π»ΡŒΠ½Ρ‹ сопротивлСниям этих ΠΏΡ€ΠΈΠ΅ΠΌΠ½ΠΈΠΊΠΎΠ², Π° ΡΠ»Π΅Π΄ΠΎΠ²Π°Ρ‚Π΅Π»ΡŒΠ½ΠΎ ΠΏΠ°Ρ€Π°Π»Π»Π΅Π»ΡŒΠ½ΠΎ соСдинённыС ΠΏΡ€ΠΈΠ΅ΠΌΠ½ΠΈΠΊΠΈ энСргии ΠΌΠΎΠΆΠ½ΠΎ Π·Π°ΠΌΠ΅Ρ‚ΡŒ ΠΎΠ΄Π½ΠΈΠΌ ΠΏΡ€ΠΈΠ΅ΠΌΠ½ΠΈΠΊΠΎΠΌ энСргии с ΡΠΎΠΎΡ‚Π²Π΅Ρ‚ΡΡ‚Π²ΡƒΡŽΡ‰ΠΈΠΌ эквивалСнтным сопротивлСниСм, согласно ΡΠ»Π΅Π΄ΡƒΡŽΡ‰ΠΈΡ… Π²Ρ‹Ρ€Π°ΠΆΠ΅Π½ΠΈΠΉ

Π’Π°ΠΊΠΈΠΌ ΠΎΠ±Ρ€Π°Π·ΠΎΠΌ, ΠΏΡ€ΠΈ ΠΏΠ°Ρ€Π°Π»Π»Π΅Π»ΡŒΠ½ΠΎΠΌ соСдинСнии эквивалСнтноС сопротивлСниС всСгда мСньшС самого ΠΌΠ°Π»ΠΎΠ³ΠΎ ΠΈΠ· ΠΏΠ°Ρ€Π°Π»Π»Π΅Π»ΡŒΠ½ΠΎ Π²ΠΊΠ»ΡŽΡ‡Π΅Π½Π½Ρ‹Ρ… сопротивлСний.

БмСшанноС соСдинСниС ΠΏΡ€ΠΈΠ΅ΠΌΠ½ΠΈΠΊΠΎΠ² энСргии

НаиболСС ΡˆΠΈΡ€ΠΎΠΊΠΎ распространСно смСшанноС соСдинСниС ΠΏΡ€ΠΈΠ΅ΠΌΠ½ΠΈΠΊΠΎΠ² элСктричСской энСргии. Π”Π°Π½Π½ΠΎΠΉ соСдинСниС прСдставляСт собой сочСтаниС ΠΏΠΎΡΠ»Π΅Π΄ΠΎΠ²Π°Ρ‚Π΅Π»ΡŒΠ½ΠΎ ΠΈ ΠΏΠ°Ρ€Π°Π»Π»Π΅Π»ΡŒΠ½ΠΎ соСдинСнных элСмСнтов. ΠžΠ±Ρ‰Π΅ΠΉ Ρ„ΠΎΡ€ΠΌΡƒΠ»Ρ‹ для расчёта Π΄Π°Π½Π½ΠΎΠ³ΠΎ Π²ΠΈΠ΄Π° соСдинСний Π½Π΅ сущСствуСт, поэтому Π² ΠΊΠ°ΠΆΠ΄ΠΎΠΌ ΠΎΡ‚Π΄Π΅Π»ΡŒΠ½ΠΎΠΌ случаС Π½Π΅ΠΎΠ±Ρ…ΠΎΠ΄ΠΈΠΌΠΎ Π²Ρ‹Π΄Π΅Π»ΡΡ‚ΡŒ участки Ρ†Π΅ΠΏΠΈ, Π³Π΄Π΅ присутствуСт Ρ‚ΠΎΠ»ΡŒΠΊΠΎ лишь ΠΎΠ΄ΠΈΠ½ Π²ΠΈΠ΄ соСдинСния ΠΏΡ€ΠΈΠ΅ΠΌΠ½ΠΈΠΊΠΎΠ² – ΠΏΠΎΡΠ»Π΅Π΄ΠΎΠ²Π°Ρ‚Π΅Π»ΡŒΠ½ΠΎΠ΅ ΠΈΠ»ΠΈ ΠΏΠ°Ρ€Π°Π»Π»Π΅Π»ΡŒΠ½ΠΎΠ΅. Π—Π°Ρ‚Π΅ΠΌ ΠΏΠΎ Ρ„ΠΎΡ€ΠΌΡƒΠ»Π°ΠΌ эквивалСнтных сопротивлСний постСпСнно ΡƒΠΏΡ€ΠΎΡ‰Π°Ρ‚ΡŒ Π΄Π°Π½Π½Ρ‹Π΅ участи ΠΈ Π² ΠΊΠΎΠ½Π΅Ρ‡Π½ΠΎΠΌ ΠΈΡ‚ΠΎΠ³Π΅ ΠΏΡ€ΠΈΠ²ΠΎΠ΄ΠΈΡ‚ΡŒ ΠΈΡ… ΠΊ ΠΏΡ€ΠΎΡΡ‚Π΅ΠΉΡˆΠ΅ΠΌΡƒ Π²ΠΈΠ΄Ρƒ с ΠΎΠ΄Π½ΠΈΠΌ сопротивлСниСм, ΠΏΡ€ΠΈ этом Ρ‚ΠΎΠΊΠΈ ΠΈ напряТСния Π²Ρ‹Ρ‡ΠΈΡΠ»ΡΡ‚ΡŒ ΠΏΠΎ Π·Π°ΠΊΠΎΠ½Ρƒ Ома. На рисункС Π½ΠΈΠΆΠ΅ прСдставлСн ΠΏΡ€ΠΈΠΌΠ΅Ρ€ смСшанного соСдинСния ΠΏΡ€ΠΈΠ΅ΠΌΠ½ΠΈΠΊΠΎΠ² энСргии


ΠŸΡ€ΠΈΠΌΠ΅Ρ€ смСшанного соСдинСния ΠΏΡ€ΠΈΠ΅ΠΌΠ½ΠΈΠΊΠΎΠ² энСргии.

Π’ качСствС ΠΏΡ€ΠΈΠΌΠ΅Ρ€Π° рассчитаСм Ρ‚ΠΎΠΊΠΈ ΠΈ напряТСния Π½Π° всСх участках Ρ†Π΅ΠΏΠΈ. Для Π½Π°Ρ‡Π°Π»Π° ΠΎΠΏΡ€Π΅Π΄Π΅Π»ΠΈΠΌ эквивалСнтноС сопротивлСниС Ρ†Π΅ΠΏΠΈ. Π’Ρ‹Π΄Π΅Π»ΠΈΠΌ Π΄Π²Π° участка с ΠΏΠ°Ρ€Π°Π»Π»Π΅Π»ΡŒΠ½Ρ‹ΠΌ соСдинСниСм ΠΏΡ€ΠΈΠ΅ΠΌΠ½ΠΈΠΊΠΎΠ² энСргии. Π­Ρ‚ΠΎ R1||R2 ΠΈ R3||R4||R5. Π’ΠΎΠ³Π΄Π° ΠΈΡ… эквивалСнтноС сопротивлСниС Π±ΡƒΠ΄Π΅Ρ‚ ΠΈΠΌΠ΅Ρ‚ΡŒ Π²ΠΈΠ΄

Π’ Ρ€Π΅Π·ΡƒΠ»ΡŒΡ‚Π°Ρ‚Π΅ ΠΏΠΎΠ»ΡƒΡ‡ΠΈΠ»ΠΈ Ρ†Π΅ΠΏΡŒ ΠΈΠ· Π΄Π²ΡƒΡ… ΠΏΠΎΡΠ»Π΅Π΄ΠΎΠ²Π°Ρ‚Π΅Π»ΡŒΠ½Ρ‹Ρ… ΠΏΡ€ΠΈΠ΅ΠΌΠ½ΠΈΠΊΠΎΠ² энСргии R 12 R 345 эквивалСнтноС сопротивлСниС ΠΈ Ρ‚ΠΎΠΊ, ΠΏΡ€ΠΎΡ‚Π΅ΠΊΠ°ΡŽΡ‰ΠΈΠΉ Ρ‡Π΅Ρ€Π΅Π· Π½ΠΈΡ…, составит

Π’ΠΎΠ³Π΄Π° ΠΏΠ°Π΄Π΅Π½ΠΈΠ΅ напряТСния ΠΏΠΎ участкам составит

Π’ΠΎΠ³Π΄Π° Ρ‚ΠΎΠΊΠΈ, ΠΏΡ€ΠΎΡ‚Π΅ΠΊΠ°ΡŽΡ‰ΠΈΠ΅ Ρ‡Π΅Ρ€Π΅Π· ΠΊΠ°ΠΆΠ΄Ρ‹ΠΉ ΠΏΡ€ΠΈΠ΅ΠΌΠ½ΠΈΠΊ энСргии, составят

Как я ΡƒΠΆΠ΅ ΡƒΠΏΠΎΠΌΠΈΠ½Π°Π», Π·Π°ΠΊΠΎΠ½Ρ‹ ΠšΠΈΡ€Ρ…Π³ΠΎΡ„Π° вмСстС с Π·Π°ΠΊΠΎΠ½ΠΎΠΌ Ома ΡΠ²Π»ΡΡŽΡ‚ΡΡ основными ΠΏΡ€ΠΈ Π°Π½Π°Π»ΠΈΠ·Π΅ ΠΈ расчётах элСктричСских Ρ†Π΅ΠΏΠ΅ΠΉ. Π—Π°ΠΊΠΎΠ½ Ома Π±Ρ‹Π» ΠΏΠΎΠ΄Ρ€ΠΎΠ±Π½ΠΎ рассмотрСн Π² Π΄Π²ΡƒΡ… ΠΏΡ€Π΅Π΄Ρ‹Π΄ΡƒΡ‰ΠΈΡ… ΡΡ‚Π°Ρ‚ΡŒΡΡ…, Ρ‚Π΅ΠΏΠ΅Ρ€ΡŒ настала ΠΎΡ‡Π΅Ρ€Π΅Π΄ΡŒ для Π·Π°ΠΊΠΎΠ½ΠΎΠ² ΠšΠΈΡ€Ρ…Π³ΠΎΡ„Π°. Π˜Ρ… всСго Π΄Π²Π°, ΠΏΠ΅Ρ€Π²Ρ‹ΠΉ описываСт ΡΠΎΠΎΡ‚Π½ΠΎΡˆΠ΅Π½ΠΈΡ Ρ‚ΠΎΠΊΠΎΠ² Π² элСктричСских цСпях, Π° Π²Ρ‚ΠΎΡ€ΠΎΠΉ – ΡΠΎΠΎΡ‚Π½ΠΎΡˆΠ΅Π½ΠΈΠ΅ Π­Π”Π‘ ΠΈ напряТСниями Π² ΠΊΠΎΠ½Ρ‚ΡƒΡ€Π΅. Начнём с ΠΏΠ΅Ρ€Π²ΠΎΠ³ΠΎ.

ΠŸΠ΅Ρ€Π²Ρ‹ΠΉ Π·Π°ΠΊΠΎΠ½ ΠšΠΈΡ€Ρ…Π³ΠΎΡ„Π° гласит, Ρ‡Ρ‚ΠΎ алгСбраичСская сумма Ρ‚ΠΎΠΊΠΎΠ² Π² ΡƒΠ·Π»Π΅ Ρ€Π°Π²Π½Π° Π½ΡƒΠ»ΡŽ. ΠžΠΏΠΈΡΡ‹Π²Π°Π΅Ρ‚ΡΡ это ΡΠ»Π΅Π΄ΡƒΡŽΡ‰ΠΈΠΌ Π²Ρ‹Ρ€Π°ΠΆΠ΅Π½ΠΈΠ΅ΠΌ

Π³Π΄Π΅ βˆ‘ β€” ΠΎΠ±ΠΎΠ·Π½Π°Ρ‡Π°Π΅Ρ‚ Π°Π»Π³Π΅Π±Ρ€Π°ΠΈΡ‡Π΅ΡΠΊΡƒΡŽ сумму.

Π‘Π»ΠΎΠ²ΠΎ «алгСбраичСская» ΠΎΠ·Π½Π°Ρ‡Π°Π΅Ρ‚, Ρ‡Ρ‚ΠΎ Ρ‚ΠΎΠΊΠΈ Π½Π΅ΠΎΠ±Ρ…ΠΎΠ΄ΠΈΠΌΠΎ Π±Ρ€Π°Ρ‚ΡŒ с ΡƒΡ‡Ρ‘Ρ‚ΠΎΠΌ Π·Π½Π°ΠΊΠ°, Ρ‚ΠΎ Π΅ΡΡ‚ΡŒ направлСния втСкания. Π’Π°ΠΊΠΈΠΌ ΠΎΠ±Ρ€Π°Π·ΠΎΠΌ, всСм Ρ‚ΠΎΠΊΠ°ΠΌ, ΠΊΠΎΡ‚ΠΎΡ€Ρ‹Π΅ Π²Ρ‚Π΅ΠΊΠ°ΡŽΡ‚ Π² ΡƒΠ·Π΅Π», присваиваСтся ΠΏΠΎΠ»ΠΎΠΆΠΈΡ‚Π΅Π»ΡŒΠ½Ρ‹ΠΉ Π·Π½Π°ΠΊ, Π° ΠΊΠΎΡ‚ΠΎΡ€Ρ‹Π΅ Π²Ρ‹Ρ‚Π΅ΠΊΠ°ΡŽΡ‚ ΠΈΠ· ΡƒΠ·Π»Π° – соотвСтствСнно ΠΎΡ‚Ρ€ΠΈΡ†Π°Ρ‚Π΅Π»ΡŒΠ½Ρ‹ΠΉ. Рисунок Π½ΠΈΠΆΠ΅ ΠΈΠ»Π»ΡŽΡΡ‚Ρ€ΠΈΡ€ΡƒΠ΅Ρ‚ ΠΏΠ΅Ρ€Π²Ρ‹ΠΉ Π·Π°ΠΊΠΎΠ½ ΠšΠΈΡ€Ρ…Π³ΠΎΡ„Π°


Π˜Π·ΠΎΠ±Ρ€Π°ΠΆΠ΅Π½ΠΈΠ΅ ΠΏΠ΅Ρ€Π²ΠΎΠ³ΠΎ Π·Π°ΠΊΠΎΠ½Π° ΠšΠΈΡ€Ρ…Π³ΠΎΡ„Π°.

На рисункС ΠΈΠ·ΠΎΠ±Ρ€Π°ΠΆΠ΅Π½ ΡƒΠ·Π΅Π», Π² ΠΊΠΎΡ‚ΠΎΡ€Ρ‹ΠΉ со стороны сопротивлСния R1 Π²Ρ‚Π΅ΠΊΠ°Π΅Ρ‚ Ρ‚ΠΎΠΊ, Π° со стороны сопротивлСний R2, R3, R4 соотвСтствСнно Π²Ρ‹Ρ‚Π΅ΠΊΠ°Π΅Ρ‚ Ρ‚ΠΎΠΊ, Ρ‚ΠΎΠ³Π΄Π° ΡƒΡ€Π°Π²Π½Π΅Π½ΠΈΠ΅ Ρ‚ΠΎΠΊΠΎΠ² для Π΄Π°Π½Π½ΠΎΠ³ΠΎ участка Ρ†Π΅ΠΏΠΈ Π±ΡƒΠ΄Π΅Ρ‚ ΠΈΠΌΠ΅Ρ‚ΡŒ Π²ΠΈΠ΄

ΠŸΠ΅Ρ€Π²Ρ‹ΠΉ Π·Π°ΠΊΠΎΠ½ ΠšΠΈΡ€Ρ…Π³ΠΎΡ„Π° примСняСтся Π½Π΅ Ρ‚ΠΎΠ»ΡŒΠΊΠΎ ΠΊ ΡƒΠ·Π»Π°ΠΌ, Π½ΠΎ ΠΈ ΠΊ Π»ΡŽΠ±ΠΎΠΌΡƒ ΠΊΠΎΠ½Ρ‚ΡƒΡ€Ρƒ ΠΈΠ»ΠΈ части элСктричСской Ρ†Π΅ΠΏΠΈ. НапримСр, ΠΊΠΎΠ³Π΄Π° я Π³ΠΎΠ²ΠΎΡ€ΠΈΠ» ΠΎ ΠΏΠ°Ρ€Π°Π»Π»Π΅Π»ΡŒΠ½ΠΎΠΌ соСдинСнии ΠΏΡ€ΠΈΠ΅ΠΌΠ½ΠΈΠΊΠΎΠ² энСргии, Π³Π΄Π΅ сумма Ρ‚ΠΎΠΊΠΎΠ² Ρ‡Π΅Ρ€Π΅Π· R1, R2 ΠΈ R3 Ρ€Π°Π²Π½Π° Π²Ρ‚Π΅ΠΊΠ°ΡŽΡ‰Π΅ΠΌΡƒ Ρ‚ΠΎΠΊΡƒ I.

Как Π³ΠΎΠ²ΠΎΡ€ΠΈΠ»ΠΎΡΡŒ Π²Ρ‹ΡˆΠ΅, Π²Ρ‚ΠΎΡ€ΠΎΠΉ Π·Π°ΠΊΠΎΠ½ ΠšΠΈΡ€Ρ…Π³ΠΎΡ„Π° опрСдСляСт ΡΠΎΠΎΡ‚Π½ΠΎΡˆΠ΅Π½ΠΈΠ΅ ΠΌΠ΅ΠΆΠ΄Ρƒ Π­Π”Π‘ ΠΈ напряТСниями Π² Π·Π°ΠΌΠΊΠ½ΡƒΡ‚ΠΎΠΌ ΠΊΠΎΠ½Ρ‚ΡƒΡ€Π΅ ΠΈ Π·Π²ΡƒΡ‡ΠΈΡ‚ ΡΠ»Π΅Π΄ΡƒΡŽΡ‰ΠΈΠΌ ΠΎΠ±Ρ€Π°Π·ΠΎΠΌ: алгСбраичСская сумма Π­Π”Π‘ Π² любом ΠΊΠΎΠ½Ρ‚ΡƒΡ€Π΅ Ρ†Π΅ΠΏΠΈ Ρ€Π°Π²Π½Π° алгСбраичСской суммС ΠΏΠ°Π΄Π΅Π½ΠΈΠΉ напряТСний Π½Π° элСмСнтах этого ΠΊΠΎΠ½Ρ‚ΡƒΡ€Π°. Π’Ρ‚ΠΎΡ€ΠΎΠΉ Π·Π°ΠΊΠΎΠ½ ΠšΠΈΡ€Ρ…Π³ΠΎΡ„Π° опрСдСляСтся ΡΠ»Π΅Π΄ΡƒΡŽΡ‰ΠΈΠΌ Π²Ρ‹Ρ€Π°ΠΆΠ΅Π½ΠΈΠ΅ΠΌ

Π’ качСствС ΠΏΡ€ΠΈΠΌΠ΅Ρ€Π° рассмотрим Π½ΠΈΠΆΠ΅ ΡΠ»Π΅Π΄ΡƒΡŽΡ‰ΡƒΡŽ схСму, ΡΠΎΠ΄Π΅Ρ€ΠΆΠ°Ρ‰ΡƒΡŽ Π½Π΅ΠΊΠΎΡ‚ΠΎΡ€Ρ‹ΠΉ ΠΊΠΎΠ½Ρ‚ΡƒΡ€


Π‘Ρ…Π΅ΠΌΠ°, ΠΈΠ»Π»ΡŽΡΡ‚Ρ€ΠΈΡ€ΡƒΡŽΡ‰Π°Ρ Π²Ρ‚ΠΎΡ€ΠΎΠΉ Π·Π°ΠΊΠΎΠ½ ΠšΠΈΡ€Ρ…Π³ΠΎΡ„Π°.

Для Π½Π°Ρ‡Π°Π»Π° Π½Π΅ΠΎΠ±Ρ…ΠΎΠ΄ΠΈΠΌΠΎ опрСдСлится с Π½Π°ΠΏΡ€Π°Π²Π»Π΅Π½ΠΈΠ΅ΠΌ ΠΎΠ±Ρ…ΠΎΠ΄Π° ΠΊΠΎΠ½Ρ‚ΡƒΡ€Π°. Π’ ΠΏΡ€ΠΈΠ½Ρ†ΠΈΠΏΠ΅ ΠΌΠΎΠΆΠ½ΠΎ Π²Ρ‹Π±Ρ€Π°Ρ‚ΡŒ ΠΊΠ°ΠΊ ΠΏΠΎ Ρ…ΠΎΠ΄Ρƒ часовой стрСлки, Ρ‚Π°ΠΊ ΠΈ ΠΏΡ€ΠΎΡ‚ΠΈΠ² Ρ…ΠΎΠ΄Π° часовой стрСлки. Π― Π²Ρ‹Π±Π΅Ρ€Ρƒ ΠΏΠ΅Ρ€Π²Ρ‹ΠΉ Π²Π°Ρ€ΠΈΠ°Π½Ρ‚, Ρ‚ΠΎ Π΅ΡΡ‚ΡŒ элСмСнты Π±ΡƒΠ΄ΡƒΡ‚ ΡΡ‡ΠΈΡ‚Π°Ρ‚ΡŒΡΡ Π² ΡΠ»Π΅Π΄ΡƒΡŽΡ‰Π΅ΠΌ порядкС E1R1R2R3E2, Ρ‚Π°ΠΊΠΈΠΌ ΠΎΠ±Ρ€Π°Π·ΠΎΠΌ, ΡƒΡ€Π°Π²Π½Π΅Π½ΠΈΠ΅ ΠΏΠΎ Π²Ρ‚ΠΎΡ€ΠΎΠΌΡƒ Π·Π°ΠΊΠΎΠ½Ρƒ ΠšΠΈΡ€Ρ…Π³ΠΎΡ„Π° Π±ΡƒΠ΄Π΅Ρ‚ ΠΈΠΌΠ΅Ρ‚ΡŒ ΡΠ»Π΅Π΄ΡƒΡŽΡ‰ΠΈΠΉ Π²ΠΈΠ΄

Π’Ρ‚ΠΎΡ€ΠΎΠΉ Π·Π°ΠΊΠΎΠ½ ΠšΠΈΡ€Ρ…Π³ΠΎΡ„Π° примСняСтся Π½Π΅ Ρ‚ΠΎΠ»ΡŒΠΊΠΎ ΠΊ цСпям постоянного Ρ‚ΠΎΠΊΠ°, Π½ΠΎ ΠΈ ΠΊ цСпям ΠΏΠ΅Ρ€Π΅ΠΌΠ΅Π½Π½ΠΎΠ³ΠΎ Ρ‚ΠΎΠΊΠ° ΠΈ ΠΊ Π½Π΅Π»ΠΈΠ½Π΅ΠΉΠ½Ρ‹ΠΌ цСпям.
Π’ ΡΠ»Π΅Π΄ΡƒΡŽΡ‰Π΅ΠΉ ΡΡ‚Π°Ρ‚ΡŒΠ΅ я Ρ€Π°ΡΡΠΌΠΎΡ‚Ρ€ΡŽ основныС способы расчёта слоТных Ρ†Π΅ΠΏΠ΅ΠΉ с использованиСм Π·Π°ΠΊΠΎΠ½Π° Ома ΠΈ Π·Π°ΠΊΠΎΠ½ΠΎΠ² ΠšΠΈΡ€Ρ…Π³ΠΎΡ„Π°.

ВСория это Ρ…ΠΎΡ€ΠΎΡˆΠΎ, Π½ΠΎ Π±Π΅Π· практичСского примСнСния это просто слова.

Π’ΠΎΠΊ Π² Ρ†Π΅ΠΏΠΈ ΠΏΡ€ΠΎΡ‚Π΅ΠΊΠ°Π΅Ρ‚ ΠΏΠΎ ΠΏΡ€ΠΎΠ²ΠΎΠ΄Π½ΠΈΠΊΠ°ΠΌ ΠΊ Π½Π°Π³Ρ€ΡƒΠ·ΠΊΠ΅ ΠΎΡ‚ источника. Π§Π°Ρ‰Π΅ всСго Π² качСствС Ρ‚Π°ΠΊΠΈΡ… элСмСнтов ΠΈΡΠΏΠΎΠ»ΡŒΠ·ΡƒΡŽΡ‚ мСдь. ЦСпь ΠΌΠΎΠΆΠ΅Ρ‚ ΠΈΠΌΠ΅Ρ‚ΡŒ нСсколько элСктричСских ΠΏΡ€ΠΈΠ΅ΠΌΠ½ΠΈΠΊΠΎΠ². Π˜Ρ… сопротивлСния разнятся. Π’ схСмС элСктроприборов ΠΏΡ€ΠΎΠ²ΠΎΠ΄Π½ΠΈΠΊΠΈ ΠΌΠΎΠ³ΡƒΡ‚ ΠΈΠΌΠ΅Ρ‚ΡŒ ΠΏΠ°Ρ€Π°Π»Π»Π΅Π»ΡŒΠ½ΠΎΠ΅ ΠΈΠ»ΠΈ ΠΏΠΎΡΠ»Π΅Π΄ΠΎΠ²Π°Ρ‚Π΅Π»ΡŒΠ½ΠΎΠ΅ соСдинСниС. Π’ΡΡ‚Ρ€Π΅Ρ‡Π°ΡŽΡ‚ΡΡ Ρ‚Π°ΠΊΠΆΠ΅ ΡΠΌΠ΅ΡˆΠ°Π½Π½Ρ‹Π΅ Π΅Π³ΠΎ Ρ‚ΠΈΠΏΡ‹. ΠžΡ‚Π»ΠΈΡ‡ΠΈΠ΅ ΠΊΠ°ΠΆΠ΄ΠΎΠ³ΠΎ ΠΈΠ· Π½ΠΈΡ… слСдуСт Π·Π½Π°Ρ‚ΡŒ ΠΏΠ΅Ρ€Π΅Π΄ Π²Ρ‹Π±ΠΎΡ€ΠΎΠΌ структуры элСктроцСпи.

ΠŸΡ€ΠΎΠ²ΠΎΠ΄Π½ΠΈΠΊΠΈ ΠΈ элСмСнты Ρ†Π΅ΠΏΠΈ

Π’ΠΎΠΊ ΠΈΠ΄Π΅Ρ‚ Ρ‡Π΅Ρ€Π΅Π· ΠΏΡ€ΠΎΠ²ΠΎΠ΄Π½ΠΈΠΊΠΈ. Он слСдуСт ΠΎΡ‚ источника ΠΊ Π½Π°Π³Ρ€ΡƒΠ·ΠΊΠ΅. ΠŸΡ€ΠΈ этом ΠΏΡ€ΠΎΠ²ΠΎΠ΄Π½ΠΈΠΊ обязан Π»Π΅Π³ΠΊΠΎ Π²Ρ‹ΡΠ²ΠΎΠ±ΠΎΠΆΠ΄Π°Ρ‚ΡŒ элСктроны.

ΠŸΡ€ΠΎΠ²ΠΎΠ΄Π½ΠΈΠΊ, ΠΈΠΌΠ΅ΡŽΡ‰ΠΈΠΉ сопротивлСниС, называСтся рСзистором. НапряТСниС этого элСмСнта — это Ρ€Π°Π·Π½ΠΎΡΡ‚ΡŒ ΠΏΠΎΡ‚Π΅Π½Ρ†ΠΈΠ°Π»ΠΎΠ² ΠΌΠ΅ΠΆΠ΄Ρƒ ΠΊΠΎΠ½Ρ†Π°ΠΌΠΈ рСзистора, ΠΊΠΎΡ‚ΠΎΡ€ΠΎΠ΅ согласовываСтся с Π½Π°ΠΏΡ€Π°Π²Π»Π΅Π½ΠΈΠ΅ΠΌ протСкания питания.

ΠŸΠΎΡΠ»Π΅Π΄ΠΎΠ²Π°Ρ‚Π΅Π»ΡŒΠ½ΠΎΠ΅ ΠΈ ΠΏΠ°Ρ€Π°Π»Π»Π΅Π»ΡŒΠ½ΠΎΠ΅ соСдинСниС ΠΏΡ€ΠΎΠ²ΠΎΠ΄Π½ΠΈΠΊΠΎΠ² характСризуСтся ΠΎΠ΄Π½ΠΈΠΌ ΠΎΠ±Ρ‰ΠΈΠΌ ΠΏΡ€ΠΈΠ½Ρ†ΠΈΠΏΠΎΠΌ. Π’ΠΎΠΊ Ρ‚Π΅Ρ‡Π΅Ρ‚ Π² Ρ†Π΅ΠΏΠΈ ΠΎΡ‚ плюса (Π΅Π³ΠΎ Π½Π°Π·Ρ‹Π²Π°ΡŽΡ‚ источником) ΠΊ минусу, Π³Π΄Π΅ ΠΏΠΎΡ‚Π΅Π½Ρ†ΠΈΠ°Π» становится всС мСньшим, ΡƒΠ±Ρ‹Π²Π°Π΅Ρ‚. На элСктричСских схСмах сопротивлСниС ΠΏΡ€ΠΎΠ²ΠΎΠ΄ΠΎΠ² считаСтся Ρ€Π°Π²Π½Ρ‹ΠΌ Π½ΡƒΠ»ΡŽ, Ρ‚Π°ΠΊ ΠΊΠ°ΠΊ ΠΎΠ½ΠΎ ΠΏΡ€Π΅Π½Π΅Π±Ρ€Π΅ΠΆΠΈΡ‚Π΅Π»ΡŒΠ½ΠΎ ΠΌΠ°Π»ΠΎ.

ΠŸΠΎΡΡ‚ΠΎΠΌΡƒ, просчитывая ΠΏΠΎΡΠ»Π΅Π΄ΠΎΠ²Π°Ρ‚Π΅Π»ΡŒΠ½ΠΎΠ΅ ΠΈΠ»ΠΈ ΠΏΠ°Ρ€Π°Π»Π»Π΅Π»ΡŒΠ½ΠΎΠ΅ соСдинСниС, ΠΏΡ€ΠΈΠ±Π΅Π³Π°ΡŽΡ‚ ΠΊ ΠΈΠ΄Π΅Π°Π»ΠΈΠ·Π°Ρ†ΠΈΠΈ. Π­Ρ‚ΠΎ ΡƒΠΏΡ€ΠΎΡ‰Π°Π΅Ρ‚ ΠΈΡ… ΠΈΠ·ΡƒΡ‡Π΅Π½ΠΈΠ΅. Π’ Ρ€Π΅Π°Π»ΡŒΠ½Ρ‹Ρ… цСпях ΠΏΠΎΡ‚Π΅Π½Ρ†ΠΈΠ°Π» постСпСнно ΡƒΠΌΠ΅Π½ΡŒΡˆΠ°Π΅Ρ‚ΡΡ ΠΏΡ€ΠΈ ΠΏΠ΅Ρ€Π΅Π΄Π²ΠΈΠΆΠ΅Π½ΠΈΠΈ ΠΏΠΎ ΠΏΡ€ΠΎΠ²ΠΎΠ΄Ρƒ ΠΈ элСмСнтам, ΠΈΠΌΠ΅ΡŽΡ‰ΠΈΠΌ ΠΏΠ°Ρ€Π°Π»Π»Π΅Π»ΡŒΠ½ΠΎΠ΅ ΠΈΠ»ΠΈ ΠΏΠΎΡΠ»Π΅Π΄ΠΎΠ²Π°Ρ‚Π΅Π»ΡŒΠ½ΠΎΠ΅ соСдинСниС.

ΠŸΠΎΡΠ»Π΅Π΄ΠΎΠ²Π°Ρ‚Π΅Π»ΡŒΠ½ΠΎΠ΅ соСдинСниС ΠΏΡ€ΠΎΠ²ΠΎΠ΄Π½ΠΈΠΊΠΎΠ²

ΠŸΡ€ΠΈ Π½Π°Π»ΠΈΡ‡ΠΈΠΈ ΠΏΠΎΡΠ»Π΅Π΄ΠΎΠ²Π°Ρ‚Π΅Π»ΡŒΠ½ΠΎΠ³ΠΎ сочСтания ΠΏΡ€ΠΎΠ²ΠΎΠ΄Π½ΠΈΠΊΠΎΠ² сопротивлСния Π²ΠΊΠ»ΡŽΡ‡Π°ΡŽΡ‚ΡΡ ΠΎΠ΄Π½ΠΎ Π·Π° Π΄Ρ€ΡƒΠ³ΠΈΠΌ. ΠŸΡ€ΠΈ Ρ‚Π°ΠΊΠΎΠΌ ΠΏΠΎΠ»ΠΎΠΆΠ΅Π½ΠΈΠΈ сила Ρ‚ΠΎΠΊΠ° Π²ΠΎ всСх элСмСнтах Ρ†Π΅ΠΏΠΈ ΠΎΠ΄ΠΈΠ½Π°ΠΊΠΎΠ²Π°. ΠŸΠΎΡΠ»Π΅Π΄ΠΎΠ²Π°Ρ‚Π΅Π»ΡŒΠ½ΠΎ соСдинСнныС ΠΏΡ€ΠΎΠ²ΠΎΠ΄Π½ΠΈΠΊΠΈ ΡΠΎΠ·Π΄Π°ΡŽΡ‚ Π½Π° участкС напряТСниС, ΠΊΠΎΡ‚ΠΎΡ€ΠΎΠ΅ Ρ€Π°Π²Π½ΠΎ ΠΈΡ… суммС Π½Π° всСх элСмСнтах.

Заряды Π½Π΅ ΠΈΠΌΠ΅ΡŽΡ‚ возмоТности Π½Π°ΠΊΠ°ΠΏΠ»ΠΈΠ²Π°Ρ‚ΡŒΡΡ Π² ΡƒΠ·Π»Π°Ρ… Ρ†Π΅ΠΏΠΈ. Π­Ρ‚ΠΎ Π±Ρ‹ ΠΏΡ€ΠΈΠ²Π΅Π»ΠΎ ΠΊ измСнСнию напряТСния элСктричСского поля ΠΈ силы Ρ‚ΠΎΠΊΠ°.

ΠŸΡ€ΠΈ Π½Π°Π»ΠΈΡ‡ΠΈΠΈ постоянного напряТСния Ρ‚ΠΎΠΊ Π±ΡƒΠ΄Π΅Ρ‚ Π·Π°Π²ΠΈΡΠ΅Ρ‚ΡŒ ΠΎΡ‚ сопротивлСния Ρ†Π΅ΠΏΠΈ. ΠŸΠΎΡΡ‚ΠΎΠΌΡƒ ΠΏΡ€ΠΈ ΠΏΠΎΡΠ»Π΅Π΄ΠΎΠ²Π°Ρ‚Π΅Π»ΡŒΠ½ΠΎΠΌ соСдинСнии сопротивлСниС Π±ΡƒΠ΄Π΅Ρ‚ ΠΌΠ΅Π½ΡΡ‚ΡŒΡΡ ΠΈΠ·-Π·Π° ΠΏΠ΅Ρ€Π΅ΠΌΠ΅Π½Ρ‹ ΠΎΠ΄Π½ΠΎΠΉ Π½Π°Π³Ρ€ΡƒΠ·ΠΊΠΈ.

ΠŸΠΎΡΠ»Π΅Π΄ΠΎΠ²Π°Ρ‚Π΅Π»ΡŒΠ½ΠΎΠ΅ соСдинСниС ΠΏΡ€ΠΎΠ²ΠΎΠ΄Π½ΠΈΠΊΠΎΠ² ΠΈΠΌΠ΅Π΅Ρ‚ нСдостаток. ΠŸΡ€ΠΈ ΠΏΠΎΠ»ΠΎΠΌΠΊΠ΅ ΠΎΠ΄Π½ΠΎΠ³ΠΎ ΠΈΠ· элСмСнтов схСмы Π±ΡƒΠ΄Π΅Ρ‚ ΠΏΡ€Π΅Ρ€Π²Π°Π½Π° Ρ€Π°Π±ΠΎΡ‚Π° всСх ΠΎΡΡ‚Π°Π»ΡŒΠ½Ρ‹Ρ… Π΅Π΅ ΡΠΎΡΡ‚Π°Π²Π»ΡΡŽΡ‰ΠΈΡ…. НапримСр, ΠΊΠ°ΠΊ Π² гирляндС. Если Π² Π½Π΅ΠΉ ΠΏΠ΅Ρ€Π΅Π³ΠΎΡ€ΠΈΡ‚ ΠΎΠ΄Π½Π° Π»Π°ΠΌΠΏΠΎΡ‡ΠΊΠ°, всС ΠΈΠ·Π΄Π΅Π»ΠΈΠ΅ Π½Π΅ Π±ΡƒΠ΄Π΅Ρ‚ Ρ€Π°Π±ΠΎΡ‚Π°Ρ‚ΡŒ.

Если ΠΏΡ€ΠΎΠ²ΠΎΠ΄Π½ΠΈΠΊΠΈ Π±Ρ‹Π»ΠΈ подсоСдинСны Π² Ρ†Π΅ΠΏΠΈ ΠΏΠΎΡΠ»Π΅Π΄ΠΎΠ²Π°Ρ‚Π΅Π»ΡŒΠ½ΠΎ, ΠΈΡ… сопротивлСниС Π² ΠΊΠ°ΠΆΠ΄ΠΎΠΉ Ρ‚ΠΎΡ‡ΠΊΠ΅ Π±ΡƒΠ΄Π΅Ρ‚ ΠΎΠ΄ΠΈΠ½Π°ΠΊΠΎΠ²Ρ‹ΠΌ. Π‘ΠΎΠΏΡ€ΠΎΡ‚ΠΈΠ²Π»Π΅Π½ΠΈΠ΅ Π² суммС всСх элСмСнтов схСмы Π±ΡƒΠ΄Π΅Ρ‚ Ρ€Π°Π²Π½ΡΡ‚ΡŒΡΡ суммС ΡƒΠΌΠ΅Π½ΡŒΡˆΠ΅Π½ΠΈΡ напряТСний Π½Π° участках Ρ†Π΅ΠΏΠΈ.

Π­Ρ‚ΠΎ ΠΌΠΎΠΆΠ΅Ρ‚ ΠΏΠΎΠ΄Ρ‚Π²Π΅Ρ€Π΄ΠΈΡ‚ΡŒ ΠΎΠΏΡ‹Ρ‚. ΠŸΠΎΡΠ»Π΅Π΄ΠΎΠ²Π°Ρ‚Π΅Π»ΡŒΠ½ΠΎΠ΅ соСдинСниС сопротивлСний подсчитываСтся ΠΏΡ€ΠΈ ΠΏΠΎΠΌΠΎΡ‰ΠΈ ΠΏΡ€ΠΈΠ±ΠΎΡ€ΠΎΠ² ΠΈ матСматичСской ΠΏΡ€ΠΎΠ²Π΅Ρ€ΠΊΠΈ. НапримСр, бСрутся Ρ‚Ρ€ΠΈ постоянных сопротивлСния извСстной Π²Π΅Π»ΠΈΡ‡ΠΈΠ½Ρ‹. Π˜Ρ… ΠΏΠΎΡΠ»Π΅Π΄ΠΎΠ²Π°Ρ‚Π΅Π»ΡŒΠ½ΠΎ ΡΠΎΠ΅Π΄ΠΈΠ½ΡΡŽΡ‚ ΠΈ ΠΏΠΎΠ΄ΠΊΠ»ΡŽΡ‡Π°ΡŽΡ‚ ΠΊ ΠΏΠΈΡ‚Π°Π½ΠΈΡŽ Π² 60 Π’.

ПослС этого ΠΏΠΎΠ΄ΡΡ‡ΠΈΡ‚Ρ‹Π²Π°ΡŽΡ‚ ΠΏΡ€Π΅Π΄ΠΏΠΎΠ»Π°Π³Π°Π΅ΠΌΡ‹Π΅ ΠΏΠΎΠΊΠ°Π·Π°Ρ‚Π΅Π»ΠΈ ΠΏΡ€ΠΈΠ±ΠΎΡ€ΠΎΠ², Ссли Π·Π°ΠΌΠΊΠ½ΡƒΡ‚ΡŒ Ρ†Π΅ΠΏΡŒ. По Π·Π°ΠΊΠΎΠ½Ρƒ Ома находится Ρ‚ΠΎΠΊ Π² Ρ†Π΅ΠΏΠΈ, Ρ‡Ρ‚ΠΎ ΠΏΠΎΠ·Π²ΠΎΠ»ΠΈΡ‚ ΠΎΠΏΡ€Π΅Π΄Π΅Π»ΠΈΡ‚ΡŒ ΠΏΠ°Π΄Π΅Π½ΠΈΠ΅ напряТСния Π½Π° всСх Π΅Π΅ участках. ПослС этого ΡΡƒΠΌΠΌΠΈΡ€ΡƒΡŽΡ‚ΡΡ ΠΏΠΎΠ»ΡƒΡ‡Π΅Π½Π½Ρ‹Π΅ Ρ€Π΅Π·ΡƒΠ»ΡŒΡ‚Π°Ρ‚Ρ‹ ΠΈ получаСтся общая Π²Π΅Π»ΠΈΡ‡ΠΈΠ½Π° сниТСния сопротивлСния Π²ΠΎ внСшнСй Ρ†Π΅ΠΏΠΈ. ΠŸΠΎΡΠ»Π΅Π΄ΠΎΠ²Π°Ρ‚Π΅Π»ΡŒΠ½ΠΎΠ΅ соСдинСниС сопротивлСний ΠΌΠΎΠΆΠ½ΠΎ ΠΏΠΎΠ΄Ρ‚Π²Π΅Ρ€Π΄ΠΈΡ‚ΡŒ ΠΏΡ€ΠΈΠΌΠ΅Ρ€Π½ΠΎ. Если Π½Π΅ Π±Ρ€Π°Ρ‚ΡŒ Π²ΠΎ Π²Π½ΠΈΠΌΠ°Π½ΠΈΠ΅ Π²Π½ΡƒΡ‚Ρ€Π΅Π½Π½Π΅Π΅ сопротивлСниС, ΡΠΎΠ·Π΄Π°ΡŽΡ‰Π΅Π΅ΡΡ источником энСргии, Ρ‚ΠΎ ΠΏΠ°Π΄Π΅Π½ΠΈΠ΅ напряТСния Π±ΡƒΠ΄Π΅Ρ‚ мСньшС, Ρ‡Π΅ΠΌ сумма сопротивлСний. По ΠΏΡ€ΠΈΠ±ΠΎΡ€Π°ΠΌ ΠΌΠΎΠΆΠ½ΠΎ ΡƒΠ±Π΅Π΄ΠΈΡ‚ΡŒΡΡ, Ρ‡Ρ‚ΠΎ равСнство ΠΏΡ€ΠΈΠ±Π»ΠΈΠ·ΠΈΡ‚Π΅Π»ΡŒΠ½ΠΎ ΡΠΎΠ±Π»ΡŽΠ΄Π°Π΅Ρ‚ΡΡ.

ΠŸΠ°Ρ€Π°Π»Π»Π΅Π»ΡŒΠ½ΠΎΠ΅ соСдинСниС ΠΏΡ€ΠΎΠ²ΠΎΠ΄Π½ΠΈΠΊΠΎΠ²

ΠŸΡ€ΠΈ ΠΏΠΎΡΠ»Π΅Π΄ΠΎΠ²Π°Ρ‚Π΅Π»ΡŒΠ½ΠΎΠΌ ΠΈ ΠΏΠ°Ρ€Π°Π»Π»Π΅Π»ΡŒΠ½ΠΎΠΌ соСдинСнии ΠΏΡ€ΠΎΠ²ΠΎΠ΄Π½ΠΈΠΊΠΎΠ² Π² Ρ†Π΅ΠΏΠΈ ΠΏΡ€ΠΈΠΌΠ΅Π½ΡΡŽΡ‚ рСзисторы. ΠŸΠ°Ρ€Π°Π»Π»Π΅Π»ΡŒΠ½ΠΎΠ΅ соСдинСниС ΠΏΡ€ΠΎΠ²ΠΎΠ΄Π½ΠΈΠΊΠΎΠ² прСдставляСт собой систСму, Π² ΠΊΠΎΡ‚ΠΎΡ€ΠΎΠΉ ΠΎΠ΄Π½ΠΈ ΠΊΠΎΠ½Ρ†Ρ‹ всСх рСзисторов сходятся Π² ΠΎΠ΄ΠΈΠ½ ΠΎΠ±Ρ‰ΠΈΠΉ ΡƒΠ·Π΅Π», Π° Π΄Ρ€ΡƒΠ³ΠΈΠ΅ — Π² Π΄Ρ€ΡƒΠ³ΠΎΠΉ ΡƒΠ·Π΅Π». Π’ этих мСстах схСмы сходятся Π±ΠΎΠ»Π΅Π΅ Π΄Π²ΡƒΡ… ΠΏΡ€ΠΎΠ²ΠΎΠ΄Π½ΠΈΠΊΠΎΠ².

ΠŸΡ€ΠΈ Ρ‚Π°ΠΊΠΎΠΌ соСдинСнии ΠΊ элСмСнтам прикладываСтся ΠΎΠ΄ΠΈΠ½Π°ΠΊΠΎΠ²ΠΎΠ΅ напряТСниС. ΠŸΠ°Ρ€Π°Π»Π»Π΅Π»ΡŒΠ½Ρ‹Π΅ участки Ρ†Π΅ΠΏΠΈ Π½Π°Π·Ρ‹Π²Π°ΡŽΡ‚ΡΡ вСтвями. Они проходят ΠΌΠ΅ΠΆΠ΄Ρƒ двумя ΡƒΠ·Π»Π°ΠΌΠΈ. ΠŸΠ°Ρ€Π°Π»Π»Π΅Π»ΡŒΠ½ΠΎΠ΅ ΠΈ ΠΏΠΎΡΠ»Π΅Π΄ΠΎΠ²Π°Ρ‚Π΅Π»ΡŒΠ½ΠΎΠ΅ соСдинСниС ΠΈΠΌΠ΅ΡŽΡ‚ свои свойства.

Если Π² элСктросхСмС Π΅ΡΡ‚ΡŒ Π²Π΅Ρ‚Π²ΠΈ, Ρ‚ΠΎ напряТСниС Π½Π° ΠΊΠ°ΠΆΠ΄ΠΎΠΉ ΠΈΠ· Π½ΠΈΡ… Π±ΡƒΠ΄Π΅Ρ‚ ΠΎΠ΄ΠΈΠ½Π°ΠΊΠΎΠ²Ρ‹ΠΌ. Оно равняСтся Π½Π°ΠΏΡ€ΡΠΆΠ΅Π½ΠΈΡŽ Π½Π° Π½Π΅Ρ€Π°Π·Π²Π΅Ρ‚Π²Π»Π΅Π½Π½ΠΎΠΌ участкС. Π’ этом мСстС сила Ρ‚ΠΎΠΊΠ° Π±ΡƒΠ΄Π΅Ρ‚ Ρ€Π°ΡΡΡ‡ΠΈΡ‚Ρ‹Π²Π°Ρ‚ΡŒΡΡ ΠΊΠ°ΠΊ сумма Π΅Π΅ Π² ΠΊΠ°ΠΆΠ΄ΠΎΠΉ Π²Π΅Ρ‚Π²ΠΈ.

Π’Π΅Π»ΠΈΡ‡ΠΈΠ½Π°, равная суммС ΠΏΠΎΠΊΠ°Π·Π°Ρ‚Π΅Π»Π΅ΠΉ, ΠΎΠ±Ρ€Π°Ρ‚Π½Ρ‹Ρ… сопротивлСниям Ρ€Π°Π·Π²Π΅Ρ‚Π²Π»Π΅Π½ΠΈΠΉ, Π±ΡƒΠ΄Π΅Ρ‚ ΠΎΠ±Ρ€Π°Ρ‚Π½Π° ΠΈ ΡΠΎΠΏΡ€ΠΎΡ‚ΠΈΠ²Π»Π΅Π½ΠΈΡŽ участка ΠΏΠ°Ρ€Π°Π»Π»Π΅Π»ΡŒΠ½ΠΎΠ³ΠΎ соСдинСния.

ΠŸΠ°Ρ€Π°Π»Π»Π΅Π»ΡŒΠ½ΠΎΠ΅ соСдинСниС сопротивлСний

ΠŸΠ°Ρ€Π°Π»Π»Π΅Π»ΡŒΠ½ΠΎΠ΅ ΠΈ ΠΏΠΎΡΠ»Π΅Π΄ΠΎΠ²Π°Ρ‚Π΅Π»ΡŒΠ½ΠΎΠ΅ соСдинСниС ΠΎΡ‚Π»ΠΈΡ‡Π°ΡŽΡ‚ΡΡ расчСтом сопротивлСний Π΅Π΅ элСмСнтов. ΠŸΡ€ΠΈ ΠΏΠ°Ρ€Π°Π»Π»Π΅Π»ΡŒΠ½ΠΎΠΌ соСдинСнии Ρ‚ΠΎΠΊ развСтвляСтся. Π­Ρ‚ΠΎ ΡƒΠ²Π΅Π»ΠΈΡ‡ΠΈΠ²Π°Π΅Ρ‚ ΠΏΡ€ΠΎΠ²ΠΎΠ΄ΠΈΠΌΠΎΡΡ‚ΡŒ Ρ†Π΅ΠΏΠΈ (ΡƒΠΌΠ΅Π½ΡŒΡˆΠ°Π΅Ρ‚ ΠΎΠ±Ρ‰Π΅Π΅ сопротивлСниС), которая Π±ΡƒΠ΄Π΅Ρ‚ Ρ€Π°Π²Π½Π° суммС проводимости Π²Π΅Ρ‚Π²Π΅ΠΉ.

Если нСсколько рСзисторов, ΠΈΠΌΠ΅ΡŽΡ‰ΠΈΡ… ΠΎΠ΄ΠΈΠ½Π°ΠΊΠΎΠ²ΡƒΡŽ Π²Π΅Π»ΠΈΡ‡ΠΈΠ½Ρƒ, соСдинСны ΠΏΠ°Ρ€Π°Π»Π»Π΅Π»ΡŒΠ½ΠΎ, Ρ‚ΠΎ суммарноС сопротивлСниС Ρ†Π΅ΠΏΠΈ Π±ΡƒΠ΄Π΅Ρ‚ мСньшС ΠΎΠ΄Π½ΠΎΠ³ΠΎ рСзистора Π²ΠΎ ΡΡ‚ΠΎΠ»ΡŒΠΊΠΎ Ρ€Π°Π·, сколько ΠΈΡ… Π²ΠΊΠ»ΡŽΡ‡Π΅Π½ΠΎ Π² схСму.

ΠŸΠΎΡΠ»Π΅Π΄ΠΎΠ²Π°Ρ‚Π΅Π»ΡŒΠ½ΠΎΠ΅ ΠΈ ΠΏΠ°Ρ€Π°Π»Π»Π΅Π»ΡŒΠ½ΠΎΠ΅ соСдинСниС ΠΏΡ€ΠΎΠ²ΠΎΠ΄Π½ΠΈΠΊΠΎΠ² ΠΈΠΌΠ΅ΡŽΡ‚ ряд особСнностСй. Π’ ΠΏΠ°Ρ€Π°Π»Π»Π΅Π»ΡŒΠ½ΠΎΠΌ ΠΏΠΎΠ΄ΠΊΠ»ΡŽΡ‡Π΅Π½ΠΈΠΈ Ρ‚ΠΎΠΊ ΠΎΠ±Ρ€Π°Ρ‚Π½ΠΎ ΠΏΡ€ΠΎΠΏΠΎΡ€Ρ†ΠΈΠΎΠ½Π°Π»Π΅Π½ ΡΠΎΠΏΡ€ΠΎΡ‚ΠΈΠ²Π»Π΅Π½ΠΈΡŽ. Π’ΠΎΠΊΠΈ Π² рСзисторах Π½Π΅ зависят Π΄Ρ€ΡƒΠ³ ΠΎΡ‚ Π΄Ρ€ΡƒΠ³Π°. ΠŸΠΎΡΡ‚ΠΎΠΌΡƒ Π²Ρ‹ΠΊΠ»ΡŽΡ‡Π΅Π½ΠΈΠ΅ ΠΎΠ΄Π½ΠΎΠ³ΠΎ ΠΈΠ· Π½ΠΈΡ… Π½Π΅ отразится Π½Π° Ρ€Π°Π±ΠΎΡ‚Π΅ ΠΎΡΡ‚Π°Π»ΡŒΠ½Ρ‹Ρ…. ΠŸΠΎΡΡ‚ΠΎΠΌΡƒ мноТСство элСктроприборов ΠΈΠΌΠ΅ΡŽΡ‚ ΠΈΠΌΠ΅Π½Π½ΠΎ этот Ρ‚ΠΈΠΏ соСдинСния элСмСнтов Ρ†Π΅ΠΏΠΈ.

БмСшанноС

ΠŸΠ°Ρ€Π°Π»Π»Π΅Π»ΡŒΠ½ΠΎΠ΅ ΠΈ ΠΏΠΎΡΠ»Π΅Π΄ΠΎΠ²Π°Ρ‚Π΅Π»ΡŒΠ½ΠΎΠ΅ соСдинСниС ΠΏΡ€ΠΎΠ²ΠΎΠ΄Π½ΠΈΠΊΠΎΠ² ΠΌΠΎΠΆΠ΅Ρ‚ ΠΊΠΎΠΌΠ±ΠΈΠ½ΠΈΡ€ΠΎΠ²Π°Ρ‚ΡŒΡΡ Π² ΠΎΠ΄Π½ΠΎΠΉ ΠΈ Ρ‚ΠΎΠΉ ΠΆΠ΅ схСмС. НапримСр, элСмСнты, ΠΏΠΎΠ΄ΠΊΠ»ΡŽΡ‡Π΅Π½Π½Ρ‹Π΅ ΠΌΠ΅ΠΆΠ΄Ρƒ собой ΠΏΠ°Ρ€Π°Π»Π»Π΅Π»ΡŒΠ½ΠΎ, ΠΌΠΎΠ³ΡƒΡ‚ Π±Ρ‹Ρ‚ΡŒ соСдинСны ΠΏΠΎΡΠ»Π΅Π΄ΠΎΠ²Π°Ρ‚Π΅Π»ΡŒΠ½ΠΎ с Π΄Ρ€ΡƒΠ³ΠΈΠΌ рСзистором ΠΈΠ»ΠΈ ΠΈΡ… Π³Ρ€ΡƒΠΏΠΏΠΎΠΉ. Π­Ρ‚ΠΎ смСшанноС соСдинСниС. ΠžΠ±Ρ‰Π΅Π΅ сопротивлСниС Ρ†Π΅ΠΏΠ΅ΠΉ вычисляСтся ΠΏΡƒΡ‚Π΅ΠΌ ΠΎΡ‚Π΄Π΅Π»ΡŒΠ½ΠΎΠ³ΠΎ суммирования Π·Π½Π°Ρ‡Π΅Π½ΠΈΠΉ для ΠΏΠ°Ρ€Π°Π»Π»Π΅Π»ΡŒΠ½ΠΎ ΠΏΠΎΠ΄ΠΊΠ»ΡŽΡ‡Π΅Π½Π½ΠΎΠ³ΠΎ Π±Π»ΠΎΠΊΠ° ΠΈ для ΠΏΠΎΡΠ»Π΅Π΄ΠΎΠ²Π°Ρ‚Π΅Π»ΡŒΠ½ΠΎΠ³ΠΎ соСдинСния.

ΠŸΡ€ΠΈΡ‡Π΅ΠΌ сначала Π²Ρ‹Ρ‡ΠΈΡΠ»ΡΡŽΡ‚ΡΡ эквивалСнтныС сопротивлСния ΠΏΠΎΡΠ»Π΅Π΄ΠΎΠ²Π°Ρ‚Π΅Π»ΡŒΠ½ΠΎ ΠΏΠΎΠ΄ΠΊΠ»ΡŽΡ‡Π΅Π½Π½Ρ‹Ρ… элСмСнтов, Π° ΠΏΠΎΡ‚ΠΎΠΌ ΡƒΠΆΠ΅ рассчитываСтся ΠΎΠ±Ρ‰Π΅Π΅ сопротивлСниС ΠΏΠ°Ρ€Π°Π»Π»Π΅Π»ΡŒΠ½Ρ‹Ρ… участков Ρ†Π΅ΠΏΠΈ. ΠŸΠΎΡΠ»Π΅Π΄ΠΎΠ²Π°Ρ‚Π΅Π»ΡŒΠ½ΠΎΠ΅ соСдинСниС Π² вычислСниях являСтся ΠΏΡ€ΠΈΠΎΡ€ΠΈΡ‚Π΅Ρ‚Π½Π΅Π΅. Π’Π°ΠΊΠΈΠ΅ Ρ‚ΠΈΠΏΡ‹ элСктросхСм довольно часто Π²ΡΡ‚Ρ€Π΅Ρ‡Π°ΡŽΡ‚ΡΡ Π² Ρ€Π°Π·Π»ΠΈΡ‡Π½Ρ‹Ρ… ΠΏΡ€ΠΈΠ±ΠΎΡ€Π°Ρ… ΠΈ ΠΎΠ±ΠΎΡ€ΡƒΠ΄ΠΎΠ²Π°Π½ΠΈΠΈ.

Ознакомившись с Π²ΠΈΠ΄Π°ΠΌΠΈ соСдинСния элСмСнтов Ρ†Π΅ΠΏΠΈ, ΠΌΠΎΠΆΠ½ΠΎ ΠΏΠΎΠ½ΡΡ‚ΡŒ ΠΏΡ€ΠΈΠ½Ρ†ΠΈΠΏ ΠΎΡ€Π³Π°Π½ΠΈΠ·Π°Ρ†ΠΈΠΈ схСм Ρ€Π°Π·Π»ΠΈΡ‡Π½Ρ‹Ρ… элСктричСских ΠΏΡ€ΠΈΠ±ΠΎΡ€ΠΎΠ². ΠŸΠ°Ρ€Π°Π»Π»Π΅Π»ΡŒΠ½ΠΎΠ΅ ΠΈ ΠΏΠΎΡΠ»Π΅Π΄ΠΎΠ²Π°Ρ‚Π΅Π»ΡŒΠ½ΠΎΠ΅ соСдинСниС ΠΎΠ±Π»Π°Π΄Π°ΡŽΡ‚ рядом особСнностСй расчСта ΠΈ функционирования всСй систСмы. Зная ΠΈΡ…, ΠΌΠΎΠΆΠ½ΠΎ ΠΏΡ€Π°Π²ΠΈΠ»ΡŒΠ½ΠΎ ΠΏΡ€ΠΈΠΌΠ΅Π½ΡΡ‚ΡŒ ΠΊΠ°ΠΆΠ΄Ρ‹ΠΉ ΠΈΠ· прСдставлСнных Π²ΠΈΠ΄ΠΎΠ² для ΠΏΠΎΠ΄ΠΊΠ»ΡŽΡ‡Π΅Π½ΠΈΡ элСмСнтов элСктричСских Ρ†Π΅ΠΏΠ΅ΠΉ.

ΠŸΠ°Ρ€Π°Π»Π»Π΅Π»ΡŒΠ½Ρ‹Π΅ соСдинСния рСзисторов, Ρ„ΠΎΡ€ΠΌΡƒΠ»Π° расчёта ΠΊΠΎΡ‚ΠΎΡ€Ρ‹Ρ… выводится ΠΈΠ· Π·Π°ΠΊΠΎΠ½Π° Ома ΠΈ ΠΏΡ€Π°Π²ΠΈΠ» ΠšΠΈΡ€Ρ…Π³ΠΎΡ„Π°, ΡΠ²Π»ΡΡŽΡ‚ΡΡ Π½Π°ΠΈΠ±ΠΎΠ»Π΅Π΅ распространённым Ρ‚ΠΈΠΏΠΎΠΌ Π²ΠΊΠ»ΡŽΡ‡Π΅Π½ΠΈΡ элСмСнтов Π² ΡΠ»Π΅ΠΊΡ‚Ρ€ΠΈΡ‡Π΅ΡΠΊΡƒΡŽ Ρ†Π΅ΠΏΡŒ. ΠŸΡ€ΠΈ ΠΏΠ°Ρ€Π°Π»Π»Π΅Π»ΡŒΠ½ΠΎΠΌ соСдинСнии ΠΏΡ€ΠΎΠ²ΠΎΠ΄Π½ΠΈΠΊΠΎΠ² Π΄Π²Π° ΠΈΠ»ΠΈ нСсколько элСмСнтов ΠΎΠ±ΡŠΠ΅Π΄ΠΈΠ½ΡΡŽΡ‚ΡΡ своими ΠΊΠΎΠ½Ρ‚Π°ΠΊΡ‚Π°ΠΌΠΈ с ΠΎΠ±Π΅ΠΈΡ… ΠΈΠ· сторон соотвСтствСнно. ΠŸΠΎΠ΄ΠΊΠ»ΡŽΡ‡Π΅Π½ΠΈΠ΅ ΠΈΡ… ΠΊ ΠΎΠ±Ρ‰Π΅ΠΉ схСмС осущСствляСтся ΠΈΠΌΠ΅Π½Π½ΠΎ этими ΡƒΠ·Π»ΠΎΠ²Ρ‹ΠΌΠΈ Ρ‚ΠΎΡ‡ΠΊΠ°ΠΌΠΈ.

Gif?x15027″ alt=»ΠžΠ±Ρ‰ΠΈΠΉ Π²ΠΈΠ΄»>

ΠžΠ±Ρ‰ΠΈΠΉ Π²ΠΈΠ΄

ΠžΡΠΎΠ±Π΅Π½Π½ΠΎΡΡ‚ΠΈ Π²ΠΊΠ»ΡŽΡ‡Π΅Π½ΠΈΡ

Π’ΠΊΠ»ΡŽΡ‡Ρ‘Π½Π½Ρ‹Π΅ Ρ‚Π°ΠΊΠΈΠΌ ΠΎΠ±Ρ€Π°Π·ΠΎΠΌ ΠΏΡ€ΠΎΠ²ΠΎΠ΄Π½ΠΈΠΊΠΈ Π½Π΅Ρ€Π΅Π΄ΠΊΠΎ входят Π² состав слоТных Ρ†Π΅ΠΏΠΎΡ‡Π΅ΠΊ, содСрТащих, ΠΏΠΎΠΌΠΈΠΌΠΎ этого, ΠΏΠΎΡΠ»Π΅Π΄ΠΎΠ²Π°Ρ‚Π΅Π»ΡŒΠ½ΠΎΠ΅ соСдинСниС ΠΎΡ‚Π΄Π΅Π»ΡŒΠ½Ρ‹Ρ… участков.

Для Ρ‚Π°ΠΊΠΎΠ³ΠΎ Π²ΠΊΠ»ΡŽΡ‡Π΅Π½ΠΈΡ Ρ‚ΠΈΠΏΠΈΡ‡Π½Ρ‹ ΡΠ»Π΅Π΄ΡƒΡŽΡ‰ΠΈΠ΅ особСнности:

  • ΠžΠ±Ρ‰Π΅Π΅ напряТСниС Π² ΠΊΠ°ΠΆΠ΄ΠΎΠΉ ΠΈΠ· Π²Π΅Ρ‚Π²Π΅ΠΉ Π±ΡƒΠ΄Π΅Ρ‚ ΠΈΠΌΠ΅Ρ‚ΡŒ ΠΎΠ΄Π½ΠΎ ΠΈ Ρ‚ΠΎ ΠΆΠ΅ Π·Π½Π°Ρ‡Π΅Π½ΠΈΠ΅;
  • ΠŸΡ€ΠΎΡ‚Π΅ΠΊΠ°ΡŽΡ‰ΠΈΠΉ Π² любом ΠΈΠ· сопротивлСний элСктричСский Ρ‚ΠΎΠΊ всСгда ΠΎΠ±Ρ€Π°Ρ‚Π½ΠΎ ΠΏΡ€ΠΎΠΏΠΎΡ€Ρ†ΠΈΠΎΠ½Π°Π»Π΅Π½ Π²Π΅Π»ΠΈΡ‡ΠΈΠ½Π΅ ΠΈΡ… Π½ΠΎΠΌΠΈΠ½Π°Π»Π°.

Π’ частном случаС, ΠΊΠΎΠ³Π΄Π° всС Π²ΠΊΠ»ΡŽΡ‡Ρ‘Π½Π½Ρ‹Π΅ Π² ΠΏΠ°Ρ€Π°Π»Π»Π΅Π»ΡŒ рСзисторы ΠΈΠΌΠ΅ΡŽΡ‚ ΠΎΠ΄ΠΈΠ½Π°ΠΊΠΎΠ²Ρ‹Π΅ Π½ΠΎΠΌΠΈΠ½Π°Π»ΡŒΠ½Ρ‹Π΅ значСния, ΠΏΡ€ΠΎΡ‚Π΅ΠΊΠ°ΡŽΡ‰ΠΈΠ΅ ΠΏΠΎ Π½ΠΈΠΌ Β«ΠΈΠ½Π΄ΠΈΠ²ΠΈΠ΄ΡƒΠ°Π»ΡŒΠ½Ρ‹Π΅Β» Ρ‚ΠΎΠΊΠΈ Ρ‚Π°ΠΊΠΆΠ΅ Π±ΡƒΠ΄ΡƒΡ‚ Ρ€Π°Π²Π½Ρ‹ ΠΌΠ΅ΠΆΠ΄Ρƒ собой.

Расчёт

БопротивлСния ряда соСдинённых Π² ΠΏΠ°Ρ€Π°Π»Π»Π΅Π»ΡŒ проводящих элСмСнтов ΠΎΠΏΡ€Π΅Π΄Π΅Π»ΡΡŽΡ‚ΡΡ ΠΏΠΎ общСизвСстной Ρ„ΠΎΡ€ΠΌΠ΅ расчёта, ΠΏΡ€Π΅Π΄ΠΏΠΎΠ»Π°Π³Π°ΡŽΡ‰Π΅ΠΉ слоТСниС ΠΈΡ… проводимостСй (ΠΎΠ±Ρ€Π°Ρ‚Π½Ρ‹Ρ… ΡΠΎΠΏΡ€ΠΎΡ‚ΠΈΠ²Π»Π΅Π½ΠΈΡŽ Π²Π΅Π»ΠΈΡ‡ΠΈΠ½).

ΠŸΡ€ΠΎΡ‚Π΅ΠΊΠ°ΡŽΡ‰ΠΈΠΉ Π² ΠΊΠ°ΠΆΠ΄ΠΎΠΌ ΠΈΠ· ΠΎΡ‚Π΄Π΅Π»ΡŒΠ½Ρ‹Ρ… ΠΏΡ€ΠΎΠ²ΠΎΠ΄Π½ΠΈΠΊΠΎΠ² Ρ‚ΠΎΠΊ Π² соотвСтствиС с Π·Π°ΠΊΠΎΠ½ΠΎΠΌ Ома, ΠΌΠΎΠΆΠ΅Ρ‚ Π±Ρ‹Ρ‚ΡŒ Π½Π°ΠΉΠ΄Π΅Π½ ΠΏΠΎ Ρ„ΠΎΡ€ΠΌΡƒΠ»Π΅:

I= U/R (ΠΎΠ΄Π½ΠΎΠ³ΠΎ ΠΈΠ· рСзисторов).

ПослС ознакомлСния с ΠΎΠ±Ρ‰ΠΈΠΌΠΈ ΠΏΡ€ΠΈΠ½Ρ†ΠΈΠΏΠ°ΠΌΠΈ обсчёта элСмСнтов слоТных Ρ†Π΅ΠΏΠΎΡ‡Π΅ΠΊ ΠΌΠΎΠΆΠ½ΠΎ ΠΏΠ΅Ρ€Π΅ΠΉΡ‚ΠΈ ΠΊ ΠΊΠΎΠ½ΠΊΡ€Π΅Ρ‚Π½Ρ‹ΠΌ ΠΏΡ€ΠΈΠΌΠ΅Ρ€Π°ΠΌ Ρ€Π΅ΡˆΠ΅Π½ΠΈΡ Π·Π°Π΄Π°Ρ‡ Π΄Π°Π½Π½ΠΎΠ³ΠΎ класса.

Π’ΠΈΠΏΠΈΡ‡Π½Ρ‹Π΅ ΠΏΠΎΠ΄ΠΊΠ»ΡŽΡ‡Π΅Π½ΠΈΡ

ΠŸΡ€ΠΈΠΌΠ΅Ρ€ β„–1

НСрСдко для Ρ€Π΅ΡˆΠ΅Π½ΠΈΡ стоящСй ΠΏΠ΅Ρ€Π΅Π΄ конструктором Π·Π°Π΄Π°Ρ‡ΠΈ трСбуСтся ΠΏΡƒΡ‚Ρ‘ΠΌ объСдинСния Π½Π΅ΡΠΊΠΎΠ»ΡŒΠΊΠΈΡ… элСмСнтов ΠΏΠΎΠ»ΡƒΡ‡ΠΈΡ‚ΡŒ Π² ΠΈΡ‚ΠΎΠ³Π΅ ΠΊΠΎΠ½ΠΊΡ€Π΅Ρ‚Π½ΠΎΠ΅ сопротивлСниС. ΠŸΡ€ΠΈ рассмотрСнии ΠΏΡ€ΠΎΡΡ‚Π΅ΠΉΡˆΠ΅Π³ΠΎ Π²Π°Ρ€ΠΈΠ°Π½Ρ‚Π° Ρ‚Π°ΠΊΠΎΠ³ΠΎ Ρ€Π΅ΡˆΠ΅Π½ΠΈΡ допустим, Ρ‡Ρ‚ΠΎ ΠΎΠ±Ρ‰Π΅Π΅ сопротивлСниС Ρ†Π΅ΠΏΠΎΡ‡ΠΊΠΈ ΠΈΠ· Π½Π΅ΡΠΊΠΎΠ»ΡŒΠΊΠΈΡ… элСмСнтов Π΄ΠΎΠ»ΠΆΠ½ΠΎ ΡΠΎΡΡ‚Π°Π²Π»ΡΡ‚ΡŒ 8 Ом. Π­Ρ‚ΠΎΡ‚ ΠΏΡ€ΠΈΠΌΠ΅Ρ€ нуТдаСтся Π² ΠΎΡ‚Π΄Π΅Π»ΡŒΠ½ΠΎΠΌ рассмотрСнии ΠΏΠΎ Ρ‚ΠΎΠΉ простой ΠΏΡ€ΠΈΡ‡ΠΈΠ½Π΅, Ρ‡Ρ‚ΠΎ Π² стандартном ряду сопротивлСний Π½ΠΎΠΌΠΈΠ½Π°Π» Π² 8 Ом отсутствуСт (Π΅ΡΡ‚ΡŒ Ρ‚ΠΎΠ»ΡŒΠΊΠΎ 7,5 ΠΈ 8,2 Ом).

РСшСниС этой ΠΏΡ€ΠΎΡΡ‚Π΅ΠΉΡˆΠ΅ΠΉ Π·Π°Π΄Π°Ρ‡ΠΈ удаётся ΠΏΠΎΠ»ΡƒΡ‡ΠΈΡ‚ΡŒ Π·Π° счёт соСдинСния Π΄Π²ΡƒΡ… ΠΎΠ΄ΠΈΠ½Π°ΠΊΠΎΠ²Ρ‹Ρ… элСмСнтов с сопротивлСниями ΠΏΠΎ 16 Ом ΠΊΠ°ΠΆΠ΄ΠΎΠ΅ (Ρ‚Π°ΠΊΠΈΠ΅ Π½ΠΎΠΌΠΈΠ½Π°Π»Ρ‹ Π² рСзистивном ряду ΡΡƒΡ‰Π΅ΡΡ‚Π²ΡƒΡŽΡ‚). Богласно ΠΏΡ€ΠΈΠ²ΠΎΠ΄ΠΈΠΌΠΎΠΉ Π²Ρ‹ΡˆΠ΅ Ρ„ΠΎΡ€ΠΌΡƒΠ»Π΅ ΠΎΠ±Ρ‰Π΅Π΅ сопротивлСниС Ρ†Π΅ΠΏΠΎΡ‡ΠΊΠΈ Π² этом случаС вычисляСтся ΠΎΡ‡Π΅Π½ΡŒ просто.

Из Π½Π΅Ρ‘ слСдуСт:

16Ρ…16/32=8 (Ом), Ρ‚ΠΎ Π΅ΡΡ‚ΡŒ ΠΊΠ°ΠΊ Ρ€Π°Π· ΡΡ‚ΠΎΠ»ΡŒΠΊΠΎ, сколько Ρ‚Ρ€Π΅Π±ΠΎΠ²Π°Π»ΠΎΡΡŒ ΠΏΠΎΠ»ΡƒΡ‡ΠΈΡ‚ΡŒ.

Π’Π°ΠΊΠΈΠΌ ΡΡ€Π°Π²Π½ΠΈΡ‚Π΅Π»ΡŒΠ½ΠΎ простым способом удаётся Ρ€Π΅ΡˆΠΈΡ‚ΡŒ Π·Π°Π΄Π°Ρ‡Ρƒ формирования ΠΎΠ±Ρ‰Π΅Π³ΠΎ сопротивлСния, Ρ€Π°Π²Π½ΠΎΠ³ΠΎ 8-ΠΌΠΈ Омам.

ΠŸΡ€ΠΈΠΌΠ΅Ρ€ β„–2

Π’ качСствС Π΅Ρ‰Ρ‘ ΠΎΠ΄Π½ΠΎΠ³ΠΎ Ρ…Π°Ρ€Π°ΠΊΡ‚Π΅Ρ€Π½ΠΎΠ³ΠΎ ΠΏΡ€ΠΈΠΌΠ΅Ρ€Π° образования Ρ‚Ρ€Π΅Π±ΡƒΠ΅ΠΌΠΎΠ³ΠΎ сопротивлСния ΠΌΠΎΠΆΠ½ΠΎ Ρ€Π°ΡΡΠΌΠΎΡ‚Ρ€Π΅Ρ‚ΡŒ построСниС схСмы, состоящСй ΠΈΠ· 3-Ρ… рСзисторов.

ΠžΠ±Ρ‰Π΅Π΅ Π·Π½Π°Ρ‡Π΅Π½ΠΈΠ΅ R Ρ‚Π°ΠΊΠΎΠ³ΠΎ Π²ΠΊΠ»ΡŽΡ‡Π΅Π½ΠΈΡ ΠΌΠΎΠΆΠ΅Ρ‚ Π±Ρ‹Ρ‚ΡŒ рассчитано ΠΏΠΎ Ρ„ΠΎΡ€ΠΌΡƒΠ»Π΅ ΠΏΠΎΡΠ»Π΅Π΄ΠΎΠ²Π°Ρ‚Π΅Π»ΡŒΠ½ΠΎΠ³ΠΎ ΠΈ ΠΏΠ°Ρ€Π°Π»Π»Π΅Π»ΡŒΠ½ΠΎΠ³ΠΎ соСдинСния Π² ΠΏΡ€ΠΎΠ²ΠΎΠ΄Π½ΠΈΠΊΠ°Ρ….

Gif?x15027″ alt=»ΠŸΡ€ΠΈΠΌΠ΅Ρ€»>

Π’ соотвСтствии с ΡƒΠΊΠ°Π·Π°Π½Π½Ρ‹ΠΌΠΈ Π½Π° ΠΊΠ°Ρ€Ρ‚ΠΈΠ½ΠΊΠ΅ значСниями Π½ΠΎΠΌΠΈΠ½Π°Π»ΠΎΠ², ΠΎΠ±Ρ‰Π΅Π΅ сопротивлСниС Ρ†Π΅ΠΏΠΎΡ‡ΠΊΠΈ Π±ΡƒΠ΄Π΅Ρ‚ Ρ€Π°Π²Π½ΠΎ:

1/R = 1/200+1/220+1/470 = 0,0117;

R=1/0,0117 = 85,67Ом.

Π’ ΠΈΡ‚ΠΎΠ³Π΅ Π½Π°Ρ…ΠΎΠ΄ΠΈΠΌ суммарноС сопротивлСниС всСй Ρ†Π΅ΠΏΠΎΡ‡ΠΊΠΈ, ΠΏΠΎΠ»ΡƒΡ‡Π°Π΅ΠΌΠΎΠΉ ΠΏΡ€ΠΈ ΠΏΠ°Ρ€Π°Π»Π»Π΅Π»ΡŒΠ½ΠΎΠΌ соСдинСнии Ρ‚Ρ€Ρ‘Ρ… элСмСнтов с Π½ΠΎΠΌΠΈΠ½Π°Π»ΡŒΠ½Ρ‹ΠΌΠΈ значСниями 200, 240 ΠΈ 470 Ом.

Π’Π°ΠΆΠ½ΠΎ! Π£ΠΊΠ°Π·Π°Π½Π½Ρ‹ΠΉ ΠΌΠ΅Ρ‚ΠΎΠ΄ ΠΏΡ€ΠΈΠΌΠ΅Π½ΠΈΠΌ ΠΈ ΠΏΡ€ΠΈ расчётС ΠΏΡ€ΠΎΠΈΠ·Π²ΠΎΠ»ΡŒΠ½ΠΎΠ³ΠΎ числа соСдинСнных Π² ΠΏΠ°Ρ€Π°Π»Π»Π΅Π»ΡŒ ΠΏΡ€ΠΎΠ²ΠΎΠ΄Π½ΠΈΠΊΠΎΠ² ΠΈΠ»ΠΈ ΠΏΠΎΡ‚Ρ€Π΅Π±ΠΈΡ‚Π΅Π»Π΅ΠΉ.

Π’Π°ΠΊΠΆΠ΅ Π½Π΅ΠΎΠ±Ρ…ΠΎΠ΄ΠΈΠΌΠΎ ΠΎΡ‚ΠΌΠ΅Ρ‚ΠΈΡ‚ΡŒ, Ρ‡Ρ‚ΠΎ ΠΏΡ€ΠΈ Ρ‚Π°ΠΊΠΎΠΌ способС Π²ΠΊΠ»ΡŽΡ‡Π΅Π½ΠΈΡ Ρ€Π°Π·Π»ΠΈΡ‡Π½Ρ‹Ρ… ΠΏΠΎ Π²Π΅Π»ΠΈΡ‡ΠΈΠ½Π΅ элСмСнтов ΠΎΠ±Ρ‰Π΅Π΅ сопротивлСниС Π±ΡƒΠ΄Π΅Ρ‚ мСньшС, Ρ‡Π΅ΠΌ Ρƒ самого ΠΌΠ°Π»ΠΎΠ³ΠΎ Π½ΠΎΠΌΠΈΠ½Π°Π»Π°.

Расчёт ΠΊΠΎΠΌΠ±ΠΈΠ½ΠΈΡ€ΠΎΠ²Π°Π½Π½Ρ‹Ρ… схСм

РассмотрСнный ΠΌΠ΅Ρ‚ΠΎΠ΄ ΠΌΠΎΠΆΠ΅Ρ‚ ΠΏΡ€ΠΈΠΌΠ΅Π½ΡΡ‚ΡŒΡΡ ΠΈ ΠΏΡ€ΠΈ расчётС сопротивлСния Π±ΠΎΠ»Π΅Π΅ слоТных ΠΈΠ»ΠΈ ΠΊΠΎΠΌΠ±ΠΈΠ½ΠΈΡ€ΠΎΠ²Π°Π½Π½Ρ‹Ρ… схСм, состоящих ΠΈΠ· Ρ†Π΅Π»ΠΎΠ³ΠΎ Π½Π°Π±ΠΎΡ€Π° ΠΊΠΎΠΌΠΏΠΎΠ½Π΅Π½Ρ‚ΠΎΠ². Π˜Ρ… ΠΈΠ½ΠΎΠ³Π΄Π° Π½Π°Π·Ρ‹Π²Π°ΡŽΡ‚ ΡΠΌΠ΅ΡˆΠ°Π½Π½Ρ‹ΠΌΠΈ, ΠΏΠΎΡΠΊΠΎΠ»ΡŒΠΊΡƒ ΠΏΡ€ΠΈ Ρ„ΠΎΡ€ΠΌΠΈΡ€ΠΎΠ²Π°Π½ΠΈΠΈ Ρ†Π΅ΠΏΠΎΡ‡Π΅ΠΊ ΠΈΡΠΏΠΎΠ»ΡŒΠ·ΡƒΡŽΡ‚ΡΡ сразу ΠΎΠ±Π° способа. БмСшанноС соСдинСниС рСзисторов прСдставлСно Π½Π° Ρ€Π°Π·ΠΌΠ΅Ρ‰Π΅Π½Π½ΠΎΠΌ Π½ΠΈΠΆΠ΅ рисункС.

Gif?x15027″ alt=»Π‘мСшанная схСма»>

БмСшанная схСма

Для упрощСния расчСта сначала Ρ€Π°Π·Π±ΠΈΠ²Π°Π΅ΠΌ всС рСзисторы ΠΏΠΎ Ρ‚ΠΈΠΏΡƒ Π²ΠΊΠ»ΡŽΡ‡Π΅Π½ΠΈΡ Π½Π° Π΄Π²Π΅ ΡΠ°ΠΌΠΎΡΡ‚ΠΎΡΡ‚Π΅Π»ΡŒΠ½Ρ‹Π΅ Π³Ρ€ΡƒΠΏΠΏΡ‹. Одна ΠΈΠ· Π½ΠΈΡ… прСдставляСт собой ΠΏΠΎΡΠ»Π΅Π΄ΠΎΠ²Π°Ρ‚Π΅Π»ΡŒΠ½ΠΎΠ΅ соСдинСниС, Π° вторая – ΠΈΠΌΠ΅Π΅Ρ‚ Π²ΠΈΠ΄ ΠΏΠΎΠ΄ΠΊΠ»ΡŽΡ‡Π΅Π½ΠΈΡ ΠΏΠ°Ρ€Π°Π»Π»Π΅Π»ΡŒΠ½ΠΎΠ³ΠΎ Ρ‚ΠΈΠΏΠ°.

Из ΠΏΡ€ΠΈΠ²Π΅Π΄Ρ‘Π½Π½ΠΎΠΉ схСмы Π²ΠΈΠ΄Π½ΠΎ, Ρ‡Ρ‚ΠΎ элСмСнты R2 ΠΈ R3 ΡΠΎΠ΅Π΄ΠΈΠ½ΡΡŽΡ‚ΡΡ ΠΏΠΎΡΠ»Π΅Π΄ΠΎΠ²Π°Ρ‚Π΅Π»ΡŒΠ½ΠΎ (ΠΎΠ½ΠΈ ΠΎΠ±ΡŠΠ΅Π΄ΠΈΠ½Π΅Π½Ρ‹ Π² Π³Ρ€ΡƒΠΏΠΏΡƒ 2), которая, Π² свою ΠΎΡ‡Π΅Ρ€Π΅Π΄ΡŒ, Π²ΠΊΠ»ΡŽΡ‡Π΅Π½Π° Π² ΠΏΠ°Ρ€Π°Π»Π»Π΅Π»ΡŒ с рСзистором R1, ΠΏΡ€ΠΈΠ½Π°Π΄Π»Π΅ΠΆΠ°Ρ‰ΠΈΠΌ Π³Ρ€ΡƒΠΏΠΏΠ΅ 1.

ΠŸΠΎΡΠ»Π΅Π΄ΠΎΠ²Π°Ρ‚Π΅Π»ΡŒΠ½Ρ‹ΠΌ называСтся Ρ‚Π°ΠΊΠΎΠ΅ соСдинСниС рСзисторов, ΠΊΠΎΠ³Π΄Π° ΠΊΠΎΠ½Π΅Ρ† ΠΎΠ΄Π½ΠΎΠ³ΠΎ ΠΏΡ€ΠΎΠ²ΠΎΠ΄Π½ΠΈΠΊΠ° соСдиняСтся с Π½Π°Ρ‡Π°Π»ΠΎΠΌ Π΄Ρ€ΡƒΠ³ΠΎΠ³ΠΎ ΠΈ Ρ‚.Π΄. (рис. 1). ΠŸΡ€ΠΈ ΠΏΠΎΡΠ»Π΅Π΄ΠΎΠ²Π°Ρ‚Π΅Π»ΡŒΠ½ΠΎΠΌ соСдинСнии сила Ρ‚ΠΎΠΊΠ° Π½Π° любом участкС элСктричСской Ρ†Π΅ΠΏΠΈ ΠΎΠ΄ΠΈΠ½Π°ΠΊΠΎΠ²Π°. Π­Ρ‚ΠΎ ΠΎΠ±ΡŠΡΡΠ½ΡΠ΅Ρ‚ΡΡ Ρ‚Π΅ΠΌ, Ρ‡Ρ‚ΠΎ заряды Π½Π΅ ΠΌΠΎΠ³ΡƒΡ‚ Π½Π°ΠΊΠ°ΠΏΠ»ΠΈΠ²Π°Ρ‚ΡŒΡΡ Π² ΡƒΠ·Π»Π°Ρ… Ρ†Π΅ΠΏΠΈ. Π˜Ρ… Π½Π°ΠΊΠΎΠΏΠ»Π΅Π½ΠΈΠ΅ ΠΏΡ€ΠΈΠ²Π΅Π»ΠΎ Π±Ρ‹ ΠΊ измСнСнию напряТСнности элСктричСского поля, Π° ΡΠ»Π΅Π΄ΠΎΠ²Π°Ρ‚Π΅Π»ΡŒΠ½ΠΎ, ΠΈ ΠΊ измСнСнию силы Ρ‚ΠΎΠΊΠ°. ΠŸΠΎΡΡ‚ΠΎΠΌΡƒ

\(~I = I_1 = I_2 .\)

АмпСрмСтр А измСряСт силу Ρ‚ΠΎΠΊΠ° Π² Ρ†Π΅ΠΏΠΈ ΠΈ ΠΎΠ±Π»Π°Π΄Π°Π΅Ρ‚ ΠΌΠ°Π»Ρ‹ΠΌ Π²Π½ΡƒΡ‚Ρ€Π΅Π½Π½ΠΈΠΌ сопротивлСниСм (R A β†’ 0).

Π’ΠΊΠ»ΡŽΡ‡Π΅Π½Π½Ρ‹Π΅ Π²ΠΎΠ»ΡŒΡ‚ΠΌΠ΅Ρ‚Ρ€Ρ‹ V 1 ΠΈ V 2 ΠΈΠ·ΠΌΠ΅Ρ€ΡΡŽΡ‚ напряТСниС U 1 ΠΈ U 2 Π½Π° сопротивлСниях R 1 ΠΈ R 2 . Π’ΠΎΠ»ΡŒΡ‚ΠΌΠ΅Ρ‚Ρ€ V измСряСт ΠΏΠΎΠ΄Π²Π΅Π΄Π΅Π½Π½ΠΎΠ΅ ΠΊ ΠΊΠ»Π΅ΠΌΠΌΠ°ΠΌ Μ ΠΈ N напряТСниС U .n R_i .\)

Если сопротивлСния ΠΎΡ‚Π΄Π΅Π»ΡŒΠ½Ρ‹Ρ… рСзисторов Ρ€Π°Π²Π½Ρ‹ ΠΌΠ΅ΠΆΠ΄Ρƒ собой, Ρ‚.Π΅. R 1 = R 2 = … = R n , Ρ‚ΠΎ ΠΎΠ±Ρ‰Π΅Π΅ сопротивлСниС этих рСзисторов ΠΏΡ€ΠΈ ΠΏΠΎΡΠ»Π΅Π΄ΠΎΠ²Π°Ρ‚Π΅Π»ΡŒΠ½ΠΎΠΌ соСдинСнии Π² n Ρ€Π°Π· большС сопротивлСния ΠΎΠ΄Π½ΠΎΠ³ΠΎ рСзистора: R = nR 1 .

ΠŸΡ€ΠΈ ΠΏΠΎΡΠ»Π΅Π΄ΠΎΠ²Π°Ρ‚Π΅Π»ΡŒΠ½ΠΎΠΌ соСдинСнии рСзисторов справСдливо ΡΠΎΠΎΡ‚Π½ΠΎΡˆΠ΅Π½ΠΈΠ΅ \(~\frac{U_1}{U_2} = \frac{R_1}{R_2}\), Ρ‚.Π΅. напряТСния Π½Π° рСзисторах прямо ΠΏΡ€ΠΎΠΏΠΎΡ€Ρ†ΠΈΠΎΠ½Π°Π»ΡŒΠ½Ρ‹ сопротивлСниям.

ΠŸΠ°Ρ€Π°Π»Π»Π΅Π»ΡŒΠ½Ρ‹ΠΌ называСтся Ρ‚Π°ΠΊΠΎΠ΅ соСдинСниС рСзисторов, ΠΊΠΎΠ³Π΄Π° ΠΎΠ΄Π½ΠΈ ΠΊΠΎΠ½Ρ†Ρ‹ всСх рСзисторов соСдинСны Π² ΠΎΠ΄ΠΈΠ½ ΡƒΠ·Π΅Π», Π΄Ρ€ΡƒΠ³ΠΈΠ΅ ΠΊΠΎΠ½Ρ†Ρ‹ — Π² Π΄Ρ€ΡƒΠ³ΠΎΠΉ ΡƒΠ·Π΅Π» (рис. 2). Π£Π·Π»ΠΎΠΌ называСтся Ρ‚ΠΎΡ‡ΠΊΠ° Ρ€Π°Π·Π²Π΅Ρ‚Π²Π»Π΅Π½Π½ΠΎΠΉ Ρ†Π΅ΠΏΠΈ, Π² ΠΊΠΎΡ‚ΠΎΡ€ΠΎΠΉ сходятся Π±ΠΎΠ»Π΅Π΅ Π΄Π²ΡƒΡ… ΠΏΡ€ΠΎΠ²ΠΎΠ΄Π½ΠΈΠΊΠΎΠ². ΠŸΡ€ΠΈ ΠΏΠ°Ρ€Π°Π»Π»Π΅Π»ΡŒΠ½ΠΎΠΌ соСдинСнии рСзисторов ΠΊ Ρ‚ΠΎΡ‡ΠΊΠ°ΠΌ Μ ΠΈ N ΠΏΠΎΠ΄ΠΊΠ»ΡŽΡ‡Π΅Π½ Π²ΠΎΠ»ΡŒΡ‚ΠΌΠ΅Ρ‚Ρ€. Он ΠΏΠΎΠΊΠ°Π·Ρ‹Π²Π°Π΅Ρ‚, Ρ‡Ρ‚ΠΎ напряТСния Π½Π° ΠΎΡ‚Π΄Π΅Π»ΡŒΠ½Ρ‹Ρ… участках Ρ†Π΅ΠΏΠΈ с сопротивлСниями R 1 ΠΈ R 2 Ρ€Π°Π²Π½Ρ‹. Π­Ρ‚ΠΎ ΠΎΠ±ΡŠΡΡΠ½ΡΠ΅Ρ‚ΡΡ Ρ‚Π΅ΠΌ, Ρ‡Ρ‚ΠΎ Ρ€Π°Π±ΠΎΡ‚Π° сил стационарного элСктричСского поля Π½Π΅ зависит ΠΎΡ‚ Ρ„ΠΎΡ€ΠΌΡ‹ Ρ‚Ρ€Π°Π΅ΠΊΡ‚ΠΎΡ€ΠΈΠΈ:

\(~U = U_1 = U_2 .n \frac{1}{R_i} .\)

Если сопротивлСния всСх n ΠΏΠ°Ρ€Π°Π»Π»Π΅Π»ΡŒΠ½ΠΎ соСдинСнных рСзисторов ΠΎΠ΄ΠΈΠ½Π°ΠΊΠΎΠ²Ρ‹ ΠΈ Ρ€Π°Π²Π½Ρ‹ R 1 Ρ‚ΠΎ \(~\frac 1R = \frac{n}{R_1}\) . ΠžΡ‚ΠΊΡƒΠ΄Π° \(~R = \frac{R_1}{n}\) .

Π‘ΠΎΠΏΡ€ΠΎΡ‚ΠΈΠ²Π»Π΅Π½ΠΈΠ΅ Ρ†Π΅ΠΏΠΈ, состоящСй ΠΈΠ· n ΠΎΠ΄ΠΈΠ½Π°ΠΊΠΎΠ²Ρ‹Ρ… ΠΏΠ°Ρ€Π°Π»Π»Π΅Π»ΡŒΠ½ΠΎ соСдинСнных рСзисторов, Π² n Ρ€Π°Π· мСньшС сопротивлСния ΠΊΠ°ΠΆΠ΄ΠΎΠ³ΠΎ ΠΈΠ· Π½ΠΈΡ….

ΠŸΡ€ΠΈ ΠΏΠ°Ρ€Π°Π»Π»Π΅Π»ΡŒΠ½ΠΎΠΌ соСдинСнии рСзисторов справСдливо ΡΠΎΠΎΡ‚Π½ΠΎΡˆΠ΅Π½ΠΈΠ΅ \(~\frac{I_1}{I_2} = \frac{R_2}{R_1}\), Ρ‚.Π΅. силы Ρ‚ΠΎΠΊΠΎΠ² Π² вСтвях ΠΏΠ°Ρ€Π°Π»Π»Π΅Π»ΡŒΠ½ΠΎ соСдинСнной Ρ†Π΅ΠΏΠΈ ΠΎΠ±Ρ€Π°Ρ‚Π½ΠΎ ΠΏΡ€ΠΎΠΏΠΎΡ€Ρ†ΠΈΠΎΠ½Π°Π»ΡŒΠ½Ρ‹ сопротивлСниям Π²Π΅Ρ‚Π²Π΅ΠΉ.

Π›ΠΈΡ‚Π΅Ρ€Π°Ρ‚ΡƒΡ€Π°

АксСнович Π›. А. Π€ΠΈΠ·ΠΈΠΊΠ° Π² срСднСй школС: ВСория. Задания. ВСсты: Π£Ρ‡Π΅Π±. пособиС для ΡƒΡ‡Ρ€Π΅ΠΆΠ΄Π΅Π½ΠΈΠΉ, ΠΎΠ±Π΅ΡΠΏΠ΅Ρ‡ΠΈΠ²Π°ΡŽΡ‰ΠΈΡ… ΠΏΠΎΠ»ΡƒΡ‡Π΅Π½ΠΈΠ΅ ΠΎΠ±Ρ‰. срСд, образования / Π›. А. АксСнович, Н.Н.Π Π°ΠΊΠΈΠ½Π°, К. Π‘. Π€Π°Ρ€ΠΈΠ½ΠΎ; Под Ρ€Π΅Π΄. К. Π‘. Π€Π°Ρ€ΠΈΠ½ΠΎ. — Мн.: Адукацыя i Π²Ρ‹Ρ…Π°Π²Π°Π½Π½Π΅, 2004. — C. 257-259.

Как Π½Π°ΠΉΡ‚ΠΈ эквивалСнтноС сопротивлСниС Ρ†Π΅ΠΏΠΈ Ρ„ΠΎΡ€ΠΌΡƒΠ»Π°

БопротивлСния Π² элСктричСских цСпях ΠΌΠΎΠ³ΡƒΡ‚ Π±Ρ‹Ρ‚ΡŒ соСдинСны ΠΏΠΎΡΠ»Π΅Π΄ΠΎΠ²Π°Ρ‚Π΅Π»ΡŒΠ½ΠΎ, ΠΏΠ°Ρ€Π°Π»Π»Π΅Π»ΡŒΠ½ΠΎ, ΠΏΠΎ смСшанной схСмС ΠΈ ΠΏΠΎ схСмам Β«Π·Π²Π΅Π·Π΄Π°Β», Β«Ρ‚Ρ€Π΅ΡƒΠ³ΠΎΠ»ΡŒΠ½ΠΈΠΊΒ». РасчСт слоТной схСмы упрощаСтся, Ссли сопротивлСния Π² этой схСмС Π·Π°ΠΌΠ΅Π½ΡΡŽΡ‚ΡΡ ΠΎΠ΄Π½ΠΈΠΌ эквивалСнтным сопротивлСниСм Rэкв, ΠΈ вся схСма прСдставляСтся Π² Π²ΠΈΠ΄Π΅ схСмы Π½Π° рис. 1.3, Π³Π΄Π΅ R=Rэкв, Π° расчСт Ρ‚ΠΎΠΊΠΎΠ² ΠΈ напряТСний производится с ΠΏΠΎΠΌΠΎΡ‰ΡŒΡŽ Π·Π°ΠΊΠΎΠ½ΠΎΠ² Ома ΠΈ ΠšΠΈΡ€Ρ…Π³ΠΎΡ„Π°.

ЭлСктричСская Ρ†Π΅ΠΏΡŒ с ΠΏΠΎΡΠ»Π΅Π΄ΠΎΠ²Π°Ρ‚Π΅Π»ΡŒΠ½Ρ‹ΠΌ соСдинСниСм элСмСнтов

Рис. 1.4

Рис. 1.5

ΠŸΠΎΡΠ»Π΅Π΄ΠΎΠ²Π°Ρ‚Π΅Π»ΡŒΠ½Ρ‹ΠΌ Π½Π°Π·Ρ‹Π²Π°ΡŽΡ‚ Ρ‚Π°ΠΊΠΎΠ΅ соСдинСниС элСмСнтов Ρ†Π΅ΠΏΠΈ, ΠΏΡ€ΠΈ ΠΊΠΎΡ‚ΠΎΡ€ΠΎΠΌ Π²ΠΎ всСх Π²ΠΊΠ»ΡŽΡ‡Π΅Π½Π½Ρ‹Ρ… Π² Ρ†Π΅ΠΏΡŒ элСмСнтах Π²ΠΎΠ·Π½ΠΈΠΊΠ°Π΅Ρ‚ ΠΎΠ΄ΠΈΠ½ ΠΈ Ρ‚ΠΎΡ‚ ΠΆΠ΅ Ρ‚ΠΎΠΊ I (рис. 1.4).

На основании Π²Ρ‚ΠΎΡ€ΠΎΠ³ΠΎ Π·Π°ΠΊΠΎΠ½Π° ΠšΠΈΡ€Ρ…Π³ΠΎΡ„Π° (1.5) ΠΎΠ±Ρ‰Π΅Π΅ напряТСниС U всСй Ρ†Π΅ΠΏΠΈ Ρ€Π°Π²Π½ΠΎ суммС напряТСний Π½Π° ΠΎΡ‚Π΄Π΅Π»ΡŒΠ½Ρ‹Ρ… участках:

Π’Π°ΠΊΠΈΠΌ ΠΎΠ±Ρ€Π°Π·ΠΎΠΌ, ΠΏΡ€ΠΈ ΠΏΠΎΡΠ»Π΅Π΄ΠΎΠ²Π°Ρ‚Π΅Π»ΡŒΠ½ΠΎΠΌ соСдинСнии элСмСнтов Ρ†Π΅ΠΏΠΈ ΠΎΠ±Ρ‰Π΅Π΅ эквивалСнтноС сопротивлСниС Ρ†Π΅ΠΏΠΈ Ρ€Π°Π²Π½ΠΎ арифмСтичСской суммС сопротивлСний ΠΎΡ‚Π΄Π΅Π»ΡŒΠ½Ρ‹Ρ… участков. Π‘Π»Π΅Π΄ΠΎΠ²Π°Ρ‚Π΅Π»ΡŒΠ½ΠΎ, Ρ†Π΅ΠΏΡŒ с Π»ΡŽΠ±Ρ‹ΠΌ числом ΠΏΠΎΡΠ»Π΅Π΄ΠΎΠ²Π°Ρ‚Π΅Π»ΡŒΠ½ΠΎ Π²ΠΊΠ»ΡŽΡ‡Π΅Π½Π½Ρ‹Ρ… сопротивлСний ΠΌΠΎΠΆΠ½ΠΎ Π·Π°ΠΌΠ΅Π½ΠΈΡ‚ΡŒ простой Ρ†Π΅ΠΏΡŒΡŽ с ΠΎΠ΄Π½ΠΈΠΌ эквивалСнтным сопротивлСниСм Rэкв (рис. 1.5). ПослС этого расчСт Ρ†Π΅ΠΏΠΈ сводится ΠΊ ΠΎΠΏΡ€Π΅Π΄Π΅Π»Π΅Π½ΠΈΡŽ Ρ‚ΠΎΠΊΠ° I всСй Ρ†Π΅ΠΏΠΈ ΠΏΠΎ Π·Π°ΠΊΠΎΠ½Ρƒ Ома

,

ΠΈ ΠΏΠΎ Π²Ρ‹ΡˆΠ΅ΠΏΡ€ΠΈΠ²Π΅Π΄Π΅Π½Π½Ρ‹ΠΌ Ρ„ΠΎΡ€ΠΌΡƒΠ»Π°ΠΌ Ρ€Π°ΡΡΡ‡ΠΈΡ‚Ρ‹Π²Π°ΡŽΡ‚ ΠΏΠ°Π΄Π΅Π½ΠΈΠ΅ напряТСний U1, U2, U3 Π½Π° ΡΠΎΠΎΡ‚Π²Π΅Ρ‚ΡΡ‚Π²ΡƒΡŽΡ‰ΠΈΡ… участках элСктричСской Ρ†Π΅ΠΏΠΈ (рис. 1.4).

НСдостаток ΠΏΠΎΡΠ»Π΅Π΄ΠΎΠ²Π°Ρ‚Π΅Π»ΡŒΠ½ΠΎΠ³ΠΎ Π²ΠΊΠ»ΡŽΡ‡Π΅Π½ΠΈΡ элСмСнтов Π·Π°ΠΊΠ»ΡŽΡ‡Π°Π΅Ρ‚ΡΡ Π² Ρ‚ΠΎΠΌ, Ρ‡Ρ‚ΠΎ ΠΏΡ€ΠΈ Π²Ρ‹Ρ…ΠΎΠ΄Π΅ ΠΈΠ· строя хотя Π±Ρ‹ ΠΎΠ΄Π½ΠΎΠ³ΠΎ элСмСнта, прСкращаСтся Ρ€Π°Π±ΠΎΡ‚Π° всСх ΠΎΡΡ‚Π°Π»ΡŒΠ½Ρ‹Ρ… элСмСнтов Ρ†Π΅ΠΏΠΈ.

ЭлСктричСская Ρ†Π΅ΠΏΡŒ с ΠΏΠ°Ρ€Π°Π»Π»Π΅Π»ΡŒΠ½Ρ‹ΠΌ соСдинСниСм элСмСнтов

ΠŸΠ°Ρ€Π°Π»Π»Π΅Π»ΡŒΠ½Ρ‹ΠΌ Π½Π°Π·Ρ‹Π²Π°ΡŽΡ‚ Ρ‚Π°ΠΊΠΎΠ΅ соСдинСниС, ΠΏΡ€ΠΈ ΠΊΠΎΡ‚ΠΎΡ€ΠΎΠΌ всС Π²ΠΊΠ»ΡŽΡ‡Π΅Π½Π½Ρ‹Π΅ Π² Ρ†Π΅ΠΏΡŒ ΠΏΠΎΡ‚Ρ€Π΅Π±ΠΈΡ‚Π΅Π»ΠΈ элСктричСской энСргии, находятся ΠΏΠΎΠ΄ ΠΎΠ΄Π½ΠΈΠΌ ΠΈ Ρ‚Π΅ΠΌ ΠΆΠ΅ напряТСниСм (рис. 1.6).

Π’ этом случаС ΠΎΠ½ΠΈ присоСдинСны ΠΊ Π΄Π²ΡƒΠΌ ΡƒΠ·Π»Π°ΠΌ Ρ†Π΅ΠΏΠΈ Π° ΠΈ b, ΠΈ Π½Π° основании ΠΏΠ΅Ρ€Π²ΠΎΠ³ΠΎ Π·Π°ΠΊΠΎΠ½Π° ΠšΠΈΡ€Ρ…Π³ΠΎΡ„Π° (1.3) ΠΌΠΎΠΆΠ½ΠΎ Π·Π°ΠΏΠΈΡΠ°Ρ‚ΡŒ, Ρ‡Ρ‚ΠΎ ΠΎΠ±Ρ‰ΠΈΠΉ Ρ‚ΠΎΠΊ I всСй Ρ†Π΅ΠΏΠΈ Ρ€Π°Π²Π΅Π½ алгСбраичСской суммС Ρ‚ΠΎΠΊΠΎΠ² ΠΎΡ‚Π΄Π΅Π»ΡŒΠ½Ρ‹Ρ… Π²Π΅Ρ‚Π²Π΅ΠΉ:

I = I1 + I2 + I3, Ρ‚.Π΅. ,

ΠΎΡ‚ΠΊΡƒΠ΄Π° слСдуСт, Ρ‡Ρ‚ΠΎ

.

Π’ Ρ‚ΠΎΠΌ случаС, ΠΊΠΎΠ³Π΄Π° ΠΏΠ°Ρ€Π°Π»Π»Π΅Π»ΡŒΠ½ΠΎ Π²ΠΊΠ»ΡŽΡ‡Π΅Π½Ρ‹ Π΄Π²Π° сопротивлСния R1 ΠΈ R2, ΠΎΠ½ΠΈ Π·Π°ΠΌΠ΅Π½ΡΡŽΡ‚ΡΡ ΠΎΠ΄Π½ΠΈΠΌ эквивалСнтным сопротивлСниСм

.

Из ΡΠΎΠΎΡ‚Π½ΠΎΡˆΠ΅Π½ΠΈΡ (1.6), слСдуСт, Ρ‡Ρ‚ΠΎ эквивалСнтная ΠΏΡ€ΠΎΠ²ΠΎΠ΄ΠΈΠΌΠΎΡΡ‚ΡŒ Ρ†Π΅ΠΏΠΈ Ρ€Π°Π²Π½Π° арифмСтичСской суммС проводимостСй ΠΎΡ‚Π΄Π΅Π»ΡŒΠ½Ρ‹Ρ… Π²Π΅Ρ‚Π²Π΅ΠΉ:

По ΠΌΠ΅Ρ€Π΅ роста числа ΠΏΠ°Ρ€Π°Π»Π»Π΅Π»ΡŒΠ½ΠΎ Π²ΠΊΠ»ΡŽΡ‡Π΅Π½Π½Ρ‹Ρ… ΠΏΠΎΡ‚Ρ€Π΅Π±ΠΈΡ‚Π΅Π»Π΅ΠΉ ΠΏΡ€ΠΎΠ²ΠΎΠ΄ΠΈΠΌΠΎΡΡ‚ΡŒ Ρ†Π΅ΠΏΠΈ gэкв возрастаСт, ΠΈ Π½Π°ΠΎΠ±ΠΎΡ€ΠΎΡ‚, ΠΎΠ±Ρ‰Π΅Π΅ сопротивлСниС Rэкв ΡƒΠΌΠ΅Π½ΡŒΡˆΠ°Π΅Ρ‚ΡΡ.

НапряТСния Π² элСктричСской Ρ†Π΅ΠΏΠΈ с ΠΏΠ°Ρ€Π°Π»Π»Π΅Π»ΡŒΠ½ΠΎ соСдинСнными сопротивлСниями (рис. 1.6)

ΠžΡ‚ΡΡŽΠ΄Π° слСдуСт, Ρ‡Ρ‚ΠΎ

,

Ρ‚.Π΅. Ρ‚ΠΎΠΊ Π² Ρ†Π΅ΠΏΠΈ распрСдСляСтся ΠΌΠ΅ΠΆΠ΄Ρƒ ΠΏΠ°Ρ€Π°Π»Π»Π΅Π»ΡŒΠ½Ρ‹ΠΌΠΈ вСтвями ΠΎΠ±Ρ€Π°Ρ‚Π½ΠΎ ΠΏΡ€ΠΎΠΏΠΎΡ€Ρ†ΠΈΠΎΠ½Π°Π»ΡŒΠ½ΠΎ ΠΈΡ… сопротивлСниям.

По ΠΏΠ°Ρ€Π°Π»Π»Π΅Π»ΡŒΠ½ΠΎ Π²ΠΊΠ»ΡŽΡ‡Π΅Π½Π½ΠΎΠΉ схСмС Ρ€Π°Π±ΠΎΡ‚Π°ΡŽΡ‚ Π² номинальном Ρ€Π΅ΠΆΠΈΠΌΠ΅ ΠΏΠΎΡ‚Ρ€Π΅Π±ΠΈΡ‚Π΅Π»ΠΈ любой мощности, рассчитанныС Π½Π° ΠΎΠ΄Π½ΠΎ ΠΈ Ρ‚ΠΎ ΠΆΠ΅ напряТСниС. ΠŸΡ€ΠΈΡ‡Π΅ΠΌ Π²ΠΊΠ»ΡŽΡ‡Π΅Π½ΠΈΠ΅ ΠΈΠ»ΠΈ ΠΎΡ‚ΠΊΠ»ΡŽΡ‡Π΅Π½ΠΈΠ΅ ΠΎΠ΄Π½ΠΎΠ³ΠΎ ΠΈΠ»ΠΈ Π½Π΅ΡΠΊΠΎΠ»ΡŒΠΊΠΈΡ… ΠΏΠΎΡ‚Ρ€Π΅Π±ΠΈΡ‚Π΅Π»Π΅ΠΉ Π½Π΅ отраТаСтся Π½Π° Ρ€Π°Π±ΠΎΡ‚Π΅ ΠΎΡΡ‚Π°Π»ΡŒΠ½Ρ‹Ρ…. ΠŸΠΎΡΡ‚ΠΎΠΌΡƒ эта схСма являСтся основной схСмой ΠΏΠΎΠ΄ΠΊΠ»ΡŽΡ‡Π΅Π½ΠΈΡ ΠΏΠΎΡ‚Ρ€Π΅Π±ΠΈΡ‚Π΅Π»Π΅ΠΉ ΠΊ источнику элСктричСской энСргии.

ЭлСктричСская Ρ†Π΅ΠΏΡŒ со ΡΠΌΠ΅ΡˆΠ°Π½Π½Ρ‹ΠΌ соСдинСниСм элСмСнтов

Π‘ΠΌΠ΅ΡˆΠ°Π½Π½Ρ‹ΠΌ называСтся Ρ‚Π°ΠΊΠΎΠ΅ соСдинСниС, ΠΏΡ€ΠΈ ΠΊΠΎΡ‚ΠΎΡ€ΠΎΠΌ Π² Ρ†Π΅ΠΏΠΈ ΠΈΠΌΠ΅ΡŽΡ‚ΡΡ Π³Ρ€ΡƒΠΏΠΏΡ‹ ΠΏΠ°Ρ€Π°Π»Π»Π΅Π»ΡŒΠ½ΠΎ ΠΈ ΠΏΠΎΡΠ»Π΅Π΄ΠΎΠ²Π°Ρ‚Π΅Π»ΡŒΠ½ΠΎ Π²ΠΊΠ»ΡŽΡ‡Π΅Π½Π½Ρ‹Ρ… сопротивлСний.

Для Ρ†Π΅ΠΏΠΈ, прСдставлСнной Π½Π° рис. 1.7, расчСт эквивалСнтного сопротивлСния начинаСтся с ΠΊΠΎΠ½Ρ†Π° схСмы. Для упрощСния расчСтов ΠΏΡ€ΠΈΠΌΠ΅ΠΌ, Ρ‡Ρ‚ΠΎ всС сопротивлСния Π² этой схСмС ΡΠ²Π»ΡΡŽΡ‚ΡΡ ΠΎΠ΄ΠΈΠ½Π°ΠΊΠΎΠ²Ρ‹ΠΌΠΈ: R1=R2=R3=R4=R5=R. БопротивлСния R4 ΠΈ R5 Π²ΠΊΠ»ΡŽΡ‡Π΅Π½Ρ‹ ΠΏΠ°Ρ€Π°Π»Π»Π΅Π»ΡŒΠ½ΠΎ, Ρ‚ΠΎΠ³Π΄Π° сопротивлСниС участка Ρ†Π΅ΠΏΠΈ cd Ρ€Π°Π²Π½ΠΎ:

.

Π’ этом случаС ΠΈΡΡ…ΠΎΠ΄Π½ΡƒΡŽ схСму (рис. 1.7) ΠΌΠΎΠΆΠ½ΠΎ ΠΏΡ€Π΅Π΄ΡΡ‚Π°Π²ΠΈΡ‚ΡŒ Π² ΡΠ»Π΅Π΄ΡƒΡŽΡ‰Π΅ΠΌ Π²ΠΈΠ΄Π΅ (рис. 1.8):

На схСмС (рис. 1.8) сопротивлСниС R3 ΠΈ Rcd соСдинСны ΠΏΠΎΡΠ»Π΅Π΄ΠΎΠ²Π°Ρ‚Π΅Π»ΡŒΠ½ΠΎ, ΠΈ Ρ‚ΠΎΠ³Π΄Π° сопротивлСниС участка Ρ†Π΅ΠΏΠΈ ad Ρ€Π°Π²Π½ΠΎ:

.

Π’ΠΎΠ³Π΄Π° схСму (рис. 1.8) ΠΌΠΎΠΆΠ½ΠΎ ΠΏΡ€Π΅Π΄ΡΡ‚Π°Π²ΠΈΡ‚ΡŒ Π² сокращСнном Π²Π°Ρ€ΠΈΠ°Π½Ρ‚Π΅ (рис. 1.9):

На схСмС (рис. 1.9) сопротивлСниС R2 ΠΈ Rad соСдинСны ΠΏΠ°Ρ€Π°Π»Π»Π΅Π»ΡŒΠ½ΠΎ, Ρ‚ΠΎΠ³Π΄Π° сопротивлСниС участка Ρ†Π΅ΠΏΠΈ Π°b Ρ€Π°Π²Π½ΠΎ

.

Π‘Ρ…Π΅ΠΌΡƒ (рис. 1.9) ΠΌΠΎΠΆΠ½ΠΎ ΠΏΡ€Π΅Π΄ΡΡ‚Π°Π²ΠΈΡ‚ΡŒ Π² ΡƒΠΏΡ€ΠΎΡ‰Π΅Π½Π½ΠΎΠΌ Π²Π°Ρ€ΠΈΠ°Π½Ρ‚Π΅ (рис. 1.10), Π³Π΄Π΅ сопротивлСния R1 ΠΈ Rab Π²ΠΊΠ»ΡŽΡ‡Π΅Π½Ρ‹ ΠΏΠΎΡΠ»Π΅Π΄ΠΎΠ²Π°Ρ‚Π΅Π»ΡŒΠ½ΠΎ.

Π’ΠΎΠ³Π΄Π° эквивалСнтноС сопротивлСниС исходной схСмы (рис. 1.7) Π±ΡƒΠ΄Π΅Ρ‚ Ρ€Π°Π²Π½ΠΎ:

.

Рис. 1.10

Рис. 1.11

Π’ Ρ€Π΅Π·ΡƒΠ»ΡŒΡ‚Π°Ρ‚Π΅ ΠΏΡ€Π΅ΠΎΠ±Ρ€Π°Π·ΠΎΠ²Π°Π½ΠΈΠΉ исходная схСма (рис. 1.7) прСдставлСна Π² Π²ΠΈΠ΄Π΅ схСмы (рис. 1.11) с ΠΎΠ΄Π½ΠΈΠΌ сопротивлСниСм Rэкв. РасчСт Ρ‚ΠΎΠΊΠΎΠ² ΠΈ напряТСний для всСх элСмСнтов схСмы ΠΌΠΎΠΆΠ½ΠΎ произвСсти ΠΏΠΎ Π·Π°ΠΊΠΎΠ½Π°ΠΌ Ома ΠΈ ΠšΠΈΡ€Ρ…Π³ΠΎΡ„Π°.

Π‘ΠΎΠ΅Π΄ΠΈΠ½Π΅Π½ΠΈΠ΅ элСмСнтов элСктричСской Ρ†Π΅ΠΏΠΈ ΠΏΠΎ схСмам Β«Π·Π²Π΅Π·Π΄Π°Β» ΠΈ Β«Ρ‚Ρ€Π΅ΡƒΠ³ΠΎΠ»ΡŒΠ½ΠΈΠΊΒ»

Π’ элСктротСхничСских ΠΈ элСктронных устройствах элСмСнты Ρ†Π΅ΠΏΠΈ ΡΠΎΠ΅Π΄ΠΈΠ½ΡΡŽΡ‚ΡΡ ΠΏΠΎ мостовой схСмС (рис. 1.12). БопротивлСния R12, R13, R24, R34 Π²ΠΊΠ»ΡŽΡ‡Π΅Π½Ρ‹ Π² ΠΏΠ»Π΅Ρ‡ΠΈ моста, Π² диагональ 1–4 Π²ΠΊΠ»ΡŽΡ‡Π΅Π½ источник питания с Π­Π”Π‘ Π•, другая диагональ 3–4 называСтся ΠΈΠ·ΠΌΠ΅Ρ€ΠΈΡ‚Π΅Π»ΡŒΠ½ΠΎΠΉ диагональю моста.

Рис. 1.12

Рис. 1.13

Π’ мостовой схСмС сопротивлСния R13, R12, R23 ΠΈ R24, R34, R23 соСдинСны ΠΏΠΎ схСмС Β«Ρ‚Ρ€Π΅ΡƒΠ³ΠΎΠ»ΡŒΠ½ΠΈΠΊΒ». Π­ΠΊΠ²ΠΈΠ²Π°Π»Π΅Π½Ρ‚Π½ΠΎΠ΅ сопротивлСниС этой схСмы ΠΌΠΎΠΆΠ½ΠΎ ΠΎΠΏΡ€Π΅Π΄Π΅Π»ΠΈΡ‚ΡŒ Ρ‚ΠΎΠ»ΡŒΠΊΠΎ послС Π·Π°ΠΌΠ΅Π½Ρ‹ ΠΎΠ΄Π½ΠΎΠ³ΠΎ ΠΈΠ· Ρ‚Ρ€Π΅ΡƒΠ³ΠΎΠ»ΡŒΠ½ΠΈΠΊΠΎΠ², Π½Π°ΠΏΡ€ΠΈΠΌΠ΅Ρ€ Ρ‚Ρ€Π΅ΡƒΠ³ΠΎΠ»ΡŒΠ½ΠΈΠΊΠ° R24 R34 R23 Π·Π²Π΅Π·Π΄ΠΎΠΉ R2 R3 R4 (рис. 1.13). Вакая Π·Π°ΠΌΠ΅Π½Π° Π±ΡƒΠ΄Π΅Ρ‚ эквивалСнтной, Ссли ΠΎΠ½Π° Π½Π΅ Π²Ρ‹Π·ΠΎΠ²Π΅Ρ‚ измСнСния Ρ‚ΠΎΠΊΠΎΠ² всСх ΠΎΡΡ‚Π°Π»ΡŒΠ½Ρ‹Ρ… элСмСнтов Ρ†Π΅ΠΏΠΈ. Для этого Π²Π΅Π»ΠΈΡ‡ΠΈΠ½Ρ‹ сопротивлСний Π·Π²Π΅Π·Π΄Ρ‹ Π΄ΠΎΠ»ΠΆΠ½Ρ‹ Ρ€Π°ΡΡΡ‡ΠΈΡ‚Ρ‹Π²Π°Ρ‚ΡŒΡΡ ΠΏΠΎ ΡΠ»Π΅Π΄ΡƒΡŽΡ‰ΠΈΠΌ ΡΠΎΠΎΡ‚Π½ΠΎΡˆΠ΅Π½ΠΈΡΠΌ:

; ; .

Для Π·Π°ΠΌΠ΅Π½Ρ‹ схСмы Β«Π·Π²Π΅Π·Π΄Π°Β» эквивалСнтным Ρ‚Ρ€Π΅ΡƒΠ³ΠΎΠ»ΡŒΠ½ΠΈΠΊΠΎΠΌ Π½Π΅ΠΎΠ±Ρ…ΠΎΠ΄ΠΈΠΌΠΎ Ρ€Π°ΡΡΡ‡ΠΈΡ‚Π°Ρ‚ΡŒ сопротивлСния Ρ‚Ρ€Π΅ΡƒΠ³ΠΎΠ»ΡŒΠ½ΠΈΠΊΠ°:

; ; .

ПослС ΠΏΡ€ΠΎΠ²Π΅Π΄Π΅Π½Π½Ρ‹Ρ… ΠΏΡ€Π΅ΠΎΠ±Ρ€Π°Π·ΠΎΠ²Π°Π½ΠΈΠΉ (рис. 1.13) ΠΌΠΎΠΆΠ½ΠΎ ΠΎΠΏΡ€Π΅Π΄Π΅Π»ΠΈΡ‚ΡŒ Π²Π΅Π»ΠΈΡ‡ΠΈΠ½Ρƒ эквивалСнтного сопротивлСния мостовой схСмы (рис. 1.12)

.

БопротивлСния Π² элСктричСских цСпях ΠΌΠΎΠ³ΡƒΡ‚ Π±Ρ‹Ρ‚ΡŒ соСдинСны ΠΏΠΎΡΠ»Π΅Π΄ΠΎΠ²Π°Ρ‚Π΅Π»ΡŒΠ½ΠΎ, ΠΏΠ°Ρ€Π°Π»Π»Π΅Π»ΡŒΠ½ΠΎ, ΠΏΠΎ смСшанной схСмС ΠΈ ΠΏΠΎ схСмам Β«Π·Π²Π΅Π·Π΄Π°Β», Β«Ρ‚Ρ€Π΅ΡƒΠ³ΠΎΠ»ΡŒΠ½ΠΈΠΊΒ». РасчСт слоТной схСмы упрощаСтся, Ссли сопротивлСния Π² этой схСмС Π·Π°ΠΌΠ΅Π½ΡΡŽΡ‚ΡΡ ΠΎΠ΄Π½ΠΈΠΌ эквивалСнтным сопротивлСниСм Rэкв, ΠΈ вся схСма прСдставляСтся Π² Π²ΠΈΠ΄Π΅ схСмы Π½Π° рис. 1.3, Π³Π΄Π΅ R=Rэкв, Π° расчСт Ρ‚ΠΎΠΊΠΎΠ² ΠΈ напряТСний производится с ΠΏΠΎΠΌΠΎΡ‰ΡŒΡŽ Π·Π°ΠΊΠΎΠ½ΠΎΠ² Ома ΠΈ ΠšΠΈΡ€Ρ…Π³ΠΎΡ„Π°.

ЭлСктричСская Ρ†Π΅ΠΏΡŒ с ΠΏΠΎΡΠ»Π΅Π΄ΠΎΠ²Π°Ρ‚Π΅Π»ΡŒΠ½Ρ‹ΠΌ соСдинСниСм элСмСнтов

Рис. 1.4

Рис. 1.5

ΠŸΠΎΡΠ»Π΅Π΄ΠΎΠ²Π°Ρ‚Π΅Π»ΡŒΠ½Ρ‹ΠΌ Π½Π°Π·Ρ‹Π²Π°ΡŽΡ‚ Ρ‚Π°ΠΊΠΎΠ΅ соСдинСниС элСмСнтов Ρ†Π΅ΠΏΠΈ, ΠΏΡ€ΠΈ ΠΊΠΎΡ‚ΠΎΡ€ΠΎΠΌ Π²ΠΎ всСх Π²ΠΊΠ»ΡŽΡ‡Π΅Π½Π½Ρ‹Ρ… Π² Ρ†Π΅ΠΏΡŒ элСмСнтах Π²ΠΎΠ·Π½ΠΈΠΊΠ°Π΅Ρ‚ ΠΎΠ΄ΠΈΠ½ ΠΈ Ρ‚ΠΎΡ‚ ΠΆΠ΅ Ρ‚ΠΎΠΊ I (рис. 1.4).

На основании Π²Ρ‚ΠΎΡ€ΠΎΠ³ΠΎ Π·Π°ΠΊΠΎΠ½Π° ΠšΠΈΡ€Ρ…Π³ΠΎΡ„Π° (1.5) ΠΎΠ±Ρ‰Π΅Π΅ напряТСниС U всСй Ρ†Π΅ΠΏΠΈ Ρ€Π°Π²Π½ΠΎ суммС напряТСний Π½Π° ΠΎΡ‚Π΄Π΅Π»ΡŒΠ½Ρ‹Ρ… участках:

Π’Π°ΠΊΠΈΠΌ ΠΎΠ±Ρ€Π°Π·ΠΎΠΌ, ΠΏΡ€ΠΈ ΠΏΠΎΡΠ»Π΅Π΄ΠΎΠ²Π°Ρ‚Π΅Π»ΡŒΠ½ΠΎΠΌ соСдинСнии элСмСнтов Ρ†Π΅ΠΏΠΈ ΠΎΠ±Ρ‰Π΅Π΅ эквивалСнтноС сопротивлСниС Ρ†Π΅ΠΏΠΈ Ρ€Π°Π²Π½ΠΎ арифмСтичСской суммС сопротивлСний ΠΎΡ‚Π΄Π΅Π»ΡŒΠ½Ρ‹Ρ… участков. Π‘Π»Π΅Π΄ΠΎΠ²Π°Ρ‚Π΅Π»ΡŒΠ½ΠΎ, Ρ†Π΅ΠΏΡŒ с Π»ΡŽΠ±Ρ‹ΠΌ числом ΠΏΠΎΡΠ»Π΅Π΄ΠΎΠ²Π°Ρ‚Π΅Π»ΡŒΠ½ΠΎ Π²ΠΊΠ»ΡŽΡ‡Π΅Π½Π½Ρ‹Ρ… сопротивлСний ΠΌΠΎΠΆΠ½ΠΎ Π·Π°ΠΌΠ΅Π½ΠΈΡ‚ΡŒ простой Ρ†Π΅ΠΏΡŒΡŽ с ΠΎΠ΄Π½ΠΈΠΌ эквивалСнтным сопротивлСниСм Rэкв (рис. 1.5). ПослС этого расчСт Ρ†Π΅ΠΏΠΈ сводится ΠΊ ΠΎΠΏΡ€Π΅Π΄Π΅Π»Π΅Π½ΠΈΡŽ Ρ‚ΠΎΠΊΠ° I всСй Ρ†Π΅ΠΏΠΈ ΠΏΠΎ Π·Π°ΠΊΠΎΠ½Ρƒ Ома

,

ΠΈ ΠΏΠΎ Π²Ρ‹ΡˆΠ΅ΠΏΡ€ΠΈΠ²Π΅Π΄Π΅Π½Π½Ρ‹ΠΌ Ρ„ΠΎΡ€ΠΌΡƒΠ»Π°ΠΌ Ρ€Π°ΡΡΡ‡ΠΈΡ‚Ρ‹Π²Π°ΡŽΡ‚ ΠΏΠ°Π΄Π΅Π½ΠΈΠ΅ напряТСний U1, U2, U3 Π½Π° ΡΠΎΠΎΡ‚Π²Π΅Ρ‚ΡΡ‚Π²ΡƒΡŽΡ‰ΠΈΡ… участках элСктричСской Ρ†Π΅ΠΏΠΈ (рис. 1.4).

НСдостаток ΠΏΠΎΡΠ»Π΅Π΄ΠΎΠ²Π°Ρ‚Π΅Π»ΡŒΠ½ΠΎΠ³ΠΎ Π²ΠΊΠ»ΡŽΡ‡Π΅Π½ΠΈΡ элСмСнтов Π·Π°ΠΊΠ»ΡŽΡ‡Π°Π΅Ρ‚ΡΡ Π² Ρ‚ΠΎΠΌ, Ρ‡Ρ‚ΠΎ ΠΏΡ€ΠΈ Π²Ρ‹Ρ…ΠΎΠ΄Π΅ ΠΈΠ· строя хотя Π±Ρ‹ ΠΎΠ΄Π½ΠΎΠ³ΠΎ элСмСнта, прСкращаСтся Ρ€Π°Π±ΠΎΡ‚Π° всСх ΠΎΡΡ‚Π°Π»ΡŒΠ½Ρ‹Ρ… элСмСнтов Ρ†Π΅ΠΏΠΈ.

ЭлСктричСская Ρ†Π΅ΠΏΡŒ с ΠΏΠ°Ρ€Π°Π»Π»Π΅Π»ΡŒΠ½Ρ‹ΠΌ соСдинСниСм элСмСнтов

ΠŸΠ°Ρ€Π°Π»Π»Π΅Π»ΡŒΠ½Ρ‹ΠΌ Π½Π°Π·Ρ‹Π²Π°ΡŽΡ‚ Ρ‚Π°ΠΊΠΎΠ΅ соСдинСниС, ΠΏΡ€ΠΈ ΠΊΠΎΡ‚ΠΎΡ€ΠΎΠΌ всС Π²ΠΊΠ»ΡŽΡ‡Π΅Π½Π½Ρ‹Π΅ Π² Ρ†Π΅ΠΏΡŒ ΠΏΠΎΡ‚Ρ€Π΅Π±ΠΈΡ‚Π΅Π»ΠΈ элСктричСской энСргии, находятся ΠΏΠΎΠ΄ ΠΎΠ΄Π½ΠΈΠΌ ΠΈ Ρ‚Π΅ΠΌ ΠΆΠ΅ напряТСниСм (рис. 1.6).

Π’ этом случаС ΠΎΠ½ΠΈ присоСдинСны ΠΊ Π΄Π²ΡƒΠΌ ΡƒΠ·Π»Π°ΠΌ Ρ†Π΅ΠΏΠΈ Π° ΠΈ b, ΠΈ Π½Π° основании ΠΏΠ΅Ρ€Π²ΠΎΠ³ΠΎ Π·Π°ΠΊΠΎΠ½Π° ΠšΠΈΡ€Ρ…Π³ΠΎΡ„Π° (1.3) ΠΌΠΎΠΆΠ½ΠΎ Π·Π°ΠΏΠΈΡΠ°Ρ‚ΡŒ, Ρ‡Ρ‚ΠΎ ΠΎΠ±Ρ‰ΠΈΠΉ Ρ‚ΠΎΠΊ I всСй Ρ†Π΅ΠΏΠΈ Ρ€Π°Π²Π΅Π½ алгСбраичСской суммС Ρ‚ΠΎΠΊΠΎΠ² ΠΎΡ‚Π΄Π΅Π»ΡŒΠ½Ρ‹Ρ… Π²Π΅Ρ‚Π²Π΅ΠΉ:

I = I1 + I2 + I3, Ρ‚.Π΅. ,

ΠΎΡ‚ΠΊΡƒΠ΄Π° слСдуСт, Ρ‡Ρ‚ΠΎ

.

Π’ Ρ‚ΠΎΠΌ случаС, ΠΊΠΎΠ³Π΄Π° ΠΏΠ°Ρ€Π°Π»Π»Π΅Π»ΡŒΠ½ΠΎ Π²ΠΊΠ»ΡŽΡ‡Π΅Π½Ρ‹ Π΄Π²Π° сопротивлСния R1 ΠΈ R2, ΠΎΠ½ΠΈ Π·Π°ΠΌΠ΅Π½ΡΡŽΡ‚ΡΡ ΠΎΠ΄Π½ΠΈΠΌ эквивалСнтным сопротивлСниСм

.

Из ΡΠΎΠΎΡ‚Π½ΠΎΡˆΠ΅Π½ΠΈΡ (1.6), слСдуСт, Ρ‡Ρ‚ΠΎ эквивалСнтная ΠΏΡ€ΠΎΠ²ΠΎΠ΄ΠΈΠΌΠΎΡΡ‚ΡŒ Ρ†Π΅ΠΏΠΈ Ρ€Π°Π²Π½Π° арифмСтичСской суммС проводимостСй ΠΎΡ‚Π΄Π΅Π»ΡŒΠ½Ρ‹Ρ… Π²Π΅Ρ‚Π²Π΅ΠΉ:

По ΠΌΠ΅Ρ€Π΅ роста числа ΠΏΠ°Ρ€Π°Π»Π»Π΅Π»ΡŒΠ½ΠΎ Π²ΠΊΠ»ΡŽΡ‡Π΅Π½Π½Ρ‹Ρ… ΠΏΠΎΡ‚Ρ€Π΅Π±ΠΈΡ‚Π΅Π»Π΅ΠΉ ΠΏΡ€ΠΎΠ²ΠΎΠ΄ΠΈΠΌΠΎΡΡ‚ΡŒ Ρ†Π΅ΠΏΠΈ gэкв возрастаСт, ΠΈ Π½Π°ΠΎΠ±ΠΎΡ€ΠΎΡ‚, ΠΎΠ±Ρ‰Π΅Π΅ сопротивлСниС Rэкв ΡƒΠΌΠ΅Π½ΡŒΡˆΠ°Π΅Ρ‚ΡΡ.

НапряТСния Π² элСктричСской Ρ†Π΅ΠΏΠΈ с ΠΏΠ°Ρ€Π°Π»Π»Π΅Π»ΡŒΠ½ΠΎ соСдинСнными сопротивлСниями (рис. 1.6)

ΠžΡ‚ΡΡŽΠ΄Π° слСдуСт, Ρ‡Ρ‚ΠΎ

,

Ρ‚.Π΅. Ρ‚ΠΎΠΊ Π² Ρ†Π΅ΠΏΠΈ распрСдСляСтся ΠΌΠ΅ΠΆΠ΄Ρƒ ΠΏΠ°Ρ€Π°Π»Π»Π΅Π»ΡŒΠ½Ρ‹ΠΌΠΈ вСтвями ΠΎΠ±Ρ€Π°Ρ‚Π½ΠΎ ΠΏΡ€ΠΎΠΏΠΎΡ€Ρ†ΠΈΠΎΠ½Π°Π»ΡŒΠ½ΠΎ ΠΈΡ… сопротивлСниям.

По ΠΏΠ°Ρ€Π°Π»Π»Π΅Π»ΡŒΠ½ΠΎ Π²ΠΊΠ»ΡŽΡ‡Π΅Π½Π½ΠΎΠΉ схСмС Ρ€Π°Π±ΠΎΡ‚Π°ΡŽΡ‚ Π² номинальном Ρ€Π΅ΠΆΠΈΠΌΠ΅ ΠΏΠΎΡ‚Ρ€Π΅Π±ΠΈΡ‚Π΅Π»ΠΈ любой мощности, рассчитанныС Π½Π° ΠΎΠ΄Π½ΠΎ ΠΈ Ρ‚ΠΎ ΠΆΠ΅ напряТСниС. ΠŸΡ€ΠΈΡ‡Π΅ΠΌ Π²ΠΊΠ»ΡŽΡ‡Π΅Π½ΠΈΠ΅ ΠΈΠ»ΠΈ ΠΎΡ‚ΠΊΠ»ΡŽΡ‡Π΅Π½ΠΈΠ΅ ΠΎΠ΄Π½ΠΎΠ³ΠΎ ΠΈΠ»ΠΈ Π½Π΅ΡΠΊΠΎΠ»ΡŒΠΊΠΈΡ… ΠΏΠΎΡ‚Ρ€Π΅Π±ΠΈΡ‚Π΅Π»Π΅ΠΉ Π½Π΅ отраТаСтся Π½Π° Ρ€Π°Π±ΠΎΡ‚Π΅ ΠΎΡΡ‚Π°Π»ΡŒΠ½Ρ‹Ρ…. ΠŸΠΎΡΡ‚ΠΎΠΌΡƒ эта схСма являСтся основной схСмой ΠΏΠΎΠ΄ΠΊΠ»ΡŽΡ‡Π΅Π½ΠΈΡ ΠΏΠΎΡ‚Ρ€Π΅Π±ΠΈΡ‚Π΅Π»Π΅ΠΉ ΠΊ источнику элСктричСской энСргии.

ЭлСктричСская Ρ†Π΅ΠΏΡŒ со ΡΠΌΠ΅ΡˆΠ°Π½Π½Ρ‹ΠΌ соСдинСниСм элСмСнтов

Π‘ΠΌΠ΅ΡˆΠ°Π½Π½Ρ‹ΠΌ называСтся Ρ‚Π°ΠΊΠΎΠ΅ соСдинСниС, ΠΏΡ€ΠΈ ΠΊΠΎΡ‚ΠΎΡ€ΠΎΠΌ Π² Ρ†Π΅ΠΏΠΈ ΠΈΠΌΠ΅ΡŽΡ‚ΡΡ Π³Ρ€ΡƒΠΏΠΏΡ‹ ΠΏΠ°Ρ€Π°Π»Π»Π΅Π»ΡŒΠ½ΠΎ ΠΈ ΠΏΠΎΡΠ»Π΅Π΄ΠΎΠ²Π°Ρ‚Π΅Π»ΡŒΠ½ΠΎ Π²ΠΊΠ»ΡŽΡ‡Π΅Π½Π½Ρ‹Ρ… сопротивлСний.

Для Ρ†Π΅ΠΏΠΈ, прСдставлСнной Π½Π° рис. 1.7, расчСт эквивалСнтного сопротивлСния начинаСтся с ΠΊΠΎΠ½Ρ†Π° схСмы. Для упрощСния расчСтов ΠΏΡ€ΠΈΠΌΠ΅ΠΌ, Ρ‡Ρ‚ΠΎ всС сопротивлСния Π² этой схСмС ΡΠ²Π»ΡΡŽΡ‚ΡΡ ΠΎΠ΄ΠΈΠ½Π°ΠΊΠΎΠ²Ρ‹ΠΌΠΈ: R1=R2=R3=R4=R5=R. БопротивлСния R4 ΠΈ R5 Π²ΠΊΠ»ΡŽΡ‡Π΅Π½Ρ‹ ΠΏΠ°Ρ€Π°Π»Π»Π΅Π»ΡŒΠ½ΠΎ, Ρ‚ΠΎΠ³Π΄Π° сопротивлСниС участка Ρ†Π΅ΠΏΠΈ cd Ρ€Π°Π²Π½ΠΎ:

.

Π’ этом случаС ΠΈΡΡ…ΠΎΠ΄Π½ΡƒΡŽ схСму (рис. 1.7) ΠΌΠΎΠΆΠ½ΠΎ ΠΏΡ€Π΅Π΄ΡΡ‚Π°Π²ΠΈΡ‚ΡŒ Π² ΡΠ»Π΅Π΄ΡƒΡŽΡ‰Π΅ΠΌ Π²ΠΈΠ΄Π΅ (рис. 1.8):

На схСмС (рис. 1.8) сопротивлСниС R3 ΠΈ Rcd соСдинСны ΠΏΠΎΡΠ»Π΅Π΄ΠΎΠ²Π°Ρ‚Π΅Π»ΡŒΠ½ΠΎ, ΠΈ Ρ‚ΠΎΠ³Π΄Π° сопротивлСниС участка Ρ†Π΅ΠΏΠΈ ad Ρ€Π°Π²Π½ΠΎ:

.

Π’ΠΎΠ³Π΄Π° схСму (рис. 1.8) ΠΌΠΎΠΆΠ½ΠΎ ΠΏΡ€Π΅Π΄ΡΡ‚Π°Π²ΠΈΡ‚ΡŒ Π² сокращСнном Π²Π°Ρ€ΠΈΠ°Π½Ρ‚Π΅ (рис. 1.9):

На схСмС (рис. 1.9) сопротивлСниС R2 ΠΈ Rad соСдинСны ΠΏΠ°Ρ€Π°Π»Π»Π΅Π»ΡŒΠ½ΠΎ, Ρ‚ΠΎΠ³Π΄Π° сопротивлСниС участка Ρ†Π΅ΠΏΠΈ Π°b Ρ€Π°Π²Π½ΠΎ

.

Π‘Ρ…Π΅ΠΌΡƒ (рис. 1.9) ΠΌΠΎΠΆΠ½ΠΎ ΠΏΡ€Π΅Π΄ΡΡ‚Π°Π²ΠΈΡ‚ΡŒ Π² ΡƒΠΏΡ€ΠΎΡ‰Π΅Π½Π½ΠΎΠΌ Π²Π°Ρ€ΠΈΠ°Π½Ρ‚Π΅ (рис. 1.10), Π³Π΄Π΅ сопротивлСния R1 ΠΈ Rab Π²ΠΊΠ»ΡŽΡ‡Π΅Π½Ρ‹ ΠΏΠΎΡΠ»Π΅Π΄ΠΎΠ²Π°Ρ‚Π΅Π»ΡŒΠ½ΠΎ.

Π’ΠΎΠ³Π΄Π° эквивалСнтноС сопротивлСниС исходной схСмы (рис. 1.7) Π±ΡƒΠ΄Π΅Ρ‚ Ρ€Π°Π²Π½ΠΎ:

.

Рис. 1.10

Рис. 1.11

Π’ Ρ€Π΅Π·ΡƒΠ»ΡŒΡ‚Π°Ρ‚Π΅ ΠΏΡ€Π΅ΠΎΠ±Ρ€Π°Π·ΠΎΠ²Π°Π½ΠΈΠΉ исходная схСма (рис. 1.7) прСдставлСна Π² Π²ΠΈΠ΄Π΅ схСмы (рис. 1.11) с ΠΎΠ΄Π½ΠΈΠΌ сопротивлСниСм Rэкв. РасчСт Ρ‚ΠΎΠΊΠΎΠ² ΠΈ напряТСний для всСх элСмСнтов схСмы ΠΌΠΎΠΆΠ½ΠΎ произвСсти ΠΏΠΎ Π·Π°ΠΊΠΎΠ½Π°ΠΌ Ома ΠΈ ΠšΠΈΡ€Ρ…Π³ΠΎΡ„Π°.

Π‘ΠΎΠ΅Π΄ΠΈΠ½Π΅Π½ΠΈΠ΅ элСмСнтов элСктричСской Ρ†Π΅ΠΏΠΈ ΠΏΠΎ схСмам Β«Π·Π²Π΅Π·Π΄Π°Β» ΠΈ Β«Ρ‚Ρ€Π΅ΡƒΠ³ΠΎΠ»ΡŒΠ½ΠΈΠΊΒ»

Π’ элСктротСхничСских ΠΈ элСктронных устройствах элСмСнты Ρ†Π΅ΠΏΠΈ ΡΠΎΠ΅Π΄ΠΈΠ½ΡΡŽΡ‚ΡΡ ΠΏΠΎ мостовой схСмС (рис. 1.12). БопротивлСния R12, R13, R24, R34 Π²ΠΊΠ»ΡŽΡ‡Π΅Π½Ρ‹ Π² ΠΏΠ»Π΅Ρ‡ΠΈ моста, Π² диагональ 1–4 Π²ΠΊΠ»ΡŽΡ‡Π΅Π½ источник питания с Π­Π”Π‘ Π•, другая диагональ 3–4 называСтся ΠΈΠ·ΠΌΠ΅Ρ€ΠΈΡ‚Π΅Π»ΡŒΠ½ΠΎΠΉ диагональю моста.

Рис. 1.12

Рис. 1.13

Π’ мостовой схСмС сопротивлСния R13, R12, R23 ΠΈ R24, R34, R23 соСдинСны ΠΏΠΎ схСмС Β«Ρ‚Ρ€Π΅ΡƒΠ³ΠΎΠ»ΡŒΠ½ΠΈΠΊΒ». Π­ΠΊΠ²ΠΈΠ²Π°Π»Π΅Π½Ρ‚Π½ΠΎΠ΅ сопротивлСниС этой схСмы ΠΌΠΎΠΆΠ½ΠΎ ΠΎΠΏΡ€Π΅Π΄Π΅Π»ΠΈΡ‚ΡŒ Ρ‚ΠΎΠ»ΡŒΠΊΠΎ послС Π·Π°ΠΌΠ΅Π½Ρ‹ ΠΎΠ΄Π½ΠΎΠ³ΠΎ ΠΈΠ· Ρ‚Ρ€Π΅ΡƒΠ³ΠΎΠ»ΡŒΠ½ΠΈΠΊΠΎΠ², Π½Π°ΠΏΡ€ΠΈΠΌΠ΅Ρ€ Ρ‚Ρ€Π΅ΡƒΠ³ΠΎΠ»ΡŒΠ½ΠΈΠΊΠ° R24 R34 R23 Π·Π²Π΅Π·Π΄ΠΎΠΉ R2 R3 R4 (рис. 1.13). Вакая Π·Π°ΠΌΠ΅Π½Π° Π±ΡƒΠ΄Π΅Ρ‚ эквивалСнтной, Ссли ΠΎΠ½Π° Π½Π΅ Π²Ρ‹Π·ΠΎΠ²Π΅Ρ‚ измСнСния Ρ‚ΠΎΠΊΠΎΠ² всСх ΠΎΡΡ‚Π°Π»ΡŒΠ½Ρ‹Ρ… элСмСнтов Ρ†Π΅ΠΏΠΈ. Для этого Π²Π΅Π»ΠΈΡ‡ΠΈΠ½Ρ‹ сопротивлСний Π·Π²Π΅Π·Π΄Ρ‹ Π΄ΠΎΠ»ΠΆΠ½Ρ‹ Ρ€Π°ΡΡΡ‡ΠΈΡ‚Ρ‹Π²Π°Ρ‚ΡŒΡΡ ΠΏΠΎ ΡΠ»Π΅Π΄ΡƒΡŽΡ‰ΠΈΠΌ ΡΠΎΠΎΡ‚Π½ΠΎΡˆΠ΅Π½ΠΈΡΠΌ:

; ; .

Для Π·Π°ΠΌΠ΅Π½Ρ‹ схСмы Β«Π·Π²Π΅Π·Π΄Π°Β» эквивалСнтным Ρ‚Ρ€Π΅ΡƒΠ³ΠΎΠ»ΡŒΠ½ΠΈΠΊΠΎΠΌ Π½Π΅ΠΎΠ±Ρ…ΠΎΠ΄ΠΈΠΌΠΎ Ρ€Π°ΡΡΡ‡ΠΈΡ‚Π°Ρ‚ΡŒ сопротивлСния Ρ‚Ρ€Π΅ΡƒΠ³ΠΎΠ»ΡŒΠ½ΠΈΠΊΠ°:

; ; .

ПослС ΠΏΡ€ΠΎΠ²Π΅Π΄Π΅Π½Π½Ρ‹Ρ… ΠΏΡ€Π΅ΠΎΠ±Ρ€Π°Π·ΠΎΠ²Π°Π½ΠΈΠΉ (рис. 1.13) ΠΌΠΎΠΆΠ½ΠΎ ΠΎΠΏΡ€Π΅Π΄Π΅Π»ΠΈΡ‚ΡŒ Π²Π΅Π»ΠΈΡ‡ΠΈΠ½Ρƒ эквивалСнтного сопротивлСния мостовой схСмы (рис. 1.12)

.

Расчёт элСктричСских схСм, содСрТащих нСсколько сопротивлСний (рСзисторов), ΠΏΡ€ΠΈ Π½Π°Ρ…ΠΎΠΆΠ΄Π΅Π½ΠΈΠΈ силы Ρ‚ΠΎΠΊΠ° Π² Ρ†Π΅ΠΏΠΈ, напряТСния ΠΈΠ»ΠΈ мощности, производится с использованиСм ΠΌΠ΅Ρ‚ΠΎΠ΄Π° свёртывания. ΠœΠ΅Ρ‚ΠΎΠ΄ Π·Π°ΠΊΠ»ΡŽΡ‡Π°Π΅Ρ‚ΡΡ Π² Ρ‚ΠΎΠΌ, Ρ‡Ρ‚ΠΎΠ±Ρ‹ Π½Π°ΠΉΡ‚ΠΈ эквивалСнтноС сопротивлСниС Π²Ρ‹Π΄Π΅Π»Π΅Π½Π½Ρ‹Ρ… участков Ρ†Π΅ΠΏΠΈ. Основная Π·Π°Π΄Π°Ρ‡Π° – Π·Π°ΠΌΠ΅Π½Π° рСзисторов, ΠΈΠΌΠ΅ΡŽΡ‰ΠΈΡ… Ρ€Π°Π·Π»ΠΈΡ‡Π½ΠΎΠ΅ ΠΏΠΎΠ΄ΠΊΠ»ΡŽΡ‡Π΅Π½ΠΈΠ΅ ΠΎΡ‚Π½ΠΎΡΠΈΡ‚Π΅Π»ΡŒΠ½ΠΎ Π΄Ρ€ΡƒΠ³ Π΄Ρ€ΡƒΠ³Π°, Π½Π° эквивалСнт (Rэкв.).

ΠžΠΏΡ€Π΅Π΄Π΅Π»Π΅Π½ΠΈΠ΅ эквивалСнтного сопротивлСния

ΠŸΡ€ΠΈ рассмотрСнии схСм Π»ΡŽΠ±Ρ‹Ρ… элСктричСских ΠΈΠ»ΠΈ элСктронных устройств ΠΌΠΎΠΆΠ½ΠΎ ΡƒΠ²ΠΈΠ΄Π΅Ρ‚ΡŒ, Ρ‡Ρ‚ΠΎ Ρ‚Π°ΠΊΠΈΠ΅ ΠΊΠΎΠΌΠΏΠΎΠ½Π΅Π½Ρ‚Ρ‹, ΠΊΠ°ΠΊ рСзисторы, ΠΈΠΌΠ΅ΡŽΡ‚ Ρ€Π°Π·Π½Ρ‹Π΅ Ρ‚ΠΈΠΏΡ‹ соСдинСний ΠΌΠ΅ΠΆΠ΄Ρƒ собой. Π§Ρ‚ΠΎΠ±Ρ‹ ΠΎΠΏΡ€Π΅Π΄Π΅Π»ΠΈΡ‚ΡŒ эквивалСнтноС соСдинСниС, Π½Π΅ΠΎΠ±Ρ…ΠΎΠ΄ΠΈΠΌΠΎ Ρ€Π°ΡΡΠΌΠ°Ρ‚Ρ€ΠΈΠ²Π°Ρ‚ΡŒ Π΄Π²Π° элСмСнта, Π²ΠΊΠ»ΡŽΡ‡Π΅Π½Π½Ρ‹Ρ… Π² ΠΎΠΏΡ€Π΅Π΄Π΅Π»Ρ‘Π½Π½ΠΎΠΌ порядкС. НСсмотря Π½Π° Ρ‚ΠΎ, Ρ‡Ρ‚ΠΎ Π½Π° Ρ‡Π΅Ρ€Ρ‚Π΅ΠΆΠ΅ ΠΈΡ… ΠΌΠΎΠΆΠ΅Ρ‚ Π±Ρ‹Ρ‚ΡŒ нСсколько дСсятков, ΠΈ соСдинСны ΠΎΠ½ΠΈ ΠΏΠΎ-Ρ€Π°Π·Π½ΠΎΠΌΡƒ, Π΅ΡΡ‚ΡŒ Ρ‚ΠΎΠ»ΡŒΠΊΠΎ Π΄Π²Π° Ρ‚ΠΈΠΏΠ° Π²ΠΊΠ»ΡŽΡ‡Π΅Π½ΠΈΡ ΠΈΡ… Π΄Ρ€ΡƒΠ³ с Π΄Ρ€ΡƒΠ³ΠΎΠΌ: ΠΏΠΎΡΠ»Π΅Π΄ΠΎΠ²Π°Ρ‚Π΅Π»ΡŒΠ½ΠΎΠ΅ ΠΈ ΠΏΠ°Ρ€Π°Π»Π»Π΅Π»ΡŒΠ½ΠΎΠ΅. ΠžΡΡ‚Π°Π»ΡŒΠ½Ρ‹Π΅ ΠΊΠΎΠ½Ρ„ΠΈΠ³ΡƒΡ€Π°Ρ†ΠΈΠΈ – это лишь ΠΈΡ… Π²Π°Ρ€ΠΈΠ°Ρ†ΠΈΠΈ.

ΠŸΠΎΡΠ»Π΅Π΄ΠΎΠ²Π°Ρ‚Π΅Π»ΡŒΠ½ΠΎΠ΅ соСдинСниС элСмСнтов

ПодобноС Π²ΠΊΠ»ΡŽΡ‡Π΅Π½ΠΈΠ΅ ΠΏΠΎΠ΄Ρ€Π°Π·ΡƒΠΌΠ΅Π²Π°Π΅Ρ‚ ΠΊΠΎΠΌΠ±ΠΈΠ½Π°Ρ†ΠΈΡŽ Π΄Π΅Ρ‚Π°Π»Π΅ΠΉ Π² прямой ΠΏΠΎΡΠ»Π΅Π΄ΠΎΠ²Π°Ρ‚Π΅Π»ΡŒΠ½ΠΎΡΡ‚ΠΈ. Π’Ρ‹Ρ…ΠΎΠ΄ ΠΎΠ΄Π½ΠΎΠ³ΠΎ сопротивлСния ΠΏΠΎΠ΄ΠΊΠ»ΡŽΡ‡Π°Π΅Ρ‚ΡΡ ΠΊ Π²Ρ…ΠΎΠ΄Ρƒ Π΄Ρ€ΡƒΠ³ΠΎΠ³ΠΎ. ΠŸΡ€ΠΈ этом ΠΎΡ‚ΡΡƒΡ‚ΡΡ‚Π²ΡƒΡŽΡ‚ ΠΊΠ°ΠΊΠΈΠ΅-Π»ΠΈΠ±ΠΎ отвСтвлСния Π½Π° участкС. Π’Π΅Π»ΠΈΡ‡ΠΈΠ½Π° Ρ‚ΠΎΠΊΠ°, ΠΊΠΎΡ‚ΠΎΡ€Ρ‹ΠΉ ΠΏΡ€ΠΎΡ…ΠΎΠ΄ΠΈΡ‚ Ρ‡Π΅Ρ€Π΅Π· всС соСдинённыС ΠΏΠΎΡΠ»Π΅Π΄ΠΎΠ²Π°Ρ‚Π΅Π»ΡŒΠ½ΠΎ ΠΊΠΎΠΌΠΏΠΎΠ½Π΅Π½Ρ‚Ρ‹, Π±ΡƒΠ΄Π΅Ρ‚ ΠΎΠ΄Π½Π° ΠΈ Ρ‚Π° ΠΆΠ΅.

Π’Π½ΠΈΠΌΠ°Π½ΠΈΠ΅! Π‘Π½ΠΈΠΆΠ΅Π½ΠΈΠ΅ ΠΏΠΎΡ‚Π΅Π½Ρ†ΠΈΠ°Π»Π° Π½Π° ΠΊΠ°ΠΆΠ΄ΠΎΠΌ рСзистивном элСмСнтС Π² суммС даст ΠΏΠΎΠ»Π½ΠΎΠ΅ напряТСниС, ΠΏΡ€ΠΈΠ»ΠΎΠΆΠ΅Π½Π½ΠΎΠ΅ ΠΊ ΠΏΠΎΡΠ»Π΅Π΄ΠΎΠ²Π°Ρ‚Π΅Π»ΡŒΠ½ΠΎΠΉ Ρ†Π΅ΠΏΠΈ.

Π’ случаС постоянного Ρ‚ΠΎΠΊΠ° Ρ„ΠΎΡ€ΠΌΡƒΠ»Π° Π·Π°ΠΊΠΎΠ½Π° Ома для ΠΎΡ‚Ρ€Π΅Π·ΠΊΠ° Ρ†Π΅ΠΏΠΈ ΠΈΠΌΠ΅Π΅Ρ‚ Π²ΠΈΠ΄:

Π‘ΠΈΠ»Π° Ρ‚ΠΎΠΊΠ° зависит ΠΎΡ‚ ΠΏΡ€ΠΈΠ»ΠΎΠΆΠ΅Π½Π½ΠΎΠ³ΠΎ напряТСния ΠΈ ΠΎΠΊΠ°Π·Π°Π½Π½ΠΎΠ³ΠΎ Π΅ΠΌΡƒ сопротивлСния. Если Π²Ρ‹Ρ€Π°Π·ΠΈΡ‚ΡŒ R, Π΅Π³ΠΎ Ρ„ΠΎΡ€ΠΌΡƒΠ»Π°:

ΠŸΠ°Ρ€Π°ΠΌΠ΅Ρ‚Ρ€Ρ‹ ΠΏΠΎΡΠ»Π΅Π΄ΠΎΠ²Π°Ρ‚Π΅Π»ΡŒΠ½ΠΎΠΉ Ρ†Π΅ΠΏΠΈ, Π²ΠΊΠ»ΡŽΡ‡Π°ΡŽΡ‰Π΅ΠΉ n соСдинённых Π΄Ρ€ΡƒΠ³ с Π΄Ρ€ΡƒΠ³ΠΎΠΌ элСмСнтов, ΠΈΠΌΠ΅ΡŽΡ‚ свои особСнности.

ΠŸΡ€ΠΎΡ…ΠΎΠ΄ΡΡ‰ΠΈΠΉ ΠΏΠΎ Ρ†Π΅ΠΏΠΈ Ρ‚ΠΎΠΊ Π²Π΅Π·Π΄Π΅ ΠΎΠ΄ΠΈΠ½Π°ΠΊΠΎΠ²Ρ‹ΠΉ:

ΠŸΡ€ΠΈΠΊΠ»Π°Π΄Ρ‹Π²Π°Π΅ΠΌΠΎΠ΅ напряТСниС являСтся суммой напряТСний Π½Π° ΠΊΠ°ΠΆΠ΄ΠΎΠΌ рСзисторС:

Π‘Π»Π΅Π΄ΠΎΠ²Π°Ρ‚Π΅Π»ΡŒΠ½ΠΎ, Ρ€Π°ΡΡΡ‡ΠΈΡ‚Π°Ρ‚ΡŒ ΠΌΠΎΠΆΠ½ΠΎ ΠΎΠ±Ρ‰Π΅Π΅:

Rэкв.= U1/I + U2/I + … +Un/I) = R1 + R2 + … +Rn.

Π’Π°ΠΆΠ½ΠΎ! ΠŸΠΎΡΠ»Π΅Π΄ΠΎΠ²Π°Ρ‚Π΅Π»ΡŒΠ½Π°Ρ Ρ†Π΅ΠΏΡŒ, ΠΈΠΌΠ΅ΡŽΡ‰Π°Ρ Π² своём составС N рСзисторов Ρ€Π°Π²Π½ΠΎΠ³ΠΎ Π½ΠΎΠΌΠΈΠ½Π°Π»Π°, ΠΈΠΌΠ΅Π΅Ρ‚ эквивалСнтноС сопротивлСниС Rэкв. = N*R.

ΠŸΠ°Ρ€Π°Π»Π»Π΅Π»ΡŒΠ½ΠΎΠ΅ соСдинСниС

Когда условныС Π²Ρ‹Ρ…ΠΎΠ΄Ρ‹ Π΄Π΅Ρ‚Π°Π»Π΅ΠΉ ΠΈΠΌΠ΅ΡŽΡ‚ ΠΎΠ±Ρ‰ΠΈΠΉ ΠΊΠΎΠ½Ρ‚Π°ΠΊΡ‚ Π² ΠΎΠ΄Π½ΠΎΠΉ Ρ‚ΠΎΡ‡ΠΊΠ΅ (ΡƒΠ·Π»Π΅) схСмы, Π° условныС Π²Ρ…ΠΎΠ΄Ρ‹ Ρ‚Π°ΠΊ ΠΆΠ΅ ΠΎΠ±ΡŠΠ΅Π΄ΠΈΠ½Π΅Π½Ρ‹ Π²ΠΎ Π²Ρ‚ΠΎΡ€ΠΎΠΉ, говорят ΠΎ ΠΏΠ°Ρ€Π°Π»Π»Π΅Π»ΡŒΠ½ΠΎΠΌ соСдинСнии. Π£Π·Π΅Π» Π½Π° Ρ‡Π΅Ρ€Ρ‚Π΅ΠΆΠ΅ обозначаСтся графичСской Ρ‚ΠΎΡ‡ΠΊΠΎΠΉ. Π­Ρ‚ΠΎ мСсто, Π³Π΄Π΅ происходят развСтвлСния Ρ†Π΅ΠΏΠ΅ΠΉ Π² схСмах. Π’Π°ΠΊΠΎΠΉ Π²Π°Ρ€ΠΈΠ°Π½Ρ‚ ΠΏΠΎΠ΄ΠΊΠ»ΡŽΡ‡Π΅Π½ΠΈΡ рСзисторов обСспСчиваСт ΠΎΠ΄ΠΈΠ½Π°ΠΊΠΎΠ²ΠΎΠ΅ ΠΏΠ°Π΄Π΅Π½ΠΈΠ΅ напряТСния U для всСх ΠΏΠ°Ρ€Π°Π»Π»Π΅Π»ΡŒΠ½Ρ‹Ρ… элСмСнтов. Π’ΠΎΠΊ Π² этой ΠΏΠΎΠ·ΠΈΡ†ΠΈΠΈ Π±ΡƒΠ΄Π΅Ρ‚ Ρ€Π°Π²Π΅Π½ суммС Ρ‚ΠΎΠΊΠΎΠ², ΠΈΠ΄ΡƒΡ‰ΠΈΡ… ΠΏΠΎ ΠΊΠ°ΠΆΠ΄ΠΎΠΌΡƒ ΠΊΠΎΠΌΠΏΠΎΠ½Π΅Π½Ρ‚Ρƒ.

Когда Π² ΠΏΠ°Ρ€Π°Π»Π»Π΅Π»ΡŒΠ½ΠΎΠ΅ ΠΏΠΎΠ΄ΠΊΠ»ΡŽΡ‡Π΅Π½ΠΈΠ΅ Π²Ρ…ΠΎΠ΄ΠΈΡ‚ n рСзистивных элСмСнтов, Ρ‚ΠΎ Ρ€Π°Π·Π½ΠΎΡΡ‚ΡŒ ΠΏΠΎΡ‚Π΅Π½Ρ†ΠΈΠ°Π»ΠΎΠ², Ρ‚ΠΎΠΊ ΠΈ ΠΎΠ±Ρ‰Π΅Π΅ сопротивлСниС Π±ΡƒΠ΄ΡƒΡ‚ ΠΈΠΌΠ΅Ρ‚ΡŒ ΡΠ»Π΅Π΄ΡƒΡŽΡ‰ΠΈΠ΅ выраТСния:

  • ΠΎΠ±Ρ‰ΠΈΠΉ Ρ‚ΠΎΠΊ: I = I1 + I2 + … + In;
  • ΠΎΠ±Ρ‰Π΅Π΅ напряТСниС: U = U1 = U2 = … = Un;
  • RΠΎΠ±Ρ‰. = Rэкв. = U/I1 + U/I2 + …+ U/In) = 1/R1 + 1/R2 +…+ 1/Rn.

Π’Π΅Π»ΠΈΡ‡ΠΈΠ½Ρƒ, ΠΎΠ±Ρ€Π°Ρ‚Π½ΠΎ ΠΏΡ€ΠΎΠΏΠΎΡ€Ρ†ΠΈΠΎΠ½Π°Π»ΡŒΠ½ΡƒΡŽ ΡΠΎΠΏΡ€ΠΎΡ‚ΠΈΠ²Π»Π΅Π½ΠΈΡŽ 1/R, Π½Π°Π·Ρ‹Π²Π°ΡŽΡ‚ ΠΏΡ€ΠΎΠ²ΠΎΠ΄ΠΈΠΌΠΎΡΡ‚ΡŒΡŽ.

Если n Ρ€Π°Π²Π½Ρ‹Ρ… ΠΏΠΎ Π½ΠΎΠΌΠΈΠ½Π°Π»Ρƒ сопротивлСний Π²ΠΊΠ»ΡŽΡ‡ΠΈΡ‚ΡŒ ΠΏΠ°Ρ€Π°Π»Π»Π΅Π»ΡŒΠ½ΠΎ, Ρ‚ΠΎ Rэкв. = (R*R)/n*R = R/n. Π€ΠΎΡ€ΠΌΡƒΠ»Π° ΠΏΠΎΠ΄Ρ…ΠΎΠ΄ΠΈΡ‚ ΠΈ для ΠΈΠ½Π΄ΡƒΠΊΡ‚ΠΈΠ²Π½Ρ‹Ρ… сопротивлСний ΠΏΡ€ΠΎΠ²ΠΎΠ»ΠΎΡ‡Π½Ρ‹Ρ… ΠΊΠ°Ρ‚ΡƒΡˆΠ΅ΠΊ ΠΈ ёмкостных сопротивлСний кондСнсаторов.

Расчёт ΠΏΡ€ΠΈ смСшанном соСдинСнии устройств

ΠŸΡ€ΠΎΠΈΠ·Π²Π΅ΡΡ‚ΠΈ расчСт сопротивлСния Ρ†Π΅ΠΏΠΈ, ΠΊΠΎΠ³Π΄Π° ΠΎΠ½Π° Ρ€Π°Π·Π²Π΅Ρ‚Π²Π»Π΅Π½Π° ΠΈ Π½Π°ΠΏΠΎΠ»Π½Π΅Π½Π° Ρ€Π°Π·Π½Ρ‹ΠΌΠΈ Π²ΠΈΠ΄Π°ΠΌΠΈ рСзистивных соСдинСний, просто Π½Π΅ получится. ЗатрудняСт Ρ€Π΅ΡˆΠ΅Π½ΠΈΠ΅ Π·Π°Π΄Π°Ρ‡ΠΈ мноТСство участков, Π³Π΄Π΅ Π΄Π΅Ρ‚Π°Π»ΠΈ ΠΏΠΎΠ΄ΠΊΠ»ΡŽΡ‡Π΅Π½Ρ‹ Π΄Ρ€ΡƒΠ³ Π΄Ρ€ΡƒΠ³Ρƒ Π² Ρ€Π°Π·Π½Ρ‹Ρ… комбинациях. Π’ Ρ‚Π°ΠΊΠΈΡ… ΠΎΠ±ΡΡ‚ΠΎΡΡ‚Π΅Π»ΡŒΡΡ‚Π²Π°Ρ… ΠΆΠ΅Π»Π°Ρ‚Π΅Π»ΡŒΠ½ΠΎ Π²Ρ‹ΠΏΠΎΠ»Π½ΡΡ‚ΡŒ ряд ΠΏΡ€Π΅ΠΎΠ±Ρ€Π°Π·ΠΎΠ²Π°Π½ΠΈΠΉ, добиваясь упрощСния схСмы Π²Π²ΠΎΠ΄ΠΎΠΌ ΠΎΡ‚Π΄Π΅Π»ΡŒΠ½Ρ‹Ρ… эквивалСнтных элСмСнтов. Π’Ρ‹ΡΠ²Π»ΡΡŽΡ‚ ΠΏΡ€ΠΈ этом подходящиС ΠΊΠΎΠ½Ρ‚ΡƒΡ€Ρ‹ ΠΏΠΎΡΠ»Π΅Π΄ΠΎΠ²Π°Ρ‚Π΅Π»ΡŒΠ½Ρ‹Ρ… ΠΈ ΠΏΠ°Ρ€Π°Π»Π»Π΅Π»ΡŒΠ½Ρ‹Ρ… присоСдинСний.

НапримСр, выискав Π½Π΅ΠΊΠΎΡ‚ΠΎΡ€ΠΎΠ΅ количСство ΠΏΠΎΡΠ»Π΅Π΄ΠΎΠ²Π°Ρ‚Π΅Π»ΡŒΠ½Ρ‹Ρ… ΠΏΠΎΠ΄ΠΊΠ»ΡŽΡ‡Π΅Π½ΠΈΠΉ рСзисторов, Π·Π°ΠΌΠ΅Π½ΡΡŽΡ‚ ΠΈΡ… Π½Π° ΠΎΠ΄ΠΈΠ½ эквивалСнтный ΠΊΠΎΠΌΠΏΠΎΠ½Π΅Π½Ρ‚. ΠžΠΏΡ€Π΅Π΄Π΅Π»ΠΈΠ² элСмСнты, соСдинённыС ΠΏΠΎΡΠ»Π΅Π΄ΠΎΠ²Π°Ρ‚Π΅Π»ΡŒΠ½ΠΎ, Ρ‚Π°ΠΊΠΆΠ΅ Ρ€ΠΈΡΡƒΡŽΡ‚ вмСсто Π½Π΅Π³ΠΎ эквивалСнт. Π’Π½ΠΎΠ²ΡŒ Π½Π°Ρ‡ΠΈΠ½Π°ΡŽΡ‚ ΠΈΡΠΊΠ°Ρ‚ΡŒ ΠΏΠΎΠ΄ΠΎΠ±Π½Ρ‹Π΅ простыС соСдинСния.

ΠœΠ΅Ρ‚ΠΎΠ΄ Π½Π°Π·Ρ‹Π²Π°ΡŽΡ‚ Β«ΠΌΠ΅Ρ‚ΠΎΠ΄ΠΎΠΌ свёртывания». Π‘Ρ…Π΅ΠΌΡƒ ΡƒΠΏΡ€ΠΎΡ‰Π°ΡŽΡ‚ Π΄ΠΎ Ρ‚Π΅Ρ… ΠΏΠΎΡ€, ΠΏΠΎΠΊΠ° Π² Π½Π΅ΠΉ Π½Π΅ останСтся ΠΎΠ΄Π½ΠΎ Rэкв.

Π’Π°ΠΆΠ½ΠΎ! ΠœΠ΅Ρ‚ΠΎΠ΄ эквивалСнтных ΠΏΡ€Π΅ΠΎΠ±Ρ€Π°Π·ΠΎΠ²Π°Π½ΠΈΠΉ примСняСтся Ρ‚ΠΎΠ³Π΄Π°, ΠΊΠΎΠ³Π΄Π° ΠΏΠΈΡ‚Π°Π½ΠΈΠ΅ рассматриваСмого участка Ρ†Π΅ΠΏΠΈ осущСствляСтся ΠΎΡ‚ ΠΎΠ΄Π½ΠΎΠ³ΠΎ источника элСктричСского Ρ‚ΠΎΠΊΠ°, Π° Ρ‚Π°ΠΊΠΆΠ΅ ΠΏΡ€ΠΈ ΠΎΠΏΡ€Π΅Π΄Π΅Π»Π΅Π½ΠΈΠΈ Rэкв. Π² Π·Π°ΠΌΠΊΠ½ΡƒΡ‚ΠΎΠΌ ΠΊΠΎΠ½Ρ‚ΡƒΡ€Π΅ с ΠΎΠ΄Π½ΠΎΠΉ Π­Π”Π‘.

Π’Π°ΠΊΠΎΠΉ ΠΎΡ‚Π½ΠΎΡΠΈΡ‚Π΅Π»ΡŒΠ½Ρ‹ΠΉ способ опрСдСлСния Rэкв ΠΈΡΠΏΠΎΠ»ΡŒΠ·ΡƒΡŽΡ‚ ΠΈ для изучСния зависимости Ρ‚ΠΎΠΊΠΎΠ² Π² Π½Π΅ΠΊΠΎΡ‚ΠΎΡ€ΠΎΠΉ Ρ†Π΅ΠΏΠΈ ΠΎΡ‚ значСния R Π½Π°Π³Ρ€ΡƒΠ·ΠΊΠΈ. Π­Ρ‚ΠΎ ΠΌΠ΅Ρ‚ΠΎΠ΄ эквивалСнтного Π³Π΅Π½Π΅Ρ€Π°Ρ‚ΠΎΡ€Π°, ΠΏΡ€ΠΈ ΠΊΠΎΡ‚ΠΎΡ€ΠΎΠΌ слоТный Π΄Π²ΡƒΡ…ΠΏΠΎΠ»ΡŽΡΠ½ΠΈΠΊ, ΡΠ²Π»ΡΡŽΡ‰ΠΈΠΉΡΡ Π°ΠΊΡ‚ΠΈΠ²Π½Ρ‹ΠΌ, ΠΏΡ€Π΅Π΄ΡΡ‚Π°Π²Π»ΡΡŽΡ‚ эквивалСнтным Π³Π΅Π½Π΅Ρ€Π°Ρ‚ΠΎΡ€ΠΎΠΌ. ΠŸΡ€ΠΈ этом ΡΡ‡ΠΈΡ‚Π°ΡŽΡ‚, Ρ‡Ρ‚ΠΎ Π­Π”Π‘ Π΅Π³ΠΎ соотвСтствуСт UΡ….Ρ…. (холостого Ρ…ΠΎΠ΄Π°) Π½Π° Π·Π°ΠΆΠΈΠΌΠ°Ρ…, R Π²Π½ΡƒΡ‚Ρ€Π΅Π½Π½Π΅Π΅ соотвСтствуСт R Π²Ρ…ΠΎΠ΄Π½ΠΎΠΌΡƒ Π΄Π²ΡƒΡ…ΠΏΠΎΠ»ΡŽΡΠ½ΠΈΠΊΠ° пассивного Π½Π° Ρ‚Π΅Ρ… ΠΆΠ΅ Π·Π°ΠΆΠΈΠΌΠ°Ρ…. Для Ρ‚Π°ΠΊΠΎΠ³ΠΎ опрСдСлСния источники Ρ‚ΠΎΠΊΠ° Ρ€Π°Π·ΡŠΠ΅Π΄ΠΈΠ½ΡΡŽΡ‚, Π° ΠΊΠ°Π½Π°Π» Π­Π”Π‘ Π·Π°ΠΊΠΎΡ€Π°Ρ‡ΠΈΠ²Π°ΡŽΡ‚.

ЀизичСскиС Ρ„ΠΎΡ€ΠΌΡƒΠ»Ρ‹ ΠΈ ΠΏΡ€ΠΈΠΌΠ΅Ρ€Ρ‹ вычислСний

Π€ΠΎΡ€ΠΌΡƒΠ»Ρ‹ для эквивалСнтных сопротивлСний Ρ†Π΅ΠΏΠΈ, состоящСй ΠΈΠ· ΠΏΠ°Ρ€Ρ‹ рСзисторов R1 ΠΈ R2, ΠΌΠΎΠΆΠ½ΠΎ Π²Ρ‹Π΄Π΅Π»ΠΈΡ‚ΡŒ Π² ΠΎΠΏΡ€Π΅Π΄Π΅Π»Ρ‘Π½Π½Ρ‹ΠΉ ряд:

  • ΠΏΠ°Ρ€Π°Π»Π»Π΅Π»ΡŒΠ½ΠΎΠ΅ присоСдинСниС ΠΎΠΏΡ€Π΅Π΄Π΅Π»ΡΡŽΡ‚ ΠΏΠΎ Ρ„ΠΎΡ€ΠΌΡƒΠ»Π΅ Rэкв. = (R1*R2)/R1+R2;
  • ΠΏΠΎΡΠ»Π΅Π΄ΠΎΠ²Π°Ρ‚Π΅Π»ΡŒΠ½ΠΎΠ΅ Π²ΠΊΠ»ΡŽΡ‡Π΅Π½ΠΈΠ΅ Π²Ρ‹Ρ‡ΠΈΡΠ»ΡΡŽΡ‚, опрСдСляя Π΅Π³ΠΎ сумму Rэкв. = R1+R2.

Π£ смСшанного соСдинСния рСзистивных элСмСнтов Π½Π΅Ρ‚ ΠΊΠΎΠ½ΠΊΡ€Π΅Ρ‚Π½ΠΎΠΉ Ρ„ΠΎΡ€ΠΌΡƒΠ»Ρ‹. Π§Ρ‚ΠΎΠ±Ρ‹ Π½Π΅ Π·Π°ΠΏΡƒΡ‚Π°Ρ‚ΡŒΡΡ ΠΏΡ€ΠΈ Π΄Π»ΠΈΡ‚Π΅Π»ΡŒΠ½Ρ‹Ρ… прСобразованиях, здСсь допустимо Π²ΠΎΡΠΏΠΎΠ»ΡŒΠ·ΠΎΠ²Π°Ρ‚ΡŒΡΡ ΡΠΏΠ΅Ρ†ΠΈΠ°Π»ΡŒΠ½ΠΎΠΉ ΠΏΡ€ΠΎΠ³Ρ€Π°ΠΌΠΌΠΎΠΉ ΠΈΠ· ΠΈΠ½Ρ‚Π΅Ρ€Π½Π΅Ρ‚Π°. Π­Ρ‚ΠΎ сСрвис Β«ΠΎΠ½Π»Π°ΠΉΠ½-ΠΊΠ°Π»ΡŒΠΊΡƒΠ»ΡΡ‚ΠΎΡ€Β». Он ΠΏΠΎΠΌΠΎΠΆΠ΅Ρ‚ Ρ€Π°Π·ΠΎΠ±Ρ€Π°Ρ‚ΡŒΡΡ со слоТными схСмами соСдинСния, Π±ΡƒΠ΄ΡŒ Ρ‚ΠΎ Ρ‚Ρ€Π΅ΡƒΠ³ΠΎΠ»ΡŒΠ½ΠΈΠΊ, ΠΊΠ²Π°Π΄Ρ€Π°Ρ‚, ΠΏΡΡ‚ΠΈΡƒΠ³ΠΎΠ»ΡŒΠ½ΠΈΠΊ ΠΈΠ»ΠΈ иная схСматичная Ρ„ΠΈΠ³ΡƒΡ€Π°, образованная рСзистивными элСмСнтами.

ΠŸΠΎΠ½ΡΡ‚ΡŒ, ΠΊΠ°ΠΊ Ρ€Π°Π±ΠΎΡ‚Π°ΡŽΡ‚ всС Ρ„ΠΎΡ€ΠΌΡƒΠ»Ρ‹ ΠΈ ΠΌΠ΅Ρ‚ΠΎΠ΄Ρ‹, ΠΌΠΎΠΆΠ½ΠΎ Π½Π° ΠΊΠΎΠ½ΠΊΡ€Π΅Ρ‚Π½ΠΎΠΉ Π·Π°Π΄Π°Ρ‡Π΅. На прСдставлСнном ΠΏΠ΅Ρ€Π²ΠΎΠΌ рисункС – смСшанная элСктричСская схСма. Она Π²ΠΊΠ»ΡŽΡ‡Π°Π΅Ρ‚ Π² сСбя 10 рСзисторов. Π­Π»Π΅ΠΌΠ΅Π½Ρ‚Ρ‹ прСдставлСны Π² ΡΠ»Π΅Π΄ΡƒΡŽΡ‰ΠΈΡ… Π½ΠΎΠΌΠΈΠ½Π°Π»Π°Ρ…:

  • R1 = 1 Ом;
  • R2 = 2 Ом;
  • R3 = 3 Ом;
  • R4 = 6 Ом;
  • R5 = 9 Ом;
  • R6 = 18 Ом;
  • R7 = 2Ом;
  • R8 = 2Ом;
  • R9 = 8 Ом;
  • R10 = 4 Ом.

НапряТСниС, ΠΏΠΎΠ΄Π°Π½Π½ΠΎΠ΅ Π½Π° схСму:

ВрСбуСтся Ρ€Π°ΡΡΡ‡ΠΈΡ‚Π°Ρ‚ΡŒ Ρ‚ΠΎΠΊΠΈ Π½Π° всСх рСзистивных элСмСнтах.

Для расчётов примСняСтся Π·Π°ΠΊΠΎΠ½ Ома:

I = U/R, подставляя вмСсто R эквивалСнтноС сопротивлСниС.

Π’Π½ΠΈΠΌΠ°Π½ΠΈΠ΅! Для Ρ€Π΅ΡˆΠ΅Π½ΠΈΡ этой Π·Π°Π΄Π°Ρ‡ΠΈ сначала Π²Ρ‹Ρ‡ΠΈΡΠ»ΡΡŽΡ‚ ΠΎΠ±Ρ‰Π΅Π΅ (эквивалСнтноС) R, послС Ρ‡Π΅Π³ΠΎ ΡƒΠΆΠ΅ Ρ€Π°ΡΡΡ‡ΠΈΡ‚Ρ‹Π²Π°ΡŽΡ‚ Ρ‚ΠΎΠΊ Π² Ρ†Π΅ΠΏΠΈ ΠΈ напряТСниС Π½Π° ΠΊΠ°ΠΆΠ΄ΠΎΠΌ рСзистивном ΠΊΠΎΠΌΠΏΠΎΠ½Π΅Π½Ρ‚Π΅.

Вычисляя Rэкв., Ρ€Π°Π·Π΄Π΅Π»ΡΡŽΡ‚ Π·Π°Π΄Π°Π½Π½ΡƒΡŽ Ρ†Π΅ΠΏΡŒ Π½Π° звСнья, Π²ΠΌΠ΅Ρ‰Π°ΡŽΡ‰ΠΈΠ΅ Π² сСбя ΠΏΠ°Ρ€Π°Π»Π»Π΅Π»ΡŒΠ½Ρ‹Π΅ ΠΈ ΠΏΠΎΡΠ»Π΅Π΄ΠΎΠ²Π°Ρ‚Π΅Π»ΡŒΠ½Ρ‹Π΅ Π²ΠΊΠ»ΡŽΡ‡Π΅Π½ΠΈΡ. Π”Π΅Π»Π°ΡŽΡ‚ расчёты для ΠΊΠ°ΠΆΠ΄ΠΎΠ³ΠΎ Ρ‚Π°ΠΊΠΎΠ³ΠΎ Π·Π²Π΅Π½Π°, послС – всСй Ρ†Π΅ΠΏΠΈ Ρ†Π΅Π»ΠΈΠΊΠΎΠΌ.

На рисункС Π²Ρ‹ΡˆΠ΅ ΠΈΠ·ΠΎΠ±Ρ€Π°ΠΆΠ΅Π½ΠΎ смСшанноС соСдинСниС сопротивлСний. Π•Π³ΠΎ ΠΌΠΎΠΆΠ½ΠΎ Ρ€Π°Π·Π±ΠΈΡ‚ΡŒ Π½Π° Ρ‚Ρ€ΠΈ участка:

  • АВ – участок, ΠΈΠΌΠ΅ΡŽΡ‰ΠΈΠΉ Π΄Π²Π΅ ΠΏΠ°Ρ€Π°Π»Π»Π΅Π»ΡŒΠ½Ρ‹Ρ… Π²Π΅Ρ‚Π²ΠΈ;
  • Π’Π‘ – ΠΎΡ‚Ρ€Π΅Π·ΠΎΠΊ, Π²ΠΌΠ΅Ρ‰Π°ΡŽΡ‰ΠΈΠΉ Π² сСбя ΠΏΠΎΡΠ»Π΅Π΄ΠΎΠ²Π°Ρ‚Π΅Π»ΡŒΠ½ΠΎΠ΅ сопряТСниС;
  • CD – ΠΎΡ‚Ρ€Π΅Π·ΠΎΠΊ схСмы с располоТСниСм Ρ‚Ρ€Ρ‘Ρ… ΠΏΠ°Ρ€Π°Π»Π»Π΅Π»ΡŒΠ½Ρ‹Ρ… Ρ†Π΅ΠΏΠΎΡ‡Π΅ΠΊ.

БопротивлСния R2 ΠΈ R3, ΠΎΠ±Ρ€Π°Π·ΡƒΡŽΡ‰ΠΈΠ΅ ниТнюю Π²Π΅Ρ‚ΠΊΡƒ ΠΎΡ‚Ρ€Π΅Π·ΠΊΠ° АВ, соСдинСны ΠΏΠΎΡΠ»Π΅Π΄ΠΎΠ²Π°Ρ‚Π΅Π»ΡŒΠ½ΠΎ, Ρ‡Ρ‚ΠΎ учитываСтся ΠΏΡ€ΠΈ расчётС.

Если ΠΏΠΎΡΠΌΠΎΡ‚Ρ€Π΅Ρ‚ΡŒ Π½Π° участок Π‘D, Ρ‚ΠΎ ΠΌΠΎΠΆΠ½ΠΎ ΠΎΡ‚ΠΌΠ΅Ρ‚ΠΈΡ‚ΡŒ смСшанноС Π²ΠΊΠ»ΡŽΡ‡Π΅Π½ΠΈΠ΅ рСзистивных элСмСнтов.

Начало расчётов состоит Π² ΠΎΠΏΡ€Π΅Π΄Π΅Π»Π΅Π½ΠΈΠΈ эквивалСнтных сопротивлСний для этих ΡΠΌΠ΅ΡˆΠ°Π½Π½Ρ‹Ρ… Ρ„Ρ€Π°Π³ΠΌΠ΅Π½Ρ‚ΠΎΠ². Π’Ρ‹ΠΏΠΎΠ»Π½ΡΡŽΡ‚ это Π² ΡΠ»Π΅Π΄ΡƒΡŽΡ‰Π΅ΠΌ порядкС:

  • Rэкв.2,3 = R2+R3=2 + 3 = 5 Ом;
  • Rэкв.7,8 = (R7*R8)/R7 + R8 = (2*2)/2 + 2 = 1 Ом;
  • Rэкв.7,8,9 = Rэкв.7,8 + R9 = 1 + 8 = 9 Ом.

Зная значСния ΠΏΠΎΠ»ΡƒΡ‡Π΅Π½Π½Ρ‹Ρ… эквивалСнтов, ΡƒΠΏΡ€ΠΎΡ‰Π°ΡŽΡ‚ ΠΏΠ΅Ρ€Π²ΠΎΠ½Π°Ρ‡Π°Π»ΡŒΠ½ΡƒΡŽ схСму. Она Π±ΡƒΠ΄Π΅Ρ‚ ΠΈΠΌΠ΅Ρ‚ΡŒ Π²ΠΈΠ΄, прСдставлСнный Π½Π° рисункС Π½ΠΈΠΆΠ΅.

Π”Π°Π»Π΅Π΅ ΠΌΠΎΠΆΠ½ΠΎ ΡƒΠΆΠ΅ ΠΎΠΏΡ€Π΅Π΄Π΅Π»ΠΈΡ‚ΡŒ Rэкв. для участков AB, BC, CD, ΠΏΠΎ Ρ„ΠΎΡ€ΠΌΡƒΠ»Π°ΠΌ:

  • Rэкв.AB = (R1*Rэкв 2,3)/R1 + Rэкв 2,3 = (1*5)/1 + 5 = 0,83 Ом;
  • Rэкв.BC = R4 + R5 = 6 + 9 = 15 Ом;
  • 1/Rэкв.CD = 1/R6 + 1/Rэкв.7,8,9 + 1/R10 = 1/18 + 1/9 + 1/4 = 0,05 + 0,11 + 0,25 = 0,41 Ом.

Π’ Ρ€Π΅Π·ΡƒΠ»ΡŒΡ‚Π°Ρ‚Π΅ Π²Ρ‹ΠΏΠΎΠ»Π½Π΅Π½Π½Ρ‹Ρ… вычислСний получаСтся эквивалСнтная схСма, Π² ΠΊΠΎΡ‚ΠΎΡ€ΡƒΡŽ входят Ρ‚Ρ€ΠΈ Rэкв. сопротивлСния. Она ΠΈΠΌΠ΅Π΅Ρ‚ Π²ΠΈΠ΄, ΠΏΠΎΠΊΠ°Π·Π°Π½Π½Ρ‹ΠΉ Π½Π° рисункС Π½ΠΈΠΆΠ΅.

Π’Π΅ΠΏΠ΅Ρ€ΡŒ ΠΌΠΎΠΆΠ½ΠΎ ΠΎΠΏΡ€Π΅Π΄Π΅Π»ΠΈΡ‚ΡŒ эквивалСнтноС сопротивлСниС всСй ΠΏΠ΅Ρ€Π²ΠΎΠ½Π°Ρ‡Π°Π»ΡŒΠ½ΠΎΠΉ схСмы, слоТив эквивалСнтныС значСния всСх Ρ‚Ρ€Ρ‘Ρ… участков:

Rэкв. = Rэкв.AB + Rэкв.BC + Rэкв.CD = 0,83 + 15 + 0,41 = 56,83 Ом.

Π”Π°Π»Π΅Π΅, ΠΈΡΠΏΠΎΠ»ΡŒΠ·ΡƒΡ Π·Π°ΠΊΠΎΠ½ Ома, находят Ρ‚ΠΎΠΊ Π² послСднСм ΠΏΠΎΡΠ»Π΅Π΄ΠΎΠ²Π°Ρ‚Π΅Π»ΡŒΠ½ΠΎΠΌ участкС:

I = U/ Rэкв. = 24/56,83 = 0,42 А.

Зная силу Ρ‚ΠΎΠΊΠ°, ΠΌΠΎΠΆΠ½ΠΎ Π½Π°ΠΉΡ‚ΠΈ, ΠΊΠ°ΠΊΠΎΠ΅ ΠΏΠ°Π΄Π΅Π½ΠΈΠ΅ напряТСния Π½Π° рассмотрСнных участках AB, BC, CD. Π­Ρ‚ΠΎ выполняСтся ΡΠ»Π΅Π΄ΡƒΡŽΡ‰ΠΈΠΌ ΠΎΠ±Ρ€Π°Π·ΠΎΠΌ:

  • UAB = I* Rэкв.AB= 0,42*0,83 = 0,35 Π’;
  • UBC = I* Rэкв.BC= 0,42*15 = 6,3Π’;
  • UCD = I* Rэкв.CD = 0,42*0,41 = 0,17 Π’.

Π‘Π»Π΅Π΄ΡƒΡŽΡ‰ΠΈΠΌ шагом станСт ΠΎΠΏΡ€Π΅Π΄Π΅Π»Π΅Π½ΠΈΠ΅ Ρ‚ΠΎΠΊΠΎΠ² Π½Π° ΠΏΠ°Ρ€Π°Π»Π»Π΅Π»ΡŒΠ½Ρ‹Ρ… ΠΎΡ‚Ρ€Π΅Π·ΠΊΠ°Ρ… AB ΠΈ CD:

  • I1 = UAB/R1 = 0,35/1 = 0,35 А;
  • I2 = UAB/Rэкв.2,3 = 0,35/5 = 0,07 А;
  • I3 = UCD/R6 = 0,17/18 = 0,009 А;
  • I6 = UCD/Rэкв.7,8,9= 0,17/9 = 0,02 А;
  • I7 = UCD/R10 = 0,17/4 = 0,04 А.

Π”Π°Π»Π΅Π΅, Ρ‡Ρ‚ΠΎΠ±Ρ‹ Π½Π°ΠΉΡ‚ΠΈ значСния Ρ‚ΠΎΠΊΠΎΠ², проходящих Ρ‡Π΅Ρ€Π΅Π· R7 ΠΈ R8, Π½ΡƒΠΆΠ½ΠΎ Ρ€Π°ΡΡΡ‡ΠΈΡ‚Π°Ρ‚ΡŒ напряТСниС Π½Π° этих Π΄Π²ΡƒΡ… рСзисторах. ΠŸΡ€Π΅Π΄Π²Π°Ρ€ΠΈΡ‚Π΅Π»ΡŒΠ½ΠΎ находят ΠΏΠ°Π΄Π΅Π½ΠΈΠ΅ напряТСния Π½Π° R9.

U9 = R9*I6 = 8*0,02 = 0,16 Π’.

Π’Π΅ΠΏΠ΅Ρ€ΡŒ напряТСниС, ΠΏΠ°Π΄Π°ΡŽΡ‰Π΅Π΅ Π½Π° Rэкв.7,8, Π±ΡƒΠ΄Π΅Ρ‚ Ρ€Π°Π·Π½ΠΎΡΡ‚ΡŒΡŽ ΠΌΠ΅ΠΆΠ΄Ρƒ U CD ΠΈ U9.

U7,8 = UCD – U9= 0,17 – 0,16 = 1 Π’.

ПослС этого ΠΌΠΎΠΆΠ½ΠΎ ΡƒΠΆΠ΅ ΡƒΠ·Π½Π°Ρ‚ΡŒ Π·Π½Π°Ρ‡Π΅Π½ΠΈΠ΅ Ρ‚ΠΎΠΊΠΎΠ², двиТущихся ΠΏΠΎ рСзисторам R7 ΠΈ R8, ΠΈΡΠΏΠΎΠ»ΡŒΠ·ΡƒΡ Ρ„ΠΎΡ€ΠΌΡƒΠ»Ρ‹:

  • I4 = U7,8/R7 = 1/2 = 0,5 A;
  • I5 = U7,8/R8 = 1/2 = 0,5 A.

Π‘Ρ‚ΠΎΠΈΡ‚ Π·Π°ΠΌΠ΅Ρ‚ΠΈΡ‚ΡŒ! Π’ΠΎΠΊ, ΠΏΡ€ΠΎΡ‚Π΅ΠΊΠ°ΡŽΡ‰ΠΈΠΉ Ρ‡Π΅Ρ€Π΅Π· R4 ΠΈ R5, ΠΏΠΎ своСму Π·Π½Π°Ρ‡Π΅Π½ΠΈΡŽ Ρ€Π°Π²Π΅Π½ Ρ‚ΠΎΠΊΡƒ Π½Π° ΠΎΡ‚Ρ€Π΅Π·ΠΊΠ΅, Π½Π΅ ΠΈΠΌΠ΅ΡŽΡ‰Π΅ΠΌ развСтвлСния.

Рассчитывая схСмы ΠΈ Ρ€Π΅ΡˆΠ°Ρ Π·Π°Π΄Π°Ρ‡ΠΈ ΠΏΠΎ Π½Π°Ρ…ΠΎΠΆΠ΄Π΅Π½ΠΈΡŽ Π·Π½Π°Ρ‡Π΅Π½ΠΈΠΉ элСктричСских ΠΏΠ°Ρ€Π°ΠΌΠ΅Ρ‚Ρ€ΠΎΠ², Π½Π΅ΠΎΠ±Ρ…ΠΎΠ΄ΠΈΠΌΠΎ ΠΈΡΠΏΠΎΠ»ΡŒΠ·ΠΎΠ²Π°Ρ‚ΡŒ эквивалСнтныС сопротивлСния. Π‘ ΠΏΠΎΠΌΠΎΡ‰ΡŒΡŽ Ρ‚Π°ΠΊΠΎΠΉ Π·Π°ΠΌΠ΅Π½Ρ‹ слоТныС построСния ΠΏΡ€Π΅Π²Ρ€Π°Ρ‰Π°ΡŽΡ‚ΡΡ Π² элСмСнтарныС Ρ†Π΅ΠΏΠΈ, ΠΊΠΎΡ‚ΠΎΡ€Ρ‹Π΅ сводятся ΠΊ ΠΏΠ°Ρ€Π°Π»Π»Π΅Π»ΡŒΠ½Ρ‹ΠΌ ΠΈ ΠΏΠΎΡΠ»Π΅Π΄ΠΎΠ²Π°Ρ‚Π΅Π»ΡŒΠ½Ρ‹ΠΌ соСдинСниям рСзистивных элСмСнтов.

Π’ΠΈΠ΄Π΅ΠΎ

Π¦Π΅ΠΏΠΈ рСзисторов сСрии

ΠΈ ΠΏΠ°Ρ€Π°Π»Π»Π΅Π»ΡŒΠ½Ρ‹Π΅

Π­Π»Π΅ΠΊΡ‚Ρ€ΠΎΠ½Π½Ρ‹Π΅ ΠΊΠΎΠΌΠΏΠΎΠ½Π΅Π½Ρ‚Ρ‹ ΠΏΠΎΠ΄ΠΊΠ»ΡŽΡ‡Π°ΡŽΡ‚ΡΡ Ρ€Π°Π·Π½Ρ‹ΠΌΠΈ способами. Π”Π²Π΅ ΠΏΡ€ΠΎΡΡ‚Π΅ΠΉΡˆΠΈΠ΅ Ρ„ΠΎΡ€ΠΌΡ‹ ΠΏΠΎΠ΄ΠΊΠ»ΡŽΡ‡Π΅Π½ΠΈΡ — ΠΏΠΎΡΠ»Π΅Π΄ΠΎΠ²Π°Ρ‚Π΅Π»ΡŒΠ½ΠΎΠ΅ ΠΈ ΠΏΠ°Ρ€Π°Π»Π»Π΅Π»ΡŒΠ½ΠΎΠ΅. соСдинСния.

Если ΠΊΠΎΠΌΠΏΠΎΠ½Π΅Π½Ρ‚Ρ‹ соСдинСны ΠΏΠΎΡΠ»Π΅Π΄ΠΎΠ²Π°Ρ‚Π΅Π»ΡŒΠ½ΠΎ Π² Ρ†Π΅ΠΏΠΈ, Ρ‚ΠΎ Ρ†Π΅ΠΏΡŒ называСтся ΠΏΠΎΡΠ»Π΅Π΄ΠΎΠ²Π°Ρ‚Π΅Π»ΡŒΠ½ΠΎΠΉ. Если рСзисторы соСдинСны ΠΏΠΎΡΠ»Π΅Π΄ΠΎΠ²Π°Ρ‚Π΅Π»ΡŒΠ½ΠΎ Π² Ρ†Π΅ΠΏΠΈ, Ρ‚ΠΎΠ³Π΄Π° Ρ†Π΅ΠΏΡŒ говорят, Ρ‡Ρ‚ΠΎ это Ρ†Π΅ΠΏΡŒ ΠΏΠΎΡΠ»Π΅Π΄ΠΎΠ²Π°Ρ‚Π΅Π»ΡŒΠ½ΠΎΠ³ΠΎ рСзистора.

Если ΠΊΠΎΠΌΠΏΠΎΠ½Π΅Π½Ρ‚Ρ‹ соСдинСны Π² Ρ†Π΅ΠΏΠΈ ΠΏΠ°Ρ€Π°Π»Π»Π΅Π»ΡŒΠ½ΠΎ, Ρ‚ΠΎ Ρ†Π΅ΠΏΡŒ называСтся ΠΏΠ°Ρ€Π°Π»Π»Π΅Π»ΡŒΠ½ΠΎΠΉ. Если рСзисторы соСдинСны ΠΏΠ°Ρ€Π°Π»Π»Π΅Π»ΡŒΠ½ΠΎ Π² Ρ†Π΅ΠΏΠΈ, Ρ‚ΠΎΠ³Π΄Π° Ρ†Π΅ΠΏΡŒ называСтся ΠΏΠ°Ρ€Π°Π»Π»Π΅Π»ΡŒΠ½ΠΎΠΉ схСмой рСзистора.

РСзистор сСрии

Ρ†Π΅ΠΏΡŒ

А Π‘Ρ…Π΅ΠΌΠ° ΠΏΠΎΡΠ»Π΅Π΄ΠΎΠ²Π°Ρ‚Π΅Π»ΡŒΠ½ΠΎΠ³ΠΎ рСзистора — это элСктронная схСма, Π² ΠΊΠΎΡ‚ΠΎΡ€ΠΎΠΉ всС рСзисторы ΠΏΠΎΠ΄ΠΊΠ»ΡŽΡ‡Π°ΡŽΡ‚ΡΡ Π΄Ρ€ΡƒΠ³ Π·Π° Π΄Ρ€ΡƒΠ³ΠΎΠΌ ΠΏΠΎ ΠΎΠ΄Π½ΠΎΠΌΡƒ ΠΈ Ρ‚ΠΎΠΌΡƒ ΠΆΠ΅ ΠΏΡƒΡ‚ΠΈ Ρ‚Π°ΠΊ Ρ‡Ρ‚ΠΎ ΠΎΠ΄ΠΈΠ½Π°ΠΊΠΎΠ²Ρ‹ΠΉ Ρ‚ΠΎΠΊ Ρ‚Π΅Ρ‡Π΅Ρ‚ Ρ‡Π΅Ρ€Π΅Π· всС ΠΈ ΠΊΠ°ΠΆΠ΄Ρ‹ΠΉ рСзистор.

ΠΏΠΎΠ»Π½ΠΎΠ΅ сопротивлСниС Ρ‚Π°ΠΊΠΎΠΉ схСмы получаСтся простым суммируя значСния сопротивлСний ΠΎΡ‚Π΄Π΅Π»ΡŒΠ½Ρ‹Ρ… рСзисторов.

R T = 1 + 2 + 3 + 4 Ρ€Π°Π½Π΄ΠΎΠ² …… ..ΠΈ Ρ‚. Π”.

Для НапримСр, Ссли ΠΏΡΡ‚ΡŒ рСзисторов соСдинСны ΠΏΠΎΡΠ»Π΅Π΄ΠΎΠ²Π°Ρ‚Π΅Π»ΡŒΠ½ΠΎ. Π’ΠΎΠ³Π΄Π° ΠΏΠΎΠ»Π½ΠΎΠ΅ сопротивлСниС Ρ†Π΅ΠΏΠΈ Ρ€Π°Π²Π½ΠΎ:

R T = 1 + 2 + 3 + 4 Ρ€Π°Π½Π΄ΠΎΠ² + Π  5

ВсС Ρ‚ΠΎΠΊ, ΠΏΡ€ΠΎΡ‚Π΅ΠΊΠ°ΡŽΡ‰ΠΈΠΉ Ρ‡Π΅Ρ€Π΅Π· ΠΏΠ΅Ρ€Π²Ρ‹ΠΉ рСзистор, Π½Π΅ ΠΈΠΌΠ΅Π΅Ρ‚ Π΄Ρ€ΡƒΠ³ΠΎΠ³ΠΎ ΠΏΡƒΡ‚ΡŒ ΠΈΠ΄Ρ‚ΠΈ.Π‘Π»Π΅Π΄ΠΎΠ²Π°Ρ‚Π΅Π»ΡŒΠ½ΠΎ, ΠΎΠ½ Ρ‚Π°ΠΊΠΆΠ΅ Π΄ΠΎΠ»ΠΆΠ΅Π½ ΠΏΡ€ΠΎΠΉΡ‚ΠΈ Ρ‡Π΅Ρ€Π΅Π· Π²Ρ‚ΠΎΡ€ΠΎΠΉ рСзистор, Ρ‚Ρ€Π΅Ρ‚ΠΈΠΉ рСзистор, Ρ‡Π΅Ρ‚Π²Π΅Ρ€Ρ‚Ρ‹ΠΉ рСзистор ΠΈ Ρ‚. Π΄.

ΠŸΡ€ΠΈΠΌΠ΅Ρ€:

А Π‘Ρ…Π΅ΠΌΠ° ΠΏΠΎΡΠ»Π΅Π΄ΠΎΠ²Π°Ρ‚Π΅Π»ΡŒΠ½ΠΎΠ³ΠΎ рСзистора ΠΏΠΎΠΊΠ°Π·Π°Π½Π° Π½Π° рисункС Π½ΠΈΠΆΠ΅. Π­Ρ‚ΠΎΡ‚ схСма состоит ΠΈΠ· пяти рСзисторов, ΠΊΠΎΡ‚ΠΎΡ€Ρ‹Π΅ Π²ΠΊΠ»ΡŽΡ‡Π΅Π½Ρ‹ Π² ΠΏΠΎΡΠ»Π΅Π΄ΠΎΠ²Π°Ρ‚Π΅Π»ΡŒΠ½Ρ‹ΠΉ ΠΈ источник постоянного напряТСния.

Если ЗначСния пяти рСзисторов Ρ€Π°Π²Π½Ρ‹: R 1 = 4 Ом, R 2 = 4 Ом, R 3 = 2 Ом, R 4 = 2 Ом, R 5 = 3 Ом ΠΈ батарСя постоянного Ρ‚ΠΎΠΊΠ° = 15 Π’, Ρ‚ΠΎΠ³Π΄Π°

ΠžΠ±Ρ‰Π΅Π΅ сопротивлСниС составляСт R T = 1 + 2 + 3 + 4 Ρ€Π°Π½Π΄ΠΎΠ² + R 5 = 4 + 4 + 2 + 2 + 3 = 15 Ом.

ВспомнитС Ρ„ΠΎΡ€ΠΌΡƒΠ»Ρƒ Π·Π°ΠΊΠΎΠ½Π° Ома, V = I R

Зная Π»ΡŽΠ±Ρ‹Π΅ Π΄Π²Π΅ ΠΏΠ΅Ρ€Π΅ΠΌΠ΅Π½Π½Ρ‹Π΅ Π² Π²Ρ‹ΡˆΠ΅ уравнСния, ΠΌΡ‹ ΠΌΠΎΠΆΠ΅ΠΌ Π»Π΅Π³ΠΊΠΎ Π½Π°ΠΉΡ‚ΠΈ ΠΎΡΡ‚Π°Π²ΡˆΠΈΠ΅ΡΡ нСизвСстныС ΠŸΠ΅Ρ€Π΅ΠΌΠ΅Π½Π½Π°Ρ.

Нам извСстно Π·Π½Π°Ρ‡Π΅Π½ΠΈΠ΅ ΠΏΠΎΠ»Π½ΠΎΠ³ΠΎ сопротивлСния, Ρ‚.Π΅. R T = 15 Ом ΠΈ Π·Π½Π°Ρ‡Π΅Π½ΠΈΠ΅ напряТСния I.e. V = 15 Π’

Π’Π΅ΠΏΠ΅Ρ€ΡŒ Π½ΡƒΠΆΠ½ΠΎ Π½Π°ΠΉΡ‚ΠΈ ΠΎΡΡ‚Π°Π²ΡˆΠ΅Π΅ΡΡ нСизвСстноС Ρ‚Π΅ΠΊΡƒΡ‰Π΅Π΅ Π·Π½Π°Ρ‡Π΅Π½ΠΈΠ΅ I.

Π’Π΅ΠΊΡƒΡ‰ΠΈΠΉ Ρ‚Π΅ΠΊΡƒΡ‰ΠΈΠΉ Ρ‡Π΅Ρ€Π΅Π· ΠΊΠ°ΠΆΠ΄Ρ‹ΠΉ рСзистор Π±ΡƒΠ΄Π΅Ρ‚ 1 А.

НапряТСниС Π½Π° ΠΊΠ°ΠΆΠ΄Ρ‹ΠΉ рСзистор Π² ΠΏΠΎΡΠ»Π΅Π΄ΠΎΠ²Π°Ρ‚Π΅Π»ΡŒΠ½ΠΎΠΉ Ρ†Π΅ΠΏΠΈ отличаСтся. Если всС рСзисторы Π² ΠΏΠΎΡΠ»Π΅Π΄ΠΎΠ²Π°Ρ‚Π΅Π»ΡŒΠ½ΠΎΠΉ Ρ†Π΅ΠΏΠΈ ΠΈΠΌΠ΅ΡŽΡ‚ ΠΎΠ΄ΠΈΠ½Π°ΠΊΠΎΠ²ΠΎΠ΅ Π·Π½Π°Ρ‡Π΅Π½ΠΈΠ΅ сопротивлСния Ρ‚ΠΎΠ³Π΄Π° напряТСниС Ρ‡Π΅Ρ€Π΅Π· ΠΊΠ°ΠΆΠ΄Ρ‹ΠΉ рСзистор Ρ‚Π°ΠΊΠΎΠΉ ΠΆΠ΅. Π‘ Π΄Ρ€ΡƒΠ³ΠΎΠΉ стороны, Ссли ΠΊΠ°ΠΆΠ΄Ρ‹ΠΉ рСзистор ΠΈΠΌΠ΅Π΅Ρ‚ Π΄Ρ€ΡƒΠ³ΠΎΠ΅ Π·Π½Π°Ρ‡Π΅Π½ΠΈΠ΅ сопротивлСния, Ρ‚ΠΎ напряТСниС Π½Π° ΠΊΠ°ΠΆΠ΄ΠΎΠΌ рСзисторС различаСтся.

НапряТСниС Π½Π° рСзистор (R 1 ) составляСт Π’ 1 = I Γ— R 1 = 1 Γ— 4 = 4 Π’

НапряТСниС Π½Π° рСзистор (R 2 ) составляСт Π’ 2 = I Γ— R 2 = 1 Γ— 4 = 4 Π’

НапряТСниС Π½Π° рСзистор (R 3 ) составляСт Π’ 3 = I Γ— R 3 = 1 Γ— 2 = 2 Π’

НапряТСниС Π½Π° рСзистор (R 4 ) составляСт Π’ 4 = I Γ— R 4 = 1 Γ— 2 = 2 Π’

НапряТСниС Π½Π° рСзистор (R 5 ) составляСт Π’ 5 = I Γ— R 5 = 1 Γ— 3 = 3 Π’

Π‘ΡƒΠΌΠΌΠ°Ρ€Π½ΠΎΠ΅ напряТСниС Π² ΠΏΠΎΡΠ»Π΅Π΄ΠΎΠ²Π°Ρ‚Π΅Π»ΡŒΠ½Π°Ρ Ρ†Π΅ΠΏΡŒ Ρ€Π°Π²Π½Π° суммС всСх ΠΎΡ‚Π΄Π΅Π»ΡŒΠ½Ρ‹Ρ… напряТСния Π² суммС

И.Π΅. Π’ Π’ = Π’ 1 + V 2 + V 3 + V 4 + ………. + V N

Π’ нашСй схСмС ΠΎΠ±Ρ‰Π΅Π΅ напряТСниС Ρ€Π°Π²Π½ΠΎ суммС разностСй ΠΏΠΎΡ‚Π΅Π½Ρ†ΠΈΠ°Π»ΠΎΠ² ΠΏΠΎ R 1, , R 2 , R 3 , 4 ΠΈ 5 .

Π’.Π΅. Π’ Π’ = Π’ 1 + Π’ 2 + Π’ 3 + Π’ 4 + Π’ 5 = 4 + 4 + 2 + 2 + 3 = 15 Π’.

ΠŸΠ°Ρ€Π°Π»Π»Π΅Π»ΡŒΠ½Ρ‹ΠΉ схСма рСзистора

А Π‘Ρ…Π΅ΠΌΠ° ΠΏΠ°Ρ€Π°Π»Π»Π΅Π»ΡŒΠ½ΠΎΠ³ΠΎ рСзистора — это элСктронная схСма, Π² ΠΊΠΎΡ‚ΠΎΡ€ΠΎΠΉ всС рСзисторы соСдинСны Π±ΠΎΠΊ ΠΎ Π±ΠΎΠΊ Π² Ρ€Π°Π·Π½Ρ‹Ρ… ΠΏΡƒΡ‚ΠΈ, Ρ‡Ρ‚ΠΎΠ±Ρ‹ ΠΎΠ΄ΠΈΠ½ ΠΈ Ρ‚ΠΎΡ‚ ΠΆΠ΅ Ρ‚ΠΎΠΊ Π½Π΅ ΠΏΡ€ΠΎΡ…ΠΎΠ΄ΠΈΠ» Ρ‡Π΅Ρ€Π΅Π· ΠΊΠ°ΠΆΠ΄Ρ‹ΠΉ рСзистор. ΠŸΠ°Ρ€Π°Π»Π»Π΅Π»ΡŒΠ½Π°Ρ схСма ΠΏΠΎΠΊΠ°Π·Ρ‹Π²Π°Π΅Ρ‚ нСсколько ΠΏΡƒΡ‚Π΅ΠΉ ΠΊ элСктричСский Ρ‚ΠΎΠΊ Ρ‚Π΅Ρ‡Π΅Ρ‚.

Ρ‚ΠΎΠΊ Π² ΠΏΠ°Ρ€Π°Π»Π»Π΅Π»ΡŒΠ½ΠΎΠΉ Ρ†Π΅ΠΏΠΈ прСрываСтся, с Π½Π΅ΠΊΠΎΡ‚ΠΎΡ€Ρ‹ΠΌ Ρ‚ΠΎΠΊΠΎΠΌ Ρ‚Π΅Ρ‡Π΅Ρ‚ вдоль ΠΊΠ°ΠΆΠ΄ΠΎΠΉ ΠΏΠ°Ρ€Π°Π»Π»Π΅Π»ΡŒΠ½ΠΎΠΉ Π²Π΅Ρ‚Π²ΠΈ ΠΈ ΠΏΠΎΠ²Ρ‚ΠΎΡ€Π½ΠΎ комбинируСтся, ΠΊΠΎΠ³Π΄Π° Π²Π΅Ρ‚Π²ΠΈ снова Π²ΡΡ‚Ρ€Π΅Ρ‡Π°ΡŽΡ‚ΡΡ.Π‘Π»Π΅Π΄ΠΎΠ²Π°Ρ‚Π΅Π»ΡŒΠ½ΠΎ, элСктричСский Ρ‚ΠΎΠΊ Ρ‡Π΅Ρ€Π΅Π· ΠΊΠ°ΠΆΠ΄Ρ‹ΠΉ рСзистор Π±ΡƒΠ΄Π΅Ρ‚ Π΄Ρ€ΡƒΠ³ΠΈΠΌ. Однако напряТСниС Π½Π° ΠΊΠ°ΠΆΠ΄Ρ‹ΠΉ рСзистор Ρ‚Π°ΠΊΠΎΠΉ ΠΆΠ΅.

ΠΏΠΎΠ»Π½ΠΎΠ΅ сопротивлСниС ΠΏΠ°Ρ€Π°Π»Π»Π΅Π»ΡŒΠ½ΠΎΠΉ Ρ†Π΅ΠΏΠΈ рСзистора получаСтся ΠΊΠ°ΠΊ слоТСниС ΠΎΠ±Ρ€Π°Ρ‚Π½Ρ‹Ρ… Π²Π΅Π»ΠΈΡ‡ΠΈΠ½ (1 / R) Π·Π½Π°Ρ‡Π΅Π½ΠΈΠΉ сопротивлСния ΠΎΡ‚Π΄Π΅Π»ΡŒΠ½Ρ‹Π΅ рСзисторы, Π° Π·Π°Ρ‚Π΅ΠΌ взяв ΠΎΠ±Ρ€Π°Ρ‚Π½ΡƒΡŽ Π²Π΅Π»ΠΈΡ‡ΠΈΠ½Ρƒ ΠΎΠ±Ρ‰Π΅Π΅.

Для НапримСр, Ссли Ρ‚Ρ€ΠΈ рСзистора ΠΏΠΎΠ΄ΠΊΠ»ΡŽΡ‡Π΅Π½Ρ‹ ΠΏΠ°Ρ€Π°Π»Π»Π΅Π»ΡŒΠ½ΠΎ.ΠΏΠΎΡ‚ΠΎΠΌ ΠΏΠΎΠ»Π½ΠΎΠ΅ сопротивлСниС Ρ†Π΅ΠΏΠΈ

ΠŸΡ€ΠΈΠΌΠ΅Ρ€:

А Π‘Ρ…Π΅ΠΌΠ° ΠΏΠ°Ρ€Π°Π»Π»Π΅Π»ΡŒΠ½ΠΎΠ³ΠΎ рСзистора ΠΏΠΎΠΊΠ°Π·Π°Π½Π° Π½Π° рисункС Π½ΠΈΠΆΠ΅. Π­Ρ‚ΠΎΡ‚ схСма состоит ΠΈΠ· Ρ‚Ρ€Π΅Ρ… рСзисторов, ΠΊΠΎΡ‚ΠΎΡ€Ρ‹Π΅ Π²ΠΊΠ»ΡŽΡ‡Π΅Π½Ρ‹ Π² ΠΏΠ°Ρ€Π°Π»Π»Π΅Π»ΡŒΠ½Ρ‹ΠΉ ΠΈ источник постоянного напряТСния.

Если ЗначСния Ρ‚Ρ€Π΅Ρ… рСзисторов Ρ€Π°Π²Π½Ρ‹: R 1 = 8 Ом, R 2 = 8 Ом, R 3 = 4 Ом ΠΈ батарСя постоянного Ρ‚ΠΎΠΊΠ° = 14 Π’, Π·Π°Ρ‚Π΅ΠΌ

ΠΎΠ±Ρ‰Π΅Π΅ сопротивлСниС



Π‘ΡƒΠΌΠΌΠ°Ρ€Π½Ρ‹ΠΉ Ρ‚ΠΎΠΊ ΠΏΠΎ ΠΊΠΎΠ½Ρ‚ΡƒΡ€Ρƒ —


Как напряТСниС Π½Π° ΠΊΠ°ΠΆΠ΄ΠΎΠΌ рСзисторС ΠΎΠ΄ΠΈΠ½Π°ΠΊΠΎΠ²ΠΎ ΠΏΠ°Ρ€Π°Π»Π»Π΅Π»ΡŒΠ½ΠΎ схСма, ΠΌΡ‹ ΠΌΠΎΠΆΠ΅ΠΌ ΠΈΡΠΏΠΎΠ»ΡŒΠ·ΠΎΠ²Π°Ρ‚ΡŒ Ом Π·Π°ΠΊΠΎΠ½ Ρ‡Ρ‚ΠΎΠ±Ρ‹ Π½Π°ΠΉΡ‚ΠΈ Ρ‚ΠΎΠΊ ΠΎΡ‚Π΄Π΅Π»ΡŒΠ½ΠΎΠΉ Π²Π΅Ρ‚Π²ΠΈ ΡΠ»Π΅Π΄ΡƒΡŽΡ‰ΠΈΠΌ ΠΎΠ±Ρ€Π°Π·ΠΎΠΌ.

Ρ‚ΠΎΠΊ Ρ‡Π΅Ρ€Π΅Π· рСзистор (R 1 ) is I 1 = V / R 1 = 14/8 = 1,75 А

Ρ‚ΠΎΠΊ Ρ‡Π΅Ρ€Π΅Π· рСзистор (R 2 ) is I 2 = V / R 2 = 14/8 = 1,75 А

Ρ‚ΠΎΠΊ Ρ‡Π΅Ρ€Π΅Π· рСзистор (R 3 ) is I 3 = V / R 3 = 14/4 = 3.5 А

Π’ΠΎΠ³Π΄Π° общая Ρ‚ΠΎΠΊ Ρ€Π°Π²Π΅Π½ суммС Ρ‚ΠΎΠΊΠΎΠ² ΠΎΡ‚Π΄Π΅Π»ΡŒΠ½Ρ‹Ρ… Π²Π΅Ρ‚Π²Π΅ΠΉ

В.С. Я В = Я 1 + I 2 + I 3 = 1,75 + 1,75 + 3,5 = 7 А

Π₯арактСристики ΠΏΠ°Ρ€Π°Π»Π»Π΅Π»ΡŒΠ½ΠΎΠΉ Ρ†Π΅ΠΏΠΈ

ОбновлСно 28 дСкабря 2020 Π³.

Автор: S.Hussain Ather

Π’ элСктричСских цСпях элСмСнты Ρ†Π΅ΠΏΠΈ ΠΌΠΎΠ³ΡƒΡ‚ Π±Ρ‹Ρ‚ΡŒ располоТСны ΠΏΠΎΡΠ»Π΅Π΄ΠΎΠ²Π°Ρ‚Π΅Π»ΡŒΠ½ΠΎ ΠΈΠ»ΠΈ ΠΏΠ°Ρ€Π°Π»Π»Π΅Π»ΡŒΠ½ΠΎ. Π’ ΠΏΠΎΡΠ»Π΅Π΄ΠΎΠ²Π°Ρ‚Π΅Π»ΡŒΠ½Ρ‹Ρ… цСпях элСмСнты ΡΠΎΠ΅Π΄ΠΈΠ½ΡΡŽΡ‚ΡΡ с ΠΏΠΎΠΌΠΎΡ‰ΡŒΡŽ ΠΎΠ΄Π½ΠΎΠΉ ΠΈ Ρ‚ΠΎΠΉ ΠΆΠ΅ Π²Π΅Ρ‚Π²ΠΈ, которая пропускаСт элСктричСский Ρ‚ΠΎΠΊ Ρ‡Π΅Ρ€Π΅Π· ΠΊΠ°ΠΆΠ΄ΡƒΡŽ ΠΈΠ· Π½ΠΈΡ… ΠΎΠ΄ΠΈΠ½ Π·Π° Π΄Ρ€ΡƒΠ³ΠΈΠΌ. Π’ ΠΏΠ°Ρ€Π°Π»Π»Π΅Π»ΡŒΠ½Ρ‹Ρ… цСпях элСмСнты ΠΈΠΌΠ΅ΡŽΡ‚ свои ΠΎΡ‚Π΄Π΅Π»ΡŒΠ½Ρ‹Π΅ отвСтвлСния. Π’ этих цСпях Ρ‚ΠΎΠΊ ΠΌΠΎΠΆΠ΅Ρ‚ ΠΏΡ€ΠΎΡ…ΠΎΠ΄ΠΈΡ‚ΡŒ ΠΏΠΎ Ρ€Π°Π·Π½Ρ‹ΠΌ путям.

ΠŸΠΎΡΠΊΠΎΠ»ΡŒΠΊΡƒ Ρ‚ΠΎΠΊ ΠΌΠΎΠΆΠ΅Ρ‚ ΠΏΡ€ΠΎΡ…ΠΎΠ΄ΠΈΡ‚ΡŒ ΠΏΠΎ Ρ€Π°Π·Π½Ρ‹ΠΌ путям Π² ΠΏΠ°Ρ€Π°Π»Π»Π΅Π»ΡŒΠ½ΠΎΠΉ Ρ†Π΅ΠΏΠΈ, Ρ‚ΠΎΠΊ Π½Π΅ являСтся постоянным Π²ΠΎ всСй ΠΏΠ°Ρ€Π°Π»Π»Π΅Π»ΡŒΠ½ΠΎΠΉ Ρ†Π΅ΠΏΠΈ.ВмСсто этого для Π²Π΅Ρ‚Π²Π΅ΠΉ, ΠΊΠΎΡ‚ΠΎΡ€Ρ‹Π΅ соСдинСны ΠΏΠ°Ρ€Π°Π»Π»Π΅Π»ΡŒΠ½ΠΎ Π΄Ρ€ΡƒΠ³ с Π΄Ρ€ΡƒΠ³ΠΎΠΌ, ΠΏΠ°Π΄Π΅Π½ΠΈΠ΅ напряТСния ΠΈΠ»ΠΈ ΠΏΠΎΡ‚Π΅Π½Ρ†ΠΈΠ°Π»Π° Π½Π° ΠΊΠ°ΠΆΠ΄ΠΎΠΉ Π²Π΅Ρ‚Π²ΠΈ являСтся постоянным. Π­Ρ‚ΠΎ связано с Ρ‚Π΅ΠΌ, Ρ‡Ρ‚ΠΎ Ρ‚ΠΎΠΊ распрСдСляСтся ΠΏΠΎ ΠΊΠ°ΠΆΠ΄ΠΎΠΉ Π²Π΅Ρ‚Π²ΠΈ Π² количСствС, ΠΎΠ±Ρ€Π°Ρ‚Π½ΠΎ ΠΏΡ€ΠΎΠΏΠΎΡ€Ρ†ΠΈΠΎΠ½Π°Π»ΡŒΠ½ΠΎΠΌ ΡΠΎΠΏΡ€ΠΎΡ‚ΠΈΠ²Π»Π΅Π½ΠΈΡŽ ΠΊΠ°ΠΆΠ΄ΠΎΠΉ Π²Π΅Ρ‚Π²ΠΈ. Π­Ρ‚ΠΎ ΠΏΡ€ΠΈΠ²ΠΎΠ΄ΠΈΡ‚ ΠΊ Ρ‚ΠΎΠΌΡƒ, Ρ‡Ρ‚ΠΎ Ρ‚ΠΎΠΊ становится наибольшим Ρ‚Π°ΠΌ, Π³Π΄Π΅ сопротивлСниС наимСньшСС, ΠΈ Π½Π°ΠΎΠ±ΠΎΡ€ΠΎΡ‚.

Π­Ρ‚ΠΈ качСства ΠΏΠΎΠ·Π²ΠΎΠ»ΡΡŽΡ‚ ΠΏΠ°Ρ€Π°Π»Π»Π΅Π»ΡŒΠ½Ρ‹ΠΌ цСпям ΠΏΡ€ΠΎΠΏΡƒΡΠΊΠ°Ρ‚ΡŒ заряд ΠΏΠΎ Π΄Π²ΡƒΠΌ ΠΈΠ»ΠΈ Π±ΠΎΠ»Π΅Π΅ путям, Ρ‡Ρ‚ΠΎ Π΄Π΅Π»Π°Π΅Ρ‚ Π΅Π³ΠΎ стандартным ΠΊΠ°Π½Π΄ΠΈΠ΄Π°Ρ‚ΠΎΠΌ Π² Π΄ΠΎΠΌΠ°Ρ… ΠΈ элСктричСских устройствах Ρ‡Π΅Ρ€Π΅Π· ΡΡ‚Π°Π±ΠΈΠ»ΡŒΠ½ΡƒΡŽ ΠΈ ΡΡ„Ρ„Π΅ΠΊΡ‚ΠΈΠ²Π½ΡƒΡŽ систСму питания.Он позволяСт элСктричСству Ρ‚Π΅Ρ‡ΡŒ Ρ‡Π΅Ρ€Π΅Π· Π΄Ρ€ΡƒΠ³ΠΈΠ΅ части Ρ†Π΅ΠΏΠΈ, ΠΊΠΎΠ³Π΄Π° какая-Π»ΠΈΠ±ΠΎ Ρ‡Π°ΡΡ‚ΡŒ ΠΏΠΎΠ²Ρ€Π΅ΠΆΠ΄Π΅Π½Π° ΠΈΠ»ΠΈ сломана, ΠΈ ΠΎΠ½ΠΈ ΠΌΠΎΠ³ΡƒΡ‚ Ρ€Π°ΡΠΏΡ€Π΅Π΄Π΅Π»ΡΡ‚ΡŒ ΠΌΠΎΡ‰Π½ΠΎΡΡ‚ΡŒ Ρ€Π°Π²Π½ΠΎΠΌΠ΅Ρ€Π½ΠΎ ΠΏΠΎ Ρ€Π°Π·Π½Ρ‹ΠΌ зданиям. Π­Ρ‚ΠΈ характСристики ΠΌΠΎΠΆΠ½ΠΎ ΠΏΡ€ΠΎΠ΄Π΅ΠΌΠΎΠ½ΡΡ‚Ρ€ΠΈΡ€ΠΎΠ²Π°Ρ‚ΡŒ с ΠΏΠΎΠΌΠΎΡ‰ΡŒΡŽ Π΄ΠΈΠ°Π³Ρ€Π°ΠΌΠΌΡ‹ ΠΈ ΠΏΡ€ΠΈΠΌΠ΅Ρ€Π° ΠΏΠ°Ρ€Π°Π»Π»Π΅Π»ΡŒΠ½ΠΎΠΉ Ρ†Π΅ΠΏΠΈ.

Π‘Ρ…Π΅ΠΌΠ° ΠΏΠ°Ρ€Π°Π»Π»Π΅Π»ΡŒΠ½ΠΎΠΉ Ρ†Π΅ΠΏΠΈ

β€’β€’β€’ Syed Hussain Ather

На схСмС ΠΏΠ°Ρ€Π°Π»Π»Π΅Π»ΡŒΠ½ΠΎΠΉ Ρ†Π΅ΠΏΠΈ Π²Ρ‹ ΠΌΠΎΠΆΠ΅Ρ‚Π΅ ΠΎΠΏΡ€Π΅Π΄Π΅Π»ΠΈΡ‚ΡŒ ΠΏΠΎΡ‚ΠΎΠΊ элСктричСского Ρ‚ΠΎΠΊΠ°, создав ΠΏΠΎΡ‚ΠΎΠΊΠΈ элСктричСского Ρ‚ΠΎΠΊΠ° ΠΎΡ‚ ΠΏΠΎΠ»ΠΎΠΆΠΈΡ‚Π΅Π»ΡŒΠ½ΠΎΠ³ΠΎ ΠΊΠΎΠ½Ρ†Π° Π±Π°Ρ‚Π°Ρ€Π΅ΠΈ ΠΊ ΠΎΡ‚Ρ€ΠΈΡ†Π°Ρ‚Π΅Π»ΡŒΠ½ΠΎΠΌΡƒ.ΠŸΠΎΠ»ΠΎΠΆΠΈΡ‚Π΅Π»ΡŒΠ½Ρ‹ΠΉ ΠΊΠΎΠ½Π΅Ρ† ΠΎΠ±ΠΎΠ·Π½Π°Ρ‡Π΅Π½ плюсом Π½Π° источникС напряТСния, Π° ΠΎΡ‚Ρ€ΠΈΡ†Π°Ρ‚Π΅Π»ΡŒΠ½Ρ‹ΠΉ -.

По ΠΌΠ΅Ρ€Π΅ прохоТдСния Ρ‚ΠΎΠΊΠ° ΠΏΠΎ вСтвям ΠΏΠ°Ρ€Π°Π»Π»Π΅Π»ΡŒΠ½ΠΎΠΉ Ρ†Π΅ΠΏΠΈ ΠΏΠΎΠΌΠ½ΠΈΡ‚Π΅, Ρ‡Ρ‚ΠΎ вСсь Ρ‚ΠΎΠΊ, входящий Π² ΠΎΠ΄ΠΈΠ½ ΡƒΠ·Π΅Π» ΠΈΠ»ΠΈ Ρ‚ΠΎΡ‡ΠΊΡƒ Π² Ρ†Π΅ΠΏΠΈ, Π΄ΠΎΠ»ΠΆΠ΅Π½ Ρ€Π°Π²Π½ΡΡ‚ΡŒΡΡ всСму Ρ‚ΠΎΠΊΡƒ, выходящСму ΠΈΠ»ΠΈ выходящСму ΠΈΠ· этой Ρ‚ΠΎΡ‡ΠΊΠΈ. Π’Π°ΠΊΠΆΠ΅ ΠΈΠΌΠ΅ΠΉΡ‚Π΅ Π² Π²ΠΈΠ΄Ρƒ, Ρ‡Ρ‚ΠΎ ΠΏΠ°Π΄Π΅Π½ΠΈΠ΅ напряТСния Π²ΠΎΠΊΡ€ΡƒΠ³ любого Π·Π°ΠΌΠΊΠ½ΡƒΡ‚ΠΎΠ³ΠΎ ΠΊΠΎΠ½Ρ‚ΡƒΡ€Π° Π² Ρ†Π΅ΠΏΠΈ Π΄ΠΎΠ»ΠΆΠ½ΠΎ Ρ€Π°Π²Π½ΡΡ‚ΡŒΡΡ Π½ΡƒΠ»ΡŽ. Π­Ρ‚ΠΈ Π΄Π²Π° утвСрТдСния ΡΠ²Π»ΡΡŽΡ‚ΡΡ Π·Π°ΠΊΠΎΠ½Π°ΠΌΠΈ Ρ†Π΅ΠΏΠ΅ΠΉ ΠšΠΈΡ€Ρ…Π³ΠΎΡ„Π° .

Π₯арактСристики ΠΏΠ°Ρ€Π°Π»Π»Π΅Π»ΡŒΠ½ΠΎΠΉ Ρ†Π΅ΠΏΠΈ

Π’ ΠΏΠ°Ρ€Π°Π»Π»Π΅Π»ΡŒΠ½Ρ‹Ρ… цСпях ΠΈΡΠΏΠΎΠ»ΡŒΠ·ΡƒΡŽΡ‚ΡΡ отвСтвлСния, ΠΊΠΎΡ‚ΠΎΡ€Ρ‹Π΅ ΠΏΠΎΠ·Π²ΠΎΠ»ΡΡŽΡ‚ Ρ‚ΠΎΠΊΡƒ ΠΏΡ€ΠΎΡ…ΠΎΠ΄ΠΈΡ‚ΡŒ ΠΏΠΎ Ρ€Π°Π·Π½Ρ‹ΠΌ ΠΌΠ°Ρ€ΡˆΡ€ΡƒΡ‚Π°ΠΌ Ρ‡Π΅Ρ€Π΅Π· Ρ†Π΅ΠΏΡŒ.Π’ΠΎΠΊ ΠΏΡ€ΠΎΡ…ΠΎΠ΄ΠΈΡ‚ ΠΎΡ‚ ΠΏΠΎΠ»ΠΎΠΆΠΈΡ‚Π΅Π»ΡŒΠ½ΠΎΠ³ΠΎ ΠΊΠΎΠ½Ρ†Π° Π±Π°Ρ‚Π°Ρ€Π΅ΠΈ ΠΈΠ»ΠΈ источника напряТСния ΠΊ ΠΎΡ‚Ρ€ΠΈΡ†Π°Ρ‚Π΅Π»ΡŒΠ½ΠΎΠΌΡƒ. НапряТСниС остаСтся постоянным ΠΏΠΎ всСй Ρ†Π΅ΠΏΠΈ, Π² Ρ‚ΠΎ врСмя ΠΊΠ°ΠΊ Ρ‚ΠΎΠΊ измСняСтся Π² зависимости ΠΎΡ‚ сопротивлСния ΠΊΠ°ΠΆΠ΄ΠΎΠΉ Π²Π΅Ρ‚Π²ΠΈ.

ΠŸΡ€ΠΈΠΌΠ΅Ρ€Ρ‹ ΠΏΠ°Ρ€Π°Π»Π»Π΅Π»ΡŒΠ½Ρ‹Ρ… Ρ†Π΅ΠΏΠ΅ΠΉ

Π§Ρ‚ΠΎΠ±Ρ‹ Π½Π°ΠΉΡ‚ΠΈ ΠΏΠΎΠ»Π½ΠΎΠ΅ сопротивлСниС рСзисторов, установлСнных ΠΏΠ°Ρ€Π°Π»Π»Π΅Π»ΡŒΠ½ΠΎ Π΄Ρ€ΡƒΠ³ Π΄Ρ€ΡƒΠ³Ρƒ, ΠΈΡΠΏΠΎΠ»ΡŒΠ·ΡƒΠΉΡ‚Π΅ Ρ„ΠΎΡ€ΠΌΡƒΠ»Ρƒ

\ frac {1} {R_ {total}} = \ frac {1} {R_1} + \ frac {1} {R_2} + \ frac {1} {R_3} + … + \ frac {1} {R_n}

, Π² ΠΊΠΎΡ‚ΠΎΡ€ΠΎΠΌ сопротивлСниС ΠΊΠ°ΠΆΠ΄ΠΎΠ³ΠΎ рСзистора суммируСтся Π² ΠΏΡ€Π°Π²ΠΎΠΉ части уравнСния.На ΠΏΡ€ΠΈΠ²Π΅Π΄Π΅Π½Π½ΠΎΠΉ Π²Ρ‹ΡˆΠ΅ Π΄ΠΈΠ°Π³Ρ€Π°ΠΌΠΌΠ΅ ΠΎΠ±Ρ‰Π΅Π΅ сопротивлСниС Π² Ом (Ом) ΠΌΠΎΠΆΠ½ΠΎ Ρ€Π°ΡΡΡ‡ΠΈΡ‚Π°Ρ‚ΡŒ ΡΠ»Π΅Π΄ΡƒΡŽΡ‰ΠΈΠΌ ΠΎΠ±Ρ€Π°Π·ΠΎΠΌ:

  1. 1 / R всСго = 1/5 Ом + 1/6 Ом + 1/10 Ом
  2. 1 / R всСго = 6/30 Ом + 5/30 Ом + 3/30 Ом
  3. 1 / R всСго = 14/30 Ом
  4. R всСго = 15/7 Ом или около 2,14 Ом

ΠžΠ±Ρ€Π°Ρ‚ΠΈΡ‚Π΅ Π²Π½ΠΈΠΌΠ°Π½ΠΈΠ΅, Ρ‡Ρ‚ΠΎ Π²Ρ‹ ΠΌΠΎΠΆΠ΅Ρ‚Π΅ Β«ΠΏΠ΅Ρ€Π΅Π²Π΅Ρ€Π½ΡƒΡ‚ΡŒΒ» ΠΎΠ±Π΅ стороны уравнСния с шага 3 Π½Π° шаг 4 Ρ‚ΠΎΠ»ΡŒΠΊΠΎ Ρ‚ΠΎΠ³Π΄Π°, ΠΊΠΎΠ³Π΄Π° Π΅ΡΡ‚ΡŒ Ρ‚ΠΎΠ»ΡŒΠΊΠΎ ΠΎΠ΄ΠΈΠ½ Ρ‡Π»Π΅Π½ с ΠΎΠ±Π΅ΠΈΡ… сторон уравнСния (Π² Π΄Π°Π½Π½ΠΎΠΌ случаС 1 / R всСго слСва ΠΈ 14/30 Ом справа).

ПослС Ρ‚ΠΎΠ³ΠΎ, ΠΊΠ°ΠΊ Π²Ρ‹ рассчитали сопротивлСниС, Ρ‚ΠΎΠΊ ΠΈ напряТСниС, ΠΌΠΎΠΆΠ½ΠΎ Ρ€Π°ΡΡΡ‡ΠΈΡ‚Π°Ρ‚ΡŒ ΠΏΠΎ Π·Π°ΠΊΠΎΠ½Ρƒ Ома V = I / R , Π² ΠΊΠΎΡ‚ΠΎΡ€ΠΎΠΌ V — напряТСниС, ΠΈΠ·ΠΌΠ΅Ρ€Π΅Π½Π½ΠΎΠ΅ Π² Π²ΠΎΠ»ΡŒΡ‚Π°Ρ…, I — Ρ‚ΠΎΠΊ, ΠΈΠ·ΠΌΠ΅Ρ€Π΅Π½Π½Ρ‹ΠΉ Π² Π°ΠΌΠΏΠ΅Ρ€Π°Ρ…. , Π° R — сопротивлСниС Π² Ом. Π’ ΠΏΠ°Ρ€Π°Π»Π»Π΅Π»ΡŒΠ½Ρ‹Ρ… цСпях сумма Ρ‚ΠΎΠΊΠΎΠ², ΠΏΡ€ΠΎΡ‚Π΅ΠΊΠ°ΡŽΡ‰ΠΈΡ… ΠΏΠΎ ΠΊΠ°ΠΆΠ΄ΠΎΠΌΡƒ ΠΏΡƒΡ‚ΠΈ, являСтся ΠΏΠΎΠ»Π½Ρ‹ΠΌ Ρ‚ΠΎΠΊΠΎΠΌ ΠΎΡ‚ источника. Π’ΠΎΠΊ Π½Π° ΠΊΠ°ΠΆΠ΄ΠΎΠΌ рСзисторС Π² Ρ†Π΅ΠΏΠΈ ΠΌΠΎΠΆΠ½ΠΎ Ρ€Π°ΡΡΡ‡ΠΈΡ‚Π°Ρ‚ΡŒ, ΡƒΠΌΠ½ΠΎΠΆΠΈΠ² напряТСниС Π½Π° сопротивлСниС рСзистора.НапряТСниС остаСтся постоянным ΠΏΠΎ всСй Ρ†Π΅ΠΏΠΈ, поэтому напряТСниС являСтся напряТСниСм Π±Π°Ρ‚Π°Ρ€Π΅ΠΈ ΠΈΠ»ΠΈ источника напряТСния.

ΠŸΠ°Ρ€Π°Π»Π»Π΅Π»ΡŒΠ½Π°Ρ ΠΈ ΠΏΠΎΡΠ»Π΅Π΄ΠΎΠ²Π°Ρ‚Π΅Π»ΡŒΠ½Π°Ρ Ρ†Π΅ΠΏΡŒ

β€’β€’β€’ Syed Hussain Ather

Π’ ΠΏΠΎΡΠ»Π΅Π΄ΠΎΠ²Π°Ρ‚Π΅Π»ΡŒΠ½Ρ‹Ρ… цСпях Ρ‚ΠΎΠΊ постоянСн Π½Π° всСм протяТСнии, ΠΏΠ°Π΄Π΅Π½ΠΈΠ΅ напряТСния зависит ΠΎΡ‚ сопротивлСния ΠΊΠ°ΠΆΠ΄ΠΎΠ³ΠΎ рСзистора, Π° ΠΎΠ±Ρ‰Π΅Π΅ сопротивлСниС складываСтся ΠΈΠ· ΠΊΠ°ΠΆΠ΄ΠΎΠ³ΠΎ ΠΎΡ‚Π΄Π΅Π»ΡŒΠ½ΠΎΠ³ΠΎ рСзистора. Π’ ΠΏΠ°Ρ€Π°Π»Π»Π΅Π»ΡŒΠ½Ρ‹Ρ… цСпях напряТСниС постоянно, Ρ‚ΠΎΠΊ зависит ΠΎΡ‚ ΠΊΠ°ΠΆΠ΄ΠΎΠ³ΠΎ рСзистора, Π° Π²Π΅Π»ΠΈΡ‡ΠΈΠ½Π°, обратная Π²Π΅Π»ΠΈΡ‡ΠΈΠ½Π΅ ΠΎΠ±Ρ‰Π΅Π³ΠΎ сопротивлСния, являСтся суммой Π²Π΅Π»ΠΈΡ‡ΠΈΠ½, ΠΎΠ±Ρ€Π°Ρ‚Π½Ρ‹Ρ… Π²Π΅Π»ΠΈΡ‡ΠΈΠ½Π΅ ΠΊΠ°ΠΆΠ΄ΠΎΠ³ΠΎ ΠΎΡ‚Π΄Π΅Π»ΡŒΠ½ΠΎΠ³ΠΎ рСзистора.

ΠšΠΎΠ½Π΄Π΅Π½ΡΠ°Ρ‚ΠΎΡ€Ρ‹ ΠΈ ΠΊΠ°Ρ‚ΡƒΡˆΠΊΠΈ индуктивности ΠΌΠΎΠ³ΡƒΡ‚ ΠΈΡΠΏΠΎΠ»ΡŒΠ·ΠΎΠ²Π°Ρ‚ΡŒΡΡ для измСнСния заряда Π² ΠΏΠΎΡΠ»Π΅Π΄ΠΎΠ²Π°Ρ‚Π΅Π»ΡŒΠ½ΠΎΠΉ ΠΈ ΠΏΠ°Ρ€Π°Π»Π»Π΅Π»ΡŒΠ½ΠΎΠΉ цСпях с Ρ‚Π΅Ρ‡Π΅Π½ΠΈΠ΅ΠΌ Π²Ρ€Π΅ΠΌΠ΅Π½ΠΈ. Π’ ΠΏΠΎΡΠ»Π΅Π΄ΠΎΠ²Π°Ρ‚Π΅Π»ΡŒΠ½ΠΎΠΉ Ρ†Π΅ΠΏΠΈ общая Π΅ΠΌΠΊΠΎΡΡ‚ΡŒ , Ρ†Π΅ΠΏΠΈ (заданная ΠΏΠ΅Ρ€Π΅ΠΌΠ΅Π½Π½ΠΎΠΉ C ), ΠΏΠΎΡ‚Π΅Π½Ρ†ΠΈΠ°Π» кондСнсатора Π½Π°ΠΊΠ°ΠΏΠ»ΠΈΠ²Π°Ρ‚ΡŒ заряд с Ρ‚Π΅Ρ‡Π΅Π½ΠΈΠ΅ΠΌ Π²Ρ€Π΅ΠΌΠ΅Π½ΠΈ, являСтся ΠΎΠ±Ρ€Π°Ρ‚Π½ΠΎΠΉ суммой ΠΎΠ±Ρ€Π°Ρ‚Π½Ρ‹Ρ… Π²Π΅Π»ΠΈΡ‡ΠΈΠ½ ΠΊΠ°ΠΆΠ΄ΠΎΠΉ ΠΎΡ‚Π΄Π΅Π»ΡŒΠ½ΠΎΠΉ Смкости, ΠΈ общая ΠΈΠ½Π΄ΡƒΠΊΡ‚ΠΈΠ²Π½ΠΎΡΡ‚ΡŒ ( I ), ΠΌΠΎΡ‰Π½ΠΎΡΡ‚ΡŒ ΠΈΠ½Π΄ΡƒΠΊΡ‚ΠΎΡ€ΠΎΠ², Π²Ρ‹Π΄Π΅Π»ΡΡŽΡ‰ΠΈΡ… заряд с Ρ‚Π΅Ρ‡Π΅Π½ΠΈΠ΅ΠΌ Π²Ρ€Π΅ΠΌΠ΅Π½ΠΈ, являСтся суммой ΠΊΠ°ΠΆΠ΄ΠΎΠΉ индуктивности. Напротив, Π² ΠΏΠ°Ρ€Π°Π»Π»Π΅Π»ΡŒΠ½ΠΎΠΉ Ρ†Π΅ΠΏΠΈ общая Π΅ΠΌΠΊΠΎΡΡ‚ΡŒ являСтся суммой ΠΊΠ°ΠΆΠ΄ΠΎΠ³ΠΎ ΠΎΡ‚Π΄Π΅Π»ΡŒΠ½ΠΎΠ³ΠΎ кондСнсатора, Π° Π²Π΅Π»ΠΈΡ‡ΠΈΠ½Π°, обратная Π²Π΅Π»ΠΈΡ‡ΠΈΠ½Π΅ ΠΏΠΎΠ»Π½ΠΎΠΉ индуктивности, являСтся суммой ΠΎΠ±Ρ€Π°Ρ‚Π½Ρ‹Ρ… Π²Π΅Π»ΠΈΡ‡ΠΈΠ½ ΠΊΠ°ΠΆΠ΄ΠΎΠΉ ΠΈΠ½Π΄ΠΈΠ²ΠΈΠ΄ΡƒΠ°Π»ΡŒΠ½ΠΎΠΉ индуктивности.

ΠŸΠΎΡΠ»Π΅Π΄ΠΎΠ²Π°Ρ‚Π΅Π»ΡŒΠ½Ρ‹Π΅ ΠΈ ΠΏΠ°Ρ€Π°Π»Π»Π΅Π»ΡŒΠ½Ρ‹Π΅ Ρ†Π΅ΠΏΠΈ Ρ‚Π°ΠΊΠΆΠ΅ ΠΈΠΌΠ΅ΡŽΡ‚ Ρ€Π°Π·Π½Ρ‹Π΅ Ρ„ΡƒΠ½ΠΊΡ†ΠΈΠΈ. Π’ ΠΏΠΎΡΠ»Π΅Π΄ΠΎΠ²Π°Ρ‚Π΅Π»ΡŒΠ½ΠΎΠΉ Ρ†Π΅ΠΏΠΈ, Ссли ΠΎΠ΄Π½Π° Ρ‡Π°ΡΡ‚ΡŒ сломана, Ρ‚ΠΎΠΊ Π²ΠΎΠΎΠ±Ρ‰Π΅ Π½Π΅ Π±ΡƒΠ΄Π΅Ρ‚ Ρ‚Π΅Ρ‡ΡŒ ΠΏΠΎ Ρ†Π΅ΠΏΠΈ. Π’ ΠΏΠ°Ρ€Π°Π»Π»Π΅Π»ΡŒΠ½ΠΎΠΉ Ρ†Π΅ΠΏΠΈ ΠΎΡ‚ΠΊΡ€Ρ‹Ρ‚ΠΈΠ΅ ΠΎΡ‚Π΄Π΅Π»ΡŒΠ½ΠΎΠΉ Π²Π΅Ρ‚Π²ΠΈ останавливаСт Ρ‚ΠΎΠ»ΡŒΠΊΠΎ Ρ‚ΠΎΠΊ Π² этой Π²Π΅Ρ‚Π²ΠΈ. ΠžΡΡ‚Π°Π»ΡŒΠ½Ρ‹Π΅ Π²Π΅Ρ‚Π²ΠΈ Π±ΡƒΠ΄ΡƒΡ‚ ΠΏΡ€ΠΎΠ΄ΠΎΠ»ΠΆΠ°Ρ‚ΡŒ Ρ€Π°Π±ΠΎΡ‚Π°Ρ‚ΡŒ, ΠΏΠΎΡ‚ΠΎΠΌΡƒ Ρ‡Ρ‚ΠΎ Ρƒ Ρ‚ΠΎΠΊΠ° Π΅ΡΡ‚ΡŒ нСсколько ΠΏΡƒΡ‚Π΅ΠΉ, ΠΊΠΎΡ‚ΠΎΡ€Ρ‹Π΅ ΠΎΠ½ ΠΌΠΎΠΆΠ΅Ρ‚ ΠΏΡ€ΠΎΠΉΡ‚ΠΈ ΠΏΠΎ Ρ†Π΅ΠΏΠΈ.

ΠŸΠΎΡΠ»Π΅Π΄ΠΎΠ²Π°Ρ‚Π΅Π»ΡŒΠ½ΠΎ-ΠΏΠ°Ρ€Π°Π»Π»Π΅Π»ΡŒΠ½Π°Ρ Ρ†Π΅ΠΏΡŒ

β€’β€’β€’ Syed Hussain Ather

Π¦Π΅ΠΏΠΈ, Ρƒ ΠΊΠΎΡ‚ΠΎΡ€Ρ‹Ρ… Π΅ΡΡ‚ΡŒ ΠΎΠ±Π° Ρ€Π°Π·Π²Π΅Ρ‚Π²Π»Π΅Π½Π½Ρ‹Ρ… элСмСнта, ΠΊΠΎΡ‚ΠΎΡ€Ρ‹Π΅ Ρ‚Π°ΠΊΠΆΠ΅ соСдинСны Ρ‚Π°ΠΊΠΈΠΌ ΠΎΠ±Ρ€Π°Π·ΠΎΠΌ, Ρ‡Ρ‚ΠΎ Ρ‚ΠΎΠΊ Ρ‚Π΅Ρ‡Π΅Ρ‚ Π² ΠΎΠ΄Π½ΠΎΠΌ Π½Π°ΠΏΡ€Π°Π²Π»Π΅Π½ΠΈΠΈ ΠΌΠ΅ΠΆΠ΄Ρƒ этими вСтвями, ΠΊΠ°ΠΊ ΠΏΠΎΡΠ»Π΅Π΄ΠΎΠ²Π°Ρ‚Π΅Π»ΡŒΠ½Ρ‹Π΅, Ρ‚Π°ΠΊ ΠΈ ΠΏΠ°Ρ€Π°Π»Π»Π΅Π»ΡŒΠ½Ρ‹Π΅.Π’ этих случаях Π²Ρ‹ ΠΌΠΎΠΆΠ΅Ρ‚Π΅ ΠΏΡ€ΠΈΠΌΠ΅Π½ΡΡ‚ΡŒ ΠΏΡ€Π°Π²ΠΈΠ»Π° ΠΊΠ°ΠΊ для ΠΏΠΎΡΠ»Π΅Π΄ΠΎΠ²Π°Ρ‚Π΅Π»ΡŒΠ½ΠΎΠ³ΠΎ, Ρ‚Π°ΠΊ ΠΈ для ΠΏΠ°Ρ€Π°Π»Π»Π΅Π»ΡŒΠ½ΠΎΠ³ΠΎ ΠΏΠΎΠ΄ΠΊΠ»ΡŽΡ‡Π΅Π½ΠΈΡ, Π² зависимости ΠΎΡ‚ схСмы. Π’ ΠΏΡ€ΠΈΠ²Π΅Π΄Π΅Π½Π½ΠΎΠΌ Π²Ρ‹ΡˆΠ΅ ΠΏΡ€ΠΈΠΌΠ΅Ρ€Π΅ R1 ΠΈ R2 ΠΏΠ°Ρ€Π°Π»Π»Π΅Π»ΡŒΠ½Ρ‹ Π΄Ρ€ΡƒΠ³ Π΄Ρ€ΡƒΠ³Ρƒ, Ρ‡Ρ‚ΠΎΠ±Ρ‹ ΡΡ„ΠΎΡ€ΠΌΠΈΡ€ΠΎΠ²Π°Ρ‚ΡŒ R5 , Π° Ρ‚Π°ΠΊΠΆΠ΅ R3 ΠΈ R4 , Ρ‡Ρ‚ΠΎΠ±Ρ‹ ΡΡ„ΠΎΡ€ΠΌΠΈΡ€ΠΎΠ²Π°Ρ‚ΡŒ R6 . Π˜Ρ… ΠΌΠΎΠΆΠ½ΠΎ ΡΡƒΠΌΠΌΠΈΡ€ΠΎΠ²Π°Ρ‚ΡŒ ΠΏΠ°Ρ€Π°Π»Π»Π΅Π»ΡŒΠ½ΠΎ ΡΠ»Π΅Π΄ΡƒΡŽΡ‰ΠΈΠΌ ΠΎΠ±Ρ€Π°Π·ΠΎΠΌ:

  1. 1 / R5 = 1/1 Ом + 1/5 Ом
  2. 1 / R5 = 5/5 Ом + 1/5 Ом
  3. 1 / R5 = 6/5 Ом
  4. R5 = 5/6 Ом или около 0,83 Ом
  1. 1 / R6 = 1/7 Ом + 1/2 Ом
  2. 1 / R6 = 2/14 Ом + 7/14 Ом
  3. 1 / R6 = 9/14 Ом
  4. R6 = 14/9 Ом или около 1.56 Ом

β€’β€’β€’ Syed Hussain Ather

Π‘Ρ…Π΅ΠΌΠ° ΠΌΠΎΠΆΠ΅Ρ‚ Π±Ρ‹Ρ‚ΡŒ ΡƒΠΏΡ€ΠΎΡ‰Π΅Π½Π° для создания схСмы, ΠΏΠΎΠΊΠ°Π·Π°Π½Π½ΠΎΠΉ нСпосрСдствСнно Π²Ρ‹ΡˆΠ΅, с R5 ΠΈ R6 . Π­Ρ‚ΠΈ Π΄Π²Π° рСзистора ΠΌΠΎΠ³ΡƒΡ‚ Π±Ρ‹Ρ‚ΡŒ Π΄ΠΎΠ±Π°Π²Π»Π΅Π½Ρ‹ просто, ΠΊΠ°ΠΊ Ссли Π±Ρ‹ Ρ†Π΅ΠΏΡŒ Π±Ρ‹Π»Π° ΠΏΠΎΡΠ»Π΅Π΄ΠΎΠ²Π°Ρ‚Π΅Π»ΡŒΠ½ΠΎΠΉ.

R_ {total} = 5/6 \ Omega + 14/9 \ Omega = 2.38 \ Omega

ΠŸΡ€ΠΈ напряТСнии 20 Π’ Π² качСствС напряТСния Π·Π°ΠΊΠΎΠ½ Ома опрСдСляСт, Ρ‡Ρ‚ΠΎ ΠΎΠ±Ρ‰ΠΈΠΉ Ρ‚ΠΎΠΊ Ρ€Π°Π²Π΅Π½ Π’ / R , ΠΈΠ»ΠΈ 20 Π’ / (43/18 Ом) = 360/43 А ΠΈΠ»ΠΈ ΠΎΠΊΠΎΠ»ΠΎ 8.37 A. Π˜ΡΠΏΠΎΠ»ΡŒΠ·ΡƒΡ этот ΠΎΠ±Ρ‰ΠΈΠΉ Ρ‚ΠΎΠΊ, Π²Ρ‹ Ρ‚Π°ΠΊΠΆΠ΅ ΠΌΠΎΠΆΠ΅Ρ‚Π΅ ΠΎΠΏΡ€Π΅Π΄Π΅Π»ΠΈΡ‚ΡŒ ΠΏΠ°Π΄Π΅Π½ΠΈΠ΅ напряТСния Π½Π° R5 ΠΈ R6, ΠΈΡΠΏΠΎΠ»ΡŒΠ·ΡƒΡ Π·Π°ΠΊΠΎΠ½ Ома ( Π’ = I / R ).

V_5 = \ frac {360} {43} \ times 5/6 = 6.98 \ text {V}

V_5 = \ frac {360} {43} \ times 14/9 = 13.02 \ text {V}

НаконСц, эти падСния напряТСния для R5 ΠΈ R6 ΠΌΠΎΠΆΠ½ΠΎ Ρ€Π°Π·Π΄Π΅Π»ΠΈΡ‚ΡŒ ΠΎΠ±Ρ€Π°Ρ‚Π½ΠΎ Π½Π° исходныС ΠΏΠ°Ρ€Π°Π»Π»Π΅Π»ΡŒΠ½Ρ‹Π΅ Ρ†Π΅ΠΏΠΈ для расчСта Ρ‚ΠΎΠΊΠ° R1 ΠΈ R2 для R5 ΠΈ R2 ΠΈ R3 для R6 с использованиСм Π·Π°ΠΊΠΎΠ½Π° Ома.

I1 = (1800/258 Π’) / 1 Ом = 1800/258 A ΠΈΠ»ΠΈ ΠΎΠΊΠΎΠ»ΠΎ Ρ‚ 6,98 А.

I2 = (1800/258 Π’) /5 Ом = 1500/43 А ΠΈΠ»ΠΈ ΠΎΠΊΠΎΠ»ΠΎ Ρ‚ 34,88 А.

I3 = ( 680/129 Π’ ) / 7 Ом = 4760/129 A ΠΈΠ»ΠΈ ΠΎΠΊΠΎΠ»ΠΎ 36,90 A .

I3 = ( 680/129 V ) / 2 Ом = 1360/129 A или около 10.54 А.

ΠŸΠΎΡΠ»Π΅Π΄ΠΎΠ²Π°Ρ‚Π΅Π»ΡŒΠ½Ρ‹Π΅ ΠΈ ΠΏΠ°Ρ€Π°Π»Π»Π΅Π»ΡŒΠ½Ρ‹Π΅ рСзисторы

Π’Π²Π΅Π΄Π΅Π½ΠΈΠ΅

Π¦Π΅ΠΏΠΈ постоянного Ρ‚ΠΎΠΊΠ° (DC) Ρ…Π°Ρ€Π°ΠΊΡ‚Π΅Ρ€ΠΈΠ·ΡƒΡŽΡ‚ΡΡ Π²Π΅Π»ΠΈΡ‡ΠΈΠ½Π°ΠΌΠΈ Ρ‚ΠΎΠΊΠ°, напряТСния ΠΈ сопротивлСния. Π’ΠΎΠΊ — это ΡΠΊΠΎΡ€ΠΎΡΡ‚ΡŒ ΠΏΠΎΡ‚ΠΎΠΊΠ° заряда. Π•Π΄ΠΈΠ½ΠΈΡ†Π° БИ — Π°ΠΌΠΏΠ΅Ρ€ (А). Условно Π½Π°ΠΏΡ€Π°Π²Π»Π΅Π½ΠΈΠ΅ Ρ‚ΠΎΠΊΠ° — это Π½Π°ΠΏΡ€Π°Π²Π»Π΅Π½ΠΈΠ΅ ΠΏΠΎΡ‚ΠΎΠΊΠ° заряда, хотя Π² мСталличСских ΠΏΡ€ΠΎΠ²ΠΎΠ΄Π½ΠΈΠΊΠ°Ρ… Ρ‚ΠΎΠΊ Π²ΠΎΠ·Π½ΠΈΠΊΠ°Π΅Ρ‚ ΠΈΠ·-Π·Π° ΠΏΠΎΡ‚ΠΎΠΊΠ° ΠΎΡ‚Ρ€ΠΈΡ†Π°Ρ‚Π΅Π»ΡŒΠ½ΠΎΠ³ΠΎ заряда (элСктронов) Π² ΠΏΡ€ΠΎΡ‚ΠΈΠ²ΠΎΠΏΠΎΠ»ΠΎΠΆΠ½ΠΎΠΌ Π½Π°ΠΏΡ€Π°Π²Π»Π΅Π½ΠΈΠΈ. Из-Π·Π° сохранСния заряда Ρ‚ΠΎΠΊ ΠΎΠ΄ΠΈΠ½Π°ΠΊΠΎΠ² Π²ΠΎ всСх Ρ‚ΠΎΡ‡ΠΊΠ°Ρ… ΠΎΠ΄Π½ΠΎΠΏΠ΅Ρ‚Π»Π΅Π²ΠΎΠΉ Ρ†Π΅ΠΏΠΈ.Π’ Ρ‚ΠΎΡ‡ΠΊΠ΅ вСтвлСния Π² Ρ†Π΅ΠΏΠΈ, Π³Π΄Π΅ проводящий ΠΏΡƒΡ‚ΡŒ раздСляСтся Π½Π° Π΄Π²Π° ΠΈΠ»ΠΈ Π±ΠΎΠ»Π΅Π΅, ΠΎΠ±Ρ‰ΠΈΠΉ Ρ‚ΠΎΠΊ Π² Ρ‚ΠΎΡ‡ΠΊΠ΅ вСтвлСния Ρ€Π°Π²Π΅Π½ ΠΏΠΎΠ»Π½ΠΎΠΌΡƒ Ρ‚ΠΎΠΊΡƒ ΠΈΠ· этой Ρ‚ΠΎΡ‡ΠΊΠΈ. ΠžΠ±Ρ‹Ρ‡Π½ΠΎ Ρ‚ΠΎΠΊ Ρ‚Π΅Ρ‡Π΅Ρ‚ ΠΎΡ‚ ΠΏΠΎΠ»ΠΎΠΆΠΈΡ‚Π΅Π»ΡŒΠ½ΠΎΠΉ ΠΊΠ»Π΅ΠΌΠΌΡ‹ Π±Π°Ρ‚Π°Ρ€Π΅ΠΈ ΠΈΠ»ΠΈ источника питания ΠΊ ΠΎΡ‚Ρ€ΠΈΡ†Π°Ρ‚Π΅Π»ΡŒΠ½ΠΎΠΉ ΠΊΠ»Π΅ΠΌΠΌΠ΅. Для поддСрТания Ρ‚ΠΎΠΊΠ° Π² Ρ†Π΅ΠΏΠΈ Π΄ΠΎΠ»ΠΆΠ΅Π½ Π±Ρ‹Ρ‚ΡŒ ΠΏΠΎΠ»Π½ΠΎΡΡ‚ΡŒΡŽ проводящий ΠΏΡƒΡ‚ΡŒ. НапряТСниС — это ΠΌΠ΅Ρ€Π° разности элСктричСских ΠΏΠΎΡ‚Π΅Π½Ρ†ΠΈΠ°Π»ΠΎΠ² ΠΌΠ΅ΠΆΠ΄Ρƒ двумя Ρ‚ΠΎΡ‡ΠΊΠ°ΠΌΠΈ Π² Ρ†Π΅ΠΏΠΈ. Π•Π΄ΠΈΠ½ΠΈΡ†Π° БИ — Π²ΠΎΠ»ΡŒΡ‚ (Π’). ΠŸΠΎΡΠΊΠΎΠ»ΡŒΠΊΡƒ элСктричСская сила являСтся консСрвативной, сумма напряТСний увСличиваСтся ΠΈ ΡƒΠΌΠ΅Π½ΡŒΡˆΠ°Π΅Ρ‚ΡΡ Π²ΠΎΠΊΡ€ΡƒΠ³ любого Π·Π°ΠΌΠΊΠ½ΡƒΡ‚ΠΎΠ³ΠΎ ΠΊΠΎΠ½Ρ‚ΡƒΡ€Π°, ΠΎΠ½Π° Ρ€Π°Π²Π½Π° Π½ΡƒΠ»ΡŽ.Π‘ΠΎΠΏΡ€ΠΎΡ‚ΠΈΠ²Π»Π΅Π½ΠΈΠ΅ — это свойство элСмСнта схСмы (ΠΏΡ€ΠΎΠ²ΠΎΠ΄Π½ΠΈΠΊΠ°) ΠΏΡ€Π΅ΠΏΡΡ‚ΡΡ‚Π²ΠΎΠ²Π°Ρ‚ΡŒ ΠΏΡ€ΠΎΡ…ΠΎΠΆΠ΄Π΅Π½ΠΈΡŽ Ρ‚ΠΎΠΊΠ°. Π‘ΠΎΠΏΡ€ΠΎΡ‚ΠΈΠ²Π»Π΅Π½ΠΈΠ΅ опрСдСляСтся Π³Π΄Π΅ Π’, — напряТСниС Π½Π° элСмСнтС схСмы, Π° I — Ρ‚ΠΎΠΊ, ΠΏΡ€ΠΎΡ‚Π΅ΠΊΠ°ΡŽΡ‰ΠΈΠΉ Ρ‡Π΅Ρ€Π΅Π· Π½Π΅Π³ΠΎ. Если R постоянный, ΠΎΠ΄ΠΈΠ½Π°ΠΊΠΎΠ²Ρ‹ΠΉ для всСх V , Ρ‚ΠΎ элСмСнт схСмы подчиняСтся Π·Π°ΠΊΠΎΠ½Ρƒ Ома. Π•Π΄ΠΈΠ½ΠΈΡ†Π΅ΠΉ измСрСния сопротивлСния Π² систСмС БИ являСтся ΠΎΠΌ (Ом). Π‘ΠΎΠΏΡ€ΠΎΡ‚ΠΈΠ²Π»Π΅Π½ΠΈΠ΅ рСзистивного элСмСнта Ρ†Π΅ΠΏΠΈ измСняСтся Π² зависимости ΠΎΡ‚ Ρ‚Π΅ΠΌΠΏΠ΅Ρ€Π°Ρ‚ΡƒΡ€Ρ‹. Π”Π²Π° рСзистора R 1 ΠΈ R 2 соСдинСны ΠΏΠΎΡΠ»Π΅Π΄ΠΎΠ²Π°Ρ‚Π΅Π»ΡŒΠ½ΠΎ, Ссли вСсь Ρ‚ΠΎΠΊ, ΠΊΠΎΡ‚ΠΎΡ€Ρ‹ΠΉ ΠΏΡ€ΠΎΡ…ΠΎΠ΄ΠΈΡ‚ Ρ‡Π΅Ρ€Π΅Π· R 1 , Ρ‚Π°ΠΊΠΆΠ΅ ΠΏΡ€ΠΎΡ…ΠΎΠ΄ΠΈΡ‚ Ρ‡Π΅Ρ€Π΅Π· R 2 .Π‘Π»Π΅Π΄ΠΎΠ²Π°Ρ‚Π΅Π»ΡŒΠ½ΠΎ, для Π΄Π²ΡƒΡ… ΠΏΠΎΡΠ»Π΅Π΄ΠΎΠ²Π°Ρ‚Π΅Π»ΡŒΠ½ΠΎ соСдинСнных рСзисторов Ρ‚ΠΎΠΊ с I 1 ΠΏΠΎ R 1 Ρ‚Π°ΠΊΠΎΠΉ ΠΆΠ΅, ΠΊΠ°ΠΊ Ρ‚ΠΎΠΊ с I 2 ΠΏΠΎ R 2 , ΠΈ этот Ρ‚ΠΎΠΊ Ρ‚Π°ΠΊΠΎΠΉ ΠΆΠ΅, ΠΊΠ°ΠΊ Ρƒ Ρ‚ΠΎΠΊ, I , ΠΊΠΎΡ‚ΠΎΡ€Ρ‹ΠΉ Π²Ρ…ΠΎΠ΄ΠΈΡ‚ Π² ΠΏΠΎΡΠ»Π΅Π΄ΠΎΠ²Π°Ρ‚Π΅Π»ΡŒΠ½ΡƒΡŽ ΡΠ΅Ρ‚ΡŒ:

I = I 1 = I 2 .

ΠžΠ±Ρ‰Π΅Π΅ напряТСниС Π’ Π² ΠΏΠΎΡΠ»Π΅Π΄ΠΎΠ²Π°Ρ‚Π΅Π»ΡŒΠ½ΠΎΠΉ сСти прСдставляСт собой сумму напряТСний Π’ 1 ΠΈ Π’ 2 Π½Π° ΠΊΠ°ΠΆΠ΄ΠΎΠΌ рСзисторС.Π’ΠΎ Π΅ΡΡ‚ΡŒ

V = V 1 + V 2 .

Π­ΠΊΠ²ΠΈΠ²Π°Π»Π΅Π½Ρ‚Π½ΠΎΠ΅ сопротивлСниС, R с , R 1 ΠΈ R 2 ΠΏΠΎΡΠ»Π΅Π΄ΠΎΠ²Π°Ρ‚Π΅Π»ΡŒΠ½ΠΎ опрСдСляСтся Π²Ρ‹Ρ€Π°ΠΆΠ΅Π½ΠΈΠ΅ΠΌ Π”Π²Π° рСзистора R 1 ΠΈ R 2 соСдинСны ΠΏΠ°Ρ€Π°Π»Π»Π΅Π»ΡŒΠ½ΠΎ, Ссли напряТСния Π’, 1 ΠΈ Π’ 2 Π½Π° ΠΊΠ°ΠΆΠ΄ΠΎΠΌ ΠΈΠ· Π½ΠΈΡ… ΠΎΠ΄ΠΈΠ½Π°ΠΊΠΎΠ²Ρ‹ ΠΈ Ρ€Π°Π²Π½Ρ‹ Π½Π°ΠΏΡ€ΡΠΆΠ΅Π½ΠΈΡŽ, Π’ , Ρ‡Π΅Ρ€Π΅Π· ΠΏΠ°Ρ€Π°Π»Π»Π΅Π»ΡŒΠ½ΡƒΡŽ ΡΠ΅Ρ‚ΡŒ.Π’ΠΎ Π΅ΡΡ‚ΡŒ

V = V 1 = V 2 .

Π’ΠΎΠΊΠΈ I 1 ΠΈ I 2 Ρ‡Π΅Ρ€Π΅Π· ΠΊΠ°ΠΆΠ΄Ρ‹ΠΉ ΠΈΠ· рСзисторов ΡΠΊΠ»Π°Π΄Ρ‹Π²Π°ΡŽΡ‚ΡΡ, Ρ‡Ρ‚ΠΎΠ±Ρ‹ ΠΏΠΎΠ»ΡƒΡ‡ΠΈΡ‚ΡŒ ΠΎΠ±Ρ‰ΠΈΠΉ Ρ‚ΠΎΠΊ, I , Ρ‚Π΅ΠΊΡƒΡ‰ΠΈΠΉ Π² ΡΠ΅Ρ‚ΡŒ ΠΈ ΠΈΠ· Π½Π΅Π΅:

I = I 1 + I 2 .

Π­ΠΊΠ²ΠΈΠ²Π°Π»Π΅Π½Ρ‚Π½ΠΎΠ΅ сопротивлСниС R p ΠΈΠ· R 1 ΠΈ R 2 ΠΏΠ°Ρ€Π°Π»Π»Π΅Π»ΡŒΠ½ΠΎ опрСдСляСтся Π²Ρ‹Ρ€Π°ΠΆΠ΅Π½ΠΈΠ΅ΠΌ Π­Ρ‚ΠΎ Ρ‚Π°ΠΊΠΆΠ΅ ΠΌΠΎΠΆΠ½ΠΎ Π·Π°ΠΏΠΈΡΠ°Ρ‚ΡŒ ΠΊΠ°ΠΊ АмпСрмСтры ΠΈΡΠΏΠΎΠ»ΡŒΠ·ΡƒΡŽΡ‚ΡΡ для измСрСния Ρ‚ΠΎΠΊΠ°.АмпСрмСтр ΠΏΠΎΠ΄ΠΊΠ»ΡŽΡ‡Π°Π΅Ρ‚ΡΡ ΠΏΠΎΡΠ»Π΅Π΄ΠΎΠ²Π°Ρ‚Π΅Π»ΡŒΠ½ΠΎ ΠΊ Ρ†Π΅ΠΏΠΈ, Ρ‚Π°ΠΊ Ρ‡Ρ‚ΠΎ вСсь измСряСмый Ρ‚ΠΎΠΊ Ρ‚Π΅Ρ‡Π΅Ρ‚ Ρ‡Π΅Ρ€Π΅Π· Π°ΠΌΠΏΠ΅Ρ€ΠΌΠ΅Ρ‚Ρ€. Π‘Π»Π΅Π΄ΠΎΠ²Π°Ρ‚Π΅Π»ΡŒΠ½ΠΎ, Π°ΠΌΠΏΠ΅Ρ€ΠΌΠ΅Ρ‚Ρ€Ρ‹ Π΄ΠΎΠ»ΠΆΠ½Ρ‹ ΠΈΠΌΠ΅Ρ‚ΡŒ ΠΎΡ‡Π΅Π½ΡŒ малСнькоС сопротивлСниС, Ρ‡Ρ‚ΠΎΠ±Ρ‹ Π½Π΅ ΠΈΠ·ΠΌΠ΅Π½ΡΡ‚ΡŒ Ρ‚ΠΎΠΊ Π² Ρ†Π΅ΠΏΠΈ. Π’ΠΎΠ»ΡŒΡ‚ΠΌΠ΅Ρ‚Ρ€Ρ‹ ΠΈΡΠΏΠΎΠ»ΡŒΠ·ΡƒΡŽΡ‚ΡΡ для измСрСния напряТСний. Π’ΠΎΠ»ΡŒΡ‚ΠΌΠ΅Ρ‚Ρ€ ΠΏΠΎΠ΄ΠΊΠ»ΡŽΡ‡Π°Π΅Ρ‚ΡΡ ΠΏΠ°Ρ€Π°Π»Π»Π΅Π»ΡŒΠ½ΠΎ Π² Π΄Π²ΡƒΡ… Ρ‚ΠΎΡ‡ΠΊΠ°Ρ…, ΠΌΠ΅ΠΆΠ΄Ρƒ ΠΊΠΎΡ‚ΠΎΡ€Ρ‹ΠΌΠΈ Π΄ΠΎΠ»ΠΆΠ½Π° Π±Ρ‹Ρ‚ΡŒ ΠΈΠ·ΠΌΠ΅Ρ€Π΅Π½Π° Ρ€Π°Π·Π½ΠΎΡΡ‚ΡŒ ΠΏΠΎΡ‚Π΅Π½Ρ†ΠΈΠ°Π»ΠΎΠ². Π‘Π»Π΅Π΄ΠΎΠ²Π°Ρ‚Π΅Π»ΡŒΠ½ΠΎ, Π²ΠΎΠ»ΡŒΡ‚ΠΌΠ΅Ρ‚Ρ€ Π΄ΠΎΠ»ΠΆΠ΅Π½ ΠΈΠΌΠ΅Ρ‚ΡŒ большоС сопротивлСниС, Ρ‡Ρ‚ΠΎΠ±Ρ‹ Ρ‡Π΅Ρ€Π΅Π· Π½Π΅Π³ΠΎ ΠΏΡ€ΠΎΡ…ΠΎΠ΄ΠΈΠ» ΠΎΡ‡Π΅Π½ΡŒ нСбольшой Ρ‚ΠΎΠΊ.

ЦСль

Π’ этой Π»Π°Π±ΠΎΡ€Π°Ρ‚ΠΎΡ€ΠΈΠΈ ΠΌΡ‹ Π±ΡƒΠ΄Π΅ΠΌ ΠΈΠ·ΠΌΠ΅Ρ€ΡΡ‚ΡŒ ΠΈ Π°Π½Π°Π»ΠΈΠ·ΠΈΡ€ΠΎΠ²Π°Ρ‚ΡŒ Ρ‚ΠΎΠΊΠΈ ΠΈ напряТСния для Ρ†Π΅ΠΏΠ΅ΠΉ, содСрТащих ΠΎΠ΄ΠΈΠ½ рСзистор, Π° Ρ‚Π°ΠΊΠΆΠ΅ для Π΄Π²ΡƒΡ… ΠΏΠΎΡΠ»Π΅Π΄ΠΎΠ²Π°Ρ‚Π΅Π»ΡŒΠ½ΠΎ Π²ΠΊΠ»ΡŽΡ‡Π΅Π½Π½Ρ‹Ρ… рСзисторов ΠΈ Π΄Π²ΡƒΡ… ΠΏΠ°Ρ€Π°Π»Π»Π΅Π»ΡŒΠ½ΠΎ.

Аппарат

  • Π˜ΡΡ‚ΠΎΡ‡Π½ΠΈΠΊ питания постоянного Ρ‚ΠΎΠΊΠ° 0-40 Π²ΠΎΠ»ΡŒΡ‚
  • Π›Π°ΠΌΠΏΠΎΡ‡ΠΊΠ° Π½Π° 12 Π²ΠΎΠ»ΡŒΡ‚ ΠΈ Ρ€ΠΎΠ·Π΅Ρ‚ΠΊΠ°
  • РСзисторы 150 ΠΈ 700 Ом
  • Π¦ΠΈΡ„Ρ€ΠΎΠ²ΠΎΠΉ ΠΌΡƒΠ»ΡŒΡ‚ΠΈΠΌΠ΅Ρ‚Ρ€

ΠŸΡ€ΠΎΡ†Π΅Π΄ΡƒΡ€Π°

РаспСчатайтС лист для этой Π»Π°Π±ΠΎΡ€Π°Ρ‚ΠΎΡ€Π½ΠΎΠΉ Ρ€Π°Π±ΠΎΡ‚Ρ‹. Π­Ρ‚ΠΎΡ‚ лист понадобится Π²Π°ΠΌ для записи Π²Π°ΡˆΠΈΡ… Π΄Π°Π½Π½Ρ‹Ρ….

Π˜Π·ΠΌΠ΅Ρ€Π΅Π½ΠΈΠ΅ напряТСния

1

Π‘Π»ΠΎΠΊ питания являСтся источником разности ΠΏΠΎΡ‚Π΅Π½Ρ†ΠΈΠ°Π»ΠΎΠ² (напряТСния).НайдитС источник питания постоянного Ρ‚ΠΎΠΊΠ° Π·Π° столом. НаТмитС ΠΊΠ½ΠΎΠΏΠΊΡƒ POWER ON / OFF Π² ΠΏΠΎΠ»ΠΎΠΆΠ΅Π½ΠΈΠ΅ ON. Π—Π°Ρ‚Π΅ΠΌ Π½Π°ΠΆΠΌΠΈΡ‚Π΅ ΠΊΠ½ΠΎΠΏΠΊΡƒ RANGE Π² ΠΏΠΎΠ»ΠΎΠΆΠ΅Π½ΠΈΠ΅ IN (0,85 A). Π­Ρ‚ΠΎ устанавливаСт источник питания Π² Π΄ΠΈΠ°ΠΏΠ°Π·ΠΎΠ½Π΅ 0-35 Π’ / 0-0,85 А. ΠŸΠΎΠ²Π΅Ρ€Π½ΠΈΡ‚Π΅ Ρ€ΡƒΡ‡ΠΊΡƒ Ρ€Π΅Π³ΡƒΠ»ΠΈΡ€ΠΎΠ²ΠΊΠΈ напряТСния ΠΈ Ρ‚ΠΎΠΊΠ° ADJUST ΠΏΡ€ΠΎΡ‚ΠΈΠ² часовой стрСлки. Π—Π°Ρ‚Π΅ΠΌ установитС ΠΌΠ°ΠΊΡΠΈΠΌΠ°Π»ΡŒΠ½Ρ‹ΠΉ Π²Ρ‹Ρ…ΠΎΠ΄Π½ΠΎΠΉ Ρ‚ΠΎΠΊ для этого экспСримСнта, Π½Π°ΠΆΠ°Π² ΠΊΠ½ΠΎΠΏΠΊΡƒ CC Set ΠΈ, удСрТивая Π΅Π΅, ΠΏΠΎΠ²Π΅Ρ€Π½ΠΈΡ‚Π΅ Ρ€ΡƒΡ‡ΠΊΡƒ Ρ€Π΅Π³ΡƒΠ»ΠΈΡ€ΠΎΠ²ΠΊΠΈ Ρ‚ΠΎΠΊΠ° ADJUST ΠΏΠΎ часовой стрСлкС Π΄ΠΎ Ρ‚Π΅Ρ… ΠΏΠΎΡ€, ΠΏΠΎΠΊΠ° Π½Π° дисплСС AMPS Π½Π΅ появится 0.30 A. ΠžΡ‚ΠΏΡƒΡΡ‚ΠΈΡ‚Π΅ ΠΊΠ½ΠΎΠΏΠΊΡƒ CC Set . НС ΠΏΠ΅Ρ€Π΅ΠΌΠ΅Ρ‰Π°ΠΉΡ‚Π΅ Ρ€ΡƒΡ‡ΠΊΡƒ Ρ‚Π΅ΠΊΡƒΡ‰Π΅ΠΉ настройки ( CC Set ) Π² любой ΠΌΠΎΠΌΠ΅Π½Ρ‚ Π²ΠΎ врСмя экспСримСнта.

2

ΠœΡƒΠ»ΡŒΡ‚ΠΈΠΌΠ΅Ρ‚Ρ€ — это ΠΈΠ·ΠΌΠ΅Ρ€ΠΈΡ‚Π΅Π»ΡŒΠ½ΠΎΠ΅ устройство, ΠΊΠΎΡ‚ΠΎΡ€ΠΎΠ΅ ΠΈΡΠΏΠΎΠ»ΡŒΠ·ΡƒΠ΅Ρ‚ΡΡ для измСрСния разности напряТСний, элСктричСских Ρ‚ΠΎΠΊΠΎΠ² ΠΈ элСктричСских сопротивлСний. Он Ρ‚Π°ΠΊΠΆΠ΅ ΠΌΠΎΠΆΠ΅Ρ‚ ΠΈΠ·ΠΌΠ΅Ρ€ΡΡ‚ΡŒ Π΄Ρ€ΡƒΠ³ΠΈΠ΅ элСктричСскиС свойства. Π‘ΠΌ. Рис. 1. Π’ Π²Π΅Ρ€Ρ…Π½Π΅ΠΉ части измСритСля находится Π–Πš-дисплСй (ТидкокристалличСский дисплСй), Π² сСрСдинС — ΠΏΠ΅Ρ€Π΅ΠΊΠ»ΡŽΡ‡Π°Ρ‚Π΅Π»ΡŒ Ρ„ΡƒΠ½ΠΊΡ†ΠΈΠΉ / Π΄ΠΈΠ°ΠΏΠ°Π·ΠΎΠ½Π° (диск), Π° Π²Π½ΠΈΠ·Ρƒ — Ρ‡Π΅Ρ‚Ρ‹Ρ€Π΅ Π²Ρ…ΠΎΠ΄Π½Ρ‹Ρ… Ρ€Π°Π·ΡŠΠ΅ΠΌΠ°.

ΠŸΡ€ΠΈΠΌΠ΅Ρ‡Π°Π½ΠΈΠ΅. Π˜Π·ΠΌΠ΅Ρ€ΠΈΡ‚Π΅Π»ΡŒ особСнно чувствитСлСн (ΠΈ склонСн ΠΊ ΠΏΠ΅Ρ€Π΅Π³ΠΎΡ€Π°Π½ΠΈΡŽ прСдохранитСля) ΠΏΡ€ΠΈ использовании Π²Ρ…ΠΎΠ΄Π½ΠΎΠ³ΠΎ Ρ€Π°Π·ΡŠΠ΅ΠΌΠ° 200 мА (см. I Π² ΠΎΠ±ΠΎΠ·Π½Π°Ρ‡Π΅Π½ΠΈΠΈ Π½Π° РисункС 1).

ΠšΠ»ΡŽΡ‡ для рисунка 1:
  • А

    3-1 / 2-разрядный Π–Πš-дисплСй с сигнализаторами.
  • Π’

    Кнопка Π’ΠšΠ› / Π’Π«ΠšΠ›: Π²ΠΊΠ»ΡŽΡ‡Π°Π΅Ρ‚ ΠΈ Π²Ρ‹ΠΊΠ»ΡŽΡ‡Π°Π΅Ρ‚ ΠΏΠΈΡ‚Π°Π½ΠΈΠ΅ измСритСля.
  • Π‘

    Кнопка HI / LO: Π²Ρ‹Π±ΠΈΡ€Π°Π΅Ρ‚ высокий ΠΈΠ»ΠΈ Π½ΠΈΠ·ΠΊΠΈΠΉ ΡƒΡ€ΠΎΠ²Π΅Π½ΡŒ запуска для измСрСния частоты.
  • D

    Кнопка MAX: Π²Ρ‹Π±ΠΈΡ€Π°Π΅Ρ‚ Ρ„ΡƒΠ½ΠΊΡ†ΠΈΡŽ удСрТания максимального показания.
  • E

    Кнопка DC / AC: Π²Ρ‹Π±ΠΈΡ€Π°Π΅Ρ‚ напряТСниС постоянного ΠΈΠ»ΠΈ ΠΏΠ΅Ρ€Π΅ΠΌΠ΅Π½Π½ΠΎΠ³ΠΎ Ρ‚ΠΎΠΊΠ°.
  • Π€

    ΠŸΠ΅Ρ€Π΅ΠΊΠ»ΡŽΡ‡Π°Ρ‚Π΅Π»ΡŒ Ρ„ΡƒΠ½ΠΊΡ†ΠΈΠΈ / Π΄ΠΈΠ°ΠΏΠ°Π·ΠΎΠ½Π°: Π²Ρ‹Π±ΠΈΡ€Π°Π΅Ρ‚ ΠΆΠ΅Π»Π°Π΅ΠΌΡƒΡŽ Ρ„ΡƒΠ½ΠΊΡ†ΠΈΡŽ ΠΈ Π΄ΠΈΠ°ΠΏΠ°Π·ΠΎΠ½.
  • G

    Π’Ρ…ΠΎΠ΄Π½ΠΎΠΉ Ρ€Π°Π·ΡŠΠ΅ΠΌ V Ξ©: Π²Ρ…ΠΎΠ΄Π½ΠΎΠΉ Ρ€Π°Π·ΡŠΠ΅ΠΌ для напряТСния, сопротивлСния, ΠΏΡ€ΠΎΠ²Π΅Ρ€ΠΊΠΈ Π΄ΠΈΠΎΠ΄ΠΎΠ², цСлостности Ρ†Π΅ΠΏΠΈ, частоты ΠΈ Π»ΠΎΠ³ΠΈΠΊΠΈ.
  • H

    Π’Ρ…ΠΎΠ΄Π½ΠΎΠΉ Ρ€Π°Π·ΡŠΠ΅ΠΌ COM: Π²Ρ…ΠΎΠ΄Π½ΠΎΠΉ Ρ€Π°Π·ΡŠΠ΅ΠΌ зазСмлСния.
  • Π―

    Π’Ρ…ΠΎΠ΄Π½ΠΎΠΉ Ρ€Π°Π·ΡŠΠ΅ΠΌ 200 мА: Π²Ρ…ΠΎΠ΄Π½ΠΎΠΉ Ρ€Π°Π·ΡŠΠ΅ΠΌ для Ρ‚ΠΎΠΊΠ° Π΄ΠΎ 200 мА, L x (ΠΈΠ½Π΄ΡƒΠΊΡ‚ΠΈΠ²Π½ΠΎΡΡ‚ΡŒ), C x (Π΅ΠΌΠΊΠΎΡΡ‚ΡŒ).
  • Π”ΠΆ

    Π’Ρ…ΠΎΠ΄Π½ΠΎΠΉ Ρ€Π°Π·ΡŠΠ΅ΠΌ Π½Π° 10 А: Π²Ρ…ΠΎΠ΄Π½ΠΎΠΉ Ρ€Π°Π·ΡŠΠ΅ΠΌ для Ρ‚ΠΎΠΊΠ° Π΄ΠΎ 10 А.
Для измСрСния Π·Π°Π΄Π°Π½Π½ΠΎΠ³ΠΎ количСства Ρ†ΠΈΡ„Π΅Ρ€Π±Π»Π°Ρ‚ Π΄ΠΎΠ»ΠΆΠ΅Π½ Π½Π°Ρ…ΠΎΠ΄ΠΈΡ‚ΡŒΡΡ Π² ΡΠΎΠΎΡ‚Π²Π΅Ρ‚ΡΡ‚Π²ΡƒΡŽΡ‰Π΅ΠΌ ΠΏΠΎΠ»ΠΎΠΆΠ΅Π½ΠΈΠΈ ΠΈ Π΄ΠΎΠ»ΠΆΠ½Ρ‹ ΠΈΡΠΏΠΎΠ»ΡŒΠ·ΠΎΠ²Π°Ρ‚ΡŒΡΡ ΡΠΎΠΎΡ‚Π²Π΅Ρ‚ΡΡ‚Π²ΡƒΡŽΡ‰ΠΈΠ΅ Π΄Π²Π° Π²Ρ…ΠΎΠ΄Π½Ρ‹Ρ… Ρ€Π°Π·ΡŠΠ΅ΠΌΠ°.Π’Π°ΠΊΠΈΠΌ ΠΎΠ±Ρ€Π°Π·ΠΎΠΌ, ΠΏΡ€ΠΈ ΠΏΠΎΠ²ΠΎΡ€ΠΎΡ‚Π΅ ΡˆΠΊΠ°Π»Ρ‹ для ΠΏΠ΅Ρ€Π΅Ρ…ΠΎΠ΄Π° ΠΎΡ‚ ΠΎΠ΄Π½ΠΎΠ³ΠΎ Ρ‚ΠΈΠΏΠ° измСрСния ΠΊ Π΄Ρ€ΡƒΠ³ΠΎΠΌΡƒ (Π½Π°ΠΏΡ€ΠΈΠΌΠ΅Ρ€, ΠΎΡ‚ разности напряТСний ΠΊ элСктричСскому Ρ‚ΠΎΠΊΡƒ) Π²Π°ΠΌ, Π²ΠΎΠ·ΠΌΠΎΠΆΠ½ΠΎ, Ρ‚Π°ΠΊΠΆΠ΅ придСтся ΠΈΠ·ΠΌΠ΅Π½ΠΈΡ‚ΡŒ Π²Ρ…ΠΎΠ΄Π½Ρ‹Π΅ Ρ€Π°Π·ΡŠΠ΅ΠΌΡ‹. ΠŸΡ€ΠΈ ΠΏΠ΅Ρ€Π΅Π³Ρ€ΡƒΠ·ΠΊΠ΅ ΠΏΡ€Π΅Π΄ΠΎΡ…Ρ€Π°Π½ΠΈΡ‚Π΅Π»ΡŒ ΠΌΠΎΠΆΠ΅Ρ‚ ΠΏΠ΅Ρ€Π΅Π³ΠΎΡ€Π΅Ρ‚ΡŒ.

ΠžΡΡ‚ΠΎΡ€ΠΎΠΆΠ½ΠΎ:
Для Π·Π°Ρ‰ΠΈΡ‚Ρ‹ счСтчик Π³ΡƒΠ΄ΠΈΡ‚ ΠΏΡ€ΠΈ ΠΏΠ΅Ρ€Π΅Π³Ρ€ΡƒΠ·ΠΊΠ΅; Ссли ΠΎΠ½ Π³ΡƒΠ΄ΠΈΡ‚, Π½Π΅ΠΌΠ΅Π΄Π»Π΅Π½Π½ΠΎ отсоСдинитС ΠΏΡ€ΠΎΠ²ΠΎΠ΄Π° счСтчика!

Π§Ρ‚ΠΎΠ±Ρ‹ Π·Π°Ρ‰ΠΈΡ‚ΠΈΡ‚ΡŒ ΠΈΠ·ΠΌΠ΅Ρ€ΠΈΡ‚Π΅Π»ΡŒ ΠΎΡ‚ ΠΏΠ΅Ρ€Π΅Π³Ρ€ΡƒΠ·ΠΊΠΈ ΠΏΡ€ΠΈ ΠΈΠ·ΠΌΠ΅Ρ€Π΅Π½ΠΈΠΈ нСизвСстного напряТСния ΠΈΠ»ΠΈ Ρ‚ΠΎΠΊΠ°, сначала Π½Π΅ΠΎΠ±Ρ…ΠΎΠ΄ΠΈΠΌΠΎ ΡƒΡΡ‚Π°Π½ΠΎΠ²ΠΈΡ‚ΡŒ ΠΈΠ·ΠΌΠ΅Ρ€ΠΈΡ‚Π΅Π»ΡŒ Π½Π° ΠΌΠ°ΠΊΡΠΈΠΌΠ°Π»ΡŒΠ½ΡƒΡŽ ΡˆΠΊΠ°Π»Ρƒ для этой Ρ„ΡƒΠ½ΠΊΡ†ΠΈΠΈ.Если показания нСдостаточно Π²Π΅Π»ΠΈΠΊΠΈ, Ρ‡Ρ‚ΠΎΠ±Ρ‹ Π΄Π°Ρ‚ΡŒ ΠΏΠΎ ΠΊΡ€Π°ΠΉΠ½Π΅ΠΉ ΠΌΠ΅Ρ€Π΅ Ρ‚Ρ€ΠΈ Π·Π½Π°Ρ‡Π°Ρ‰ΠΈΡ… Ρ†ΠΈΡ„Ρ€Ρ‹, ΡˆΠΊΠ°Π»Ρƒ слСдуСт ΠΏΠ΅Ρ€Π΅ΠΊΠ»ΡŽΡ‡ΠΈΡ‚ΡŒ (Ссли Π²ΠΎΠ·ΠΌΠΎΠΆΠ½ΠΎ) Π½Π° Ρ‚Π°ΠΊΡƒΡŽ, которая позволяСт ΠΏΡ€ΠΎΠ²ΠΎΠ΄ΠΈΡ‚ΡŒ Ρ‚ΠΎΡ‡Π½ΠΎΠ΅ ΠΈΠ·ΠΌΠ΅Ρ€Π΅Π½ΠΈΠ΅.

3

Π§Ρ‚ΠΎΠ±Ρ‹ Π²ΠΊΠ»ΡŽΡ‡ΠΈΡ‚ΡŒ ΠΌΡƒΠ»ΡŒΡ‚ΠΈΠΌΠ΅Ρ‚Ρ€, Π½Π°ΠΆΠΈΠΌΠ°ΠΉΡ‚Π΅ Π²Π΅Ρ€Ρ…Π½ΡŽΡŽ Π»Π΅Π²ΡƒΡŽ ΠΊΠ½ΠΎΠΏΠΊΡƒ Π½Π° ΠΈΠ·ΠΌΠ΅Ρ€ΠΈΡ‚Π΅Π»Π΅, ΠΏΠΎΠΊΠ° Π½Π° Π΅Π³ΠΎ Ρ†ΠΈΡ„Π΅Ρ€Π±Π»Π°Ρ‚Π΅ Π½Π΅ появится дисплСй. Π§Ρ‚ΠΎΠ±Ρ‹ Π½Π°ΡΡ‚Ρ€ΠΎΠΈΡ‚ΡŒ ΠΌΡƒΠ»ΡŒΡ‚ΠΈΠΌΠ΅Ρ‚Ρ€ Π½Π° ΠΈΠ·ΠΌΠ΅Ρ€Π΅Π½ΠΈΠ΅ постоянного напряТСния, Π’, , ΠΏΠ΅Ρ€Π΅ΠΊΠ»ΡŽΡ‡ΠΈΡ‚Π΅ Π²Π΅Ρ€Ρ…Π½ΡŽΡŽ ΠΏΡ€Π°Π²ΡƒΡŽ ΠΊΠ½ΠΎΠΏΠΊΡƒ Π½Π° постоянный Ρ‚ΠΎΠΊ. Π£Π±Π΅Π΄ΠΈΡ‚Π΅ΡΡŒ, Ρ‡Ρ‚ΠΎ Π½Π° дисплСС Π³Π»ΡŽΠΊΠΎΠΌΠ΅Ρ‚Ρ€Π° отобраТаСтся постоянный Ρ‚ΠΎΠΊ. ΠŸΠΎΠ²Π΅Ρ€Π½ΠΈΡ‚Π΅ ΠΏΠ΅Ρ€Π΅ΠΊΠ»ΡŽΡ‡Π°Ρ‚Π΅Π»ΡŒ Ρ„ΡƒΠ½ΠΊΡ†ΠΈΠΉ / Π΄ΠΈΠ°ΠΏΠ°Π·ΠΎΠ½ΠΎΠ² Π² ΠΏΠΎΠ»ΠΎΠΆΠ΅Π½ΠΈΠ΅ напряТСния (Π’) ΠΈ установитС Π·Π½Π°Ρ‡Π΅Π½ΠΈΠ΅ 20.Π’Π΅ΠΏΠ΅Ρ€ΡŒ ΠΈΠ·ΠΌΠ΅Ρ€ΠΈΡ‚Π΅Π»ΡŒ настроСн Π½Π° считываниС напряТСний Π΄ΠΎ 20 Π²ΠΎΠ»ΡŒΡ‚ постоянного Ρ‚ΠΎΠΊΠ°. ΠŸΠΎΠ΄ΠΊΠ»ΡŽΡ‡ΠΈΡ‚Π΅ Π±Π°Π½Π°Π½ ΠΊ Π±Π°Π½Π°Π½ΠΎΠ²Ρ‹ΠΌ Π²Ρ‹Π²ΠΎΠ΄Π°ΠΌ ΠΊ ΠΎΠ±Ρ‰Π΅ΠΌΡƒ Ρ€Π°Π·ΡŠΠ΅ΠΌΡƒ (COM) ΠΈ ΠΊ Ρ€Π°Π·ΡŠΠ΅ΠΌΡƒ напряТСния (V).

4

ΠŸΠΎΠ΄ΠΊΠ»ΡŽΡ‡ΠΈΡ‚Π΅ Π²Ρ‹Π²ΠΎΠ΄Ρ‹ ΠΌΡƒΠ»ΡŒΡ‚ΠΈΠΌΠ΅Ρ‚Ρ€Π° ΠΊ ΠΊΠ»Π΅ΠΌΠΌΠ°ΠΌ + ΠΈ — источника питания. Π‘ΠΌ. Рисунок 2. На Π±Π»ΠΎΠΊΠ΅ питания ΠΏΠΎΠ²Π΅Ρ€Π½ΠΈΡ‚Π΅ Ρ€ΡƒΡ‡ΠΊΡƒ Ρ€Π΅Π³ΡƒΠ»ΠΈΡ€ΠΎΠ²ΠΊΠΈ напряТСния ADJUST ΠΏΠΎ часовой стрСлкС, ΠΏΠΎΠΊΠ° Π½Π° дисплСС Π²ΠΎΠ»ΡŒΡ‚ Π½Π΅ отобразится 5,0 Π²ΠΎΠ»ΡŒΡ‚. Π‘Ρ€Π°Π²Π½ΠΈΡ‚Π΅ показания напряТСния Π½Π° ΠΌΡƒΠ»ΡŒΡ‚ΠΈΠΌΠ΅Ρ‚Ρ€Π΅ ΠΈ Π½Π° ΠΈΠ·ΠΌΠ΅Ρ€ΠΈΡ‚Π΅Π»Π΅ Π±Π»ΠΎΠΊΠ° питания. Π­Ρ‚ΠΈ Π΄Π²Π° показания ΠΌΠΎΠ³ΡƒΡ‚ Π½Π΅ ΡΠΎΠ²ΠΏΠ°Π΄Π°Ρ‚ΡŒ.ΠžΠΆΠΈΠ΄Π°Π΅Ρ‚ΡΡ, Ρ‡Ρ‚ΠΎ ΠΌΡƒΠ»ΡŒΡ‚ΠΈΠΌΠ΅Ρ‚Ρ€ Π±ΡƒΠ΄Π΅Ρ‚ Ρ‚ΠΎΡ‡Π½Π΅Π΅.

Π’ΠΎΠΊ ΠΈ напряТСниС для ΠΎΠ΄ΠΈΠ½ΠΎΡ‡Π½ΠΎΠ³ΠΎ рСзистора

1

Π£ΠΌΠ΅Π½ΡŒΡˆΠΈΡ‚Π΅ напряТСниС Π±Π»ΠΎΠΊΠ° питания (ΠΏΡ€ΠΎΡ‚ΠΈΠ² часовой стрСлки) Π΄ΠΎ нуля Π²ΠΎΠ»ΡŒΡ‚. ΠŸΠΎΠ΄ΠΊΠ»ΡŽΡ‡ΠΈΡ‚Π΅ Π±Π»ΠΎΠΊ питания ΠΊ рСзистору Π½Π° ΠΏΠ΅Ρ‡Π°Ρ‚Π½ΠΎΠΉ ΠΏΠ»Π°Ρ‚Π΅ с ΠΌΠ°Ρ€ΠΊΠΈΡ€ΠΎΠ²ΠΊΠΎΠΉ 700 Ом. (НС пСрСнастраивайтС ΠΈ Π½Π΅ измСняйтС настройку Ρ‚ΠΎΠΊΠ° Π½Π° источникС питания.) ΠœΡ‹ Π±ΡƒΠ΄Π΅ΠΌ ΠΈΡΠΏΠΎΠ»ΡŒΠ·ΠΎΠ²Π°Ρ‚ΡŒ ΠΌΡƒΠ»ΡŒΡ‚ΠΈΠΌΠ΅Ρ‚Ρ€ для измСрСния постоянного Ρ‚ΠΎΠΊΠ° Ρ‡Π΅Ρ€Π΅Π· рСзистор 700 Ом Π² зависимости ΠΎΡ‚ ΠΏΡ€ΠΈΠ»ΠΎΠΆΠ΅Π½Π½ΠΎΠ³ΠΎ напряТСния. Для этого ΠΌΡ‹ Π΄ΠΎΠ»ΠΆΠ½Ρ‹ ΡΠΎΠ΅Π΄ΠΈΠ½ΠΈΡ‚ΡŒ ΠΌΡƒΠ»ΡŒΡ‚ΠΈΠΌΠ΅Ρ‚Ρ€ сСрии с рСзистором, Ρ‡Ρ‚ΠΎΠ±Ρ‹ ΠΎΠ΄ΠΈΠ½Π°ΠΊΠΎΠ²Ρ‹ΠΉ Ρ‚ΠΎΠΊ ΠΏΡ€ΠΎΡ…ΠΎΠ΄ΠΈΠ» Ρ‡Π΅Ρ€Π΅Π· ΠΎΠ±Π°. Π’Π°ΠΊ ΠΊΠ°ΠΊ ΠΏΡ€Π΅Π΄ΠΎΡ…Ρ€Π°Π½ΠΈΡ‚Π΅Π»ΡŒ Π»Π΅Π³ΠΊΠΎ ΠΏΠ΅Ρ€Π΅Π³ΠΎΡ€ΠΈΡ‚, ΠΊΠΎΠ³Π΄Π° ΠΌΡƒΠ»ΡŒΡ‚ΠΈΠΌΠ΅Ρ‚Ρ€ настроСн Π½Π° Ρ‚Π΅ΠΊΡƒΡ‰ΡƒΡŽ настройку, Π²Π½ΠΈΠΌΠ°Ρ‚Π΅Π»ΡŒΠ½ΠΎ слСдуйтС инструкциям. УстановитС ΡˆΠΊΠ°Π»Ρƒ ΠΌΡƒΠ»ΡŒΡ‚ΠΈΠΌΠ΅Ρ‚Ρ€Π° Π½Π° ΡˆΠΊΠ°Π»Ρƒ Ρ‚ΠΎΠΊΠ° 20 мА ΠΈ ΠΏΠΎΠ΄ΠΊΠ»ΡŽΡ‡ΠΈΡ‚Π΅ Π±Π°Π½Π°Π½ΠΎΠ²Ρ‹Π΅ ΡˆΡ‚Π΅ΠΊΠ΅Ρ€Ρ‹ ΠΊ Π³Π½Π΅Π·Π΄Π°ΠΌ COM ΠΈ мА Π½Π° ΠΈΠ·ΠΌΠ΅Ρ€ΠΈΡ‚Π΅Π»Π΅.

ΠžΡΡ‚ΠΎΡ€ΠΎΠΆΠ½ΠΎ:
НС ΠΏΠΎΠ²Ρ‹ΡˆΠ°ΠΉΡ‚Π΅ напряТСниС Π½Π° Π±Π»ΠΎΠΊΠ΅ питания, ΠΏΠΎΠΊΠ° ВА Π½Π΅ ΠΏΡ€ΠΎΠ²Π΅Ρ€ΠΈΡ‚ Π²Π°ΡˆΡƒ Ρ†Π΅ΠΏΡŒ.

2

ПослС Ρ‚ΠΎΠ³ΠΎ, ΠΊΠ°ΠΊ ваш ВА даст Π΄ΠΎΠ±Ρ€ΠΎ, установитС источник питания Π½Π° 1 Π’ ΠΈ Π·Π°ΠΏΠΈΡˆΠΈΡ‚Π΅ Π² Π’Π°Π±Π»ΠΈΡ†Ρƒ 1 Ρ‚ΠΎΠΊ Ρ‡Π΅Ρ€Π΅Π· рСзистор, ΠΊΠ°ΠΊ ΠΏΠΎΠΊΠ°Π·Π°Π½ΠΎ Π½Π° ΠΌΡƒΠ»ΡŒΡ‚ΠΈΠΌΠ΅Ρ‚Ρ€Π΅.ΠŸΠΎΠ²Ρ‚ΠΎΡ€ΠΈΡ‚Π΅ Ρ‚ΠΎ ΠΆΠ΅ самоС с источником питания, установлСнным Π½Π° 2, 3, 4 ΠΈ 5 Π²ΠΎΠ»ΡŒΡ‚.

3

Π˜ΡΠΏΠΎΠ»ΡŒΠ·ΡƒΠΉΡ‚Π΅ Excel для построСния Π³Ρ€Π°Ρ„ΠΈΠΊΠ° Π΄Π°Π½Π½Ρ‹Ρ… с Ρ‚ΠΎΠΊΠΎΠΌ ΠΏΠΎ Π²Π΅Ρ€Ρ‚ΠΈΠΊΠ°Π»ΡŒΠ½ΠΎΠΉ оси ΠΈ напряТСниСм ΠΏΠΎ Π³ΠΎΡ€ΠΈΠ·ΠΎΠ½Ρ‚Π°Π»ΡŒΠ½ΠΎΠΉ оси. Π˜Π½ΡΡ‚Ρ€ΡƒΠΊΡ†ΠΈΠΈ ΠΏΠΎ ΠΏΠΎΡΡ‚Ρ€ΠΎΠ΅Π½ΠΈΡŽ Π³Ρ€Π°Ρ„ΠΈΠΊΠΎΠ² Π² Excel см. Π’ ΠΏΡ€ΠΈΠ»ΠΎΠΆΠ΅Π½ΠΈΠΈ ΠΊ ΠΈΠ½Ρ‚Π΅Ρ€Π°ΠΊΡ‚ΠΈΠ²Π½ΠΎΠΌΡƒ руководству Π»Π°Π±ΠΎΡ€Π°Ρ‚ΠΎΡ€ΠΈΠΈ. Если Π²Ρ‹ ΠΏΠΎΠ»ΡƒΡ‡ΠΈΡ‚Π΅ ΠΎΠΆΠΈΠ΄Π°Π΅ΠΌΡ‹Π΅ Ρ€Π΅Π·ΡƒΠ»ΡŒΡ‚Π°Ρ‚Ρ‹, Π΄Π°Π½Π½Ρ‹Π΅ Π±ΡƒΠ΄ΡƒΡ‚ Ρ€Π°ΡΠΏΠΎΠ»Π°Π³Π°Ρ‚ΡŒΡΡ Π±Π»ΠΈΠ·ΠΊΠΎ ΠΊ прямой Π»ΠΈΠ½ΠΈΠΈ, проходящСй Ρ‡Π΅Ρ€Π΅Π· Π½Π°Ρ‡Π°Π»ΠΎ ΠΊΠΎΠΎΡ€Π΄ΠΈΠ½Π°Ρ‚. Π˜ΡΠΏΠΎΠ»ΡŒΠ·ΡƒΠΉΡ‚Π΅ Excel, Ρ‡Ρ‚ΠΎΠ±Ρ‹ Π½Π°ΠΉΡ‚ΠΈ Π½Π°ΠΊΠ»ΠΎΠ½ прямой Π»ΠΈΠ½ΠΈΠΈ, ΠΊΠΎΡ‚ΠΎΡ€Ρ‹ΠΉ Π»ΡƒΡ‡ΡˆΠ΅ всСго соотвСтствуСт вашим Π΄Π°Π½Π½Ρ‹ΠΌ, ΠΈ Π·Π°ΠΏΠΈΡˆΠΈΡ‚Π΅ Ρ€Π΅Π·ΡƒΠ»ΡŒΡ‚Π°Ρ‚, Π²ΠΊΠ»ΡŽΡ‡Π°Ρ Π΅Π΄ΠΈΠ½ΠΈΡ†Ρ‹ измСрСния.

4

Π˜ΡΠΏΠΎΠ»ΡŒΠ·ΡƒΠΉΡ‚Π΅ Π·Π°ΠΊΠΎΠ½ Ома ΠΈ Π½Π°ΠΊΠ»ΠΎΠ½ Π³Ρ€Π°Ρ„ΠΈΠΊΠ°, Ρ‡Ρ‚ΠΎΠ±Ρ‹ Ρ€Π°ΡΡΡ‡ΠΈΡ‚Π°Ρ‚ΡŒ сопротивлСниС рСзистора R Π² Π΅Π΄ΠΈΠ½ΠΈΡ†Π°Ρ… Ом (Ом). Π—Π°ΠΏΠΈΡˆΠΈΡ‚Π΅ свой Ρ€Π΅Π·ΡƒΠ»ΡŒΡ‚Π°Ρ‚.

Π’ΠΎΠΊ ΠΈ напряТСниС для Π»Π°ΠΌΠΏΠΎΡ‡ΠΊΠΈ

1

Π£Π‘Π’ΠΠΠžΠ’Π˜Π’Π• ΠΠΠŸΠ Π―Π–Π•ΠΠ˜Π• ПИВАНИЯ НА НУЛЬ, Π½ΠΎ Π½Π΅ Π²Ρ‹ΠΊΠ»ΡŽΡ‡Π°ΠΉΡ‚Π΅ ΠΏΠΈΡ‚Π°Π½ΠΈΠ΅. НС измСняйтС настройку Ρ‚ΠΎΠΊΠ° Π½Π° источникС питания ( CC Set ). ΠœΡ‹ Π±ΡƒΠ΄Π΅ΠΌ ΠΈΡΠΏΠΎΠ»ΡŒΠ·ΠΎΠ²Π°Ρ‚ΡŒ ΠΌΡƒΠ»ΡŒΡ‚ΠΈΠΌΠ΅Ρ‚Ρ€ для измСрСния постоянного Ρ‚ΠΎΠΊΠ° Ρ‡Π΅Ρ€Π΅Π· Π»Π°ΠΌΠΏΠΎΡ‡ΠΊΡƒ Π² зависимости ΠΎΡ‚ ΠΏΡ€ΠΈΠ»ΠΎΠΆΠ΅Π½Π½ΠΎΠ³ΠΎ напряТСния.Для этого ΠΌΡ‹ Π΄ΠΎΠ»ΠΆΠ½Ρ‹ ΡΠΎΠ΅Π΄ΠΈΠ½ΠΈΡ‚ΡŒ ΠΌΡƒΠ»ΡŒΡ‚ΠΈΠΌΠ΅Ρ‚Ρ€ сСрии с Π»Π°ΠΌΠΏΠΎΡ‡ΠΊΠΎΠΉ, Ρ‡Ρ‚ΠΎΠ±Ρ‹ Ρ‡Π΅Ρ€Π΅Π· ΠΎΠ±Π° ΠΏΡ€ΠΎΡ…ΠΎΠ΄ΠΈΠ» ΠΎΠ΄ΠΈΠ½Π°ΠΊΠΎΠ²Ρ‹ΠΉ Ρ‚ΠΎΠΊ. Π’Π°ΠΊ ΠΊΠ°ΠΊ ΠΏΡ€Π΅Π΄ΠΎΡ…Ρ€Π°Π½ΠΈΡ‚Π΅Π»ΡŒ Π»Π΅Π³ΠΊΠΎ ΠΏΠ΅Ρ€Π΅Π³ΠΎΡ€ΠΈΡ‚, ΠΊΠΎΠ³Π΄Π° ΠΌΡƒΠ»ΡŒΡ‚ΠΈΠΌΠ΅Ρ‚Ρ€ настроСн Π½Π° Ρ‚Π΅ΠΊΡƒΡ‰ΡƒΡŽ настройку, Π²Π½ΠΈΠΌΠ°Ρ‚Π΅Π»ΡŒΠ½ΠΎ слСдуйтС инструкциям.

2

УстановитС ΡˆΠΊΠ°Π»Ρƒ ΠΌΡƒΠ»ΡŒΡ‚ΠΈΠΌΠ΅Ρ‚Ρ€Π° Π½Π° ΡˆΠΊΠ°Π»Ρƒ постоянного Ρ‚ΠΎΠΊΠ° 10 А. Π˜ΡΠΏΠΎΠ»ΡŒΠ·ΡƒΠΉΡ‚Π΅ Π²Ρ…ΠΎΠ΄Π½Ρ‹Π΅ Π³Π½Π΅Π·Π΄Π° COM ΠΈ 10 A. ΠŸΠΎΠ΄ΠΊΠ»ΡŽΡ‡ΠΈΡ‚Π΅ схСму, ΠΊΠ°ΠΊ ΠΏΠΎΠΊΠ°Π·Π°Π½ΠΎ Π½Π° рисункС 3.

ΠžΡΡ‚ΠΎΡ€ΠΎΠΆΠ½ΠΎ:
НС ΠΏΠΎΠ²Ρ‹ΡˆΠ°ΠΉΡ‚Π΅ напряТСниС Π½Π° Π±Π»ΠΎΠΊΠ΅ питания, ΠΏΠΎΠΊΠ° ВА Π½Π΅ ΠΏΡ€ΠΎΠ²Π΅Ρ€ΠΈΡ‚ Π²Π°ΡˆΡƒ Ρ†Π΅ΠΏΡŒ.

3

ПослС Ρ‚ΠΎΠ³ΠΎ, ΠΊΠ°ΠΊ ваш ВА даст Π΄ΠΎΠ±Ρ€ΠΎ, установитС напряТСниС источника питания Π½Π° 2 Π²ΠΎΠ»ΡŒΡ‚Π°. Π’ Ρ‚Π°Π±Π»ΠΈΡ†Π΅ 2 Π·Π°ΠΏΠΈΡˆΠΈΡ‚Π΅ Ρ‚Π΅ΠΊΡƒΡ‰Π΅Π΅ ΠΏΠΎΠΊΠ°Π·Π°Π½ΠΈΠ΅ ΠΌΡƒΠ»ΡŒΡ‚ΠΈΠΌΠ΅Ρ‚Ρ€Π°. ΠŸΠΎΠ²Ρ‚ΠΎΡ€ΠΈΡ‚Π΅ эти дСйствия для напряТСний источника питания 4, 6, 8, 10 ΠΈ 12 Π²ΠΎΠ»ΡŒΡ‚.

4

Π˜ΡΠΏΠΎΠ»ΡŒΠ·ΡƒΠΉΡ‚Π΅ Excel для построСния Π³Ρ€Π°Ρ„ΠΈΠΊΠ° Π΄Π°Π½Π½Ρ‹Ρ… с Ρ‚ΠΎΠΊΠΎΠΌ ΠΏΠΎ Π²Π΅Ρ€Ρ‚ΠΈΠΊΠ°Π»ΡŒΠ½ΠΎΠΉ оси ΠΈ напряТСниСм ΠΏΠΎ Π³ΠΎΡ€ΠΈΠ·ΠΎΠ½Ρ‚Π°Π»ΡŒΠ½ΠΎΠΉ оси. ΠžΠΆΠΈΠ΄Π°Π΅Ρ‚ΡΡ, Ρ‡Ρ‚ΠΎ ваши Π΄Π°Π½Π½Ρ‹Π΅ Π½Π΅ Π±ΡƒΠ΄ΡƒΡ‚ ΠΏΡ€ΠΈΠ±Π»ΠΈΠΆΠ°Ρ‚ΡŒΡΡ ΠΊ прямой. ΠžΠΏΡ€Π΅Π΄Π΅Π»ΠΈΡ‚Π΅ ΠΈ вычислитС R для ΠΊΠ°ΠΆΠ΄ΠΎΠ³ΠΎ Π½Π°Π±ΠΎΡ€Π° Π·Π½Π°Ρ‡Π΅Π½ΠΈΠΉ V ΠΈ I Π² Ρ‚Π°Π±Π»ΠΈΡ†Π΅ 2 ΠΈ Π·Π°ΠΏΠΈΡˆΠΈΡ‚Π΅ Π² Ρ‚Ρ€Π΅Ρ‚ΠΈΠΉ столбСц Ρ‚Π°Π±Π»ΠΈΡ†Ρ‹ 2.УвСличиваСтся Π»ΠΈ R , ΡƒΠΌΠ΅Π½ΡŒΡˆΠ°Π΅Ρ‚ΡΡ ΠΈΠ»ΠΈ остаСтся Ρ‚Π°ΠΊΠΈΠΌ ΠΆΠ΅, ΠΊΠ°ΠΊ Ρ‚ΠΎΠΊ I Ρ‡Π΅Ρ€Π΅Π· Π»Π°ΠΌΠΏΠΎΡ‡ΠΊΡƒ увСличиваСтся?

Π”Π²Π° ΠΏΠΎΡΠ»Π΅Π΄ΠΎΠ²Π°Ρ‚Π΅Π»ΡŒΠ½ΠΎ ΠΏΠΎΠ΄ΠΊΠ»ΡŽΡ‡Π΅Π½Π½Ρ‹Ρ… рСзистора

1

ΠŸΠΎΠ΄ΠΊΠ»ΡŽΡ‡ΠΈΡ‚Π΅ Π½Π° ΠΏΠ΅Ρ‡Π°Ρ‚Π½ΠΎΠΉ ΠΏΠ»Π°Ρ‚Π΅ Π΄Π²Π° рСзистора с ΠΌΠ°Ρ€ΠΊΠΈΡ€ΠΎΠ²ΠΊΠΎΠΉ 150 Ом ΠΈ 700 Ом, ΠΊΠ°ΠΊ ΠΏΠΎΠΊΠ°Π·Π°Π½ΠΎ Π½Π° рисункС 4. Говорят, Ρ‡Ρ‚ΠΎ ΠΎΠ½ΠΈ ΠΏΠΎΠ΄ΠΊΠ»ΡŽΡ‡Π΅Π½Ρ‹ ΠΏΠΎΡΠ»Π΅Π΄ΠΎΠ²Π°Ρ‚Π΅Π»ΡŒΠ½ΠΎ, ΠΏΠΎΡ‚ΠΎΠΌΡƒ Ρ‡Ρ‚ΠΎ вСсь Ρ‚ΠΎΠΊ, проходящий Ρ‡Π΅Ρ€Π΅Π· ΠΎΠ΄ΠΈΠ½, Ρ‚Π°ΠΊΠΆΠ΅ ΠΏΡ€ΠΎΡ…ΠΎΠ΄ΠΈΡ‚ Ρ‡Π΅Ρ€Π΅Π· Π΄Ρ€ΡƒΠ³ΠΎΠΉ. Π£Π±Π΅Π΄ΠΈΡ‚Π΅ΡΡŒ, Ρ‡Ρ‚ΠΎ напряТСниС источника питания установлСно Π½Π° ноль. ΠŸΠΎΠ΄ΠΊΠ»ΡŽΡ‡ΠΈΡ‚Π΅ источник питания ΠΊ ΠΊΠΎΠΌΠ±ΠΈΠ½Π°Ρ†ΠΈΠΈ ΠΏΠΎΡΠ»Π΅Π΄ΠΎΠ²Π°Ρ‚Π΅Π»ΡŒΠ½Ρ‹Ρ… рСзисторов, ΠΊΠ°ΠΊ ΠΏΠΎΠΊΠ°Π·Π°Π½ΠΎ Π½Π° рисункС 4.УстановитС Π±Π»ΠΎΠΊ питания Π½Π° 5 Π²ΠΎΠ»ΡŒΡ‚. УстановитС ΡˆΠΊΠ°Π»Ρƒ ΠΌΡƒΠ»ΡŒΡ‚ΠΈΠΌΠ΅Ρ‚Ρ€Π° Π½Π° Π΄ΠΈΠ°ΠΏΠ°Π·ΠΎΠ½ 20 Π’ ΠΈ ΠΈΡΠΏΠΎΠ»ΡŒΠ·ΡƒΠΉΡ‚Π΅ Π³Π½Π΅Π·Π΄Π° COM ΠΈ V. Π‘ ΠΏΠΎΠΌΠΎΡ‰ΡŒΡŽ ΠΌΡƒΠ»ΡŒΡ‚ΠΈΠΌΠ΅Ρ‚Ρ€Π° ΠΈΠ·ΠΌΠ΅Ρ€ΡŒΡ‚Π΅ ΠΈ Π·Π°ΠΏΠΈΡˆΠΈΡ‚Π΅ разности ΠΏΠΎΡ‚Π΅Π½Ρ†ΠΈΠ°Π»ΠΎΠ² (напряТСниС) Π’ 150 ΠΈ Π’ 700 Π½Π° ΠΊΠ°ΠΆΠ΄ΠΎΠΌ рСзисторС ΠΈ напряТСниС Π’ Π½Π° ΠΊΠΎΠΌΠ±ΠΈΠ½Π°Ρ†ΠΈΠΈ ΠΈΠ· Π΄Π²ΡƒΡ… рСзисторов.

2

Когда Π²Ρ‹ Π·Π°ΠΊΠΎΠ½Ρ‡ΠΈΡ‚Π΅ эти измСрСния, установитС напряТСниС источника питания Π½Π° ноль ΠΈ ΠΎΡ‚ΠΊΠ»ΡŽΡ‡ΠΈΡ‚Π΅ ΠΌΡƒΠ»ΡŒΡ‚ΠΈΠΌΠ΅Ρ‚Ρ€ ΠΎΡ‚ Ρ†Π΅ΠΏΠΈ.Π§Ρ‚ΠΎ ΠΈΠ· ΡΠ»Π΅Π΄ΡƒΡŽΡ‰Π΅Π³ΠΎ Π»ΡƒΡ‡ΡˆΠ΅ ΠΎΡ‚Ρ€Π°ΠΆΠ°Π΅Ρ‚ ваши Ρ€Π΅Π·ΡƒΠ»ΡŒΡ‚Π°Ρ‚Ρ‹?
  • β€’

    Π’ = Π’ 150 + Π’ 700

  • β€’

    Π’ = Π’ 150 = Π’ 700

3

Когда напряТСниС источника питания установлСно Π½Π° ноль, ΠΏΠΎΠ΄ΠΊΠ»ΡŽΡ‡ΠΈΡ‚Π΅ ΠΌΡƒΠ»ΡŒΡ‚ΠΈΠΌΠ΅Ρ‚Ρ€ ΠΏΠΎΡΠ»Π΅Π΄ΠΎΠ²Π°Ρ‚Π΅Π»ΡŒΠ½ΠΎ с рСзисторами, ΠΊΠ°ΠΊ ΠΏΠΎΠΊΠ°Π·Π°Π½ΠΎ Π½Π° рисункС 5. УстановитС ΠΌΡƒΠ»ΡŒΡ‚ΠΈΠΌΠ΅Ρ‚Ρ€ Π½Π° Π΄ΠΈΠ°ΠΏΠ°Π·ΠΎΠ½ постоянного Ρ‚ΠΎΠΊΠ° 200 мА ΠΈ ΠΏΠΎΠ΄ΠΊΠ»ΡŽΡ‡ΠΈΡ‚Π΅ Π²Ρ‹Π²ΠΎΠ΄Ρ‹ ΠΌΡƒΠ»ΡŒΡ‚ΠΈΠΌΠ΅Ρ‚Ρ€Π° ΠΊ ΠΏΡ€Π°Π²ΠΈΠ»ΡŒΠ½Ρ‹ΠΌ Π³Π½Π΅Π·Π΄Π°ΠΌ.

ΠžΡΡ‚ΠΎΡ€ΠΎΠΆΠ½ΠΎ:
ΠŸΠ΅Ρ€Π΅Π΄ Ρ‚Π΅ΠΌ, ΠΊΠ°ΠΊ ΠΏΡ€ΠΎΠ΄ΠΎΠ»ΠΆΠΈΡ‚ΡŒ, попроситС инструктора Π»Π°Π±ΠΎΡ€Π°Ρ‚ΠΎΡ€ΠΈΠΈ ΠΏΡ€ΠΎΠ²Π΅Ρ€ΠΈΡ‚ΡŒ ΠΏΡ€Π°Π²ΠΈΠ»ΡŒΠ½ΠΎΡΡ‚ΡŒ настройки: ΠΏΡ€Π΅Π΄ΠΎΡ…Ρ€Π°Π½ΠΈΡ‚Π΅Π»ΡŒ ΠΌΠΎΠΆΠ΅Ρ‚ ΠΏΠ΅Ρ€Π΅Π³ΠΎΡ€Π΅Ρ‚ΡŒ, Ссли ΠΈΠ·ΠΌΠ΅Ρ€ΠΈΡ‚Π΅Π»ΡŒ Π½Π΅ ΠΏΠΎΠ΄ΠΊΠ»ΡŽΡ‡Π΅Π½ ΠΊ Π½ΡƒΠΆΠ½Ρ‹ΠΌ Ρ‚ΠΎΡ‡ΠΊΠ°ΠΌ Π² Ρ†Π΅ΠΏΠΈ.

ΠŸΠΎΠ»ΡƒΡ‡ΠΈΠ² Π΄ΠΎΠ±Ρ€ΠΎ ΠΎΡ‚ ВА, Π²ΠΊΠ»ΡŽΡ‡ΠΈΡ‚Π΅ источник питания ΠΈ установитС Π΅Π³ΠΎ Π½Π° 5 Π²ΠΎΠ»ΡŒΡ‚. Π˜Π·ΠΌΠ΅Ρ€ΡŒΡ‚Π΅ Ρ‚ΠΎΠΊ Π² ΠΏΡ€ΠΎΠ²ΠΎΠ΄Π΅ ΠΌΠ΅ΠΆΠ΄Ρƒ двумя рСзисторами, ΠΏΡ€ΠΎΠ²ΠΎΠ΄Π΅ ΠΌΠ΅ΠΆΠ΄Ρƒ рСзистором 700 Ом ΠΈ источником питания ΠΈ ΠΏΡ€ΠΎΠ²ΠΎΠ΄Π΅ ΠΌΠ΅ΠΆΠ΄Ρƒ рСзистором 150 Ом ΠΈ источником питания.Π£Π±Π΅Π΄ΠΈΡ‚Π΅ΡΡŒ, Ρ‡Ρ‚ΠΎ эти Ρ‚Ρ€ΠΈ Ρ‚ΠΎΠΊΠ° Ρ€Π°Π²Π½Ρ‹.

4

ΠŸΡ€ΠΎΠ΄ΠΎΠ»ΠΆΠ°Ρ схСму, ΠΏΠΎΠΊΠ°Π·Π°Π½Π½ΡƒΡŽ Π½Π° рисункС 5, установитС напряТСниС источника питания Π½Π° 2 Π²ΠΎΠ»ΡŒΡ‚Π°. Π’ Ρ‚Π°Π±Π»ΠΈΡ†Π΅ 3 Π·Π°ΠΏΠΈΡˆΠΈΡ‚Π΅ Ρ‚Π΅ΠΊΡƒΡ‰ΠΈΠ΅ показания ΠΌΡƒΠ»ΡŒΡ‚ΠΈΠΌΠ΅Ρ‚Ρ€Π°. ΠŸΠΎΠ²Ρ‚ΠΎΡ€ΠΈΡ‚Π΅ эти дСйствия для напряТСний источника питания 4, 6, 8, 10 ΠΈ 12 Π²ΠΎΠ»ΡŒΡ‚.

5

Π˜ΡΠΏΠΎΠ»ΡŒΠ·ΡƒΠΉΡ‚Π΅ Excel для построСния Π³Ρ€Π°Ρ„ΠΈΠΊΠ° Π΄Π°Π½Π½Ρ‹Ρ… с Ρ‚ΠΎΠΊΠΎΠΌ ΠΏΠΎ Π²Π΅Ρ€Ρ‚ΠΈΠΊΠ°Π»ΡŒΠ½ΠΎΠΉ оси ΠΈ напряТСниСм ΠΏΠΎ Π³ΠΎΡ€ΠΈΠ·ΠΎΠ½Ρ‚Π°Π»ΡŒΠ½ΠΎΠΉ оси. Π˜ΡΠΏΠΎΠ»ΡŒΠ·ΡƒΠΉΡ‚Π΅ Excel, Ρ‡Ρ‚ΠΎΠ±Ρ‹ Π½Π°ΠΉΡ‚ΠΈ Π½Π°ΠΊΠ»ΠΎΠ½ прямой Π»ΠΈΠ½ΠΈΠΈ, ΠΊΠΎΡ‚ΠΎΡ€Ρ‹ΠΉ Π»ΡƒΡ‡ΡˆΠ΅ всСго соотвСтствуСт вашим Π΄Π°Π½Π½Ρ‹ΠΌ, ΠΈ Π·Π°ΠΏΠΈΡˆΠΈΡ‚Π΅ Ρ€Π΅Π·ΡƒΠ»ΡŒΡ‚Π°Ρ‚, Π²ΠΊΠ»ΡŽΡ‡Π°Ρ Π΅Π΄ΠΈΠ½ΠΈΡ†Ρ‹ измСрСния.

6

Π˜ΡΠΏΠΎΠ»ΡŒΠ·ΡƒΠΉΡ‚Π΅ Π·Π°ΠΊΠΎΠ½ Ома ΠΈ Π½Π°ΠΊΠ»ΠΎΠ½ Π³Ρ€Π°Ρ„ΠΈΠΊΠ°, Ρ‡Ρ‚ΠΎΠ±Ρ‹ Ρ€Π°ΡΡΡ‡ΠΈΡ‚Π°Ρ‚ΡŒ эквивалСнтноС сопротивлСниС R с Π΄Π²ΡƒΡ… ΠΏΠΎΡΠ»Π΅Π΄ΠΎΠ²Π°Ρ‚Π΅Π»ΡŒΠ½ΠΎ соСдинСнных рСзисторов Π² Π΅Π΄ΠΈΠ½ΠΈΡ†Π°Ρ… Ом (Ом). Π—Π°ΠΏΠΈΡˆΠΈΡ‚Π΅ свой Ρ€Π΅Π·ΡƒΠ»ΡŒΡ‚Π°Ρ‚.

Π”Π²Π° ΠΏΠ°Ρ€Π°Π»Π»Π΅Π»ΡŒΠ½Ρ‹Ρ… рСзистора

1

ΠŸΠΎΠ΄ΠΊΠ»ΡŽΡ‡ΠΈΡ‚Π΅ Π½Π° ΠΏΠ΅Ρ‡Π°Ρ‚Π½ΠΎΠΉ ΠΏΠ»Π°Ρ‚Π΅ Π΄Π²Π° рСзистора с ΠΌΠ°Ρ€ΠΊΠΈΡ€ΠΎΠ²ΠΊΠΎΠΉ 150 Ом ΠΈ 700 Ом, ΠΊΠ°ΠΊ ΠΏΠΎΠΊΠ°Π·Π°Π½ΠΎ Π½Π° РисункС 6a. Они ΡΡ‡ΠΈΡ‚Π°ΡŽΡ‚ΡΡ ΠΏΠ°Ρ€Π°Π»Π»Π΅Π»ΡŒΠ½Ρ‹ΠΌΠΈ, ΠΏΠΎΡΠΊΠΎΠ»ΡŒΠΊΡƒ напряТСниС Π½Π° ΠΊΠ°ΠΆΠ΄ΠΎΠΌ рСзисторС Ρ€Π°Π²Π½ΠΎ Π½Π°ΠΏΡ€ΡΠΆΠ΅Π½ΠΈΡŽ источника питания, Π° рСзисторы ΠΎΠ±Π΅ΡΠΏΠ΅Ρ‡ΠΈΠ²Π°ΡŽΡ‚ ΠΏΠ°Ρ€Π°Π»Π»Π΅Π»ΡŒΠ½Ρ‹Π΅ ΠΏΡƒΡ‚ΠΈ для прохоТдСния Ρ‚ΠΎΠΊΠ°.

2

Π£Π±Π΅Π΄ΠΈΡ‚Π΅ΡΡŒ, Ρ‡Ρ‚ΠΎ напряТСниС источника питания установлСно Π½Π° ноль. ΠŸΠΎΠ΄ΠΊΠ»ΡŽΡ‡ΠΈΡ‚Π΅ источник питания ΠΊ ΠΊΠΎΠΌΠ±ΠΈΠ½Π°Ρ†ΠΈΠΈ ΠΏΠ°Ρ€Π°Π»Π»Π΅Π»ΡŒΠ½Ρ‹Ρ… рСзисторов, ΠΊΠ°ΠΊ ΠΏΠΎΠΊΠ°Π·Π°Π½ΠΎ Π½Π° рисункС 6b. УстановитС Π±Π»ΠΎΠΊ питания Π½Π° 5 Π²ΠΎΠ»ΡŒΡ‚. Π‘ ΠΏΠΎΠΌΠΎΡ‰ΡŒΡŽ ΠΌΡƒΠ»ΡŒΡ‚ΠΈΠΌΠ΅Ρ‚Ρ€Π° ΠΈΠ·ΠΌΠ΅Ρ€ΡŒΡ‚Π΅ ΠΈ Π·Π°ΠΏΠΈΡˆΠΈΡ‚Π΅ Ρ‚ΠΎΠΊΠΈ I 150 ΠΈ I 700 , ΠΏΡ€ΠΎΡ‚Π΅ΠΊΠ°ΡŽΡ‰ΠΈΠ΅ Ρ‡Π΅Ρ€Π΅Π· ΠΊΠ°ΠΆΠ΄Ρ‹ΠΉ рСзистор, ΠΈ ΠΎΠ±Ρ‰ΠΈΠΉ Ρ‚ΠΎΠΊ, I , ΠΏΡ€ΠΎΡ‚Π΅ΠΊΠ°ΡŽΡ‰ΠΈΠΉ Ρ‡Π΅Ρ€Π΅Π· источник питания.

3

Когда Π²Ρ‹ Π·Π°ΠΊΠΎΠ½Ρ‡ΠΈΡ‚Π΅ эти измСрСния, установитС напряТСниС источника питания Π½Π° ноль ΠΈ ΠΎΡ‚ΠΊΠ»ΡŽΡ‡ΠΈΡ‚Π΅ ΠΌΡƒΠ»ΡŒΡ‚ΠΈΠΌΠ΅Ρ‚Ρ€ ΠΎΡ‚ Ρ†Π΅ΠΏΠΈ.

4

Π§Ρ‚ΠΎ ΠΈΠ· ΡΠ»Π΅Π΄ΡƒΡŽΡ‰Π΅Π³ΠΎ Π»ΡƒΡ‡ΡˆΠ΅ ΠΎΡ‚Ρ€Π°ΠΆΠ°Π΅Ρ‚ ваши Ρ€Π΅Π·ΡƒΠ»ΡŒΡ‚Π°Ρ‚Ρ‹?
  • β€’

    Π― = Π― 150 + Π― 700

  • β€’

    Π― = Π― 150 = Π― 700

5

ΠŸΡ€ΠΎΠ΄ΠΎΠ»ΠΆΠΈΡ‚Π΅ ΠΏΠ°Ρ€Π°Π»Π»Π΅Π»ΡŒΠ½ΡƒΡŽ ΡΠ΅Ρ‚ΡŒ рСзисторов. Когда напряТСниС источника питания установлСно Π½Π° ноль, ΠΏΠΎΠ΄ΠΊΠ»ΡŽΡ‡ΠΈΡ‚Π΅ ΠΌΡƒΠ»ΡŒΡ‚ΠΈΠΌΠ΅Ρ‚Ρ€ для измСрСния ΠΏΠΎΠ»Π½ΠΎΠ³ΠΎ Ρ‚ΠΎΠΊΠ°, I , ΠΏΡ€ΠΎΡ‚Π΅ΠΊΠ°ΡŽΡ‰Π΅Π³ΠΎ Ρ‡Π΅Ρ€Π΅Π· источник питания.УстановитС ΠΌΡƒΠ»ΡŒΡ‚ΠΈΠΌΠ΅Ρ‚Ρ€ Π½Π° Π΄ΠΈΠ°ΠΏΠ°Π·ΠΎΠ½ постоянного Ρ‚ΠΎΠΊΠ° 200 мА.

ΠžΡΡ‚ΠΎΡ€ΠΎΠΆΠ½ΠΎ:
ΠŸΠ΅Ρ€Π΅Π΄ Ρ‚Π΅ΠΌ, ΠΊΠ°ΠΊ ΠΏΡ€ΠΎΠ΄ΠΎΠ»ΠΆΠΈΡ‚ΡŒ, попроситС инструктора Π»Π°Π±ΠΎΡ€Π°Ρ‚ΠΎΡ€ΠΈΠΈ ΠΏΡ€ΠΎΠ²Π΅Ρ€ΠΈΡ‚ΡŒ ΠΏΡ€Π°Π²ΠΈΠ»ΡŒΠ½ΠΎΡΡ‚ΡŒ настройки: ΠΏΡ€Π΅Π΄ΠΎΡ…Ρ€Π°Π½ΠΈΡ‚Π΅Π»ΡŒ ΠΌΠΎΠΆΠ΅Ρ‚ ΠΏΠ΅Ρ€Π΅Π³ΠΎΡ€Π΅Ρ‚ΡŒ, Ссли ΠΈΠ·ΠΌΠ΅Ρ€ΠΈΡ‚Π΅Π»ΡŒ Π½Π΅ ΠΏΠΎΠ΄ΠΊΠ»ΡŽΡ‡Π΅Π½ ΠΊ Π½ΡƒΠΆΠ½Ρ‹ΠΌ Ρ‚ΠΎΡ‡ΠΊΠ°ΠΌ Π² Ρ†Π΅ΠΏΠΈ.

ΠŸΠΎΠ»ΡƒΡ‡ΠΈΠ² Π΄ΠΎΠ±Ρ€ΠΎ ΠΎΡ‚ ВА, Π²ΠΊΠ»ΡŽΡ‡ΠΈΡ‚Π΅ источник питания ΠΈ установитС Π΅Π³ΠΎ Π½Π° 2 Π²ΠΎΠ»ΡŒΡ‚Π°.

6

Π’ Ρ‚Π°Π±Π»ΠΈΡ†Π΅ 4 Π·Π°ΠΏΠΈΡˆΠΈΡ‚Π΅ Ρ‚Π΅ΠΊΡƒΡ‰Π΅Π΅ ΠΏΠΎΠΊΠ°Π·Π°Π½ΠΈΠ΅ ΠΌΡƒΠ»ΡŒΡ‚ΠΈΠΌΠ΅Ρ‚Ρ€Π°.ΠŸΠΎΠ²Ρ‚ΠΎΡ€ΠΈΡ‚Π΅ эти дСйствия для напряТСний источника питания 4, 6, 8, 10 ΠΈ 12 Π²ΠΎΠ»ΡŒΡ‚.

7

Π˜ΡΠΏΠΎΠ»ΡŒΠ·ΡƒΠΉΡ‚Π΅ Excel для построСния Π³Ρ€Π°Ρ„ΠΈΠΊΠ° Π΄Π°Π½Π½Ρ‹Ρ… с Ρ‚ΠΎΠΊΠΎΠΌ ΠΏΠΎ Π²Π΅Ρ€Ρ‚ΠΈΠΊΠ°Π»ΡŒΠ½ΠΎΠΉ оси ΠΈ напряТСниСм ΠΏΠΎ Π³ΠΎΡ€ΠΈΠ·ΠΎΠ½Ρ‚Π°Π»ΡŒΠ½ΠΎΠΉ оси. Π˜ΡΠΏΠΎΠ»ΡŒΠ·ΡƒΠΉΡ‚Π΅ Excel, Ρ‡Ρ‚ΠΎΠ±Ρ‹ Π½Π°ΠΉΡ‚ΠΈ Π½Π°ΠΊΠ»ΠΎΠ½ прямой Π»ΠΈΠ½ΠΈΠΈ, ΠΊΠΎΡ‚ΠΎΡ€Ρ‹ΠΉ Π»ΡƒΡ‡ΡˆΠ΅ всСго соотвСтствуСт вашим Π΄Π°Π½Π½Ρ‹ΠΌ, ΠΈ Π·Π°ΠΏΠΈΡˆΠΈΡ‚Π΅ Ρ€Π΅Π·ΡƒΠ»ΡŒΡ‚Π°Ρ‚, Π²ΠΊΠ»ΡŽΡ‡Π°Ρ Π΅Π΄ΠΈΠ½ΠΈΡ†Ρ‹ измСрСния.

8

Π˜ΡΠΏΠΎΠ»ΡŒΠ·ΡƒΠΉΡ‚Π΅ Π·Π°ΠΊΠΎΠ½ Ома ΠΈ Π½Π°ΠΊΠ»ΠΎΠ½ Π³Ρ€Π°Ρ„ΠΈΠΊΠ°, Ρ‡Ρ‚ΠΎΠ±Ρ‹ Ρ€Π°ΡΡΡ‡ΠΈΡ‚Π°Ρ‚ΡŒ эквивалСнтноС сопротивлСниС R p Π΄Π²ΡƒΡ… ΠΏΠ°Ρ€Π°Π»Π»Π΅Π»ΡŒΠ½ΠΎ Π²ΠΊΠ»ΡŽΡ‡Π΅Π½Π½Ρ‹Ρ… рСзисторов Π² Π΅Π΄ΠΈΠ½ΠΈΡ†Π°Ρ… Ом (Ом). Π—Π°ΠΏΠΈΡˆΠΈΡ‚Π΅ свой Ρ€Π΅Π·ΡƒΠ»ΡŒΡ‚Π°Ρ‚.

АвторскиС ΠΏΡ€Π°Π²Π° Β© 2012-2013 Advanced Instructional Systems Inc. ΠΈ | ΠšΡ€Π΅Π΄ΠΈΡ‚Ρ‹

ΠΏΠ°Ρ€Π°Π»Π»Π΅Π»ΡŒΠ½Ρ‹Ρ… рСзисторов

ΠžΠ±Π·ΠΎΡ€

РСзисторы Π² Ρ†Π΅ΠΏΠΈ ΠΌΠΎΠ³ΡƒΡ‚ Π±Ρ‹Ρ‚ΡŒ ΠΏΠΎΠ΄ΠΊΠ»ΡŽΡ‡Π΅Π½Ρ‹ ΠΏΠΎΡΠ»Π΅Π΄ΠΎΠ²Π°Ρ‚Π΅Π»ΡŒΠ½ΠΎ ΠΈΠ»ΠΈ ΠΏΠ°Ρ€Π°Π»Π»Π΅Π»ΡŒΠ½ΠΎ. ΠžΠ±Ρ‰Π΅Π΅ сопротивлСниС ΠΊΠΎΠΌΠ±ΠΈΠ½Π°Ρ†ΠΈΠΈ рСзисторов зависит ΠΊΠ°ΠΊ ΠΎΡ‚ ΠΈΡ… ΠΈΠ½Π΄ΠΈΠ²ΠΈΠ΄ΡƒΠ°Π»ΡŒΠ½Ρ‹Ρ… Π·Π½Π°Ρ‡Π΅Π½ΠΈΠΉ, Ρ‚Π°ΠΊ ΠΈ ΠΎΡ‚ способа ΠΈΡ… ΠΏΠΎΠ΄ΠΊΠ»ΡŽΡ‡Π΅Π½ΠΈΡ.

ΠŸΠ°Ρ€Π°Π»Π»Π΅Π»ΡŒΠ½Ρ‹Π΅ рСзисторы

БопротивлСния ΠΏΠ°Ρ€Π°Π»Π»Π΅Π»ΡŒΠ½Ρ‹, ΠΊΠΎΠ³Π΄Π° ΠΊΠ°ΠΆΠ΄Ρ‹ΠΉ рСзистор ΠΏΠΎΠ΄ΠΊΠ»ΡŽΡ‡Π΅Π½ нСпосрСдствСнно ΠΊ источнику напряТСния ΠΏΡƒΡ‚Π΅ΠΌ соСдинСния ΠΏΡ€ΠΎΠ²ΠΎΠ΄ΠΎΠ² с Π½Π΅Π·Π½Π°Ρ‡ΠΈΡ‚Π΅Π»ΡŒΠ½Ρ‹ΠΌ сопротивлСниСм.Π’Π°ΠΊΠΈΠΌ ΠΎΠ±Ρ€Π°Π·ΠΎΠΌ, ΠΊ ΠΊΠ°ΠΆΠ΄ΠΎΠΌΡƒ рСзистору ΠΏΡ€ΠΈΠ»ΠΎΠΆΠ΅Π½ΠΎ ΠΏΠΎΠ»Π½ΠΎΠ΅ напряТСниС источника.

ΠŸΠ°Ρ€Π°Π»Π»Π΅Π»ΡŒΠ½Ρ‹Π΅ рСзисторы

ΠŸΠ°Ρ€Π°Π»Π»Π΅Π»ΡŒΠ½ΠΎΠ΅ соСдинСниС рСзисторов.

ΠšΠ°ΠΆΠ΄Ρ‹ΠΉ рСзистор потрСбляСт Ρ‚Π°ΠΊΠΎΠΉ ΠΆΠ΅ Ρ‚ΠΎΠΊ, ΠΊΠ°ΠΊ Ссли Π±Ρ‹ ΠΎΠ½ Π±Ρ‹Π» СдинствСнным рСзистором, ΠΏΠΎΠ΄ΠΊΠ»ΡŽΡ‡Π΅Π½Π½Ρ‹ΠΌ ΠΊ источнику напряТСния. Π­Ρ‚ΠΎ Π²Π΅Ρ€Π½ΠΎ для схСм Π² Π΄ΠΎΠΌΠ΅ ΠΈΠ»ΠΈ ΠΊΠ²Π°Ρ€Ρ‚ΠΈΡ€Π΅. КаТдая Ρ€ΠΎΠ·Π΅Ρ‚ΠΊΠ°, ΠΏΠΎΠ΄ΠΊΠ»ΡŽΡ‡Π΅Π½Π½Π°Ρ ΠΊ устройству («рСзистор»), ΠΌΠΎΠΆΠ΅Ρ‚ Ρ€Π°Π±ΠΎΡ‚Π°Ρ‚ΡŒ нСзависимо, ΠΈ Ρ‚ΠΎΠΊ Π½Π΅ Π΄ΠΎΠ»ΠΆΠ΅Π½ ΠΏΡ€ΠΎΡ…ΠΎΠ΄ΠΈΡ‚ΡŒ Ρ‡Π΅Ρ€Π΅Π· ΠΊΠ°ΠΆΠ΄ΠΎΠ΅ устройство ΠΏΠΎΡΠ»Π΅Π΄ΠΎΠ²Π°Ρ‚Π΅Π»ΡŒΠ½ΠΎ.

Π—Π°ΠΊΠΎΠ½ Ома ΠΈ ΠΏΠ°Ρ€Π°Π»Π»Π΅Π»ΡŒΠ½Ρ‹Π΅ рСзисторы

ΠšΠ°ΠΆΠ΄Ρ‹ΠΉ рСзистор Π² Ρ†Π΅ΠΏΠΈ ΠΈΠΌΠ΅Π΅Ρ‚ ΠΏΠΎΠ»Π½ΠΎΠ΅ напряТСниС. Богласно Π·Π°ΠΊΠΎΠ½Ρƒ Ома Ρ‚ΠΎΠΊΠΈ, ΠΏΡ€ΠΎΡ‚Π΅ΠΊΠ°ΡŽΡ‰ΠΈΠ΅ Ρ‡Π΅Ρ€Π΅Π· ΠΎΡ‚Π΄Π΅Π»ΡŒΠ½Ρ‹Π΅ рСзисторы, Ρ€Π°Π²Π½Ρ‹ $ I_1 = \ frac {V} {R_1} $, $ I_2 = \ frac {V} {R_2} $ ΠΈ $ I_3 = \ frac {V} {R_3 } $. Π‘ΠΎΡ…Ρ€Π°Π½Π΅Π½ΠΈΠ΅ заряда ΠΏΠΎΠ΄Ρ€Π°Π·ΡƒΠΌΠ΅Π²Π°Π΅Ρ‚, Ρ‡Ρ‚ΠΎ ΠΏΠΎΠ»Π½Ρ‹ΠΉ Ρ‚ΠΎΠΊ являСтся суммой этих Ρ‚ΠΎΠΊΠΎΠ²:

РСзисторы ΠΏΠ°Ρ€Π°Π»Π»Π΅Π»ΡŒΠ½Ρ‹Π΅

Π’Ρ€ΠΈ рСзистора, ΠΏΠΎΠ΄ΠΊΠ»ΡŽΡ‡Π΅Π½Π½Ρ‹Ρ… ΠΏΠ°Ρ€Π°Π»Π»Π΅Π»ΡŒΠ½ΠΎ ΠΊ Π±Π°Ρ‚Π°Ρ€Π΅Π΅, ΠΈ эквивалСнтноС ΠΎΠ΄ΠΈΠ½ΠΎΡ‡Π½ΠΎΠ΅ ΠΈΠ»ΠΈ ΠΏΠ°Ρ€Π°Π»Π»Π΅Π»ΡŒΠ½ΠΎΠ΅ сопротивлСниС.

$ I = I_1 + I_2 + I_3. $

ΠŸΠΎΠ΄ΡΡ‚Π°Π½ΠΎΠ²ΠΊΠ° Π²Ρ‹Ρ€Π°ΠΆΠ΅Π½ΠΈΠΉ для ΠΎΡ‚Π΄Π΅Π»ΡŒΠ½Ρ‹Ρ… Ρ‚ΠΎΠΊΠΎΠ² Π΄Π°Π΅Ρ‚:

$ I = \ frac {V} {R_1} + \ frac {V} {R_2} + \ frac {V} {R_3} $

ΠΈΠ»ΠΈ

$ I = V (\ frac {1} {R_1} + \ frac {1} {R_2} + \ frac {1} {R_3}) $

Π­Ρ‚ΠΎ ΠΎΠ·Π½Π°Ρ‡Π°Π΅Ρ‚, Ρ‡Ρ‚ΠΎ ΠΏΠΎΠ»Π½ΠΎΠ΅ сопротивлСниС Π² ΠΏΠ°Ρ€Π°Π»Π»Π΅Π»ΡŒΠ½ΠΎΠΉ Ρ†Π΅ΠΏΠΈ Ρ€Π°Π²Π½ΠΎ суммС инвСрсии ΠΊΠ°ΠΆΠ΄ΠΎΠ³ΠΎ ΠΎΡ‚Π΄Π΅Π»ΡŒΠ½ΠΎΠ³ΠΎ сопротивлСния. Π‘Π»Π΅Π΄ΠΎΠ²Π°Ρ‚Π΅Π»ΡŒΠ½ΠΎ, для ΠΊΠ°ΠΆΠ΄ΠΎΠΉ Ρ†Π΅ΠΏΠΈ с числом $ n $ ΠΈΠ»ΠΈ ΠΏΠ°Ρ€Π°Π»Π»Π΅Π»ΡŒΠ½ΠΎ ΠΏΠΎΠ΄ΠΊΠ»ΡŽΡ‡Π΅Π½Π½Ρ‹ΠΌΠΈ рСзисторами

$ R_ {n \; (parallel)} = \ frac {1} {R_1} + \ frac {1} {R_2} + \ frac {1} {R_3}… + \ frac {1} {R_n}. $

Π­Ρ‚ΠΎ ΡΠΎΠΎΡ‚Π½ΠΎΡˆΠ΅Π½ΠΈΠ΅ ΠΏΡ€ΠΈΠ²ΠΎΠ΄ΠΈΡ‚ ΠΊ ΠΎΠ±Ρ‰Π΅ΠΌΡƒ ΡΠΎΠΏΡ€ΠΎΡ‚ΠΈΠ²Π»Π΅Π½ΠΈΡŽ, ΠΊΠΎΡ‚ΠΎΡ€ΠΎΠ΅ мСньшС наимСньшСго ΠΈΠ· ΠΎΡ‚Π΄Π΅Π»ΡŒΠ½Ρ‹Ρ… сопротивлСний. Когда рСзисторы соСдинСны ΠΏΠ°Ρ€Π°Π»Π»Π΅Π»ΡŒΠ½ΠΎ, ΠΎΡ‚ источника Ρ‚Π΅Ρ‡Π΅Ρ‚ большС Ρ‚ΠΎΠΊΠ°, Ρ‡Π΅ΠΌ ΠΏΡ€ΠΎΡ‚Π΅ΠΊΠ°Π΅Ρ‚ для любого ΠΈΠ· Π½ΠΈΡ… ΠΏΠΎ ΠΎΡ‚Π΄Π΅Π»ΡŒΠ½ΠΎΡΡ‚ΠΈ, поэтому ΠΎΠ±Ρ‰Π΅Π΅ сопротивлСниС Π½ΠΈΠΆΠ΅.

ΠšΠ°ΠΆΠ΄Ρ‹ΠΉ рСзистор, Π²ΠΊΠ»ΡŽΡ‡Π΅Π½Π½Ρ‹ΠΉ ΠΏΠ°Ρ€Π°Π»Π»Π΅Π»ΡŒΠ½ΠΎ, ΠΈΠΌΠ΅Π΅Ρ‚ ΠΎΠ΄ΠΈΠ½Π°ΠΊΠΎΠ²ΠΎΠ΅ ΠΏΠΎΠ»Π½ΠΎΠ΅ напряТСниС источника, ΠΏΡ€ΠΈΠ»ΠΎΠΆΠ΅Π½Π½ΠΎΠ³ΠΎ ΠΊ Π½Π΅ΠΌΡƒ, Π½ΠΎ Π΄Π΅Π»ΠΈΡ‚ ΠΎΠ±Ρ‰ΠΈΠΉ Ρ‚ΠΎΠΊ ΠΌΠ΅ΠΆΠ΄Ρƒ Π½ΠΈΠΌΠΈ. ΠŸΡ€ΠΈΠΌΠ΅Ρ€ΠΎΠΌ ΠΌΠΎΠΆΠ΅Ρ‚ ΡΠ»ΡƒΠΆΠΈΡ‚ΡŒ соСдинСниС Π΄Π²ΡƒΡ… Π»Π°ΠΌΠΏΠΎΡ‡Π΅ΠΊ Π² ΠΏΠ°Ρ€Π°Π»Π»Π΅Π»ΡŒΠ½ΡƒΡŽ Ρ†Π΅ΠΏΡŒ с 1.Аккумулятор 5 Π’. Π’ ΠΏΠΎΡΠ»Π΅Π΄ΠΎΠ²Π°Ρ‚Π΅Π»ΡŒΠ½ΠΎΠΉ Ρ†Π΅ΠΏΠΈ Π΄Π²Π΅ Π»Π°ΠΌΠΏΠΎΡ‡ΠΊΠΈ Π±ΡƒΠ΄ΡƒΡ‚ Π²Π΄Π²ΠΎΠ΅ ΠΌΠ΅Π½Π΅Π΅ тусклыми ΠΏΡ€ΠΈ ΠΏΠΎΠ΄ΠΊΠ»ΡŽΡ‡Π΅Π½ΠΈΠΈ ΠΊ ΠΎΠ΄Π½ΠΎΠΌΡƒ источнику Π±Π°Ρ‚Π°Ρ€Π΅ΠΈ. Однако, Ссли Π±Ρ‹ Π΄Π²Π΅ Π»Π°ΠΌΠΏΠΎΡ‡ΠΊΠΈ Π±Ρ‹Π»ΠΈ ΠΏΠΎΠ΄ΠΊΠ»ΡŽΡ‡Π΅Π½Ρ‹ ΠΏΠ°Ρ€Π°Π»Π»Π΅Π»ΡŒΠ½ΠΎ, ΠΎΠ½ΠΈ Π±Ρ‹Π»ΠΈ Π±Ρ‹ ΡΡ‚ΠΎΠ»ΡŒ ΠΆΠ΅ яркими, ΠΊΠ°ΠΊ Ссли Π±Ρ‹ ΠΎΠ½ΠΈ Π±Ρ‹Π»ΠΈ ΠΏΠΎΠ΄ΠΊΠ»ΡŽΡ‡Π΅Π½Ρ‹ ΠΊ Π±Π°Ρ‚Π°Ρ€Π΅Π΅ ΠΏΠΎ ΠΎΡ‚Π΄Π΅Π»ΡŒΠ½ΠΎΡΡ‚ΠΈ. ΠŸΠΎΡΠΊΠΎΠ»ΡŒΠΊΡƒ ΠΊ ΠΎΠ±Π΅ΠΈΠΌ Π»Π°ΠΌΠΏΠΎΡ‡ΠΊΠ°ΠΌ подаСтся ΠΎΠ΄ΠΈΠ½Π°ΠΊΠΎΠ²ΠΎΠ΅ ΠΏΠΎΠ»Π½ΠΎΠ΅ напряТСниС, батарСя Ρ‚Π°ΠΊΠΆΠ΅ разряТаСтся быстрСС, ΠΏΠΎΡΠΊΠΎΠ»ΡŒΠΊΡƒ ΠΎΠ½Π° ΠΏΠΎ сущСству обСспСчиваСт ΠΏΠΎΠ»Π½ΡƒΡŽ ΡΠ½Π΅Ρ€Π³ΠΈΡŽ ΠΎΠ±Π΅ΠΈΠΌΠΈ Π»Π°ΠΌΠΏΠΎΡ‡ΠΊΠ°ΠΌΠΈ. Π’ ΠΏΠΎΡΠ»Π΅Π΄ΠΎΠ²Π°Ρ‚Π΅Π»ΡŒΠ½ΠΎΠΉ Ρ†Π΅ΠΏΠΈ батарСя Π±ΡƒΠ΄Π΅Ρ‚ Ρ€Π°Π±ΠΎΡ‚Π°Ρ‚ΡŒ ΡΡ‚ΠΎΠ»ΡŒΠΊΠΎ ΠΆΠ΅, сколько ΠΈ с ΠΎΠ΄Π½ΠΎΠΉ Π»Π°ΠΌΠΏΠΎΡ‡ΠΊΠΎΠΉ, Ρ‚ΠΎΠ»ΡŒΠΊΠΎ Ρ‚ΠΎΠ³Π΄Π° ΡΡ€ΠΊΠΎΡΡ‚ΡŒ Π±ΡƒΠ΄Π΅Ρ‚ Ρ€Π°Π·Π΄Π΅Π»Π΅Π½Π° ΠΌΠ΅ΠΆΠ΄Ρƒ Π»Π°ΠΌΠΏΠΎΡ‡ΠΊΠ°ΠΌΠΈ.

рСзисторов, Π²ΠΊΠ»ΡŽΡ‡Π΅Π½Π½Ρ‹Ρ… ΠΏΠ°Ρ€Π°Π»Π»Π΅Π»ΡŒΠ½ΠΎ | Π€ΠΎΡ€ΠΌΡƒΠ»Π° эквивалСнтного сопротивлСния

Π’Π²Π΅Π΄Π΅Π½ΠΈΠ΅

Π”Π²Π° рСзистора ΡΡ‡ΠΈΡ‚Π°ΡŽΡ‚ΡΡ ΠΏΠΎΠ΄ΠΊΠ»ΡŽΡ‡Π΅Π½Π½Ρ‹ΠΌΠΈ ΠΏΠ°Ρ€Π°Π»Π»Π΅Π»ΡŒΠ½ΠΎ, Ссли ΠΎΠ±Π° Π²Ρ‹Π²ΠΎΠ΄Π° рСзистора ΠΏΠΎΠ΄ΠΊΠ»ΡŽΡ‡Π΅Π½Ρ‹ ΠΊ ΠΊΠ°ΠΆΠ΄ΠΎΠΌΡƒ ΡΠΎΠΎΡ‚Π²Π΅Ρ‚ΡΡ‚Π²ΡƒΡŽΡ‰Π΅ΠΌΡƒ Π²Ρ‹Π²ΠΎΠ΄Ρƒ Π΄Ρ€ΡƒΠ³ΠΎΠ³ΠΎ рСзистора. Π’ сСти ΠΏΠ°Ρ€Π°Π»Π»Π΅Π»ΡŒΠ½Ρ‹Ρ… рСзисторов Ρ‚ΠΎΠΊ ΠΌΠΎΠΆΠ΅Ρ‚ ΠΏΡ€ΠΎΡ…ΠΎΠ΄ΠΈΡ‚ΡŒ ΠΏΠΎ нСскольким путям, Π² ΠΎΡ‚Π»ΠΈΡ‡ΠΈΠ΅ ΠΎΡ‚ сСти с ΠΏΠΎΡΠ»Π΅Π΄ΠΎΠ²Π°Ρ‚Π΅Π»ΡŒΠ½Ρ‹ΠΌΠΈ рСзисторами, ΠΏΠΎΡΠΊΠΎΠ»ΡŒΠΊΡƒ сущСствуСт нСсколько ΠΏΡƒΡ‚Π΅ΠΉ для прохоТдСния Ρ‚ΠΎΠΊΠ°. Π‘Π»Π΅Π΄ΠΎΠ²Π°Ρ‚Π΅Π»ΡŒΠ½ΠΎ, ΠΏΠ°Ρ€Π°Π»Π»Π΅Π»ΡŒΠ½Ρ‹Π΅ рСзистивныС Ρ†Π΅ΠΏΠΈ ΡΠ²Π»ΡΡŽΡ‚ΡΡ дСлитСлями Ρ‚ΠΎΠΊΠ°.

ΠŸΠ°Ρ€Π°Π»Π»Π΅Π»ΡŒΠ½Ρ‹Π΅ рСзисторы

Если Π΄Π²Π° ΠΈΠ»ΠΈ Π±ΠΎΠ»Π΅Π΅ рСзистора ΠΏΠΎΠ΄ΠΊΠ»ΡŽΡ‡Π΅Π½Ρ‹ ΠΏΠ°Ρ€Π°Π»Π»Π΅Π»ΡŒΠ½ΠΎ, Ρ‚ΠΎ Ρ€Π°Π·Π½ΠΎΡΡ‚ΡŒ ΠΏΠΎΡ‚Π΅Π½Ρ†ΠΈΠ°Π»ΠΎΠ² Π½Π° ΠΊΠ°ΠΆΠ΄ΠΎΠΌ рСзисторС ΠΎΠ΄ΠΈΠ½Π°ΠΊΠΎΠ²Π°.ΠŸΡ€ΠΈ ΠΏΠ°Ρ€Π°Π»Π»Π΅Π»ΡŒΠ½ΠΎΠΌ соСдинСнии рСзисторы ΠΏΠΎΠ΄ΠΊΠ»ΡŽΡ‡Π°ΡŽΡ‚ΡΡ ΠΊ ΠΎΠ΄Π½ΠΈΠΌ ΠΈ Ρ‚Π΅ΠΌ ΠΆΠ΅ ΡƒΠ·Π»Π°ΠΌ. Π­Ρ‚ΠΎ ΠΌΠΎΠΆΠ½ΠΎ ΠΎΠΏΡ€Π΅Π΄Π΅Π»ΠΈΡ‚ΡŒ ΠΏΠΎ Π½Π°Π»ΠΈΡ‡ΠΈΡŽ Π±ΠΎΠ»Π΅Π΅ Ρ‡Π΅ΠΌ ΠΎΠ΄Π½ΠΎΠ³ΠΎ ΠΏΡƒΡ‚ΠΈ прохоТдСния Ρ‚ΠΎΠΊΠ°. НапримСр, привСдСнная Π½ΠΈΠΆΠ΅ схСма прСдставляСт собой ΠΏΠ°Ρ€Π°Π»Π»Π΅Π»ΡŒΠ½ΠΎΠ΅ соСдинСниС рСзисторов. Π Π°Π·Π½ΠΎΡΡ‚ΡŒ ΠΏΠΎΡ‚Π΅Π½Ρ†ΠΈΠ°Π»ΠΎΠ² Π½Π° рСзисторС R1 такая ΠΆΠ΅, ΠΊΠ°ΠΊ ΠΈ Π½Π° рСзисторС R2, ΠΊΠΎΡ‚ΠΎΡ€Ρ‹ΠΉ Ρ€Π°Π²Π΅Π½ ΠΏΠΎΡ‚Π΅Π½Ρ†ΠΈΠ°Π»Ρƒ питания V AB .

Если V AB — это ΠΏΠΎΠ΄Π°Π²Π°Π΅ΠΌΡ‹ΠΉ ΠΏΠΎΡ‚Π΅Π½Ρ†ΠΈΠ°Π», Ρ‚ΠΎ

V R1 = V R2 = V AB

Π’ ΡΠ»Π΅Π΄ΡƒΡŽΡ‰Π΅ΠΉ схСмС рСзисторы R1, R2 ΠΈ R3 ΠΏΠΎΠ΄ΠΊΠ»ΡŽΡ‡Π΅Π½Ρ‹ ΠΏΠ°Ρ€Π°Π»Π»Π΅Π»ΡŒΠ½ΠΎ комбинация.

Π—Π΄Π΅ΡΡŒ ΠΏΠΎΡ‚Π΅Π½Ρ†ΠΈΠ°Π» питания составляСт V AB ΠΌΠ΅ΠΆΠ΄Ρƒ Ρ‚ΠΎΡ‡ΠΊΠ°ΠΌΠΈ A ΠΈ B. ΠŸΠΎΡΠΊΠΎΠ»ΡŒΠΊΡƒ рСзисторы R1, R2 ΠΈ R3 соСдинСны ΠΏΠ°Ρ€Π°Π»Π»Π΅Π»ΡŒΠ½ΠΎ, Ρ€Π°Π·Π½ΠΎΡΡ‚ΡŒ ΠΏΠΎΡ‚Π΅Π½Ρ†ΠΈΠ°Π»ΠΎΠ² Π½Π° ΠΊΠ°ΠΆΠ΄ΠΎΠΌ рСзисторС такая ΠΆΠ΅, ΠΊΠ°ΠΊ ΠΈ Ρƒ источника питания. Π‘Π»Π΅Π΄ΠΎΠ²Π°Ρ‚Π΅Π»ΡŒΠ½ΠΎ, V AB = V R1 = V R2 = V R3 .

Π“Π΄Π΅

Π’ R1 — ΠΏΠΎΡ‚Π΅Π½Ρ†ΠΈΠ°Π» Π½Π° рСзисторС R1.

Π’ R2 — это ΠΏΠΎΡ‚Π΅Π½Ρ†ΠΈΠ°Π» Π½Π° рСзисторС R2.

Π’ R3 — ΠΏΠΎΡ‚Π΅Π½Ρ†ΠΈΠ°Π» Π½Π° рСзисторС R3.

Но Ρ‚ΠΎΠΊ, ΠΏΡ€ΠΎΡ‚Π΅ΠΊΠ°ΡŽΡ‰ΠΈΠΉ Ρ‡Π΅Ρ€Π΅Π· эти Ρ‚Ρ€ΠΈ рСзистора, Ρ€Π°Π·Π½Ρ‹ΠΉ. Если I — это Ρ‚ΠΎΠΊ, ΠΏΠΎΠΊΠΈΠ΄Π°ΡŽΡ‰ΠΈΠΉ ΡƒΠ·Π΅Π» A, Ρ‚ΠΎ ΠΎΠ½ ΠΈΠΌΠ΅Π΅Ρ‚ Ρ‚Ρ€ΠΈ ΠΏΡƒΡ‚ΠΈ, Ρ‡Ρ‚ΠΎΠ±Ρ‹ Π΄ΠΎΡΡ‚ΠΈΡ‡ΡŒ ΡƒΠ·Π»Π° B. Π’ΠΎΠΊ, ΠΏΡ€ΠΎΡ‚Π΅ΠΊΠ°ΡŽΡ‰ΠΈΠΉ Ρ‡Π΅Ρ€Π΅Π· ΠΊΠ°ΠΆΠ΄Ρ‹ΠΉ рСзистор, зависит ΠΎΡ‚ Π΅Π³ΠΎ сопротивлСния. Π‘Π»Π΅Π΄ΠΎΠ²Π°Ρ‚Π΅Π»ΡŒΠ½ΠΎ, Π² случаС ΠΏΠ°Ρ€Π°Π»Π»Π΅Π»ΡŒΠ½Ρ‹Ρ… рСзистивных Ρ†Π΅ΠΏΠ΅ΠΉ Ρ‚ΠΎΠΊ Π²ΠΎ всСх рСзисторах Π½Π΅ΠΎΠ΄ΠΈΠ½Π°ΠΊΠΎΠ². Если I1 — это Ρ‚ΠΎΠΊ, ΠΏΡ€ΠΎΡ‚Π΅ΠΊΠ°ΡŽΡ‰ΠΈΠΉ Ρ‡Π΅Ρ€Π΅Π· рСзистор R1, I2 — это Ρ‚ΠΎΠΊ, ΠΏΡ€ΠΎΡ‚Π΅ΠΊΠ°ΡŽΡ‰ΠΈΠΉ Ρ‡Π΅Ρ€Π΅Π· рСзистор R2, Π° I3 — это Ρ‚ΠΎΠΊ, ΠΏΡ€ΠΎΡ‚Π΅ΠΊΠ°ΡŽΡ‰ΠΈΠΉ Ρ‡Π΅Ρ€Π΅Π· рСзистор R3, Ρ‚ΠΎΠ³Π΄Π° Ρ‚ΠΎΠΊΠΈ I, I1, I2 ΠΈ I3 ΠΌΠΎΠ³ΡƒΡ‚ Π±Ρ‹Ρ‚ΡŒ связаны с ΠΏΠΎΠΌΠΎΡ‰ΡŒΡŽ Π·Π°ΠΊΠΎΠ½Π° Ρ‚ΠΎΠΊΠ° ΠšΠΈΡ€Ρ…Π³ΠΎΡ„Π°. .Богласно Ρ‚Π΅ΠΊΡƒΡ‰Π΅ΠΌΡƒ Π·Π°ΠΊΠΎΠ½Ρƒ ΠšΠΈΡ€Ρ…Π³ΠΎΡ„Π°, «сумма Ρ‚ΠΎΠΊΠΎΠ², входящих Π² ΡƒΠ·Π΅Π», Ρ€Π°Π²Π½Π° суммС Ρ‚ΠΎΠΊΠΎΠ², выходящих ΠΈΠ· ΡƒΠ·Π»Π°Β».

ΠžΡ‚ΡΡŽΠ΄Π°

I = I1 + I2 + I3.

Π€ΠΎΡ€ΠΌΡƒΠ»Π° эквивалСнтного сопротивлСния

Π›ΡŽΠ±ΠΎΠ΅ количСство рСзисторов, Π²ΠΊΠ»ΡŽΡ‡Π΅Π½Π½Ρ‹Ρ… Π² ΠΏΠ°Ρ€Π°Π»Π»Π΅Π»ΡŒΠ½ΡƒΡŽ ΠΊΠΎΠΌΠ±ΠΈΠ½Π°Ρ†ΠΈΡŽ, ΠΌΠΎΠΆΠ΅Ρ‚ Π±Ρ‹Ρ‚ΡŒ Π·Π°ΠΌΠ΅Π½Π΅Π½ΠΎ ΠΎΠ΄Π½ΠΈΠΌ рСзистором с сопротивлСниСм, Ρ€Π°Π²Π½Ρ‹ΠΌ эквивалСнтному ΡΠΎΠΏΡ€ΠΎΡ‚ΠΈΠ²Π»Π΅Π½ΠΈΡŽ рСзисторов ΠΏΠ°Ρ€Π°Π»Π»Π΅Π»ΡŒΠ½ΠΎΠΉ ΠΊΠΎΠΌΠ±ΠΈΠ½Π°Ρ†ΠΈΠΈ.

Π‘Ρ‹Π»ΠΎ установлСно, Ρ‡Ρ‚ΠΎ напряТСниС Π½Π° ΠΊΠ°ΠΆΠ΄ΠΎΠΌ рСзисторС Π² ΠΏΠ°Ρ€Π°Π»Π»Π΅Π»ΡŒΠ½ΠΎΠΉ ΠΊΠΎΠΌΠ±ΠΈΠ½Π°Ρ†ΠΈΠΈ ΠΎΠ΄ΠΈΠ½Π°ΠΊΠΎΠ²ΠΎ, Π° ΠΎΠ±Ρ‰ΠΈΠΉ Ρ‚ΠΎΠΊ Ρ€Π°Π²Π΅Π½ суммС ΠΎΡ‚Π΄Π΅Π»ΡŒΠ½Ρ‹Ρ… Ρ‚ΠΎΠΊΠΎΠ².Рассмотрим ΡΠ»Π΅Π΄ΡƒΡŽΡ‰ΡƒΡŽ схСму.

Π—Π΄Π΅ΡΡŒ I = I1 + I2 + I3

I1 = V / R1

I2 = V / R2

I3 = V / R3

Если R T — ΠΏΠΎΠ»Π½ΠΎΠ΅ сопротивлСниС Ρ†Π΅ΠΏΠΈ, Ρ‚ΠΎ

I = V / R T

Π‘Π»Π΅Π΄ΠΎΠ²Π°Ρ‚Π΅Π»ΡŒΠ½ΠΎ, V / R T = V / R 1 + V / R 2 + V / R 3

1 / R T = 1 / R 1 + 1 / R 2 + 1 / R 3

Если R eq являСтся эквивалСнтным сопротивлСниСм Ρ†Π΅ΠΏΠΈ, Ρ‚ΠΎ ΠΎΠ½ΠΎ рассчитываСтся ΠΏΡƒΡ‚Π΅ΠΌ слоТСния ΠΎΠ±Ρ€Π°Ρ‚Π½Ρ‹Ρ… Π·Π½Π°Ρ‡Π΅Π½ΠΈΠΉ ΠΎΡ‚Π΄Π΅Π»ΡŒΠ½Ρ‹Ρ… сопротивлСний (1 / R).ΠžΠ±Ρ€Π°Ρ‚Π½Π°Ρ Π²Π΅Π»ΠΈΡ‡ΠΈΠ½Π° этой алгСбраичСской суммы даст эквивалСнтноС сопротивлСниС. Π£Ρ€Π°Π²Π½Π΅Π½ΠΈΠ΅ эквивалСнтного сопротивлСния R eq ΠΏΠΎΠΊΠ°Π·Π°Π½ΠΎ Π½ΠΈΠΆΠ΅ для ΠΏΠ°Ρ€Π°Π»Π»Π΅Π»ΡŒΠ½ΠΎΠΉ рСзистивной Ρ†Π΅ΠΏΠΈ ΠΈΠ· n рСзисторов.

(1 / R экв. ) = (1 / R1) + (1 / R2) + (1 / R3) + ……… + (1 / Rn)

Из ΠΏΡ€ΠΈΠ²Π΅Π΄Π΅Π½Π½ΠΎΠ³ΠΎ Π²Ρ‹ΡˆΠ΅ уравнСния ΠΌΠΎΠΆΠ½ΠΎ ΡΠ΄Π΅Π»Π°Ρ‚ΡŒ Π²Ρ‹Π²ΠΎΠ΄, Ρ‡Ρ‚ΠΎ эквивалСнтноС сопротивлСниС рСзисторов, соСдинСнных ΠΏΠ°Ρ€Π°Π»Π»Π΅Π»ΡŒΠ½ΠΎ, всСгда мСньшС, Ρ‡Π΅ΠΌ сопротивлСниС самого малСнького рСзистора.

Если Π΄Π²Π° рСзистора ΠΏΠΎΠ΄ΠΊΠ»ΡŽΡ‡Π΅Π½Ρ‹ ΠΏΠ°Ρ€Π°Π»Π»Π΅Π»ΡŒΠ½ΠΎ, Ρ‚ΠΎ эквивалСнтноС сопротивлСниС Ρ€Π°Π²Π½ΠΎ

(1 / R eq ) = (1 / R1) + (1 / R2)

R EQ = R 1 * R 1 / (R 1 + R 2 )

Если Π΄Π²Π° рСзистора Ρ€Π°Π²Π½ΠΎΠ³ΠΎ сопротивлСния R соСдинСны ΠΏΠ°Ρ€Π°Π»Π»Π΅Π»ΡŒΠ½ΠΎ, Ρ‚ΠΎ эквивалСнтноС сопротивлСниС ΠΊΠΎΠΌΠ±ΠΈΠ½Π°Ρ†ΠΈΠΈ Π±ΡƒΠ΄Π΅Ρ‚ R / 2.

Аналогично, Ссли Ρ‚Ρ€ΠΈ рСзистора Ρ€Π°Π²Π½ΠΎΠ³ΠΎ сопротивлСния R соСдинСны Π² ΠΏΠ°Ρ€Π°Π»Π»Π΅Π»ΡŒΠ½ΠΎΠΌ соСдинСнии, Ρ‚ΠΎΠ³Π΄Π° эквивалСнтноС сопротивлСниС ΠΊΠΎΠΌΠ±ΠΈΠ½Π°Ρ†ΠΈΠΈ Π±ΡƒΠ΄Π΅Ρ‚ R / 3.

ΠŸΠ°Ρ€Π°Π»Π»Π΅Π»ΡŒΠ½ΠΎΠ΅ соСдинСниС рСзисторов Π΄Π°Π΅Ρ‚ Π·Π½Π°Ρ‡Π΅Π½ΠΈΠ΅ проводимости. ΠŸΡ€ΠΎΠ²ΠΎΠ΄ΠΈΠΌΠΎΡΡ‚ΡŒ — это Π²Π΅Π»ΠΈΡ‡ΠΈΠ½Π°, обратная ΡΠΎΠΏΡ€ΠΎΡ‚ΠΈΠ²Π»Π΅Π½ΠΈΡŽ. ΠžΠ±Ρ‹Ρ‡Π½ΠΎ ΠΎΠ½ обозначаСтся символом G. Π•Π΄ΠΈΠ½ΠΈΡ†Ρ‹ проводимости — БимСнс, прСдставлСнныС символом S. Π Π°Π½Π΅Π΅ Π΅Π΄ΠΈΠ½ΠΈΡ†Π°ΠΌΠΈ проводимости Π±Ρ‹Π»ΠΈ Mho (℧), Ρ‡Ρ‚ΠΎ ΠΎΠ·Π½Π°Ρ‡Π°Π΅Ρ‚ ΠΎΠ±Ρ€Π°Ρ‚Π½ΠΎΠ΅ написаниС Ом, Π° символ — ΠΏΠ΅Ρ€Π΅Π²Π΅Ρ€Π½ΡƒΡ‚ΠΎΠ΅ прСдставлСниС Ξ©.

НСсмотря Π½Π° Ρ‚ΠΎ, Ρ‡Ρ‚ΠΎ рСзисторы ΠΏΠ°Ρ€Π°Π»Π»Π΅Π»ΡŒΠ½ΠΎ ΠΏΠΎΠ΄ΠΊΠ»ΡŽΡ‡Π΅Π½Ρ‹ ΠΌΠ΅ΠΆΠ΄Ρƒ двумя ΡƒΠ·Π»Π°ΠΌΠΈ, прСдставлСниС этого ΠΏΠΎΠ΄ΠΊΠ»ΡŽΡ‡Π΅Π½ΠΈΡ ΠΌΠΎΠΆΠ΅Ρ‚ ΠΈΠΌΠ΅Ρ‚ΡŒ Π»ΡŽΠ±ΡƒΡŽ ΠΈΠ· ΡΠ»Π΅Π΄ΡƒΡŽΡ‰ΠΈΡ… Ρ„ΠΎΡ€ΠΌ.

ВсС Π²Ρ‹ΡˆΠ΅ΡƒΠΏΠΎΠΌΡΠ½ΡƒΡ‚Ρ‹Π΅ ΠΊΠΎΠΌΠ±ΠΈΠ½Π°Ρ†ΠΈΠΈ ΠΏΡ€Π΅Π΄ΡΡ‚Π°Π²Π»ΡΡŽΡ‚ собой ΠΏΠ°Ρ€Π°Π»Π»Π΅Π»ΡŒΠ½Ρ‹Π΅ рСзистивныС схСмы, ΠΈ всС ΠΏΡ€Π°Π²ΠΈΠ»Π° ΠΏΠ°Ρ€Π°Π»Π»Π΅Π»ΡŒΠ½Ρ‹Ρ… рСзисторов ΠΏΡ€ΠΈΠΌΠ΅Π½ΠΈΠΌΡ‹ ΠΈ ΠΊ Π²Ρ‹ΡˆΠ΅ΡƒΠΏΠΎΠΌΡΠ½ΡƒΡ‚Ρ‹ΠΌ комбинациям.

РасчСт Ρ‚ΠΎΠΊΠ°

Π’ΠΎΠΊ Π² ΠΊΠ°ΠΆΠ΄ΠΎΠΉ Π²Π΅Ρ‚Π²ΠΈ ΠΏΠ°Ρ€Π°Π»Π»Π΅Π»ΡŒΠ½ΠΎΠΉ рСзистивной Ρ†Π΅ΠΏΠΈ отличаСтся ΠΎΡ‚ Π΄Ρ€ΡƒΠ³ΠΎΠΉ. ΠŸΠΎΡΠΊΠΎΠ»ΡŒΠΊΡƒ напряТСниС Π½Π° ΠΊΠ°ΠΆΠ΄ΠΎΠΌ рСзисторС ΠΎΠ΄ΠΈΠ½Π°ΠΊΠΎΠ²ΠΎ, Ρ‚ΠΎΠΊ, ΠΏΡ€ΠΎΡ‚Π΅ΠΊΠ°ΡŽΡ‰ΠΈΠΉ Ρ‡Π΅Ρ€Π΅Π· ΠΊΠ°ΠΆΠ΄Ρ‹ΠΉ рСзистор, зависит ΠΎΡ‚ сопротивлСния этого рСзистора.Π‘Π»Π΅Π΄ΠΎΠ²Π°Ρ‚Π΅Π»ΡŒΠ½ΠΎ, Ссли Π·Π½Π°Ρ‡Π΅Π½ΠΈΠ΅ сопротивлСния Π² ΠΎΠ΄Π½ΠΎΠΉ Π²Π΅Ρ‚Π²ΠΈ отличаСтся ΠΎΡ‚ Π΄Ρ€ΡƒΠ³ΠΎΠΉ, Ρ‚ΠΎΠ³Π΄Π° Ρ‚ΠΎΠΊ Π² этих вСтвях Π±ΡƒΠ΄Π΅Ρ‚ Π΄Ρ€ΡƒΠ³ΠΈΠΌ. Π’Π΅Π»ΠΈΡ‡ΠΈΠ½Ρƒ этого Ρ‚ΠΎΠΊΠ° ΠΌΠΎΠΆΠ½ΠΎ ΠΎΠΏΡ€Π΅Π΄Π΅Π»ΠΈΡ‚ΡŒ с ΠΏΠΎΠΌΠΎΡ‰ΡŒΡŽ Π·Π°ΠΊΠΎΠ½Π° Ома.

Рассмотрим ΠΏΠ°Ρ€Π°Π»Π»Π΅Π»ΡŒΠ½ΡƒΡŽ ΡΠ΅Ρ‚ΡŒ ΠΈΠ· Π΄Π²ΡƒΡ… рСзисторов с напряТСниСм питания V ΠΌΠ΅ΠΆΠ΄Ρƒ двумя Ρ‚ΠΎΡ‡ΠΊΠ°ΠΌΠΈ A ΠΈ B.

ΠŸΡƒΡΡ‚ΡŒ I Π±ΡƒΠ΄Π΅Ρ‚ ΠΏΠΎΠ»Π½Ρ‹ΠΌ Ρ‚ΠΎΠΊΠΎΠΌ Π² ΡΠ»Π΅Π΄ΡƒΡŽΡ‰Π΅ΠΉ Ρ†Π΅ΠΏΠΈ.

ΠŸΡƒΡΡ‚ΡŒ Ρ‚ΠΎΠΊ, ΠΏΡ€ΠΎΡ‚Π΅ΠΊΠ°ΡŽΡ‰ΠΈΠΉ Ρ‡Π΅Ρ€Π΅Π· рСзистор R 1 , Ρ€Π°Π²Π΅Π½ I R1 , Π° Ρ‚ΠΎΠΊ, ΠΏΡ€ΠΎΡ‚Π΅ΠΊΠ°ΡŽΡ‰ΠΈΠΉ Ρ‡Π΅Ρ€Π΅Π· рСзистор R 2 , Ρ€Π°Π²Π΅Π½ I R2 .

Π’ΠΎΠ³Π΄Π°, согласно Π·Π°ΠΊΠΎΠ½Ρƒ ΠšΠΈΡ€Ρ…Π³ΠΎΡ„Π°, Β«ΠΏΠΎΠ»Π½Ρ‹ΠΉ Ρ‚ΠΎΠΊ, входящий Π² Ρ†Π΅ΠΏΡŒ, Ρ€Π°Π²Π΅Π½ Ρ‚ΠΎΠΊΡƒ Π½Π° Π²Ρ‹Ρ…ΠΎΠ΄Π΅ ΠΈΠ· Ρ†Π΅ΠΏΠΈΒ».

Если I T — это ΠΎΠ±Ρ‰ΠΈΠΉ Ρ‚ΠΎΠΊ, Ρ‚ΠΎ

I T = I R1 + I R2

ΠŸΠΎΡΠΊΠΎΠ»ΡŒΠΊΡƒ ΠΏΠ°Π΄Π΅Π½ΠΈΠ΅ напряТСния Π½Π° ΠΊΠ°ΠΆΠ΄ΠΎΠΌ рСзисторС ΠΎΠ΄ΠΈΠ½Π°ΠΊΠΎΠ²ΠΎ

I R1 = V / R 1

И I R2 = V / R 2

Если Ρ€Π°ΡΡΠΌΠ°Ρ‚Ρ€ΠΈΠ²Π°Ρ‚ΡŒ ΠΏΠ°Ρ€Π°Π»Π»Π΅Π»ΡŒΠ½ΡƒΡŽ Ρ€Π΅Π·ΠΈΡΡ‚ΠΈΠ²Π½ΡƒΡŽ Ρ†Π΅ΠΏΡŒ, ΡΠΎΡΡ‚ΠΎΡΡ‰ΡƒΡŽ ΠΈΠ· n рСзисторов, Ρ‚ΠΎ ΠΎΠ±Ρ‰ΠΈΠΉ Ρ‚ΠΎΠΊ Π² Ρ†Π΅ΠΏΠΈ Ρ€Π°Π²Π΅Π½

I ВсСго = I R1 + Π― R2 +….+ I Rn

Если ΠΏΠΎΡΠ»Π΅Π΄ΠΎΠ²Π°Ρ‚Π΅Π»ΡŒΠ½Ρ‹Π΅ рСзистивныС Ρ†Π΅ΠΏΠΈ Π½Π°Π·Ρ‹Π²Π°ΡŽΡ‚ΡΡ цСпями дСлитСля напряТСния, Ρ‚ΠΎ Π°Π½Π°Π»ΠΎΠ³ΠΈΡ‡Π½ΠΎ ΠΏΠ°Ρ€Π°Π»Π»Π΅Π»ΡŒΠ½Ρ‹Π΅ рСзистивныС Ρ†Π΅ΠΏΠΈ Π½Π°Π·Ρ‹Π²Π°ΡŽΡ‚ΡΡ цСпями дСлитСля Ρ‚ΠΎΠΊΠ°.

Если рассматриваСтся ΠΏΠ°Ρ€Π°Π»Π»Π΅Π»ΡŒΠ½Π°Ρ рСзистивная Ρ†Π΅ΠΏΡŒ ΠΈΠ· n рСзисторов с Ρ€Π°Π·Π½Ρ‹ΠΌ сопротивлСниСм, Ρ‚ΠΎ ΠΌΠΎΠΆΠ½ΠΎ ΠΈΠΌΠ΅Ρ‚ΡŒ n Ρ€Π°Π·Π½Ρ‹Ρ… ΠΏΡƒΡ‚Π΅ΠΉ для прохоТдСния Ρ‚ΠΎΠΊΠ° ΠΈ n Ρ€Π°Π·Π½Ρ‹Ρ… Π·Π½Π°Ρ‡Π΅Π½ΠΈΠΉ Ρ‚ΠΎΠΊΠ° Ρ‡Π΅Ρ€Π΅Π· эти ΠΏΡƒΡ‚ΠΈ. РСзисторы Π² ΠΏΠ°Ρ€Π°Π»Π»Π΅Π»ΡŒΠ½ΠΎΠΉ ΠΊΠΎΠΌΠ±ΠΈΠ½Π°Ρ†ΠΈΠΈ ΠΌΠΎΠΆΠ½ΠΎ ΠΌΠ΅Π½ΡΡ‚ΡŒ мСстами, Π½Π΅ влияя Π½Π° ΠΎΠ±Ρ‰ΠΈΠΉ Ρ‚ΠΎΠΊ ΠΈ эквивалСнтноС сопротивлСниС.

ΠŸΡ€ΠΈΠΌΠ΅Ρ€ ΠΏΠ°Ρ€Π°Π»Π»Π΅Π»ΡŒΠ½ΠΎΠ³ΠΎ ΠΏΠΎΠ΄ΠΊΠ»ΡŽΡ‡Π΅Π½ΠΈΡ рСзисторов
  1. Рассмотрим ΡΠ»Π΅Π΄ΡƒΡŽΡ‰ΡƒΡŽ схСму, Π² ΠΊΠΎΡ‚ΠΎΡ€ΠΎΠΉ Ρ‡Π΅Ρ‚Ρ‹Ρ€Π΅ рСзистора R1, R2, R3 ΠΈ R4 ΠΏΠΎΠ΄ΠΊΠ»ΡŽΡ‡Π΅Π½Ρ‹ ΠΏΠ°Ρ€Π°Π»Π»Π΅Π»ΡŒΠ½ΠΎ.

ЗначСния сопротивлСния ΠΊΠ°ΠΆΠ΄ΠΎΠ³ΠΎ рСзистора:

R1 = 10 Ом

R2 = 20 Ом

R3 = 30 Ом

R4 = 40 Ом

НапряТСниС питания V = 24 Π’

ΠžΠ±Ρ‰ΠΈΠΉ Ρ‚ΠΎΠΊ Π² схСмС ΠΌΠΎΠΆΠ½ΠΎ Ρ€Π°ΡΡΡ‡ΠΈΡ‚Π°Ρ‚ΡŒ двумя способами.

ΠŸΠ΅Ρ€Π²Ρ‹ΠΉ ΠΌΠ΅Ρ‚ΠΎΠ΄ — Π²Ρ‹Ρ‡ΠΈΡΠ»ΠΈΡ‚ΡŒ ΠΎΡ‚Π΄Π΅Π»ΡŒΠ½Ρ‹Π΅ Ρ‚ΠΎΠΊΠΈ, ΠΏΡ€ΠΎΡ‚Π΅ΠΊΠ°ΡŽΡ‰ΠΈΠ΅ Ρ‡Π΅Ρ€Π΅Π· ΠΊΠ°ΠΆΠ΄Ρ‹ΠΉ рСзистор.

Если I1 — это Ρ‚ΠΎΠΊ, ΠΏΡ€ΠΎΡ‚Π΅ΠΊΠ°ΡŽΡ‰ΠΈΠΉ Ρ‡Π΅Ρ€Π΅Π· рСзистор R1, Ρ‚ΠΎ согласно Π·Π°ΠΊΠΎΠ½Ρƒ Ома

I1 = V / R 1 = 24/10 = 2,4 A

Аналогично, Ссли I2 — это Ρ‚ΠΎΠΊ, ΠΏΡ€ΠΎΡ‚Π΅ΠΊΠ°ΡŽΡ‰ΠΈΠΉ Ρ‡Π΅Ρ€Π΅Π· рСзистор R2, Ρ‚ΠΎ согласно ΠΏΠΎ Π·Π°ΠΊΠΎΠ½Ρƒ Ома

I2 = V / R 2 = 24/20 = 1,2 A

Если I3 — это Ρ‚ΠΎΠΊ, ΠΏΡ€ΠΎΡ‚Π΅ΠΊΠ°ΡŽΡ‰ΠΈΠΉ Ρ‡Π΅Ρ€Π΅Π· рСзистор R3, Ρ‚ΠΎ ΠΏΠΎ Π·Π°ΠΊΠΎΠ½Ρƒ Ома

I3 = V / R 3 = 24 / 30 = 0,8 А

И Ссли I4 — это Ρ‚ΠΎΠΊ, ΠΏΡ€ΠΎΡ‚Π΅ΠΊΠ°ΡŽΡ‰ΠΈΠΉ Ρ‡Π΅Ρ€Π΅Π· рСзистор R4, Ρ‚ΠΎ ΠΏΠΎ Π·Π°ΠΊΠΎΠ½Ρƒ Ома

I4 = V / R 4 = 24/40 = 0.6 A

Если I Π˜Π’ΠžΠ“Πž — это ΠΏΠΎΠ»Π½Ρ‹ΠΉ Ρ‚ΠΎΠΊ Π² Ρ†Π΅ΠΏΠΈ, Ρ‚ΠΎ согласно Π·Π°ΠΊΠΎΠ½Ρƒ ΠšΠΈΡ€Ρ…Π³ΠΎΡ„Π°

I Π˜Π’ΠžΠ“Πž = I1 + I2 + I3 + I4 = 2,4 + 1,2 + 0,8 + 0,6 = 5A

Π’Ρ‚ΠΎΡ€ΠΎΠΉ ΠΌΠ΅Ρ‚ΠΎΠ΄ расчСта Ρ‚ΠΎΠΊΠ° — это ΠΎΠΏΡ€Π΅Π΄Π΅Π»Π΅Π½ΠΈΠ΅ эквивалСнтного сопротивлСния Ρ†Π΅ΠΏΠΈ.

Π­ΠΊΠ²ΠΈΠ²Π°Π»Π΅Π½Ρ‚Π½ΠΎΠ΅ сопротивлСниС Ρ†Π΅ΠΏΠΈ составляСт

1 / R EQ = (1 / R 1 ) + (1 / R 2 ) + (1 / R 3 ) + (1 / R 4 )

1 / R EQ = (1/10) + (1/20) + (1/30) + (1/40)

R EQ = 1/2.083 = 4,8 Ом

Π­Ρ‚ΠΎΡ‚ СдинствСнный рСзистор ΠΌΠΎΠΆΠ½ΠΎ ΠΈΡΠΏΠΎΠ»ΡŒΠ·ΠΎΠ²Π°Ρ‚ΡŒ для Π·Π°ΠΌΠ΅Π½Ρ‹ всСх рСзисторов Π² ΠΏΠ°Ρ€Π°Π»Π»Π΅Π»ΡŒΠ½ΠΎΠΉ ΠΊΠΎΠΌΠ±ΠΈΠ½Π°Ρ†ΠΈΠΈ.

∴ I Π˜Π’ΠžΠ“Πž = V / R EQ = 24 / 4,8 = 5 А.

Рассмотрим ΡΠ»Π΅Π΄ΡƒΡŽΡ‰ΡƒΡŽ схСму, Π² ΠΊΠΎΡ‚ΠΎΡ€ΠΎΠΉ Ρ‚Ρ€ΠΈ рСзистора R1, R2 ΠΈ R3 соСдинСны ΠΏΠ°Ρ€Π°Π»Π»Π΅Π»ΡŒΠ½ΠΎ.

Π’ΠΎΠΊ, ΠΏΡ€ΠΎΡ‚Π΅ΠΊΠ°ΡŽΡ‰ΠΈΠΉ Ρ‡Π΅Ρ€Π΅Π· R1, Ρ€Π°Π²Π΅Π½ I1 = 6A

Π’ΠΎΠΊ, ΠΏΡ€ΠΎΡ‚Π΅ΠΊΠ°ΡŽΡ‰ΠΈΠΉ Ρ‡Π΅Ρ€Π΅Π· R2, Ρ€Π°Π²Π΅Π½ I2 = 4A

Π’ΠΎΠΊ, ΠΏΡ€ΠΎΡ‚Π΅ΠΊΠ°ΡŽΡ‰ΠΈΠΉ Ρ‡Π΅Ρ€Π΅Π· R3, Ρ€Π°Π²Π΅Π½ I3 = 2A

Π’ ΠΏΠ°Ρ€Π°Π»Π»Π΅Π»ΡŒΠ½Ρ‹Ρ… рСзистивных цСпях напряТСниС Π½Π° ΠΊΠ°ΠΆΠ΄ΠΎΠΌ рСзисторС ΠΎΠ΄ΠΈΠ½Π°ΠΊΠΎΠ²ΠΎΠ΅ ΠΈ Ρ€Π°Π²Π½ΠΎ Π½Π°ΠΏΡ€ΡΠΆΠ΅Π½ΠΈΡŽ питания.

Π—Π΄Π΅ΡΡŒ напряТСниС питания V = 12 Π’.

Если V1 — это напряТСниС Π½Π° рСзисторС R1, V2 — это напряТСниС Π½Π° рСзисторС R2, Π° V3 — это напряТСниС Π½Π° рСзисторС R3, Ρ‚ΠΎΠ³Π΄Π°

V = V1 = V2 = V3 = 12 Π’

Π’ΠΎΠ³Π΄Π° ΠΏΠΎ Π·Π°ΠΊΠΎΠ½Ρƒ Ома

R1 = V 1 / I 1

R1 = 12/6

R1 = 2 Ом

R2 = V 2 / I 2

R2 = 12/4

R2 = 3 Ом

R3 = V 3 / I 3

R3 = 12/2

R3 = 6 Ом

ΠŸΡ€ΠΈΠΌΠ΅Π½Π΅Π½ΠΈΡ

ΠšΠΎΠ½Ρ†Π΅ΠΏΡ†ΠΈΡ рСзисторов Π² parallel ΠΈΡΠΏΠΎΠ»ΡŒΠ·ΡƒΠ΅Ρ‚ΡΡ ΠΏΡ€ΠΈ Π°Π½Π°Π»ΠΈΠ·Π΅ мостовой схСмы Уитстона.ΠŸΠ°Ρ€Π°Π»Π»Π΅Π»ΡŒΠ½ΠΎ соСдинСнныС рСзисторы Π΄Π΅ΠΉΡΡ‚Π²ΡƒΡŽΡ‚ ΠΊΠ°ΠΊ Ρ†Π΅ΠΏΡŒ дСлитСля Ρ‚ΠΎΠΊΠ°. Π­Ρ‚Π° тСкущая концСпция дСлитСля ΠΈΡΠΏΠΎΠ»ΡŒΠ·ΡƒΠ΅Ρ‚ΡΡ Π² Ρ‚Π°ΠΊΠΈΡ… прилоТСниях, ΠΊΠ°ΠΊ Π°Π½Π°Π»ΠΎΠ³ΠΎ-Ρ†ΠΈΡ„Ρ€ΠΎΠ²Ρ‹Π΅ ΠΏΡ€Π΅ΠΎΠ±Ρ€Π°Π·ΠΎΠ²Π°Ρ‚Π΅Π»ΠΈ ΠΈ Ρ†ΠΈΡ„Ρ€ΠΎ-Π°Π½Π°Π»ΠΎΠ³ΠΎΠ²Ρ‹Π΅ ΠΏΡ€Π΅ΠΎΠ±Ρ€Π°Π·ΠΎΠ²Π°Ρ‚Π΅Π»ΠΈ.

ΠΏΠ°Ρ€Π°Π»Π»Π΅Π»ΡŒΠ½Ρ‹Ρ… рСзисторов — Π²ΠΈΠ΄Π΅ΠΎ ΠΏΠΎ Ρ„ΠΈΠ·ΠΈΠΊΠ΅ ΠΎΡ‚ Brightstorm

ΠŸΠ°Ρ€Π°Π»Π»Π΅Π»ΡŒΠ½ΠΎ ΠΏΠΎΠ΄ΠΊΠ»ΡŽΡ‡Π΅Π½Π½Ρ‹Π΅ рСзисторы состоят ΠΈΠ· Π΄Π²ΡƒΡ… ΠΎΡ‚Π΄Π΅Π»ΡŒΠ½Ρ‹Ρ… нСзависимых Ρ†Π΅ΠΏΠ΅ΠΉ, Ρ‚Π°ΠΊ Ρ‡Ρ‚ΠΎ, ΠΊΠΎΠ³Π΄Π° Ρ‚ΠΎΠΊ достигаСт рСзистора, Ρ‚ΠΎΠΊ ΠΌΠΎΠΆΠ΅Ρ‚ Π²Ρ‹Π±ΠΈΡ€Π°Ρ‚ΡŒ, Ρ‡Π΅Ρ€Π΅Π· Ρ‡Ρ‚ΠΎ ΠΏΡ€ΠΎΡ…ΠΎΠ΄ΠΈΡ‚ΡŒ. Π‘ΠΎΠ»ΡŒΡˆΠΈΠ½ΡΡ‚Π²ΠΎ Π΄ΠΎΠΌΠΎΠ² ΠΏΠΎΠ΄ΠΊΠ»ΡŽΡ‡Π΅Π½Ρ‹ ΠΏΠ°Ρ€Π°Π»Π»Π΅Π»ΡŒΠ½ΠΎ, поэтому, Π² ΠΎΡ‚Π»ΠΈΡ‡ΠΈΠ΅ ΠΎΡ‚ рСзисторов, ΠΏΠΎΠ΄ΠΊΠ»ΡŽΡ‡Π΅Π½Π½Ρ‹Ρ… ΠΏΠΎΡΠ»Π΅Π΄ΠΎΠ²Π°Ρ‚Π΅Π»ΡŒΠ½ΠΎ, Π½Π΅ всС ΠΏΡ€ΠΈΠ±ΠΎΡ€Ρ‹ Π² Π½Π°ΡˆΠΈΡ… Π΄ΠΎΠΌΠ°Ρ… Π΄ΠΎΠ»ΠΆΠ½Ρ‹ Π±Ρ‹Ρ‚ΡŒ Π²ΠΊΠ»ΡŽΡ‡Π΅Π½Ρ‹, Ρ‡Ρ‚ΠΎΠ±Ρ‹ Ρ€Π°Π±ΠΎΡ‚Π°Π»Π° хотя Π±Ρ‹ ΠΎΠ΄Π½Π°.Π”ΠΎΠ±Π°Π²Π»Π΅Π½ΠΈΠ΅ рСзисторов ΠΏΠ°Ρ€Π°Π»Π»Π΅Π»ΡŒΠ½ΠΎ сниТаСт ΠΎΠ±Ρ‰Π΅Π΅ сопротивлСниС. НаимСньший Ρ‚ΠΎΠΊ ΠΏΡ€ΠΎΠΉΠ΄Π΅Ρ‚ Ρ‡Π΅Ρ€Π΅Π· самый большой рСзистор. Π§Ρ‚ΠΎΠ±Ρ‹ Π·Π°ΠΏΠΈΡΠ°Ρ‚ΡŒ всС рСзисторы ΠΊΠ°ΠΊ ΠΎΠ΄ΠΈΠ½ рСзистор, ΠΈΡΠΏΠΎΠ»ΡŒΠ·ΡƒΠΉΡ‚Π΅ ΡƒΡ€Π°Π²Π½Π΅Π½ΠΈΠ΅ 1 / (эффСктивноС сопротивлСниС) = 1 / (рСзистор 1) + 1 / (рСзистор 2). Π­Ρ„Ρ„Π΅ΠΊΡ‚ΠΈΠ²Π½ΠΎΠ΅ сопротивлСниС мСньшС, Ρ‡Π΅ΠΌ Ρƒ любого ΠΈΠ· Π΄Π²ΡƒΡ… ΠΎΡ‚Π΄Π΅Π»ΡŒΠ½Ρ‹Ρ… рСзисторов.

Π˜Ρ‚Π°ΠΊ, Π΄Π°Π²Π°ΠΉΡ‚Π΅ ΠΏΠΎΠ³ΠΎΠ²ΠΎΡ€ΠΈΠΌ ΠΎ рСзисторных цСпях, соСдинСнных ΠΏΠ°Ρ€Π°Π»Π»Π΅Π»ΡŒΠ½ΠΎ.Π˜Ρ‚Π°ΠΊ, Π²ΠΎ-ΠΏΠ΅Ρ€Π²Ρ‹Ρ…, Ρ‡Ρ‚ΠΎ ΠΎΠ·Π½Π°Ρ‡Π°Π΅Ρ‚ ΠΏΠ°Ρ€Π°Π»Π»Π΅Π»ΡŒ? ΠŸΠ°Ρ€Π°Π»Π»Π΅Π»ΡŒΠ½ΠΎΡΡ‚ΡŒ ΠΎΠ·Π½Π°Ρ‡Π°Π΅Ρ‚, Ρ‡Ρ‚ΠΎ Π΅ΡΡ‚ΡŒ Π²Ρ‹Π±ΠΎΡ€, это ΠΎΠ·Π½Π°Ρ‡Π°Π΅Ρ‚, Ρ‡Ρ‚ΠΎ Π² Ρ†Π΅ΠΏΠΈ Π΅ΡΡ‚ΡŒ ΠΎΡ‚Π²Π΅Ρ‚Π²Π»Π΅Π½ΠΈΠ΅, ΠΈ ΠΌΠΎΠΆΠ΅Ρ‚ Π»ΠΈ Ρ‚ΠΎΠΊ ΠΈΠ΄Ρ‚ΠΈ Π² Ρ‚ΠΎΠΌ ΠΈΠ»ΠΈ ΠΈΠ½ΠΎΠΌ Π½Π°ΠΏΡ€Π°Π²Π»Π΅Π½ΠΈΠΈ. ΠŸΠ°Ρ€Π°Π»Π»Π΅Π»ΡŒΠ½ΠΎ ΡΠ΅Ρ‚ΡŒ рСзисторов выглядит Ρ‚Π°ΠΊ. Π’Π΅Ρ‡Π΅Π½ΠΈΠ΅ Π²Ρ…ΠΎΠ΄ΠΈΡ‚, ΠΈ Ρ‚ΠΎΠ³Π΄Π° Ρƒ Π½Π΅Π³ΠΎ Π΅ΡΡ‚ΡŒ Π²Ρ‹Π±ΠΎΡ€, ΠΎΠ½ ΠΏΡ€ΠΎΡ…ΠΎΠ΄ΠΈΡ‚ ΠΌΠ΅ΠΆΠ΄Ρƒ Ρ‚Π΅ΠΌΠΈ ΠΆΠ΅ двумя Ρ‚ΠΎΡ‡ΠΊΠ°ΠΌΠΈ, Π½ΠΎ Π΅ΡΡ‚ΡŒ 2 Ρ€Π°Π·Π½Ρ‹Π΅ Π΄ΠΎΡ€ΠΎΠ³ΠΈ, ΠΏΠΎ ΠΊΠΎΡ‚ΠΎΡ€Ρ‹ΠΌ ΠΎΠ½ ΠΌΠΎΠΆΠ΅Ρ‚ ΠΈΠ΄Ρ‚ΠΈ. Π˜Ρ‚Π°ΠΊ, Π² основном ΠΏΠ°Ρ€Π°Π»Π»Π΅Π»ΡŒΠ½Ρ‹Π΅ сСти состоят ΠΈΠ· 2 ΠΎΡ‚Π΄Π΅Π»ΡŒΠ½Ρ‹Ρ… Ρ†Π΅ΠΏΠ΅ΠΉ, нСзависимых Π΄Ρ€ΡƒΠ³ ΠΎΡ‚ Π΄Ρ€ΡƒΠ³Π°. Π˜Ρ‚Π°ΠΊ, ΠΏΠ°Ρ€Π°Π»Π»Π΅Π»ΡŒΠ½Π°Ρ Ρ€Π°Π·Π½ΠΎΡΡ‚ΡŒ ΠΏΠΎΡ‚Π΅Π½Ρ†ΠΈΠ°Π»ΠΎΠ² ΠΎΠ΄ΠΈΠ½Π°ΠΊΠΎΠ²Π°, поэтому Ρ€Π°Π·Π½ΠΎΡΡ‚ΡŒ ΠΏΠΎΡ‚Π΅Π½Ρ†ΠΈΠ°Π»ΠΎΠ² ΠΌΠ΅ΠΆΠ΄Ρƒ этими двумя Ρ‚ΠΎΡ‡ΠΊΠ°ΠΌΠΈ ΠΎΠ΄ΠΈΠ½Π°ΠΊΠΎΠ²Π°, ΠΏΠΎΡ‚ΠΎΠΌΡƒ Ρ‡Ρ‚ΠΎ это ΠΎΠ΄Π½ΠΈ ΠΈ Ρ‚Π΅ ΠΆΠ΅ Π΄Π²Π΅ Ρ‚ΠΎΡ‡ΠΊΠΈ.Π­Ρ‚ΠΎ Π±Ρ‹Π»ΠΈ всСго лишь 2 Ρ€Π°Π·Π½Ρ‹Π΅ Π²Π΅Ρ‚Π²ΠΈ, ΠΊΠΎΡ‚ΠΎΡ€Ρ‹Π΅ ΠΏΡ€ΠΎΡˆΠ»ΠΈ. Π’ΠΎΠΊ Π±ΡƒΠ΄Π΅Ρ‚ Π΄ΠΎΠ±Π°Π²Π»ΡΡ‚ΡŒΡΡ, поэтому ΠΌΡ‹ Π±Ρ‹ сказали, Ρ‡Ρ‚ΠΎ i1 плюс i2 Ρ€Π°Π²Π½ΠΎ ΠΎΠ±Ρ‰Π΅ΠΌΡƒ Ρ‚ΠΎΠΊΡƒ i. i1 плюс i2 Ρ…ΠΎΡ€ΠΎΡˆΠΎ, ΠΊΠ°ΠΊ всСгда, Ρ‡Ρ‚ΠΎ ΠΌΡ‹ Ρ…ΠΎΡ‚Π΅Π»ΠΈ Π±Ρ‹ ΡΠ΄Π΅Π»Π°Ρ‚ΡŒ, Ρ‚Π°ΠΊ это Ρ€Π°ΡΡΠΌΠ°Ρ‚Ρ€ΠΈΠ²Π°Ρ‚ΡŒ эту ΠΏΠ°Ρ€Π°Π»Π»Π΅Π»ΡŒΠ½ΡƒΡŽ ΡΠ΅Ρ‚ΡŒ ΠΊΠ°ΠΊ ΠΎΠ΄ΠΈΠ½ эффСктивный рСзистор. Π˜Ρ‚Π°ΠΊ, я Ρ…ΠΎΡ‡Ρƒ, Ρ‡Ρ‚ΠΎΠ±Ρ‹ Ρ€Π°Π·Π½ΠΎΡΡ‚ΡŒ ΠΏΠΎΡ‚Π΅Π½Ρ†ΠΈΠ°Π»ΠΎΠ² Π±Ρ‹Π»Π° Ρ‚Π°ΠΊΠΎΠΉ ΠΆΠ΅, ΠΎΡ‡Π΅Π²ΠΈΠ΄Π½ΠΎ, Ρ‚ΠΎΠΊ, ΠΈΠ΄ΡƒΡ‰ΠΈΠΉ здСсь Π² ΠΌΠΎΠΉ эффСктивный рСзистор, Π±ΡƒΠ΄Π΅Ρ‚ Ρ€Π°Π²Π΅Π½ i, ΠΈ поэтому ΠΌΡ‹ собираСмся Π·Π°ΠΏΠΈΡΠ°Ρ‚ΡŒ Ρ‚ΠΎΠΊ ΠΊΠ°ΠΊ Π΄Π΅Π»ΡŒΡ‚Π° v Π½Π°Π΄ ΠΏΠΎΡ‚Π΅Π½Ρ†ΠΈΠ°Π»ΠΎΠΌ. Π Π°Π·Π½ΠΈΡ†Π° дСлится Π½Π° сопротивлСниС с нашим ΠΎΠ±ΡΠ·Π°Ρ‚Π΅Π»ΡŒΠ½Ρ‹ΠΌ Π·Π½Π°ΠΊΠΎΠΌ минус.Π’Π°ΠΊΠΈΠΌ ΠΎΠ±Ρ€Π°Π·ΠΎΠΌ, минус Ρ€Π°Π·Π½ΠΎΡΡ‚ΡŒ ΠΏΠΎΡ‚Π΅Π½Ρ†ΠΈΠ°Π»ΠΎΠ² ΠΏΠΎ r ΠΏΠ°Ρ€Π°Π»Π»Π΅Π»ΠΈ Π±ΡƒΠ΄Π΅Ρ‚ Ρ€Π°Π²Π½Π° минус разности ΠΏΠΎΡ‚Π΅Π½Ρ†ΠΈΠ°Π»ΠΎΠ² ΠΏΠΎ r1 минус Ρ€Π°Π·Π½ΠΎΡΡ‚ΡŒ ΠΏΠΎΡ‚Π΅Π½Ρ†ΠΈΠ°Π»ΠΎΠ² ΠΏΠΎ r2.

Π’Π΅ΠΏΠ΅Ρ€ΡŒ Ρ€Π°Π·Π½ΠΎΡΡ‚ΡŒ ΠΏΠΎΡ‚Π΅Π½Ρ†ΠΈΠ°Π»ΠΎΠ² такая ΠΆΠ΅, это ΠΎΠ·Π½Π°Ρ‡Π°Π΅Ρ‚, Ρ‡Ρ‚ΠΎ ΠΎΠ½Π° Π±ΡƒΠ΄Π΅Ρ‚ ΡΠΎΠΊΡ€Π°Ρ‰Π°Ρ‚ΡŒΡΡ вмСстС со Π·Π½Π°ΠΊΠΎΠΌ минус, ΠΈ это Π΄Π°Π΅Ρ‚ Π½Π°ΠΌ Ρ„ΠΎΡ€ΠΌΡƒΠ»Ρƒ для добавлСния рСзисторов ΠΏΠ°Ρ€Π°Π»Π»Π΅Π»ΡŒΠ½ΠΎ. 1 ΠΏΠΎ r ΠΏΠ°Ρ€Π°Π»Π»Π΅Π»ΠΈ Ρ€Π°Π²Π½ΠΎ 1 ΠΏΠΎ r1 плюс 1 ΠΏΠΎ r2. Π’Π°ΠΊΠΈΠΌ ΠΎΠ±Ρ€Π°Π·ΠΎΠΌ, Π΄ΠΎΠ±Π°Π²Π»Π΅Π½ΠΈΠ΅ рСзисторов ΠΏΠ°Ρ€Π°Π»Π»Π΅Π»ΡŒΠ½ΠΎ слоТнСС, Ρ‡Π΅ΠΌ ΠΈΡ… Π΄ΠΎΠ±Π°Π²Π»Π΅Π½ΠΈΠ΅ ΠΏΠΎΡΠ»Π΅Π΄ΠΎΠ²Π°Ρ‚Π΅Π»ΡŒΠ½ΠΎ. Когда я добавляю ΠΈΡ… ΠΏΠΎΡΠ»Π΅Π΄ΠΎΠ²Π°Ρ‚Π΅Π»ΡŒΠ½ΠΎ, я просто добавляю, ΠΊΠΎΠ³Π΄Π° я добавляю ΠΈΡ… ΠΏΠ°Ρ€Π°Π»Π»Π΅Π»ΡŒΠ½ΠΎ, ΠΌΡ‹ Π³ΠΎΠ²ΠΎΡ€ΠΈΠΌ, Ρ‡Ρ‚ΠΎ ΠΎΠ±Ρ€Π°Ρ‚Π½Ρ‹Π΅ суммы ΡΠΊΠ»Π°Π΄Ρ‹Π²Π°ΡŽΡ‚ΡΡ.Π’Π°ΠΊΠΈΠΌ ΠΎΠ±Ρ€Π°Π·ΠΎΠΌ, 1 ΠΏΠΎ эффСктивному ΡΠΎΠΏΡ€ΠΎΡ‚ΠΈΠ²Π»Π΅Π½ΠΈΡŽ равняСтся 1 ΠΏΠΎ r1 ΠΈ 1 ΠΏΠΎ r2. ΠžΡ‡Π΅Π½ΡŒ, ΠΎΡ‡Π΅Π½ΡŒ, ΡΠΎΠ±Π»Π°Π·Π½ΠΈΡ‚Π΅Π»ΡŒΠ½ΠΎ, Π½ΠΎ Π°Π±ΡΠΎΠ»ΡŽΡ‚Π½ΠΎ Π½Π΅Π²Π΅Ρ€Π½ΠΎ ΠΏΡ‹Ρ‚Π°Ρ‚ΡŒΡΡ ΠΏΠ΅Ρ€Π΅Π²Π΅Ρ€Π½ΡƒΡ‚ΡŒ это Π²Π²Π΅Ρ€Ρ… Π΄Π½ΠΎΠΌ ΠΈ ΡΠΊΠ°Π·Π°Ρ‚ΡŒ, Ρ‡Ρ‚ΠΎ это просто ΠΎΠ·Π½Π°Ρ‡Π°Π΅Ρ‚, Ρ‡Ρ‚ΠΎ r parallel Ρ€Π°Π²Π½ΠΎ r1 плюс r2, Ρ‡Ρ‚ΠΎ Π½Π΅Π²Π΅Ρ€Π½ΠΎ. Π’Π΅ΠΏΠ΅Ρ€ΡŒ ΠΎΠ΄Π½Π° Π²Π΅Ρ‰ΡŒ, ΠΊΠΎΡ‚ΠΎΡ€ΡƒΡŽ ΠΌΡ‹ ΠΌΠΎΠΆΠ΅ΠΌ Π΄ΡƒΠΌΠ°Ρ‚ΡŒ ΠΎ ΠΏΠ°Ρ€Π°Π»Π»Π΅Π»ΡŒΠ½Ρ‹Ρ… цСпях, ΠΊΠ°ΠΊ Π±ΡƒΠ΄Ρ‚ΠΎ Π²Ρ‹ ΠΏΡ‹Ρ‚Π°Π΅Ρ‚Π΅ΡΡŒ Π΄ΠΎΠ±Ρ€Π°Ρ‚ΡŒΡΡ ΠΈΠ· Ρ‚ΠΎΡ‡ΠΊΠΈ a Π² Ρ‚ΠΎΡ‡ΠΊΡƒ b. Если Π΅ΡΡ‚ΡŒ Ρ‚ΠΎΠ»ΡŒΠΊΠΎ ΠΎΠ΄Π½Π° Π΄ΠΎΡ€ΠΎΠ³Π°, это Π±ΡƒΠ΄Π΅Ρ‚ Ρ€Π°Π·Π΄Ρ€Π°ΠΆΠ°Ρ‚ΡŒ, любой, ΠΊΡ‚ΠΎ Ρ…ΠΎΡ‡Π΅Ρ‚ ΠΏΠ΅Ρ€Π΅ΠΉΡ‚ΠΈ ΠΈΠ· ΠΏΡƒΠ½ΠΊΡ‚Π° А Π² ΠΏΡƒΠ½ΠΊΡ‚ Π‘, Π΄ΠΎΠ»ΠΆΠ΅Π½ ΠΏΠΎΠΉΡ‚ΠΈ ΠΏΠΎ этой Π΄ΠΎΡ€ΠΎΠ³Π΅.

Если я добавлю ΠΏΠ°Ρ€Π°Π»Π»Π΅Π»ΡŒΠ½ΠΎ, это Π±ΡƒΠ΄Π΅Ρ‚ ΠΏΠΎΡ…ΠΎΠΆΠ΅ Π½Π° ΡΡ‚Ρ€ΠΎΠΈΡ‚Π΅Π»ΡŒΡΡ‚Π²ΠΎ Π½ΠΎΠ²ΠΎΠΉ Π΄ΠΎΡ€ΠΎΠ³ΠΈ, которая Ρ‚Π°ΠΊΠΆΠ΅ ΠΈΠ΄Π΅Ρ‚ ΠΎΡ‚ Ρ‚ΠΎΡ‡ΠΊΠΈ a ΠΊ Ρ‚ΠΎΡ‡ΠΊΠ΅ b, ΠΈ это Π΄Π΅ΠΉΡΡ‚Π²ΠΈΡ‚Π΅Π»ΡŒΠ½ΠΎ ΡƒΠΌΠ΅Π½ΡŒΡˆΠΈΡ‚ Ρ‚Ρ€Π°Ρ„ΠΈΠΊ.Π’Π°ΠΊ Ρ‡Ρ‚ΠΎ Π²Π°ΠΆΠ½ΠΎ ΠΏΠΎΠΌΠ½ΠΈΡ‚ΡŒ ΠΎ ΠΏΠ°Ρ€Π°Π»Π»Π΅Π»ΡŒΠ½ΠΎΡΡ‚ΠΈ, ΠΈ ΠΎΠ½Π° сразу ΠΆΠ΅ скаТСт Π½Π°ΠΌ, Ρ‡Ρ‚ΠΎ Π²Ρ‹ Π½Π΅ ΠΌΠΎΠΆΠ΅Ρ‚Π΅ просто Π΄ΠΎΠ±Π°Π²ΠΈΡ‚ΡŒ. Когда я добавляю рСзисторы ΠΏΠ°Ρ€Π°Π»Π»Π΅Π»ΡŒΠ½ΠΎ, я ΡƒΠΌΠ΅Π½ΡŒΡˆΠ°ΡŽ ΠΎΠ±Ρ‰Π΅Π΅ сопротивлСниС прямо сСйчас, Ρ‚Π°ΠΊ ΠΊΠ°ΠΊ эта ΠΏΠ°Ρ€Π°Π»Π»Π΅Π»ΡŒΠ½Π°Ρ ΡΠ΅Ρ‚ΡŒ Π±ΡƒΠ΄Π΅Ρ‚ Ρ€Π°Π±ΠΎΡ‚Π°Ρ‚ΡŒ, Ρƒ мСня Π΅ΡΡ‚ΡŒ 5 Π°ΠΌΠΏΠ΅Ρ€, ΠΈ Ρƒ мСня Π΅ΡΡ‚ΡŒ 2 рСзистора Π½Π° 15 Ом ΠΈ 10 Ом. Π Π°Π·Π½ΠΈΡ†Π° ΠΏΠΎΡ‚Π΅Π½Ρ†ΠΈΠ°Π»ΠΎΠ² такая ΠΆΠ΅, Π²Ρ‹ ΠΌΠΎΠΆΠ΅Ρ‚Π΅ Π΄ΡƒΠΌΠ°Ρ‚ΡŒ ΠΎΠ± этом ΠΊΠ°ΠΊ ΠΎ Ρ‚ΠΎΡ‡ΠΊΠ΅ b 2 Ρ€Π°Π·Π½Ρ‹Ρ… Π΄ΠΎΡ€ΠΎΠ³Π°Ρ…, Π΄ΡƒΠΌΠ°ΠΉΡ‚Π΅ ΠΎ рСзисторах ΠΊΠ°ΠΊ ΠΎ свСтофорС. Π§Π΅ΠΌ большС сопротивлСниС, Ρ‚Π΅ΠΌ большС свСтофоров Π½Π° этом ΠΏΡƒΡ‚ΠΈ ΠΈ Ρ‚Π΅ΠΌ мСньшС машин ΠΏΠΎΠ΅Π΄Π΅Ρ‚ Ρ‚ΡƒΠ΄Π°.Π’Π°ΠΊΠΈΠΌ ΠΎΠ±Ρ€Π°Π·ΠΎΠΌ, наимСньший Ρ‚ΠΎΠΊ Π±ΡƒΠ΄Π΅Ρ‚ ΠΏΡ€ΠΎΡ…ΠΎΠ΄ΠΈΡ‚ΡŒ Ρ‡Π΅Ρ€Π΅Π· наибольший рСзистор Π² ΠΏΠ°Ρ€Π°Π»Π»Π΅Π»ΡŒΠ½ΠΎΠΉ ΠΊΠΎΠΌΠ±ΠΈΠ½Π°Ρ†ΠΈΠΈ. Π˜Ρ‚Π°ΠΊ, ΠΊΠ°ΠΊ ΠΌΡ‹ собираСмся ΠΎΠΏΡ€Π΅Π΄Π΅Π»ΠΈΡ‚ΡŒ, сколько Ρ‚ΠΎΠΊΠ° ΠΏΡ€ΠΎΡ…ΠΎΠ΄ΠΈΡ‚ Ρ‡Π΅Ρ€Π΅Π· 15 ΠΈ сколько Ρ‚ΠΎΠΊΠ° ΠΏΡ€ΠΎΡ…ΠΎΠ΄ΠΈΡ‚ Ρ‡Π΅Ρ€Π΅Π· 10? На самом Π΄Π΅Π»Π΅ это Π΄Π΅ΠΉΡΡ‚Π²ΠΈΡ‚Π΅Π»ΡŒΠ½ΠΎ просто, сначала ΠΌΡ‹ собираСмся Π΄ΠΎΠ±Π°Π²ΠΈΡ‚ΡŒ эти Π΄Π²Π° рСзистора ΠΏΠ°Ρ€Π°Π»Π»Π΅Π»ΡŒΠ½ΠΎ. Π’Π΅ΠΏΠ΅Ρ€ΡŒ ΠΏΠΎΠΌΠ½ΠΈΡ‚Π΅, Ρ‡Ρ‚ΠΎ слоТСниС ΠΏΠ°Ρ€Π°Π»Π»Π΅Π»ΡŒΠ½ΠΎ — это Π½Π΅ΠΌΠ½ΠΎΠ³ΠΎ слоТно, поэтому ΠΌΡ‹ скаТСм, Ρ‡Ρ‚ΠΎ 1 ΠΏΠΎ r ΠΏΠ°Ρ€Π°Π»Π»Π΅Π»ΡŒΠ½ΠΎΠΌΡƒ Ρ€Π°Π²Π½ΠΎ 1 ΠΏΠΎ 15 плюс 1 ΠΏΠΎ 10.

Π₯ΠΎΡ€ΠΎΡˆΠΎ, Ρ‚Π΅ΠΏΠ΅Ρ€ΡŒ, ΠΊΠ°ΠΊ Π² срСднСй школС, Π½Π°ΠΌ Π½ΡƒΠΆΠ½ΠΎ ΠΏΠ΅Ρ€Π΅ΡΠ΅Ρ‡ΡŒ ΡƒΠΌΠ½ΠΎΠΆΠ΅Π½ΠΈΠ΅.Π’Π°ΠΊ Ρ‡Ρ‚ΠΎ Π±ΡƒΠ΄Π΅Ρ‚ 10 плюс 15 большС 150, Ρ…ΠΎΡ€ΠΎΡˆΠΎ. 10 плюс 15 — это 25 Π½Π° 150, ΠΈ Ссли ΠΌΡ‹ сдСлаСм это остороТно, ΠΌΡ‹ ΠΌΠΎΠΆΠ΅ΠΌ ΠΎΡ‚ΠΌΠ΅Π½ΠΈΡ‚ΡŒ. Π­Ρ‚ΠΎ Π΄Π°Π΅Ρ‚ Π½Π°ΠΌ ΠΎΠ΄Π½Ρƒ ΡˆΠ΅ΡΡ‚ΡƒΡŽ. 1 ΠΏΠΎ r ΠΏΠ°Ρ€Π°Π»Π»Π΅Π»ΠΈ — это 1 ΠΏΠΎ 6, это ΠΎΠ·Π½Π°Ρ‡Π°Π΅Ρ‚, Ρ‡Ρ‚ΠΎ r ΠΏΠ°Ρ€Π°Π»Π»Π΅Π»ΡŒ равняСтся 6, Ρ…ΠΎΡ€ΠΎΡˆΠΎ. ΠžΠ±Ρ€Π°Ρ‚ΠΈΡ‚Π΅ Π²Π½ΠΈΠΌΠ°Π½ΠΈΠ΅, Ρ‡Ρ‚ΠΎ нашС эффСктивноС сопротивлСниС мСньшС, Ρ‡Π΅ΠΌ любоС ΠΈΠ· Π΄Π²ΡƒΡ… сопротивлСний, ΠΊΠΎΡ‚ΠΎΡ€Ρ‹Π΅ ΠΌΡ‹ слоТили вмСстС, мСньшС, Ρ‡Π΅ΠΌ любоС ΠΈΠ· Π½ΠΈΡ…, Ρ…ΠΎΡ€ΠΎΡˆΠΎ. И Π½Π° самом Π΄Π΅Π»Π΅ это число всСгда Π±ΡƒΠ΄Π΅Ρ‚ Π»Π΅ΠΆΠ°Ρ‚ΡŒ ΠΌΠ΅ΠΆΠ΄Ρƒ этим, Π΄Π΅Π»Π΅Π½Π½Ρ‹ΠΌ Π½Π° 2, ΠΈ этим, Π΄Π΅Π»Π΅Π½Π½Ρ‹ΠΌ Π½Π° 2, 5, 7 с ΠΏΠΎΠ»ΠΎΠ²ΠΈΠ½ΠΎΠΉ, 6. Π₯ΠΎΡ€ΠΎΡˆΠΎ, это всСгда Π±ΡƒΠ΄Π΅Ρ‚ Ρ€Π°Π±ΠΎΡ‚Π°Ρ‚ΡŒ Ρ‚Π°ΠΊΠΈΠΌ ΠΎΠ±Ρ€Π°Π·ΠΎΠΌ. Π₯ΠΎΡ€ΠΎΡˆΠΎ, Ρ‚Π΅ΠΏΠ΅Ρ€ΡŒ, ΠΊΠΎΠ³Π΄Π° Ρƒ нас Π΅ΡΡ‚ΡŒ эффСктивноС сопротивлСниС, ΠΊΠ°ΠΊ ΠΌΡ‹ собираСмся ΠΎΠΏΡ€Π΅Π΄Π΅Π»ΡΡ‚ΡŒ Ρ‚ΠΎΠΊ Ρ‡Π΅Ρ€Π΅Π· ΠΊΠ°ΠΆΠ΄ΠΎΠ³ΠΎ ΠΈΠ· этих ΠΏΠ°Ρ€Π½Π΅ΠΉ? Π§Ρ‚ΠΎ ΠΆ, ΠΌΡ‹ собираСмся ΠΈΡΠΏΠΎΠ»ΡŒΠ·ΠΎΠ²Π°Ρ‚ΡŒ идСю ΠΎ Ρ‚ΠΎΠΌ, Ρ‡Ρ‚ΠΎ Ρ€Π°Π·Π½ΠΎΡΡ‚ΡŒ ΠΏΠΎΡ‚Π΅Π½Ρ†ΠΈΠ°Π»ΠΎΠ² Π΄ΠΎΠ»ΠΆΠ½Π° Π±Ρ‹Ρ‚ΡŒ ΠΎΠ΄ΠΈΠ½Π°ΠΊΠΎΠ²ΠΎΠΉ ΠΏΠ°Ρ€Π°Π»Π»Π΅Π»ΡŒΠ½ΠΎ всСгда, всСгда, всСгда.Π’Π°ΠΊ Π² Ρ‡Π΅ΠΌ ΠΆΠ΅ Ρ€Π°Π·Π½ΠΈΡ†Π° ΠΏΠΎΡ‚Π΅Π½Ρ†ΠΈΠ°Π»ΠΎΠ²? Ну, Ρ€Π°Π·Π½ΠΎΡΡ‚ΡŒ ΠΏΠΎΡ‚Π΅Π½Ρ†ΠΈΠ°Π»ΠΎΠ² Π΄ΠΎΠ»ΠΆΠ½Π° Π±Ρ‹Ρ‚ΡŒ 5 умноТСнная Π½Π° 6, ΠΏΠΎΡ‚ΠΎΠΌΡƒ Ρ‡Ρ‚ΠΎ эта ΡΠ΅Ρ‚ΡŒ Π½Π° самом Π΄Π΅Π»Π΅ ΠΈΠΌΠ΅Π΅Ρ‚ Ρ‚ΠΎΠ»ΡŒΠΊΠΎ 6 Ом ΠΈ 5 Π°ΠΌΠΏΠ΅Ρ€. Ну, 5 ΡƒΠΌΠ½ΠΎΠΆΠΈΡ‚ΡŒ Π½Π° 6 Ρ€Π°Π²Π½ΠΎ 30, Ρ‚Π°ΠΊ Ρ‡Ρ‚ΠΎ это ΠΎΠ·Π½Π°Ρ‡Π°Π΅Ρ‚, Ρ‡Ρ‚ΠΎ Ρ€Π°Π·Π½ΠΎΡΡ‚ΡŒ ΠΏΠΎΡ‚Π΅Π½Ρ†ΠΈΠ°Π»ΠΎΠ² Π΄ΠΎΠ»ΠΆΠ½Π° Π±Ρ‹Ρ‚ΡŒ 30.

Π₯ΠΎΡ€ΠΎΡˆΠΎ, Ρ‡Ρ‚ΠΎΠ±Ρ‹ ΡΠ΄Π΅Π»Π°Ρ‚ΡŒ Ρ€Π°Π·Π½ΠΎΡΡ‚ΡŒ ΠΏΠΎΡ‚Π΅Π½Ρ†ΠΈΠ°Π»ΠΎΠ² 30 Π²ΠΎΠ»ΡŒΡ‚ Π½Π° рСзисторС 15 Ом, 2 Π°ΠΌΠΏΠ΅Ρ€Π°, ΠΏΠΎΡ‚ΠΎΠΌΡƒ Ρ‡Ρ‚ΠΎ 2 ΡƒΠΌΠ½ΠΎΠΆΠΈΡ‚ΡŒ Π½Π° 15 Ρ€Π°Π²Π½ΠΎ 30, ΠΊΠ°ΠΊ насчСт Ρ‚ΠΎΠ³ΠΎ, Π²Π½ΠΈΠ·Ρƒ, я Ρ…ΠΎΡ‡Ρƒ 30 Π²ΠΎΠ»ΡŒΡ‚ Π½Π° Π½Π΅ΠΌ, Ρ‚Π°ΠΊ сколько Π°ΠΌΠΏΠ΅Ρ€ ΠΌΠ½Π΅ Π½ΡƒΠΆΠ½ΠΎ? 3 Π°ΠΌΠΏΠ΅Ρ€Π°, ΠΈ, ΠΊΠΎΠ½Π΅Ρ‡Π½ΠΎ ΠΆΠ΅, это слСдуСт Π½Π°ΡˆΠ΅ΠΌΡƒ ΠΏΡ€Π΅Π΄Ρ‹Π΄ΡƒΡ‰Π΅ΠΌΡƒ Ρ€Π΅ΡˆΠ΅Π½ΠΈΡŽ, Ρ‡Ρ‚ΠΎ Ρ‚Π΅ΠΊΡƒΡ‰ΠΈΠ΅ Π΄ΠΎΠ±Π°Π²Π»Π΅Π½Π½Ρ‹Π΅ 5 Π°ΠΌΠΏΠ΅Ρ€ ΠΈΠ΄ΡƒΡ‚ Π²ΠΎ 2 ΠΈ 1 Π²Π΅Ρ‚Π²ΠΈ, 3 — Π² Π΄Ρ€ΡƒΠ³ΡƒΡŽ.Π₯ΠΎΡ€ΠΎΡˆΠΎ, Π΅Ρ‰Π΅ ΠΎΠ΄ΠΈΠ½ способ Π΄ΡƒΠΌΠ°Ρ‚ΡŒ ΠΎ ΠΏΠ°Ρ€Π°Π»Π»Π΅Π»ΡŒΠ½Ρ‹Ρ… цСпях — это Π΄ΠΎΠΌΠ°, это способ ΠΏΠΎΠ΄ΠΊΠ»ΡŽΡ‡Π΅Π½ΠΈΡ Π΄ΠΎΠΌΠΎΠ², ΠΏΠΎΡ‚ΠΎΠΌΡƒ Ρ‡Ρ‚ΠΎ я Π½Π΅ Ρ…ΠΎΡ‡Ρƒ, Ρ‡Ρ‚ΠΎΠ±Ρ‹ ΠΌΠ½Π΅ Π±Ρ‹Π»ΠΎ Π½Π΅ΠΎΠ±Ρ…ΠΎΠ΄ΠΈΠΌΠΎ Π²ΠΊΠ»ΡŽΡ‡Π°Ρ‚ΡŒ Ρ‚Π΅Π»Π΅Π²ΠΈΠ·ΠΎΡ€, Ρ‡Ρ‚ΠΎΠ±Ρ‹ посудомоСчная машина Ρ€Π°Π±ΠΎΡ‚Π°Π»Π° ΠΏΡ€Π°Π²ΠΈΠ»ΡŒΠ½ΠΎ, поэтому ΠΌΡ‹ ΠΏΠΎΠ΄ΠΊΠ»ΡŽΡ‡Π°Π΅ΠΌ эти 2 схСмы ΠΎΡ‚Π΄Π΅Π»ΡŒΠ½ΠΎ. КаТдая Ρ€ΠΎΠ·Π΅Ρ‚ΠΊΠ° Π² вашСм Π΄ΠΎΠΌΠ΅ — это ΠΎΡ‚Π΄Π΅Π»ΡŒΠ½Π°Ρ ΠΏΠ°Ρ€Π°Π»Π»Π΅Π»ΡŒΠ½Π°Ρ Π²Π΅Ρ‚Π²ΡŒ ΠΎΠ΄Π½ΠΎΠΉ ΠΈ Ρ‚ΠΎΠΉ ΠΆΠ΅ Ρ†Π΅ΠΏΠΈ, поэтому ΠΌΡ‹ заставляСм ТильС Ρ€Π°Π±ΠΎΡ‚Π°Ρ‚ΡŒ. Π₯ΠΎΡ€ΠΎΡˆΠΎ, это ΠΏΠ°Ρ€Π°Π»Π»Π΅Π»ΡŒΠ½Ρ‹Π΅ схСмы рСзисторов.

Как Ρ€Π°ΡΡΡ‡ΠΈΡ‚Π°Ρ‚ΡŒ ΠΏΠΎΠ»Π½ΠΎΠ΅ сопротивлСниС ΠΏΠ°Ρ€Π°Π»Π»Π΅Π»ΡŒΠ½ΠΎΠΉ Ρ†Π΅ΠΏΠΈ?

Как Ρ€Π°ΡΡΡ‡ΠΈΡ‚Π°Ρ‚ΡŒ ΠΏΠΎΠ»Π½ΠΎΠ΅ сопротивлСниС ΠΏΠ°Ρ€Π°Π»Π»Π΅Π»ΡŒΠ½ΠΎΠΉ Ρ†Π΅ΠΏΠΈ?

Π­Ρ„Ρ„Π΅ΠΊΡ‚ΠΈΠ²Π½ΠΎΠ΅ сопротивлСниС рСзисторов, ΠΏΠΎΠ΄ΠΊΠ»ΡŽΡ‡Π΅Π½Π½Ρ‹Ρ… ΠΏΠ°Ρ€Π°Π»Π»Π΅Π»ΡŒΠ½ΠΎ

  1. Π’ ΠΏΠ°Ρ€Π°Π»Π»Π΅Π»ΡŒΠ½ΠΎΠΉ Ρ†Π΅ΠΏΠΈ Π΅ΡΡ‚ΡŒ Ρ‚Ρ€ΠΈ Π²Π°ΠΆΠ½Ρ‹Ρ… характСристики:
    (a) Π Π°Π·Π½ΠΎΡΡ‚ΡŒ ΠΏΠΎΡ‚Π΅Π½Ρ†ΠΈΠ°Π»ΠΎΠ² ΠΎΠ΄ΠΈΠ½Π°ΠΊΠΎΠ²Π° Π½Π° ΠΊΠ°ΠΆΠ΄ΠΎΠΌ рСзисторС.
    (b) Π’ΠΎΠΊ, ΠΊΠΎΡ‚ΠΎΡ€Ρ‹ΠΉ ΠΏΡ€ΠΎΡ…ΠΎΠ΄ΠΈΡ‚ Ρ‡Π΅Ρ€Π΅Π· ΠΊΠ°ΠΆΠ΄Ρ‹ΠΉ рСзистор, ΠΎΠ±Ρ€Π°Ρ‚Π½ΠΎ ΠΏΡ€ΠΎΠΏΠΎΡ€Ρ†ΠΈΠΎΠ½Π°Π»Π΅Π½ ΡΠΎΠΏΡ€ΠΎΡ‚ΠΈΠ²Π»Π΅Π½ΠΈΡŽ рСзистора.
    (c) ΠŸΠΎΠ»Π½Ρ‹ΠΉ Ρ‚ΠΎΠΊ Π² Ρ†Π΅ΠΏΠΈ Ρ€Π°Π²Π΅Π½ суммС Ρ‚ΠΎΠΊΠΎΠ², проходящих Ρ‡Π΅Ρ€Π΅Π· рСзисторы Π² Π΅Π΅ ΠΏΠ°Ρ€Π°Π»Π»Π΅Π»ΡŒΠ½Ρ‹Ρ… вСтвях.
  2. Когда Π΄Π²Π° ΠΈΠ»ΠΈ Π±ΠΎΠ»Π΅Π΅ сопротивлСния ΠΏΠΎΠ΄ΠΊΠ»ΡŽΡ‡Π΅Π½Ρ‹ ΠΌΠ΅ΠΆΠ΄Ρƒ двумя ΠΎΠ±Ρ‰ΠΈΠΌΠΈ Ρ‚ΠΎΡ‡ΠΊΠ°ΠΌΠΈ Ρ‚Π°ΠΊ, Ρ‡Ρ‚ΠΎ ΠΊ ΠΊΠ°ΠΆΠ΄ΠΎΠΉ ΠΈΠ· Π½ΠΈΡ… ΠΏΡ€ΠΈΠ»ΠΎΠΆΠ΅Π½Π° одинаковая Ρ€Π°Π·Π½ΠΎΡΡ‚ΡŒ ΠΏΠΎΡ‚Π΅Π½Ρ†ΠΈΠ°Π»ΠΎΠ², ΠΎΠ½ΠΈ ΡΡ‡ΠΈΡ‚Π°ΡŽΡ‚ΡΡ ΠΏΠΎΠ΄ΠΊΠ»ΡŽΡ‡Π΅Π½Π½Ρ‹ΠΌΠΈ ΠΏΠ°Ρ€Π°Π»Π»Π΅Π»ΡŒΠ½ΠΎ.
    Когда такая комбинация сопротивлСний ΠΏΠΎΠ΄ΠΊΠ»ΡŽΡ‡Π΅Π½Π° ΠΊ Π±Π°Ρ‚Π°Ρ€Π΅Π΅, всС сопротивлСния ΠΈΠΌΠ΅ΡŽΡ‚ ΠΎΠ΄ΠΈΠ½Π°ΠΊΠΎΠ²ΡƒΡŽ Ρ€Π°Π·Π½ΠΎΡΡ‚ΡŒ ΠΏΠΎΡ‚Π΅Π½Ρ†ΠΈΠ°Π»ΠΎΠ² Π½Π° ΠΊΠΎΠ½Ρ†Π°Ρ….
  3. Π’Ρ‹Π²ΠΎΠ΄ матСматичСского выраТСния ΠΏΠ°Ρ€Π°Π»Π»Π΅Π»ΡŒΠ½ΠΎΠΉ ΠΊΠΎΠΌΠ±ΠΈΠ½Π°Ρ†ΠΈΠΈ:
    ΠŸΡƒΡΡ‚ΡŒ, V Π±ΡƒΠ΄Π΅Ρ‚ Ρ€Π°Π·Π½ΠΎΡΡ‚ΡŒΡŽ ΠΏΠΎΡ‚Π΅Π½Ρ†ΠΈΠ°Π»ΠΎΠ² ΠΌΠ΅ΠΆΠ΄Ρƒ двумя ΠΎΠ±Ρ‰ΠΈΠΌΠΈ Ρ‚ΠΎΡ‡ΠΊΠ°ΠΌΠΈ A ΠΈ B. Π—Π°Ρ‚Π΅ΠΌ ΠΈΠ· Π·Π°ΠΊΠΎΠ½Π° Ома
    Π’ΠΎΠΊ, проходящий Ρ‡Π΅Ρ€Π΅Π· R 1 , I 1 = V / R 1 … (i)
    Π’ΠΎΠΊ, ΠΏΡ€ΠΎΡ‚Π΅ΠΊΠ°ΡŽΡ‰ΠΈΠΉ Ρ‡Π΅Ρ€Π΅Π· R 2 , I 2 = V / R 2 … (ii)
    Π’ΠΎΠΊ, ΠΏΡ€ΠΎΡ‚Π΅ΠΊΠ°ΡŽΡ‰ΠΈΠΉ Ρ‡Π΅Ρ€Π΅Π· R 3 , I 3 = V / R 3 … (iii)
  4. Если R — эквивалСнтноС сопротивлСниС, Ρ‚ΠΎ ΠΏΠΎ Π·Π°ΠΊΠΎΠ½Ρƒ Ома ΠΏΠΎΠ»Π½Ρ‹ΠΉ Ρ‚ΠΎΠΊ, ΠΏΡ€ΠΎΡ‚Π΅ΠΊΠ°ΡŽΡ‰ΠΈΠΉ ΠΏΠΎ Ρ†Π΅ΠΏΠΈ, опрСдСляСтся Π²Ρ‹Ρ€Π°ΠΆΠ΅Π½ΠΈΠ΅ΠΌ
    I = V / R… (iv)
    ΠΈ I = I 1 + I 2 + I 3 … (v)
  5. ΠŸΠΎΠ΄ΡΡ‚Π°Π²Π»ΡΡ значСния I, I 1 , I 2 ΠΈ I 3 Π² ΡƒΡ€Π°Π²Π½Π΅Π½ΠΈΠ΅.(v),
    \ (\ frac {\ text {V}} {\ text {R}} = \ frac {\ text {V}} {{{\ text {R}} _ {\ text {1}} }} + \ frac {\ text {V}} {{{\ text {R}} _ {\ text {2}}}} + \ frac {\ text {V}} {{{\ text {R}} _ {\ text {3}}}} \ text {} …… .. \ text {(vi)} \)
  6. ΠžΡ‚ΠΌΠ΅Π½ΡΡ ΠΎΠ±Ρ‰ΠΈΠΉ Ρ‚Π΅Ρ€ΠΌΠΈΠ½ V, ΠΏΠΎΠ»ΡƒΡ‡Π°Π΅ΠΌ
    \ (\ frac {\ text {1}} {\ text {R}} = \ frac {\ text {1}} {{{\ text {R}} _ {\ тСкст {1}}}} + \ frac {\ text {1}} {{{\ text {R}} _ {2}}} + \ frac {\ text {1}} {{{\ text {R} } _ {3}}} \)
    Π­ΠΊΠ²ΠΈΠ²Π°Π»Π΅Π½Ρ‚Π½ΠΎΠ΅ сопротивлСниС ΠΏΠ°Ρ€Π°Π»Π»Π΅Π»ΡŒΠ½ΠΎΠΉ ΠΊΠΎΠΌΠ±ΠΈΠ½Π°Ρ†ΠΈΠΈ сопротивлСний мСньшС, Ρ‡Π΅ΠΌ ΠΊΠ°ΠΆΠ΄ΠΎΠ΅ ΠΈΠ· всСх ΠΎΡ‚Π΄Π΅Π»ΡŒΠ½Ρ‹Ρ… сопротивлСний.
  7. ЭквивалСнтная схСма ΠΏΠΎΠΊΠ°Π·Π°Π½Π° Π½Π° рисункС.

Π’Π°ΠΆΠ½Ρ‹Π΅ Ρ€Π΅Π·ΡƒΠ»ΡŒΡ‚Π°Ρ‚Ρ‹ ΠΎ ΠΏΠ°Ρ€Π°Π»Π»Π΅Π»ΡŒΠ½ΠΎΠΉ ΠΊΠΎΠΌΠ±ΠΈΠ½Π°Ρ†ΠΈΠΈ:

  1. ΠŸΠΎΠ»Π½Ρ‹ΠΉ Ρ‚ΠΎΠΊ Π² Ρ†Π΅ΠΏΠΈ Ρ€Π°Π²Π΅Π½ суммС ΠΏΡ€ΠΎΡ‚Π΅ΠΊΠ°ΡŽΡ‰ΠΈΡ… ΠΏΠΎ Π½Π΅ΠΉ Ρ‚ΠΎΠΊΠΎΠ².
  2. Π’ ΠΏΠ°Ρ€Π°Π»Π»Π΅Π»ΡŒΠ½ΠΎΠΉ ΠΊΠΎΠΌΠ±ΠΈΠ½Π°Ρ†ΠΈΠΈ рСзисторов напряТСниС (ΠΈΠ»ΠΈ Ρ€Π°Π·Π½ΠΎΡΡ‚ΡŒ ΠΏΠΎΡ‚Π΅Π½Ρ†ΠΈΠ°Π»ΠΎΠ²) Π½Π° ΠΊΠ°ΠΆΠ΄ΠΎΠΌ рСзисторС ΠΎΠ΄ΠΈΠ½Π°ΠΊΠΎΠ²ΠΎ ΠΈ Ρ€Π°Π²Π½ΠΎ ΠΏΡ€ΠΈΠ»ΠΎΠΆΠ΅Π½Π½ΠΎΠΌΡƒ Π½Π°ΠΏΡ€ΡΠΆΠ΅Π½ΠΈΡŽ, Ρ‚ΠΎ Π΅ΡΡ‚ΡŒ V 1 = V 2 = V 3 = V
  3. Π’ΠΎΠΊ, ΠΏΡ€ΠΎΡ‚Π΅ΠΊΠ°ΡŽΡ‰ΠΈΠΉ Ρ‡Π΅Ρ€Π΅Π· ΠΊΠ°ΠΆΠ΄Ρ‹ΠΉ рСзистор, ΠΎΠ±Ρ€Π°Ρ‚Π½ΠΎ ΠΏΡ€ΠΎΠΏΠΎΡ€Ρ†ΠΈΠΎΠ½Π°Π»Π΅Π½ Π΅Π³ΠΎ ΡΠΎΠΏΡ€ΠΎΡ‚ΠΈΠ²Π»Π΅Π½ΠΈΡŽ, поэтому Ρ‡Π΅ΠΌ Π²Ρ‹ΡˆΠ΅ сопротивлСниС рСзистора, Ρ‚Π΅ΠΌ мСньшС Ρ‚ΠΎΠΊ, ΠΏΡ€ΠΎΡ‚Π΅ΠΊΠ°ΡŽΡ‰ΠΈΠΉ Ρ‡Π΅Ρ€Π΅Π· Π½Π΅Π³ΠΎ.

Π”ΠΎΠ±Π°Π²ΠΈΡ‚ΡŒ ΠΊΠΎΠΌΠΌΠ΅Π½Ρ‚Π°Ρ€ΠΈΠΉ

Π’Π°Ρˆ адрСс email Π½Π΅ Π±ΡƒΠ΄Π΅Ρ‚ ΠΎΠΏΡƒΠ±Π»ΠΈΠΊΠΎΠ²Π°Π½. ΠžΠ±ΡΠ·Π°Ρ‚Π΅Π»ΡŒΠ½Ρ‹Π΅ поля ΠΏΠΎΠΌΠ΅Ρ‡Π΅Π½Ρ‹ *